aap mcq 2007

537
2007 PREP SA on CD-ROM Question: 1 You are precepting a resident who has just evaluated a 4-year-old incompletely immunized immigrant boy who has classic varicella lesions and a history that is consistent with this diagnosis. Of the following, the MOST accurate statement is that A. lesions of both varicella and smallpox follow a 7- to 10-day course from eruption to resolution B. lesions of both varicella and smallpox frequently produce deep, pitted scars C. varicella lesions appear in stages or crops; smallpox lesions are uniformly in the same stage of development D. varicella lesions are concentrated on the face; smallpox lesions are concentrated over bony prominences E. varicella lesions are transient vesicles; smallpox lesions are persistent pustules until resolution of the illness Copyright © 2007 by the American Academy of Pediatrics page 1

Upload: kriti-gupta

Post on 21-Apr-2015

1.456 views

Category:

Documents


18 download

TRANSCRIPT

Page 1: AAP MCQ 2007

2007 PREP SA on CD-ROM

Question: 1

You are precepting a resident who has just evaluated a 4-year-old incompletely immunizedimmigrant boy who has classic varicella lesions and a history that is consistent with thisdiagnosis.

Of the following, the MOST accurate statement is that

A. lesions of both varicella and smallpox follow a 7- to 10-day course from eruption to resolution

B. lesions of both varicella and smallpox frequently produce deep, pitted scars

C. varicella lesions appear in stages or crops; smallpox lesions are uniformly in the same stageof development

D. varicella lesions are concentrated on the face; smallpox lesions are concentrated over bonyprominences

E. varicella lesions are transient vesicles; smallpox lesions are persistent pustules until resolutionof the illness

Copyright © 2007 by the American Academy of Pediatrics page 1

Page 2: AAP MCQ 2007

2007 PREP SA on CD-ROM

Preferred Response: CCritique: 1

Historically, the disease most often confused with severe varicella was smallpox. Smallpox,caused by the variola virus, a member of the orthopoxvirus family, has an incubation period of 7to 17 days. During the incubation period, virus replicates in the upper respiratory tract. A primaryviremia ensues, during which the liver and spleen are seeded. A secondary viremia follows, theskin is seeded, and the classic eruption appears.

Initial symptoms of smallpox may include fever as high as 104°F (40°C), generalizedmalaise, severe headache, vomiting, and backache. Characteristic skin findings appear 1 to 2days after the onset of fever. The rash begins on the face and then spreads to involve theextremities and trunk. Initially, lesions are erythematous macules; they evolve into papules,vesicles, and firm pustules (Item C1A). Crusts form at 8 or 9 days and persist for 3 to 4 weeks.As crusts separate, patients often are left with significant scars or depigmentation. Aspects ofsmallpox that differentiate it from severe varicella are that the majority of lesions are observedon the face and extremities (with lesser numbers on the trunk) and that all lesions are in a similarstage of development.

In contrast, the lesions of varicella erupt initially on the trunk and later appear on the faceand extremities. Lesions are erythematous papules that evolve to form superficial vesicles,pustules, and crusts. In varicella (unlike smallpox), lesions are observed in varying stages ofdevelopment (ie, as older lesions crust, new lesions appear) (Item C1B). By 7 to 10 days afterinfection, all lesions have crusted. Permanent scars are rare, occurring only when lesions havebeen secondarily infected with bacteria.

References:

American Academy of Pediatrics. Smallpox (variola). In: Pickering LK, ed. Red Book: 2006Report of the Committee on Infectious Diseases. 27th ed. Elk Grove Village, Ill: AmericanAcademy of Pediatrics; 2006: 591-595

Cieslak J, Henretig FM. Biologic and chemical terrorism. In: Behrman RE, Kliegman RM, JensonHB, eds. Nelson Textbook of Pediatrics. 17th ed. Philadelphia, Pa: WB Saunders Co; 2004:2378-2385

Myers MG, Stanberry LR, Sevard JF. Varicella-zoster virus. In: Behrman RE, Kliegman RM,Jenson HB, eds. Nelson Textbook of Pediatrics. 17th ed. Philadelphia, Pa: WB Saunders Co;2004:1057-1062

Paller AS, Mancini AJ. Viral diseases of the skin. In: Hurwitz Clinical Pediatric Dermatology. 3rded. Philadelphia, Pa: Elsevier Inc; 2006: 397-423

Copyright © 2007 by the American Academy of Pediatrics page 2

Page 3: AAP MCQ 2007

2007 PREP SA on CD-ROM

Question: 2

A 15-year-old boy presents with melena and anemia. Endoscopy demonstrates a nodulargastritis of the antrum (Item Q2A) and an ulcer. Biopsies of the antrum demonstrate spiral-shaped organisms consistent with Helicobacter pylori (Item Q2B). You prescribe amoxicillin,clarithromycin, and lansoprazole for 2 weeks. At a follow-up visit, the family asks whether thetreatment has been successful in eradicating the organism.

Of the following, the PREFERRED noninvasive test to evaluate whether the pathogen has beeneradicated is

A. fecal Campylobacter-like organisms (CLO) test

B. fecal H pylori antigen

C. salivary H pylori antibody concentrations

D. serum H pylori immunoglobulin G serology

E. serum H pylori urease concentrations

Copyright © 2007 by the American Academy of Pediatrics page 3

Page 4: AAP MCQ 2007

2007 PREP SA on CD-ROM

Preferred Response: BCritique: 2

Helicobacter pylori infection is a known risk factor for gastritis and duodenal ulcers in childrenand adults. Rarely, and primarily in older adulthood, H pylori also is associated with a gastriclymphoma of the mucosal-associated lymphoid tissue (MALToma). The “gold standard” for thediagnosis of H pylori infection of the stomach is endoscopy with biopsy. Endoscopy may show anodular gastritis of the antrum (Item C2A), and histology of the gastric mucosa demonstrates thecharacteristic curved organisms (Item C2B) in the gastric glands.

Because endoscopy is invasive, other surrogate markers of infection have been identified.Of the options offered, the H pylori fecal antigen is the best to document eradication in apreviously treated host. Patients colonized with H pylori have detectable antigen in their stoolthat disappears upon eradication of the organism. H pylori immunoglobulin G serology (serumantibody) is a useful marker for epidemiologic studies of past or current infection, but itssensitivity and positive predictive value in children is suboptimal. The same is true for salivaryantibody. Accordingly, a positive antibody screen should be confirmed by a second test (eitherfecal antigen, urea breath test, or endoscopy). The Campylobacter-like organisms (CLO) test isperformed on a duodenal biopsy. In the CLO test, the duodenal biopsy specimen is placed in atest tube containing chemical reagents. The H pylori bacteria convert urea to ammonia andcarbon dioxide via their urease enzyme, and the alkalinity of the ammonia can be detected usingan indicator dye. The CLO test cannot be performed on feces. Serum urease concentrationsare not helpful in identifying H pylori, which is a mucosal bacterium.

Diagnosis, treatment, and eradication of H pylori are well summarized in the AmericanAcademy of Pediatrics Red Book and in the North American Society for PediatricGastroenterology practice guideline. Patients who have documented ulcers should be tested forH pylori and the organism eradicated if found, but it is unclear if asymptomatic children colonizedwith H pylori need to be treated. Therapy is given for 14 days and should include a proton pumpinhibitor (eg, omeprazole, lansoprazole, pantoprazole) and two antibiotics (eg, tetracycline +clarithromycin, amoxicillin + metronidazole, amoxicillin + clarithromycin) (Item C2C). Treatmentfailures are common, either because of resistant bacteria or because of poor compliance withthe regimen. Therefore, testing for eradication of the organism (either by fecal antigen, ureasebreath test, or endoscopy) should be performed more than 1 month after therapy has beencompleted.

References:

American Academy of Pediatrics. Helicobacter pylori infections. In: Pickering LK, ed. Red Book:2006 Report of the Committee on Infectious Diseases. 27th ed. Elk Grove Village, Ill: AmericanAcademy of Pediatrics; 2006:321-322

Gold BD, Colletti RB, Abbott M, et al. Helicobacter pylori infection in children: recommendationsfor diagnosis and treatment. J Pediatr Gastroenterol Nutr. 2000;31:490-497. Available at:http://www.jpgn.org/pt/re/jpgn/fulltext.00005176-200011000-00007.htm

Copyright © 2007 by the American Academy of Pediatrics page 4

Page 5: AAP MCQ 2007

2007 PREP SA on CD-ROM

Question: 3

A term newborn is delivered by emergent cesarean section because of intrauterine growthrestriction, oligohydramnios, and nonreassuring fetal heart rate monitoring in labor. Deliveryroom resuscitation includes endotracheal intubation and assisted ventilation with 100% oxygen,chest compressions, intravenous epinephrine, and volume expansion. Apgar scores are 1, 2,and 3 at 1, 5, and 10 minutes, respectively. An umbilical cord arterial blood gas measurementdocuments a pH of 6.9 and a base deficit of 20 mmol/L. At 12 hours of age, the infantdemonstrates tonic-clonic convulsive activity of the arms and legs with a concomitant decreasein heart rate and bedside pulse oximetry saturation.

Of the following, the MOST likely cause for this infant's seizure is

A. hypercalcemia

B. hypercarbia

C. hyperglycemia

D. hypomagnesemia

E. hypoxia

Copyright © 2007 by the American Academy of Pediatrics page 5

Page 6: AAP MCQ 2007

2007 PREP SA on CD-ROM

Preferred Response: ECritique: 3

Seizures are the most frequent sign of central nervous system injury in the newborn. Whenseizures occur in a newborn who has depressed neuromotor tone, reflexes, andcardiopulmonary function at birth that requires assisted ventilation, perinatal asphyxia is likely. Inthis event, Apgar scores typically are depressed to less than 3 at 5 or more minutes after birth,and there is a severely acidotic umbilical cord arterial pH (<7.0), with evidence of metabolicacidemia. Poor tolerance of labor and asphyxia are more common in fetuses that haveexperienced intrauterine growth restriction. Because the infant in the vignette has the previouslydescribed features, hypoxic-ischemic encephalopathy (HIE) must be considered as a cause forthe seizures.

HIE is the most common cause of seizures occurring in the first 24 hours of postnatal lifeand accounts for up to 67% of early neonatal seizures. Other causes of neonatal seizureinclude intracranial hemorrhage, cerebrovascular accidents (stroke), or hemorrhagic infarction(10% to 15%); intracranial malformation (<10%); transient hypoglycemia or hypocalcemia(<10%); drug withdrawal (<5%); and inborn errors of metabolism (<5%).

When seizures occur beyond the first 24 hours after birth, especially in the absence of anyhistory of fetal or neonatal asphyxia, the evaluation should focus on potential causes other thanHIE. An additional cause for later seizures is infection (meningitis, encephalitis).

Asphyxia may result in hypocalcemia and hypoglycemia; hyperglycemia and hypercalcemiaare not associated with HIE and do not typically cause seizures. Hypomagnesemia mayaccompany hypocalcemia in the infant of a diabetic mother, but it is not common followingasphyxia and is not associated with neonatal seizures. Hypercarbia may occur in the depressednewborn who has inadequate ventilation, but it is not associated with seizures unless there iscorresponding hypoxia.

References:

Allan WC. The clinical spectrum and prediction of outcome in hypoxic-ischemic encephalopathy.NeoReviews. 2002;3:e108-e115. Available at:http://neoreviews.aappublications.org/cgi/content/full/3/6/e108

Hahn JS, Olson DM. Etiology of neonatal seizures. NeoReviews. 2004;5:e327-e335. Availableat: http://neoreviews.aappublications.org/cgi/content/full/5/8/e327

Riviello JJ, Jr. Pharmacology review: drug therapy for neonatal seizures: part 2. NeoReviews.2004;5:e262-e268. Available at: http://neoreviews.aappublications.org/cgi/content/full/5/6/e262

Thureen PJ, Anderson MS, Hay WW, Jr. The small-for-gestational age infant. NeoReviews.2001;2:e139-e149. Available at: http://neoreviews.aappublications.org/cgi/content/full/2/6/e139

Wu YW, Backstrand KH, Zhao S, Fullerton HJ, Johnston SC. Declining diagnosis of birthasphyxia in California: 1991-2000. Pediatrics. 2004;114:1584-1590. Available at:http://pediatrics.aappublications.org/cgi/content/full/114/6/1584

Copyright © 2007 by the American Academy of Pediatrics page 6

Page 7: AAP MCQ 2007

2007 PREP SA on CD-ROM

Question: 4

A 2-year-old boy presents with a 3-day history of diarrhea and vomiting. He has been able totolerate small amounts of fluids. He is moderately dehydrated, with dry mucous membranes anda heart rate of 145 beats/min.

Of the following, the BEST management for this patient's fluid status is

A. hospitalization with intravenous fluids and a restrictive bland diet

B. hospitalization with intravenous fluids and gut rest for 24 hours

C. oral rehydration therapy at home followed by a clear liquid diet for 24 hours

D. oral rehydration therapy at home followed by a diet of fruits, vegetables, and meats

E. oral rehydration therapy at home followed by a restrictive bland diet

Copyright © 2007 by the American Academy of Pediatrics page 7

Page 8: AAP MCQ 2007

2007 PREP SA on CD-ROM

Preferred Response: DCritique: 4

Dehydration results from a total body loss of water and sodium. Acute infectious gastroenteritisis among the most common causes of dehydration in infants and young children. Both mild andmoderate dehydration may be managed at home with oral rehydration therapy, even if the childcontinues to have intermittent vomiting. Commercial oral rehydration solutions (ORS) are widelyavailable and should be used for this purpose. All ORS are designed to replace lost electrolytes(sodium, chloride, potassium, and bicarbonate) glucose, and water. Young children who havemild dehydration have an estimated water loss of 50 mL/kg, and this amount of ORS can begiven via a spoon or syringe in small amounts over 2 to 4 hours. Those who have moderatedehydration should receive 100 mL/kg over 2 to 4 hours. Care should be taken to monitorongoing losses from stool and emesis, and intravenous or nasogastric rehydration therapyshould be considered if losses are excessive or if dehydration worsens or does not improve.Oral rehydration therapy should not be used for children who have severe dehydration, shock,suspected intestinal obstruction, obtundation, or ileus.

Once adequate hydration has been assured or rehydration is complete, a normal diet shouldbe given to the child to ensure adequate caloric and nutrient intake. A period withoutgastrointestinal intake is unnecessary and may delay nutritional recovery. Clear liquid and blanddiets also should not be used because they do not provide adequate nutrition. Infants should begiven human milk or their usual formula at full strength because diluted formula or human milk(eg, one-half or one-quarter strength formula) will not meet the child’s caloric requirements andmay worsen electrolyte abnormalities. Lactose-free formulas are generally unnecessarybecause most children do not develop lactase deficiency. Older children should be given aregular diet of complex carbohydrates, fruits, vegetables, and meats. High sugar-containingliquids should be avoided because the osmotic load of these liquids may worsen diarrhea.

References:

Finberg L. Dehydration in infancy and childhood. Pediatr Rev. 2002;23:277-282. Available at:http://pedsinreview.aappublications.org/cgi/content/full/23/8/277

King CK, Glass R, Bresee JS, Duggan C, Centers for Disease Control and Prevention.Managing acute gastroenteritis among children: oral rehydration, maintenance, and nutritionaltherapy. MMWR Recomm Rep. 2003;52(RR-16):1-16. Available at:http://www.cdc.gov/mmwr/preview/mmwrhtml/rr5216a1.htm

Copyright © 2007 by the American Academy of Pediatrics page 8

Page 9: AAP MCQ 2007

2007 PREP SA on CD-ROM

Question: 5

A worried grandmother brings her 2-year-old grandchild to the emergency departmentimmediately upon finding the boy with an open bottle of 81-mg chewable aspirin (which is usedby the grandfather for coronary artery disease prophylaxis). She is unsure of the number oftablets in the bottle prior to ingestion, but the original number was 30, and there are now threeremaining. The child has vomited once and is fussy and lethargic. Physical evaluation reveals a12-kg child who has tachypnea and tachycardia. Laboratory results include a pH of 7.45, carbondioxide of 25 mEq/L (25 mmol/L), and bicarbonate of 18 mEq/L (18 mmol/L). A salicylatemeasurement result is pending.

Of the following, the next BEST step in the management of this child is to

A. administer activated charcoal

B. administer sodium bicarbonate intravenously

C. administer syrup of ipecac

D. observe the child clinically in the emergency department

E. remeasure the salicylate level in 6 hours

Copyright © 2007 by the American Academy of Pediatrics page 9

Page 10: AAP MCQ 2007

2007 PREP SA on CD-ROM

Preferred Response: ACritique: 5

Salicylates are common in the homes of many children because many older adults (who may beparents or grandparents) use salicylates for prophylaxis against cardiovascular disease and forarthritis pain. Methylsalicylate present in liniments is especially concentrated. The toddler mayingest such preparations accidentally, as described for the boy in the vignette, and theadolescent may use salicylates in suicide gestures or attempts.

Toxic effects of salicylates can include gastritis, anticoagulant effects, increasedmetabolism, hyperventilation and respiratory alkalosis, and hepatitis. Reye syndrome, which ischaracterized by hepatitis and encephalopathy, may occur if aspirin is given during certain viralinfections. Signs and symptoms include lethargy or coma, vomiting, tachypnea, and tachycardia.

The next best step in the management of the child described in the vignette is theadministration of activated charcoal. Multiple doses of activated charcoal adsorb salicylates fromboth the intestinal tract and the systemic circulation.

For a child who has ingested a potentially toxic dose of salicylates, serum salicylateconcentrations should be measured 2 to 6 hours after the ingestion. However, administration ofactivated charcoal should not be delayed until the salicylate concentration has been measured ifa toxic dose has been ingested. The child in the vignette may have ingested 27 81-mg tablets fora total dose of more than 2,100 mg or 182 mg/kg. This dose can be expected to cause mild-to-moderate toxicity. A serum salicylate value higher than 30 mg/dL is considered toxic,concentrations higher than 70 mg/dL reflect severe toxicity, and those greater than 100 mg/dLare life-threatening.

Because gastric emptying time is prolonged with salicylate ingestion, gastrointestinaldecontamination by lavage may be effective up to 6 hours after ingestion. However, the childdescribed in the vignette is fussy and lethargic. Therefore, syrup of ipecac is not indicated.Administration of sodium bicarbonate to alkalinize the urine as well as correction of acidosis,hyperkalemia, and hypocalcemia are important adjuncts to gastric decontamination.

Cautious administration of intravenous sodium bicarbonate may be indicated clinically insevere acidosis because acidosis enhances brain toxicity, but the patient must be monitored forworsening hypernatremia and hypokalemia as a response to alkalinization. Although the child inthe vignette should be observed and repeat salicylate concentrations obtained, gastrointestinaldecontamination with activated charcoal is therapeutic and, thus, the next best step.

References:

Mariscalco MM. Salicylism. In: McMillin JA, DeAngelis CD, Feigin RD, Warshaw JB, eds. Oski’sPediatrics: Principles and Practice. 3rd ed. Philadelphia, Pa: Lippincott, Williams & Wilkins;1999:623-625

Woolf AD. Poisoning in children and adolescents. Pediatr Rev. 1993;14:411-422

Copyright © 2007 by the American Academy of Pediatrics page 10

Page 11: AAP MCQ 2007

2007 PREP SA on CD-ROM

Question: 6

A mother brings her 10-month-old son to the emergency department because he has beenvomiting for the past 10 days. The child has not experienced any diarrhea. On physicalexamination, he is lethargic and has dry mucous membranes, reduced tears, a full anteriorfontanelle, and 2-second capillary refill. After a second intravenous bolus of 20 mL/kg of normalsaline, the boy extends his arms and legs forcefully for 10 seconds.

Of the following, the MOST appropriate next step in the management of this child isadministration of

A. additional intravenous normal saline bolus of 20 mL/kg

B. intravenous dexamethasone of 1 mg/kg

C. intravenous fosphenytoin bolus at 20 mg/kg phenytoin equivalents over 10 minutes

D. intravenous prochlorperazine of 5 mg

E. rapid intravenous lorazepam of 0.05 mg/kg

Copyright © 2007 by the American Academy of Pediatrics page 11

Page 12: AAP MCQ 2007

2007 PREP SA on CD-ROM

Preferred Response: BCritique: 6

Brain tumors occur rarely in the pediatric population; the incidence is just under 1 in 25,000children annually. Despite its rarity, the ominous nature of such a diagnosis requires thatpractitioners be able to recognize readily the clinical manifestations of a brain tumor. Clinicalprodromes may include features of increased intracranial pressure (ICP) (Item C6A), findings ofa localizing nature, or symptoms and signs without a localizing quality.

Elevated ICP often is insidious and nonspecific initially. Among school-age children, decliningacademic performance, fatigue, behavioral changes, and vague intermittent headaches arecommon. Over time, morning headaches, especially pain at the occipital or frontal region, alongwith vomiting and lethargy ensue. Horizontal diplopia from abducens nerve paresis is common.Papilledema may develop if the pressure is longstanding. Among infants, irritability, anorexia,failure to thrive, and even developmental regression can be early signs of increased ICP.Macrocephaly, splitting of the cranial sutures, or a bulging anterior fontanelle can follow. The“setting sun” sign, a downward deviation of the eyes, may be seen with attendanthydrocephalus.

Localizing findings depend on the region of the brain involved. Supratentorial (cerebrum,basal ganglia, thalamus, hypothalamus, and optic chiasm) tumors can produce hemiparesis,hemisensory loss, hyperreflexia, seizures, or visual complaints. Infratentorial (cerebellum andbrainstem) tumors lead to ataxia, hemiparesis, hyperreflexia, or cranial neuropathies, but notseizures.

Nonlocalizing symptoms and signs are perhaps the most subtle manifestations of a braintumor. The child may display changes in affect, energy level, behavior, or weight. Sexualprecocity or delayed puberty, growth failure, and somnolence can suggest hypothalamic orpituitary dysfunction. Vomiting can occur with direct irritation of the area postrema in the floor ofthe fourth ventricle or from a generalized increase in ICP.

The vomiting without diarrhea displayed by the 10-month-old in the vignette, along with a fullanterior fontanelle in the face of mild dehydration, points to an intracranial process. In addition,the child is exhibiting posturing, not a seizure, from continued overhydration and worseningintracranial pressure following the saline administration. Accordingly, administration ofintravenous dexamethasone, along with other measures to treat increased ICP, is mostappropriate. Treatment with prochlorperazine will only mask the child’s vomiting. Administrationof fosphenytoin or lorazepam is not indicated in the absence of seizures. Continued aggressivehydration will worsen this child’s ICP.

References:

Kuttesch JF Jr, Alter JL. Brain tumors in childhood. In: Behrman RE, Kliegman RM, Jenson HB,eds. Nelson Textbook of Pediatrics. 17th ed. Philadelphia, Pa: WB Saunders; 2004:1702-1708

Strother DR, Pollock IF, Fisher PG, et al. Tumors of the central nervous system. In: Pizzo PA,Poplack DG, eds. Principles and Practice of Pediatric Oncology. 4th ed. Philadelphia, Pa:Lippincott, Williams & Wilkins; 2002:751-824

Copyright © 2007 by the American Academy of Pediatrics page 12

Page 13: AAP MCQ 2007

2007 PREP SA on CD-ROM

Question: 7

An 18-month-old child has been brought to your urgent care clinic for evaluation. He and hismother are in town visiting his grandmother. His mother tells you that she found him playing withan open bottle of his grandmother's medication. On physical examination, he is sleepy butarousable, pale, mildly diaphoretic, and afebrile. His respiratory rate is 20 breaths/min, heart rateis 60 beats/min, and blood pressure is 65/40 mm Hg. His lungs are clear, there are no murmurs,and his pulses are weak.

Of the following, the MOST likely cause for this patient's presentation is ingestion of

A. beta blocker

B. captopril

C. hydralazine

D. pseudoephedrine

E. tricyclic antidepressant

Copyright © 2007 by the American Academy of Pediatrics page 13

Page 14: AAP MCQ 2007

2007 PREP SA on CD-ROM

Preferred Response: ACritique: 7

Accidental ingestion of medications continues to be a significant cause of pediatric morbidity andpotential mortality. The problem is greatest in toddlers and is exacerbated when medications areleft within reach of children or not appropriately secured in a locked cabinet. The reaction toingestion of a given medication depends on the class and dose.

Beta blockers can have multiple and varying effects on the heart and other organ systems.In the heart, they typically exhibit some degree of negative chronotropic (slowing of the heartrate), dromotropic (slowing of the conduction through the AV node), and inotropic (decrease inthe ventricular force of contraction) effects. As a result, this class of medications generally isused to control certain types of arrhythmia and hypertension and to reduce myocardial work andoxygen demand. The depressed sensorium, bradycardia for age, and hypotension withdiminished pulses described for the child in the vignette are most consistent with beta blockeringestion.

Captopril is an angiotensin-converting enzyme (ACE) inhibitor. ACE activity convertsangiotensin I to angiotensin II, which is a potent vasoconstrictor. Therefore, ACE inhibitors lowerthe systemic vascular resistance and the systemic blood pressure, decreasing the afterload(work) of the left ventricle. Hydralazine is an arteriolar dilator that may have some positiveinotropic effect. It often is used as antihypertensive therapy and typically increases the heartrate. Pseudoephedrine is an alpha agonist that is used commonly for relief of symptoms causedby nasal congestion. Its effects on the cardiovascular system are to cause systemicvasoconstriction that may increase blood pressure, and it also may cause tachycardia. Tricyclicantidepressants are a diverse group of medications that can be used for a variety of noncardiacindications, including depression. When taken in high doses, they may affect the cardiovascularsystem by causing vasodilation, flushing, hypotension, and tachyarrhythmias.

Although captopril, hydralazine, and tricyclic antidepressants may be associated withhypotension, they all would be expected to cause (or be associated with) a correspondingtachycardia.

References:

Dobson JV, Webb SA. Life-threatening pediatric poisonings. J S C Med Assoc. 2004;100:327-332. Abstract available at:http://www.ncbi.nlm.nih.gov/entrez/query.fcgi?cmd=Retrieve&db=pubmed&dopt=Abstract&list_uids=15835193&query_hl=11&itool=pubmed_docsum

Opie L, Gersh B. Drugs for the Heart. 6th ed. Philadelphia, Pa: WB Saunders Co; 2004

Riordan M, Rylance G, Berry K. Poisoning in children 1: general management. Arch Dis Child.2002;87:392-396. Available at: http://adc.bmjjournals.com/cgi/content/full/87/5/392

Copyright © 2007 by the American Academy of Pediatrics page 14

Page 15: AAP MCQ 2007

2007 PREP SA on CD-ROM

Question: 8

The parents of a child who has Down syndrome and a 47,XX+21 karyotype come to you forcounseling about future pregnancies.

Of the following, their risk for giving birth to another child who has trisomy is CLOSEST to

A. no greater than the general population at risk

B. 1% added to the mother's age-related risk

C. 5% added to the mother's age-related risk

D. 10% added to the mother's age-related risk

E. 25% added to the mother's age-related risk

Copyright © 2007 by the American Academy of Pediatrics page 15

Page 16: AAP MCQ 2007

2007 PREP SA on CD-ROM

Preferred Response: BCritique: 8

Approximately 94% of individuals who have Down syndrome have trisomy of chromosome 21.Studies of the origins of trisomy 21 (47 chromosomes with trisomy for chromosome 21) showthat 90% to 95% of cases are due to a maternal meiotic error, with 75% of these occurring inmeiosis I. Approximately 3% to 5% are due to paternal meiotic errors, and the remainder are dueto mitotic nondisjunction.

Recurrence risk estimates for trisomy are based on empiric data. The overall recurrencerisk for having a live-born child who has any trisomy is approximately 1% added to the mother’sage-related risk for having a child who has a trisomy, which increases over time. Risk estimatesvary slightly from time of amniocentesis to time of term delivery because there is natural loss ofsome trisomic fetuses between the two events. As a woman’s age increases, her 1%recurrence risk becomes relatively less significant compared with her age-related risk. Forexample, at the age of 40 years, a woman’s risk for having a child who has Down syndrome is 1in 90 compared with a 1 in 1,500 risk for a 22-year-old woman. Thus, a 22-year-old woman whohas a child affected with trisomy 21 has a 1/100 plus a 1/1,500 risk (approximately 1.1%) risk forhaving another affected child compared with a 0.07% risk for other 22-year-old women whohave not had children affected with trisomy. In contrast, a 40-year-old woman who has a childwith trisomy 21 has a 1/100 plus a 1/90 (approximately 2.1%) risk for having another affectedchild, which is similar to the 1.1% risk for 40-year-old women who have not had children withtrisomy.

References:

Hall JG. Chromosomal clinical abnormalities. In: Behrman RE, Kliegman RM, Jenson HB, eds.Nelson Textbook of Pediatrics. 17th ed. Philadelphia Pa: Saunders; 2004:382-391

Nussbaum RL, McInnes RR, Willard HF. Clinical cytogenetics: disorders of the autosomes andthe sex chromosomes. In: Thompson & Thompson Genetics in Medicine. 6th ed, revised reprint.Philadelphia, Pa: Saunders; 2004:157-179

Roizen NJ, Stark AR. Epidemiology and genetics of Down syndrome. UpToDate. 2006:14.1.Available at:http://www.utdol.com/utd/content/topic.do?topicKey=dis_chld/2412&type=P&selectedTitle=27~54

Copyright © 2007 by the American Academy of Pediatrics page 16

Page 17: AAP MCQ 2007

2007 PREP SA on CD-ROM

Question: 9

A 15-month-old infant has been breastfed since birth. He eats finger foods (eg, peas, carrots)and occasionally some cereal. His mother adheres to a vegan diet and plans the same for herchild. A complete blood count documents anemia.

Of the following, the MOST likely cause of this infant's anemia is a deficiency of

A. folic acid

B. niacin

C. riboflavin

D. thiamine

E. vitamin B12

Copyright © 2007 by the American Academy of Pediatrics page 17

Page 18: AAP MCQ 2007

2007 PREP SA on CD-ROM

Preferred Response: ECritique: 9

The infant described in the vignette has been exclusively breastfed by a strict vegan mother andis at risk for anemia due to vitamin B12 deficiency. The appearance of hypersegmentedneutrophils in the peripheral blood precedes the development of classic megaloblastic(macrocytic) anemia. The infant’s deficiency may be caused by reduced placental transfer ofvitamin B12 in utero and lower vitamin content in the mother’s milk. In addition, the infant is beingoffered other foods that are consistent with a vegan diet and contain little vitamin B12. Deficiencyof vitamin B12, including dietary deficiency, is unusual in infants and children in the UnitedStates. Other causes of deficiency are pernicious anemia; impaired gastric and small bowelabsorption, including resections; and rare inherited metabolic disorders such as methylmalonicaciduria and intrinsic factor deficiency. Vitamin B12 deficiency is diagnosed by recognition of riskfactors, documentation of low serum B12 concentrations in the infant, and a subsequentresponse to treatment. Demonstration of low serum vitamin B12 values in the mother supportsthe diagnosis.

Folic acid deficiency is an unlikely cause of the anemia for the infant described in thevignette. Folate is present in many foods consumed by vegans, and human milk providesadequate folate for the breastfed infant. Folate deficiency rarely is present in newborns becausethe fetus extracts adequate folate from the mother, even in the presence of maternal deficiency.Defects of folate metabolism are rare, although infants exclusively fed goat milk are at risk forfolate deficiency and possible megaloblastic anemia. A diagnosis of folate deficiency should beconfirmed, including the exclusion of vitamin B12 deficiency, before initiating folate replacementtherapy. Treatment with therapeutic doses of folate may delay the diagnosis of vitamin B12deficiency and increase the risk of neurologic complications. Erythrocyte folic acid concentrationis a better measure of folate sufficiency than serum folate concentrations.

Deficiencies of thiamine (vitamin B1), riboflavin (vitamin B2), and niacin (vitamin B3) are rareand not associated with anemia. Thiamine deficiency is associated with beriberi(cardiomyopathy, peripheral neuropathy, and encephalopathy). Riboflavin deficiency isassociated with cheilosis and glossitis. Pellagra (dementia, diarrhea, and dermatitis) isassociated with niacin deficiency.

References:

Glader B. Megaloblastic anemias. In: Behrman RE, Kliegman RM, Jenson HB, eds. NelsonTextbook of Pediatrics. Philadelphia, Pa: Saunders; 2004:1611-1613

Nutritional aspects of vegetarian diets. In: Kleinman RE, ed. Pediatric Nutrition Handbook. 5th ed.Elk Grove Village, Ill: American Academy of Pediatrics; 2004:191-208

Vitamins. In: Kleinman RE, ed. Pediatric Nutrition Handbook. 5th ed. Elk Grove Village, Ill:American Academy of Pediatrics; 2004:339-365

Copyright © 2007 by the American Academy of Pediatrics page 18

Page 19: AAP MCQ 2007

2007 PREP SA on CD-ROM

Question: 10

A 15-year-old boy comes to your office for a health supervision visit. He expresses concern thathe is only 5 ft, 2 in tall and is not competitive in track. On physical examination, he appearshealthy, has a height of 62 in, and weighs 96 lb. His testes are 8 mL in volume bilaterally, there isslight pubertal phallic enlargement, and he has Sexual Maturity Rating 3 pubic hair. He has asmall amount of subareolar breast tissue. His last health supervision visit was 2 years ago. Hedid not have pubic hair at the last visit, and his testes were described as "prepubertal" in size.

Of the following, the MOST likely cause of his short stature is

A. constitutional delayed puberty

B. exercise-induced growth delay

C. Klinefelter syndrome

D. prolactinoma

E. undernutrition

Copyright © 2007 by the American Academy of Pediatrics page 19

Page 20: AAP MCQ 2007

2007 PREP SA on CD-ROM

Preferred Response: ACritique: 10

Puberty is considered delayed in boys if there is no testicular enlargement by age 14 years.Unless other features are present on physical examination or history, it is very difficult toseparate boys who have delayed puberty clinically from those who have true hypogonadotropichypogonadism until failure of pubertal progression persists for several years. After pubertycommences, adult testicular size usually is achieved by 3.2±1.8 years.

The boy described in the vignette is of normal height for age and is progressing into puberty,based on review of his previous visit and his present appearance. Most likely, he has only mildconstitutional delayed puberty. Most boys who have constitutional delayed puberty areunderweight and do not have other signs of chronic illness.

Exercise and weight loss rarely induce the type of marked growth delay in boys that is seenin girls who are competitive gymnasts or active in ballet. Although Klinefelter syndrome may leadto failure to progress through puberty, the increasing testicular size of the boy in the vignettemakes this diagnosis unlikely. Prolactinomas can inhibit pubertal growth, but are not common.Underweight caused by undernutrition must be severe before it inhibits growth and puberty.

Only close follow-up is required for a boy who seems to be progressing into puberty.However, if there is evidence of failure to progress into puberty, a full evaluation to determine ifthe boy has an underlying chronic illness such as celiac disease or inflammatory bowel disease,hypergonadotropic hypogonadism as seen in Klinefelter syndrome, or other endocrine disordersuch as a prolactinoma, is indicated.

References:

Cannavo S, Venturino M, Curto L, et al. Clinical presentation and outcome of pituitary adenomasin teenagers. Clin Endocrinol (0xf). 2003;58:519-527. Abstract available at:http://www.ncbi.nlm.nih.gov/entrez/query.fcgi?db=pubmed&cmd=Retrieve&dopt=Abstract&list_uids=12641637&query_hl=5&itool=pubmed_docsum

Israel EJ, Levitsky LL, Anupindi SA, Pitman MB. Case records of the Massachusetts GeneralHospital. Weekly clinicopathological exercises. Case 3-2005. A 14-year-old boy with recentslowing of growth and delayed puberty. N Engl J Med. 2005;352:393-403

Lanfranco F, Kamischke A, Zitzmann M, Nieschlag E. Klinefelter’s syndrome. Lancet.2004;364:273-283. Abstract available at:http://www.ncbi.nlm.nih.gov/entrez/query.fcgi?db=pubmed&cmd=Retrieve&dopt=Abstract&list_uids=15262106&query_hl=3&itool=pubmed_docsum

Misra M, Park-Bennett S. Disorders of puberty. In: Burg FD, Ingelfinger JR, Polin RA, GershonAA, eds. Gellis & Kagan’s Current Pediatric Therapy. Philadelphia, Pa: WB Saunders; 2002:706-710

Rogol AD, Clark PA, Roemmich JN. Growth and pubertal development in children andadolescents: effects of diet and physical activity. Am J Clin Nutr. 2000;72(suppl):521S-528SAvailable at: http://www.ajcn.org/cgi/content/full/72/2/521S

Sedlmeyer IL, Palmert MR. Delayed puberty: analysis of a large case series from an academiccenter. J Clin Endocrinol Metab. 2002;87:1613-1620. Available at:http://jcem.endojournals.org/cgi/content/full/87/4/1613

Copyright © 2007 by the American Academy of Pediatrics page 20

Page 21: AAP MCQ 2007

2007 PREP SA on CD-ROM

Question: 11

You are examining a 2-year-old girl who has a 6-month history of developmental regression.During her first postnatal year, she met all motor, language, and social milestones. Her headcircumference, which currently is at the 3rd percentile, was at the 75th percentile at birth. Onphysical examination, she makes poor eye contact and repetitively wrings her hands.

Of the following, the MOST appropriate diagnostic test is

A. arylsulfatase A

B. fragile X

C. hexosaminidase A

D. MECP2 gene testing

E. urine N-acetyl-aspartic acid

Copyright © 2007 by the American Academy of Pediatrics page 21

Page 22: AAP MCQ 2007

2007 PREP SA on CD-ROM

Preferred Response: DCritique: 11

The developmental regression, acquired microcephaly, and hand-wringing movementsdescribed for the girl in the vignette are typical findings in the X-linked condition Rett syndrome.Rett syndrome is a neurodegenerative disorder that is associated with developmentalregression as well as generalized tonic-clonic seizures, poor feeding, constipation, sleepdisorder, breath-holding spells, scoliosis, muscle wasting, cardiac arrhythmias, and death in lateadolescence. The diagnosis can be made by testing for the MECP2 gene (chromosome X q28),a gene for a transcription factor that binds to methylated CpG island and silences transcription.

Deficiency of arylsulfatase A causes the degenerative lysosomal disorder metachromaticleukodystrophy. Fragile X testing is performed primarily in boys who have cognitive impairmentand dysmorphic features. Hexosaminidase A deficiency causes Tay-Sachs disease.Accumulation of N-acetyl-aspartic acid in the urine is associated with the leukodystrophyCanavan disease.

References:

Johnston MV. Neurodegenerative disorders of childhood. In: Behrman RE, Kliegman RM,Jenson HB, eds. Nelson Textbook of Pediatrics. 17th ed. Philadelphia, Pa: WB Saunders Co;2004:2029-2034

Rich J. In brief: degenerative central nervous system (CNS) disease. Pediatr Rev. 2001;22:175-176. Available at: http://pedsinreview.aappublications.org/cgi/content/full/22/5/175

Copyright © 2007 by the American Academy of Pediatrics page 22

Page 23: AAP MCQ 2007

2007 PREP SA on CD-ROM

Question: 12

A 7-year-old girl presents to your clinic with a 2- to 3-day history of a nonproductive cough,malaise, and temperature to 101°F (38.3°C). On physical examination, you note that the girldoes not appear ill and are surprised to hear widespread crackles in the lungs bilaterally. Chestradiography demonstrates bilateral diffuse infiltrates (Item Q12A).

Of the following, the MOST appropriate antimicrobial agent to treat this girl's infection is

A. amoxicillin

B. azithromycin

C. doxycycline

D. levofloxacin

E. trimethoprim-sulfamethoxazole

Copyright © 2007 by the American Academy of Pediatrics page 23

Page 24: AAP MCQ 2007

2007 PREP SA on CD-ROM

Preferred Response: BCritique: 12

School-age children are at risk for lower respiratory tract infections (LRTIs) from Mycoplasmapneumoniae. The highest incidence of LRTI from M pneumoniae occurs in children 5 to 14 yearsof age. Frequent symptoms associated with pneumonia due to Mycoplasma include fever,cough, and malaise. The lack of coryza can be useful in differentiating illness due toMycoplasma from other common viral agents. The radiographic pattern associated with thispathogen varies, but bilateral, diffuse infiltrates (Item C12A), as reported for the girl in thevignette, are common.

Macrolides (eg, erythromycin, clarithromycin, azithromycin) are the preferred antimicrobialagents to treat infections due to Mycoplasma. Because azithromycin can be administered oncedaily and has few adverse effects compared with the other macrolides, it has become thepreferred agent. These agents also are effective against common respiratory flora such asMoraxella catarrhalis, Haemophilus influenzae, Streptococcus pyogenes, and viridansstreptococci. Other atypical pathogens, such as Chlamydia sp, Bordetella pertussis, andLegionella pneumophila, also can be treated with the macrolides. Clarithromycin andazithromycin have activity against many nontuberculous mycobacterial species (eg, M aviumcomplex, M kansasii), as well.

Doxycycline is effective against Mycoplasma sp, but should not be used in children youngerthan 8 years of age due to potential bone and teeth staining. Similarly, levofloxacin is effectiveagainst many atypical agents, including Mycoplasma sp, but should not be used routinely forchildren younger than 18 years of age because of the interference with cartilage growthdemonstrated in puppies treated with the fluoroquinolones. Amoxicillin and trimethoprim-sulfamethoxazole are not effective against Mycoplasma.

References:

Gaston B. Pneumonia. Pediatr Rev. 2002;23:132-140. Available at:http://pedsinreview.aappublications.org/cgi/content/full/23/4/132

James LP, Rahman SM, Farrar HC, Jacobs RF. Antimicrobial agents. In: Long SS, Pickering LK,Prober CG, eds. Principles and Practice of Pediatric Infectious Diseases. 2nd ed. Philadelphia,Pa: Churchill Livingstone; 2003:1458-1510

Copyright © 2007 by the American Academy of Pediatrics page 24

Page 25: AAP MCQ 2007

2007 PREP SA on CD-ROM

Question: 13

A 7-year-old boy comes to your office with complaints of daily bedwetting for 2 months. He wascompletely toilet trained by 4 years of age and had been dry at night except for occasional(about once per month) minor bedwetting until recently. He denies daytime enuresis, dysuria,frequency, urgency, fever, abdominal pain, or constipation. He has no history of urinary tractinfections. His physical examination reveals weight and height at the 75th percentiles and noabnormalities.

Of the following, the MOST important next step in this child's evaluation is to obtain

A. abdominal radiography

B. renal ultrasonography

C. serum electrolyte measurement

D. urinalysis

E. voiding cystourethrography

Copyright © 2007 by the American Academy of Pediatrics page 25

Page 26: AAP MCQ 2007

2007 PREP SA on CD-ROM

Preferred Response: DCritique: 13

After achieving daytime and nighttime dryness, most children experience occasional episodes ofnocturnal enuresis until the age of 5 to 6 years. Boys generally achieve dryness later than girls,and some boys continue to experience occasional bedwetting episodes to the age of 7 to 8years. Indeed, about 15% of 7-year-old boys experience occasional bedwetting, although thesymptoms resolve spontaneously in most.

History, physical examination, and urinalysis can identify the cause of new-onset nocturnalenuresis for most children who have secondary nocturnal enuresis (ie, achieved generalnighttime dryness once toilet trained). A simple review of recent voiding patterns and generallifestyle issues may disclose the cause of the bedwetting in many patients. It is vital to considerthe common causes of new-onset bedwetting, including psychosocial stress, urinary tractinfection, and new disease (eg, diabetes mellitus or insipidus). For children who have normalfindings on history, physical examination, and urinalysis, imaging studies such as abdominalradiography, renal ultrasonography, and voiding cystourethrography are unlikely to aid indetermining the cause of bedwetting. Measurement of serum electrolytes rarely is warranted atthe outset if the history, physical examination, and urinalysis results are normal.

If a specific cause is identified (eg, urinary tract infection), treatment of the primary conditionoften results in resolution of the bedwetting. However, behavioral or pharmacotherapy may berequired for many children in whom a primary cause cannot be identified and who experienceunabated episodes of enuresis. Therapeutic options include counseling, hypnosis, urine alarm,imipramine, no treatment, or desmopressin. Counseling may be helpful if the child’s bedwetting isdue to psychosocial stress, but it has mixed results. Behavioral therapy such as hypnosis alsoinduces resolution in some patients but success is very dependent on the expertise of thetherapist. Results also have been mixed with urine alarms. Success requires strict compliance,but if the child and parents are motivated, many children experience at least a moderatereduction in bedwetting episodes. Anticholinergic medications (eg, oxybutynin) or medicationsthat have anticholinergic properties (eg, imipramine) that alter bladder physiology in favor ofprolonged filling and decreased contractions are most successful in children who haveunderlying bladder dyssynergy. Common adverse effects of imipramine include dry mouth andconstipation. Rarely, it may cause blood pressure changes or arrhythmias. Common adverseeffects of oxybutynin include gastrointestinal symptoms and urinary retention. Finally, intranasaldesmopressin may improve water reabsorption during the nighttime, but the ability toconcentrate the urine is not impaired in most children who experience bedwetting, thus limitingthe overall effectiveness of this medication. Common adverse effects of desmopressin includegastrointestinal symptoms, headache, flushing, and rarely, hyponatremia.

References:

Bower WF, Yip SK, Yeung CK. Dysfunctional elimination symptoms in childhood and adulthood. JUrol. 2005;174:1623-1627. Abstract available at:http://www.ncbi.nlm.nih.gov/entrez/query.fcgi?cmd=Retrieve&db=pubmed&dopt=Abstract&list_uids=16148668&query_hl=21&itool=pubmed_docsum

Lee T, Suh HJ, Lee HJ, Lee JE. Comparison of effects of treatment of primary nocturnalenuresis with oxybutynin plus desmopressin, desmopressin alone or imipramine alone: arandomized controlled clinical trial. J Urol. 2005;174:1084-1087. Abstract available at:http://www.ncbi.nlm.nih.gov/entrez/query.fcgi?cmd=Retrieve&db=pubmed&dopt=Abstract&list_uids=16094064&query_hl=18&itool=pubmed_docsum

Yagci S, Kibar Y, Akay O, et al. The effect of biofeedback treatment on voiding and urodynamicparameters in children with voiding dysfunction. J Urol. 2005;174:1994-1997. Abstract availableat:http://www.ncbi.nlm.nih.gov/entrez/query.fcgi?db=pubmed&cmd=Retrieve&dopt=AbstractPlus&list_uids=16217376

Copyright © 2007 by the American Academy of Pediatrics page 26

Page 27: AAP MCQ 2007

2007 PREP SA on CD-ROM

Question: 14

You are speaking to the mother of a child who attends a junior high school where one of thestudents was diagnosed with meningococcal disease 24 hours ago. Her child does not have anyclasses with the index patient and, except for passing him in the hall during lunch 3 days ago,has had no other contact with the patient. The child's mother is frantic because the school senthome a notice asking parents to bring their children to the public health department or theirprivate physician to receive antibiotic prophylaxis.

Of the following, the MOST appropriate advice for this parent is that her child

A. does not require antibiotic prophylaxis and does not need to be seen

B. does not require antibiotic prophylaxis but needs to be evaluated to determine if she isdeveloping symptoms of meningococcal disease

C. needs to be seen to obtain nasopharyngeal cultures for meningococcal organisms and if thecultures are positive, may require antibiotic prophylaxis

D. requires antibiotic prophylaxis and should be seen immediately

E. should be seen immediately to determine if she needs to be hospitalized and treated forpossible meningococcal disease

Copyright © 2007 by the American Academy of Pediatrics page 27

Page 28: AAP MCQ 2007

2007 PREP SA on CD-ROM

Preferred Response: ACritique: 14

Close contacts of all persons who have invasive meningococcal disease, whether sporadic or inan outbreak, are at high risk for infection and should receive chemoprophylaxis within 24 hoursof diagnosis of the primary case, regardless of vaccination status. Close contacts include allhousehold contacts, child care and nursery school contacts during the previous 7 days,persons who have had direct contact with the patient’s oral secretions, and persons whofrequently eat or sleep in the same dwelling as the index patient. However, classroom contactsof students who have meningococcal disease, such as the child described in the vignette, areconsidered casual contacts (no history of direct exposure to the index patient’s oral secretions),and the use of prophylactic antibiotics is not recommended. In view of this and the fact that thecontact is asymptomatic, she does not require medical evaluation at this time, andnasopharyngeal cultures are not indicated.

Because secondary cases of meningococcal disease can occur several weeks after theonset of disease in the index case, the use of meningococcal vaccine is a possible adjunct tochemoprophylaxis if the serogroup is contained in the vaccine.

Other infections can spread easily in the household setting and may require the use ofpostexposure prophylactic immunoglobulin, antibiotics, or vaccines to prevent development ofdisease in individuals exposed to the index case. Selected diseases and prophylacticinterventions are summarized in Item C14A. For detailed advice regarding management ofspecific cases, pediatricians should consult the most recent Report of the Committee onInfectious Diseases (Red Book), the Centers for Disease Control and Prevention, or a pediatricinfectious disease specialist.

References:

American Academy of Pediatrics. Haemophilus influenzae infections. In: Pickering LK, ed. RedBook: 2006 Report of the Committee on Infectious Diseases. 27th ed. Elk Grove Village, Ill:American Academy of Pediatrics; 2006:310-318

American Academy of Pediatrics. Hepatitis A. In: Pickering LK, ed. Red Book: 2006 Report of theCommittee on Infectious Diseases. 27th ed. Elk Grove Village, Ill: American Academy ofPediatrics; 2006:326-335

American Academy of Pediatrics. Meningococcal infections. In: Pickering LK, ed. Red Book:2006 Report of the Committee on Infectious Diseases. 27th ed. Elk Grove Village, Ill: AmericanAcademy of Pediatrics; 2006:452-460

American Academy of Pediatrics. Pertussis. In: Pickering LK, ed. Red Book: 2006 Report of theCommittee on Infectious Diseases. 27th ed. Elk Grove Village, Ill: American Academy ofPediatrics; 2006:498-520

American Academy of Pediatrics. Varicella-zoster infections. In: Pickering LK, ed. Red Book:2006 Report of the Committee on Infectious Diseases. 27th ed. Elk Grove Village, Ill: AmericanAcademy of Pediatrics; 2006:711-725

Robinson J. Infectious diseases in schools and child care facilities. Pediatr Rev. 2001;22:39-46.Available at: http://pedsinreview.aappublications.org/cgi/content/full/22/2/39

Copyright © 2007 by the American Academy of Pediatrics page 28

Page 29: AAP MCQ 2007

2007 PREP SA on CD-ROM

Question: 15

An 18-year-old boy presents to the emergency department 30 minutes after eating at a seafoodrestaurant. He states that approximately 10 minutes into his meal he developed generalizedhives, pruritus, and difficulty breathing. He has a history of shellfish food allergy, although he hadordered steak and denies eating any crab, lobster, or shrimp. On physical examination, thepatient appears to have labored breathing, audible wheezing, and diffuse raised erythematouslesions (Item Q15A) on his trunk and extremities. His vital signs include a temperature of 98.5°F(37°C), heart rate of 100 beats/min, respiratory rate of 22 breaths/min, blood pressure of 110/60mm Hg, and pulse oximetry of 92% on room air.

Of the following, the MOST appropriate immediate action is

A. administration of 100% oxygen

B. administration of 1 L intravenous normal saline

C. administration of intramuscular epinephrine

D. administration of beta-2 agonist nebulization

E. observation

Copyright © 2007 by the American Academy of Pediatrics page 29

Page 30: AAP MCQ 2007

2007 PREP SA on CD-ROM

Preferred Response: CCritique: 15

The adolescent described in the vignette most likely is experiencing an adverse food reaction,specifically anaphylaxis to shellfish. Patients who have known food allergies, such as this boy,must remain vigilant regarding food choices and reading of food labels because foods maycontain hidden ingredients or may be contaminated with allergenic proteins. Most likely, thenonallergenic food (steak) in this case was contaminated with an allergenic food (shellfish) whilebeing prepared. For highly allergic individuals, only a small amount of antigen is required tocause a reaction. Because of the risk of cross-contamination, many allergists recommend strictavoidance of the allergenic food within the home. The rapid (<30 min) onset of urticaria andwheezing in a shellfish-allergic patient who is eating in a seafood restaurant is likely anaphylaxisand warrants prompt administration of intramuscular epinephrine. Other supportive measures,such as administration of oxygen, intravenous fluids, and beta-2 agonists, may improve someclinical symptoms, but they do not reverse anaphylaxis. Simple observation clearly is notindicated in a person who has food allergy and is experiencing anaphylaxis with respiratorysymptoms.

Other causes that should be considered during an acute food reaction include foodpoisoning, particularly scombroid fish poisoning. Members of the Scomberesocidae orScombridae families (eg, albacore, mackerel, tuna, kingfish) that have spoiled can have bacterialovergrowth. The bacterial overgrowth is responsible for converting histidine, found in highconcentrations in the flesh of scombroid fish, to histamine. Reactions may be indistinguishablefrom anaphylaxis, with affected individuals developing rapid-onset flushing, tachycardia, and arash, but food poisoning is more likely to involve many restaurant patrons. Observation may beall that is needed in these cases, although more severe symptoms of hypotension and wheezingcan be managed with epinephrine, intravenous fluids, antihistamines, and steroids.

References:

Chegini S, Metcalfe DD. Contemporary issues in food allergy: seafood toxin-induced disease inthe differential diagnosis of allergic reactions. Allergy Asthma Proc. 2005;26:183-190. Abstractavailable at:http://www.ncbi.nlm.nih.gov/entrez/query.fcgi?db=pubmed&cmd=Retrieve&dopt=AbstractPlus&list_uids=16119031

Sampson HA, Leung DYM. Adverse reactions to foods. In: Behrman RE, Kliegman RM, JensonHB, eds. Nelson Textbook of Pediatrics. 17th ed. Philadelpha, Pa: WB Saunders Co; 2004:789-792

Copyright © 2007 by the American Academy of Pediatrics page 30

Page 31: AAP MCQ 2007

2007 PREP SA on CD-ROM

Question: 16

A 3-year-old child is rushed to the emergency department after the mother found her with anopen and empty bottle of acetaminophen. The mother has no idea how many tablets were in thebottle. She estimates that no more than 1 hour has passed since the child ingested the tablets.The child began to vomit during the trip to the emergency department, and has vomited threetimes more since her arrival. The child is awake and alert but clearly unhappy, crying even inher mother's arms. She appears pale and diaphoretic. Her heart rate is 110 beats/min,respiratory rate is 26 breaths/min, temperature is 98.6°F (37°C), and blood pressure is 90/60mm Hg.

Of the following, the MOST appropriate statement about acetaminophen toxicity is that

A. an antidote is available, but its use can be deferred until further information is gathered

B. given the short duration since the ingestion, it will be helpful to administer syrup of ipecac

C. multiple episodes of vomiting indicate that irreversible liver damage already has occurred

D. the administration of activated charcoal is contraindicated in acetaminophen toxicity

E. the contents of one bottle of acetaminophen are not sufficient to cause life-threatening toxicityin a child

Copyright © 2007 by the American Academy of Pediatrics page 31

Page 32: AAP MCQ 2007

2007 PREP SA on CD-ROM

Preferred Response: ACritique: 16

N-acetyl cysteine is a highly effective antidote to acetaminophen (APAP) toxicity if administeredwithin 10 hours of ingestion, but there is time to evaluate the level of APAP exposure beforeresorting to this management option for the patient described in the vignette. The level of APAPexposure is determined by measuring serum APAP concentrations 4 or more hours afteringestion and plotting the result on a nomogram (Item C16A). Serum APAP concentrations belowthe lower line are not likely to result in toxicity, levels between the two lines represent possibletoxicity, and levels above the upper line indicate a significant risk of toxicity. If it can beestablished definitively that a child has ingested more than the minimal toxic dose of 140 mg/kgfrom the history alone, immediate administration of the antidote is appropriate, especially if serumAPAP concentrations are not immediately available.

The primary toxicity of APAP is severe hepatic damage due to the binding of toxicmetabolites to the hepatocytes. Normally, the metabolites conjugate with glutathione, renderingthem harmless, but in severe overdose, glutathione is depleted. The benefit of N-acetylcysteinestems from its ability to act as a glutathione precursor.

The first stage of APAP toxicity occurs within several hours of ingestion and includesanorexia, nausea, and vomiting. These symptoms generally resolve within 12 to 24 hours. In thecase of significant toxicity, a latent phase develops and lasts 1 to 4 days, during which time liverenzyme concentrations may begin to rise. In severe toxicity, jaundice and liver tendernessdevelop at the end of the latent phase. About 2% to 4% of patients who develop toxic plasmaconcentrations and do not receive antidotal treatment progress to hepatic failure and death.

Since 2003, the American Academy of Pediatrics has recommended against the use ofipecac in toxic ingestions, partly because the prolonged vomiting that can result interferes withother management options. Although toxicologists advised against the administration of activatedcharcoal in the recent past due to concerns about interference with the absorption of N-acetylcysteine, recent studies have shown clinically insignificant reductions in N-acetylcysteineabsorption following charcoal administration. Charcoal now is recommended after most cases ofAPAP overdose.

Vomiting is a nonspecific response to APAP ingestion and does not predict hepatic toxicity.Toxicity is confirmed by an increase in serum transaminases and a prolonged prothrombin time.There are many different APAP-containing products on the market, with a wide range of totalmedication per bottle. The potential for toxicity depends on the weight of the child and thecontents of the bottle. In the absence of that information, it is not possible to state that the child inthe vignette has received a subtoxic dose.

References:

American Academy of Pediatrics Committee on Drugs. Acetaminophen toxicity in children.Pediatrics. 2001;108:1030-1024. Available at:http://pediatrics.aappublications.org/cgi/content/full/108/4/1020

American Academy of Pediatrics Committee on Injury, Violence, and Poison Prevention. Poisontreatment in the home. Pediatrics. 2003;112:1182-1185 Available at:http://pediatrics.aappublications.org/cgi/content/full/112/5/1182

Bond GR. Home syrup of ipecac use does not reduce emergency department use or improveoutcome. Pediatrics. 2003;112:1061-1064. Available at:http://pediatrics.aappublications.org/cgi/content/full/112/5/1061

McGuigan ME: Poisoning potpourri. Pediatr Rev. 2001:22:295-302. Available at:http://pedsinreview.aappublications.org/cgi/content/full/22/9/295

Osterhoudt KC, Ewald MB, Shannon M, Henretig FM. Toxicologic emergencies. In: Fleisher GR,Ludwig S, Henretig FM, eds. Textbook of Pediatric Emergency Medicine. 5th ed. Philadelphia,Pa: Lippicott Williams & Wilkins; 2006:951-1008

Copyright © 2007 by the American Academy of Pediatrics page 32

Page 33: AAP MCQ 2007

2007 PREP SA on CD-ROM

Question: 17

A sexually active adolescent male presents with the primary complaint of pubic and perianalpruritus. Careful examination reveals pubic or "crab" lice infestation (Item Q17A).

Of the following, the MOST characteristic feature of this infestation is that the lice

A. are difficult to detect because of their rapid movement

B. lay viable eggs that may hatch up to 7 days after being attached to a hair shaft

C. may survive for 36 hours without a blood meal

D. only infest pubic and perianal regions

E. rarely infest African-American patients

Copyright © 2007 by the American Academy of Pediatrics page 33

Page 34: AAP MCQ 2007

2007 PREP SA on CD-ROM

Preferred Response: CCritique: 17

Pediculosis, the infestation of humans by lice, has been documented for millennia. Three speciesof lice infest humans: Pediculus humanus humanus, the body louse; Pediculus humanus capitis,the head louse; and Pthirus pubis, the crab louse. The hallmark of louse infestation is pruritus atthe site of bites. Lice are more active at night, frequently disrupting sleep of the host, which is thederivation of the term “feeling lousy.”

Adult crab lice can survive without a blood meal for 36 hours. Unlike head lice, which maytravel up to 23 cm/min, pubic lice are sluggish, traveling a maximum of 10 cm/d. Viable eggs onpubic hairs may hatch up to 10 days later. Crab louse infestation is localized most frequently tothe pubic and perianal regions but may spread to the mustache, beard, axillae, eyelashes, orscalp hair. Infestation usually is acquired through sexual contact, and the finding of pubic lice inchildren (often limited to the eyelashes) should raise concern for possible sexual abuse.Maculae caeruleae (Item C17A), blue-gray macules observed on the abdomen and thighs atsites where lice have fed, is a useful, although less common finding. Crab lice affect all racesand ethnic groups. This is in contrast to head lice, which rarely infest African-Americans,perhaps because the oval cross-sectional shape of their scalp hair does not permit lice to graspthe hair effectively.

The preferred treatments for pubic lice infestation are permethrin 1% or pyrethrin withpiperonyl butoxide. Other options include permethrin 5% or malathion (although the latter agentis expensive, potentially flammable, and may be irritating when applied to the groin). Lindane iseffective, but concerns about toxicity if used improperly or ingested inadvertently limit its use. Itis considered a second-line therapy, is contraindicated for use in neonates and pregnantwomen, and should be used with caution in those weighing less than 110 lb. If the eyelashes areaffected, petrolatum may be applied to them three to five times daily for 10 days.

References:

Meinking T, Taplin D. Infestations: lice. In: Schachner LA, Hansen RC, eds. PediatricDermatology. 3rd ed. St. Louis, Mo: Mosby; 2003:1141-1160

Paller AS, Mancini AJ. Bites and infestations. In: Hurwitz Clinical Pediatric Dermatology. 3rd ed.Philadelphia, Pa: Elsevier Inc; 2006:479-502

Copyright © 2007 by the American Academy of Pediatrics page 34

Page 35: AAP MCQ 2007

2007 PREP SA on CD-ROM

Question: 18

A 6-year-old girl presents with a 1-year history of periumbilical, nonradiating abdominal pain. Thepain occurs at least three times per week and lasts up to 30 minutes. There is no history ofheartburn, constipation, or diarrhea. Physical examination, complete blood count, erythrocytesedimentation rate, and urinalysis yield normal results. A Helicobacter pylori serology(immunoglobulin G antibody) is positive.

Of the following, a TRUE statement regarding this patient is that

A. empiric therapy with omeprazole and trimethoprim-sulfamethoxazole should be instituted

B. the H pylori antibody test is more sensitive in younger children than older children

C. the positive serology should be confirmed by another diagnostic test

D. the prevalence of H pylori increases with higher socioeconomic status

E. this patient most likely has a gastric ulcer

Copyright © 2007 by the American Academy of Pediatrics page 35

Page 36: AAP MCQ 2007

2007 PREP SA on CD-ROM

Preferred Response: CCritique: 18

There is no clear association between Helicobacter pylori and chronic recurrent abdominal painof childhood. Chronic recurrent abdominal pain affects approximately 10% to 15% of school-agechildren. No structural or inflammatory cause of the pain is identified in most cases. Affectedchildren usually have functional bowel disease. Functional bowel disease in children and teenscan be categorized as: nonulcer dyspepsia (epigastric discomfort, early satiety, and bloating),irritable bowel syndrome (abdominal cramps associated with diarrhea or constipation), andclassic functional pain of childhood (periumbilical, crampy, and nonradiating).

A subset of children who have chronic recurrent abdominal pain also have concurrent Hpylori infection. Commonly, this infection is identified on routine serologic screening by theirprimary care physicians, as described for the girl in the vignette. In most such children,especially in those who have no epigastric symptoms, the H pylori probably representsasymptomatic colonization rather than the cause of the pain. The only firm indications foreradicating H pylori in adults are duodenal ulcer and gastric lymphoma (MALToma).Randomized, controlled trials in adults who have nonulcer dyspepsia suggest that eradication ofH pylori does not resolve dyspeptic symptoms. Similar large-scale trials have not beenconducted in children. Therefore, it remains controversial whether children who have chronicabdominal pain and H pylori infection should receive therapy for their colonization.

Nevertheless, some open-label studies in children do suggest that eradicating H pylori mayalleviate pain. In addition, eradicating H pylori may prevent the development of subsequentpeptic ulcer disease or (far less commonly) gastric lymphoma. However, the serology has apoor predictive value in a low-prevalence population. Therefore, one approach is to confirm apositive serology with a second diagnostic test (fecal antigen, urea breath test, or endoscopy). Ifresults of the second test are positive, therapy should be considered. Although endoscopy is the“gold standard” diagnostic test for H pylori and can identify other causes of abdominal pain (eg,esophagitis, gastritis, ulcers, celiac disease), it is also the most invasive test. Thus, thephysician must determine benefits, risks, and cost of diagnostic testing and treatment inindividual patients.

Omeprazole and trimethoprim/sulfisoxazole do not eradicate H pylori. The H pylori antibodytest is less sensitive in younger children. The prevalence of H pylori decreases with increasingsocioeconomic status. Gastric ulcers are uncommon in children, and a child who hasperiumbilical abdominal pain most likely has functional abdominal pain without peptic ulcerdisease.

References:

AAP Subcommittee on Chronic Abdominal Pain. Chronic abdominal pain in children. Pediatrics.2005;115:e370-e381. Available at:http://pediatrics.aappublications.org/cgi/content/full/115/3/e370

Gold BD, Colletti RB, Abbott M, et al. Helicobacter pylori infection in children: recommendationsfor diagnosis and treatment. J Pediatr Gastroenterol Nutr. 2000;31:490-497 Available at:http://www.jpgn.org/pt/re/jpgn/fulltext.00005176-200011000-00007.htm

McNamara DA, Buckley M, O’Morain CA. Nonulcer dyspepsia. Current concepts andmanagement. Gastroenterol Clin North Am. 2000;29:807-818 Abstract available at:http://www.ncbi.nlm.nih.gov/entrez/query.fcgi?cmd=Retrieve&db=pubmed&dopt=Abstract&list_uids=11190065&query_hl=11&itool=pubmed_docsum

Copyright © 2007 by the American Academy of Pediatrics page 36

Page 37: AAP MCQ 2007

2007 PREP SA on CD-ROM

Question: 19

You are conducting rounds in the newborn nursery with a group of residents. You describe thechoices for infant nutrition that might optimize growth and development.

Of the following, you are MOST likely to state that

A. preterm and term infants both require 100 to 120 kcal/kg per day of energy to grow

B. preterm infants require less caloric intake per kilogram to grow than do term infants

C. term infants require 60 to 80 kcal/kg per day of energy to grow

D. term infants require 30 to 50 mL/kg per day of fluid intake

E. term infants whose birthweights are greater than 2,500 g require more energy per kilogram togrow than those whose birthweights are less than 2,500 g

Copyright © 2007 by the American Academy of Pediatrics page 37

Page 38: AAP MCQ 2007

2007 PREP SA on CD-ROM

Preferred Response: ACritique: 19

The energy (caloric) requirement for newborns to grow and develop varies with gestational age,presence of illness, a history of surgery and need for wound healing, and the neonatalenvironment. Energy use is partitioned into that required for basic metabolic function (basalmetabolic rate), thermic expenditure of enteral feeding and digestion, physical activity, and thesynthesis of new tissue. For term and preterm infants, these categories have the followingrequirements:

Categorykcal/kg per dayResting metabolic rate50 to 60Activity0 to 10Temperature regulation0 to 10Growth of new tissue10 to 15Storage of energy (mostly fat)20 to 30Energy excreted (urine and stool)10 to 15TOTAL90 to 140

Special considerations for preterm infants and low-birthweight (LBW) term infants (<2,500 g)warrant administration of the higher end of this caloric range. The preterm and LBW infant havelower fat stores and cannot conduct thermoregulation well, resulting in greater expenditure inheat production than at term. Similarly, such infants must accomplish a greater proportion oforgan system development and new tissue synthesis (as well as production of storage fat) thanthe term infant, and this uses more energy. All of these considerations are greater still in thevery low-birthweight (<1,500 g) or extremely low-birthweight (<1,000 g) infant. Further,conditions such as heart or lung disease, surgery, or infection increase energy expenditure.Because digestive or absorptive problems also may increase the excretion of unused energy,attention must be paid to the source of energy (carbohydrate, fat, and protein composition andcontent) in the milk provided to infants. The correct fluid volume for term newborns is 60 to 100mL/kg per day.

References:

Adamkin D. Feeding the preterm infant. In: Bhatia J, ed. Perinatal Nutrition: Optimizing InfantHealth and Development. New York, NY: Marcel Dekker; 2005:165-190

Denne SC, Poindexter BB, Leitch CA, Ernst JA, Lemons PK, Lemons JA. Nutrition andmetabolism in the high-risk neonate: enteral nutrition. In: Martin RJ, Fanaroff AA, Walsh MC,eds. Fanaroff and Martin’s Neonatal-Perinatal Medicine: Diseases of the Fetus and Infant. 8thed. Philadelphia, Pa: Mosby-Elsevier; 2006:661-678

Kleinman RE. Nutritional needs of the preterm infant. In: AAP Nutrition Handbook. 5th ed. ElkGrove Village, Ill: American Academy of Pediatrics; 2004:23-46

Kuzma-O’Reilly B, Duenas ML, Greecher C, et al. Evaluation, development, and implementationof potentially better practices in neonatal intensive care nutrition. Pediatrics. 2003;111: e461-e470. Available at: http://pediatrics.aappublications.org/cgi/content/full/111/4/SE1/e461

Schanler RJ. The low birth weight infant. In: Walker WA, Watkins JB, Duggan C, eds. Nutrition inPediatrics: Basic Science and Clinical Applications. 3rd ed. Hamilton, Ontario, Canada: BCDecker, Inc; 2003:491-514

Copyright © 2007 by the American Academy of Pediatrics page 38

Page 39: AAP MCQ 2007

2007 PREP SA on CD-ROM

Question: 20

A 3-year-old child who has a history of recurrent otitis media with effusion (OME) in infancy isbrought to the clinic. His mother is afraid that he has a hearing loss because he does not talk asmuch as his brother did at the same age. He speaks in three-word sentences, and you canunderstand fewer than 50% of his words. Results of his physical examination, including the ears,are normal.

Of the following, the MOST appropriate statement regarding this child's condition is that

A. even mild conductive hearing loss could affect his later school performance without frankspeech delay

B. OME does not cause conductive hearing loss severe enough to cause speech delay

C. performing hearing screening solely in response to parental concern is not recommended

D. testing air and bone conduction thresholds in the office will help you rule out hearing loss

E. the absence of middle ear fluid rules out conductive hearing loss

Copyright © 2007 by the American Academy of Pediatrics page 39

Page 40: AAP MCQ 2007

2007 PREP SA on CD-ROM

Preferred Response: ACritique: 20

Some children who have hearing loss may be diagnosed during neonatal screening, but manydevelop hearing loss later in infancy or early childhood and usually are brought to medicalattention because of a delay in language development. The 3-year-old child described in thevignette has language that is less than 50% understood by the practitioner, which is abnormaland necessitates an audiologic evaluation. In addition, a parent’s concern about a child’s speechshould prompt the pediatrician to investigate the child’s hearing abilities and should not beignored. A history of poor school performance and inattention at school are other reasons toconsider a hearing evaluation.

Hearing impairment can be divided into three primary categories: sensorineural, conductive,and mixed. Causes of sensorineural losses include genetic diseases, infections, ototoxicmedications, and anatomic anomalies. Conductive hearing loss is more common thansensorineural hearing loss in childhood. It results from interrupted sound transmission in theexternal or middle ear, and middle ear effusion is the most common cause. The hearing loss canvary between 20 and 60 dB, which may be substantial enough to cause significant speechdelay.

It has been shown that even mild speech delay can impair school performance. Childrenwho have borderline hearing loss (16 to 25 dB) may miss 10% of speech in an environment withnoise (eg, a noisy classroom), and interaction with peers may be affected. Children who havemild hearing loss (26 to 40 dB) may miss up to 50% of speech in a noisy classroom and maydevelop poor self-esteem because of being labeled as a “problem student.” Moderate hearingloss (41 to 55 dB) can cause significant problems with communication.

Early diagnosis, with a prompt, complete speech and language evaluation, is the key tomaximizing speech improvement. The earlier the cause of conductive hearing loss is discoveredand corrected (eg, through tympanostomy tube placement), the more likely the child will developnormal language.

Testing air and bone conduction thresholds with a tuning fork may help distinguish betweenconductive and sensorineural hearing loss, but it should not be used to diagnose hearing loss,and results may be unreliable if the hearing loss is bilateral. The absence of middle ear fluid atthe time of the examination does not rule out hearing loss. Therefore, audiometry testing shouldbe performed if hearing loss is suspected.

References:

Gregg RB, Wiorek LS, Arvedson JC. Pediatric audiology: a review. Pediatr Rev. 2004;25:224-234. Available at: http://pedsinreview.aappublications.org/cgi/content/full/25/7/224

Haddad J Jr. Hearing loss. In: Behrman RE, Kliegman RM, Jenson HB, eds. Nelson Textbook ofPediatrics. 17th ed. Philadelphia, Pa: WB Saunders Co; 2004:2129-2135

Jarvelin MR, Mati-Torkko E, Sorri MJ, Rantakallio PT. Effect of hearing impairment oneducational outcomes and employment up to the age of 25 years in northern Finland. Br JAudiol. 1997;31:165-175. Abstract available at:http://www.ncbi.nlm.nih.gov/entrez/query.fcgi?cmd=Retrieve&db=pubmed&dopt=Abstract&list_uids=9276099&query_hl=10&itool=pubmed_docsum

Copyright © 2007 by the American Academy of Pediatrics page 40

Page 41: AAP MCQ 2007

2007 PREP SA on CD-ROM

Question: 21

The pediatric resident at the emergency department notifies you that one of your patients, an 18-month-old boy, has been admitted to the hospital for observation after being found with an openbottle of lemon-scented furniture polish. On arrival at the emergency department, the child hadlemon-scented polish on his clothing, and his breath smelled of the substance. His mother statedthat he had vomited once, but there has been no choking, gagging, or coughing since theingestion.

Of the following, the MOST appropriate management includes

A. administration of activated charcoal

B. consultation for urgent bronchoscopy

C. gastric lavage

D. observation for 6 hours for signs of distress

E. serial chest radiographs

Copyright © 2007 by the American Academy of Pediatrics page 41

Page 42: AAP MCQ 2007

2007 PREP SA on CD-ROM

Preferred Response: DCritique: 21

Ingestion of a hydrocarbon can cause irritation of the orogastric mucosa and the respiratoryepithelium. Because hydrocarbons often are volatile, simple inhalation of vapor may causechoking and gagging, even if no liquid is ingested. Initial physical examination may reveal thesmell of the hydrocarbon substance on the patient’s breath, as described for the boy in thevignette; other symptoms of inhalation or ingestion may include wheezing, laryngospasm, andother signs of respiratory distress. Ingestion may result in coughing and vomiting, which cancause further aspiration or lung irritation from the gastric contents. Absence of tachypnea is agood prognostic sign. Fulminant chemical pneumonitis associated with hypoxia may occur andreaches a peak within 3 days of ingestion. Fever occurs early (usually within 8 hours) and isdue to direct chemical inflammation, not infection.

Management of asymptomatic patients, such as the boy described in the vignette, mayinclude only observation for 4 to 6 hours. Gastric lavage or administration of either ipecac oractivated charcoal is contraindicated (unless the hydrocarbon is mixed with certain high-risktoxins or the volume ingested is large) because of the risk of vomiting and additional aspiration.Symptomatic patients should have respiratory distress managed with oxygen, a trial ofbronchodilators, and supportive care. There is no evidence-based support for empiric treatmentwith antibiotics or steroids. Chest radiographs initially appear normal, especially in asymptomaticchildren, although they may reveal signs of pneumonitis (Item C21A) by 6 hours after ingestion.Radiography is not necessary in asymptomatic children who have normal findings on physicalexamination. Bronchoscopy rarely is indicated or helpful.

For most children, the outcome following hydrocarbon ingestion is good. In rare cases,bronchospasm may continue or pneumatoceles may form.

References:

Fortenberry JD, Mariscalco MM. General principles of poisoning. In: McMIllan JA, DeAngelisCD, Feigin RD, Warshaw JB, eds. Oski’s Pediatrics: Principles and Practice. 3rd ed.Philadelphia, Pa: Lippincott, Williams & Wilkins; 1999:621-622

Copyright © 2007 by the American Academy of Pediatrics page 42

Page 43: AAP MCQ 2007

2007 PREP SA on CD-ROM

Question: 22

A 17-year-old girl complains of clumsiness over the past 3 days. She has had moderateheadaches for 1 month and neck discomfort for 3 days. Physical examination reveals right-sided dysmetria and left upper and lower extremity numbness to pinprick and weakness, gradedas 4/5. The remainder of her examination results are normal.

Of the following, the MOST appropriate evaluation to establish this patient's diagnosis is

A. computed tomography scan of the brain

B. lumbar puncture

C. measurement of nerve conduction velocities

D. measurement of somatosensory evoked potentials

E. urine toxicology screen

Copyright © 2007 by the American Academy of Pediatrics page 43

Page 44: AAP MCQ 2007

2007 PREP SA on CD-ROM

Preferred Response: ACritique: 22

Headache is a frequent primary or secondary complaint in pediatric practice. The clinician mustbe able to distinguish primary, benign headaches (ie, migraines or tension headaches) fromsecondary, pathologic headaches, which can stem from a tumor, abscess, or other causes ofincreased intracranial pressure.

Secondary or ominous headaches are characterized by occipital or frontal pain. Theheadache frequently is worse in the morning, after the child has been recumbent at night andintracranial pressure rises with less effect from gravity. The child may wake in the early morningwith a complaint of head pain. Vomiting may occur at that time, too. Headaches that areassociated with persistent vomiting are worrisome, especially when diarrhea is absent.Secondary headache also may be precipitated by the Valsalva maneuver.

New-onset neurologic findings in the first 2 to 6 months after onset of pain are especiallyconcerning for a secondary headache. Specifically, the finding of papilledema, strabismus,unilateral weakness, or ataxia should alert the practitioner to the likelihood of a structural lesion.Neurologic deficits such as weakness, strabismus, or ataxia also can be seen rarely withmigraine variants, such as hemiplegic migraine, ophthalmoplegic migraine, or basilar migraine,respectively, but such atypical headache syndromes are diagnoses of exclusion. Urgentneuroimaging should be obtained for children who have headache and neurologic deficits.Computed tomography scan of the head without contrast can be obtained quickly withoutsedation to identify a structual lesion causing headache from mass effect. Alernatively, magneticresonance imaging of the head can be obtained if the child is stable and monitored closely duringany sedation that may be required.

The headaches exibited by the 17-year-old girl in the vignette, combined with her ataxia, lefthemisensory loss, right dyscoordination, and neck pain, point to a posterior fossa mass. Headcomputed tomography scan is the most appropriate next step. Lumbar puncture iscontraindicated when there is suspicion of a mass. Nerve conduction velocities are useful onlyto evaluate a peripheral neuropathy, and somatosensory evoked potentials are used to evaluatedemyelination in the peripheral or central nervous system. A toxin would not produce thisconstellation of symptoms and signs.

References:

Fisher PG. Help for headaches: a strategy for your busy practice. Contemp Pediatr. 2005;22:34-41

Haslam RHA. Headaches. In: Behrman RE, Kliegman RM, Jenson HB, eds. Nelson Textbook ofPediatrics. 17th ed. Philadelphia, Pa: WB Saunders; 2004:2012-2015

Honig PJ, Charney EB. Children with brain tumor headaches. Distinguishing features. Am J DisChild. 1982;136:121-124. Abstract available at:http://www.ncbi.nlm.nih.gov/entrez/query.fcgi?cmd=Retrieve&db=pubmed&dopt=Abstract&list_uids=7064925&query_hl=7&itool=pubmed_DocSum

Lewis DW, Ashwal S, Dahl G, et al, Quality Standards Subcommittee of the American Academyof Neurology; Practice Committee of the Child Neurology Society. Practice parameter: evaluationof children and adolescents with recurrent headaches: report of the Quality StandardsSubcommittee of the American Academy of Neurology and the Practice Committee of the ChildNeurology Society. Neurology. 2002;59:490-498. Available at:http://www.neurology.org/cgi/content/full/59/4/490

Copyright © 2007 by the American Academy of Pediatrics page 44

Page 45: AAP MCQ 2007

2007 PREP SA on CD-ROM

Question: 23

You are supervising a pediatric resident in her continuity clinic. She is evaluating a 4-week-oldmale infant who has had projectile vomiting after feeding for the past week. After reviewing thepatient's electrolyte levels, she obtains an electrocardiogram (ECG) and asks you to helpinterpret it (Item Q23A). The ECG reveals flat T waves.

Of the following, the MOST likely electrolyte abnormality suggested by the electrocardiographicfindings is

A. hypercalcemia

B. hyperkalemia

C. hypernatremia

D. hypocalcemia

E. hypokalemia

Copyright © 2007 by the American Academy of Pediatrics page 45

Page 46: AAP MCQ 2007

2007 PREP SA on CD-ROM

Preferred Response: ECritique: 23

Hypokalemia generally is defined as a serum potassium concentration of less than 3.5 mEq/L(3.5 mmol/L) in infants and children. Potassium is an intracellular cation that is vital in maintainingthe normal transmembrane charge in cells. Hypokalemia may be precipitated by gastrointestinalloss from vomiting or diarrhea, especially when there is inadequate oral intake. Other primarycauses include aldosterone excess or renal tubular disorders. Abnormalities of potassiumhomeostasis can affect cardiac conduction and often are observed with defined changes insurface electrocardiography.

The typical electrocardiographic findings seen in a child who has hypokalemia include STsegment depression, flattening of the T waves, and the appearance of a U wave (Item C23A),as seen for the infant in the vignette. Generally, there are broadened T and U waves, and rarelythe QT interval can be prolonged, which places the patient at greater risk for the development ofventricular dysrhythmias. However, arrhythmias with hypokalemia are unusual unless thepatient is receiving digoxin, whose arrhythmogenic effects are potentiated by hypokalemia.

Hyperkalemia most commonly is due to renal failure, adrenal insufficiency, or an iatrogenicoverdose. As levels exceed 5.5 mEq/L (5.5 mmol/L), the T waves on the electrocardiogram maybecome tall and peaked (Item C23B). Prolonged conduction delay also may develop aspotassium concentrations increase. Hypercalcemia may be due to a variety of causes andtypically shortens the QT interval by shortening the ST segment. Hypercalcemia also may affectthe sinus node, causing sinus slowing or sinus arrest. Hypocalcemia, in contrast, prolongs theST segment, thereby prolonging the QT interval. Hypernatremia is not associated withelectrocardiographic abnormalities.

References:

Garson A Jr. Electrocardiography. In: Garson A Jr, Bricker JT, Fisher DJ, Neish SR, eds. TheScience and Practice of Pediatric Cardiology. 2nd ed. Baltimore, Md: Williams & Wilkins;1998:735-788

Schaefer TJ, Wolford RW. Disorders of potassium. Emerg Med Clin North Am. 2005;23:723-74.Abstract available at:http://www.ncbi.nlm.nih.gov/entrez/query.fcgi?orig_db=PubMed&db=PubMed&cmd=Search&term=%22Emergency+medicine+clinics+of+North+America%22[Jour]+AND+2005[pdat]+AND+Schaefer+TJ[author]

Copyright © 2007 by the American Academy of Pediatrics page 46

Page 47: AAP MCQ 2007

2007 PREP SA on CD-ROM

Question: 24

A female infant is born with unilateral cleft lip and palate. Findings on the remainder of thephysical examination are normal, and you inform the parents that this apparently is an isolatedbirth defect.

During counseling of the family about their risk for having another similarly affected child, thestatement that you are MOST likely to include is that

A. because the defect is isolated, their risk is no greater than that of any other couple

B. cleft lip with/without cleft palate is a multifactorial trait that has a 4% risk of recurrence

C. the recurrence risk can be estimated only after a chromosome analysis has been obtained

D. the risk is increased for future females, but not males

E. the risk is increased only if one of the parents had cleft lip with/without cleft palate

Copyright © 2007 by the American Academy of Pediatrics page 47

Page 48: AAP MCQ 2007

2007 PREP SA on CD-ROM

Preferred Response: BCritique: 24

Isolated cleft lip with/without cleft palate is inherited as a multifactorial trait that is due to theinteraction of genetic and environmental factors. Multifactorial inheritance commonly refers totraits that occur with greater frequency among individuals in a particular family than they do inthe general population, but at a lower rate than would be expected for single-gene traits.Recurrence risks for multifactorial traits have been derived by analysis of many families thatexpress the trait. For cleft lip with/without cleft palate, the recurrence risk for a family in whichneither parent is affected and there is one affected child is approximately 4% in futurepregnancies.

In general, disorders that are inherited as multifactorial traits include most of the commonbirth defects (Item C24A) as well as many adult-onset diseases. For most multifactorial birthdefects, the recurrence risk for parents who have had one affected child ranges from 2% to 5%,although some traits are associated with higher risks (eg, 10% for hypospadias). Risks may bemodified based on other characteristics, such as the severity of the defect, the sex of theaffected individual, and the number of affected individuals in the family. For example, unilateralcleft lip has a recurrence risk of 4%, while bilateral cleft lip has a recurrence risk of 6%. Similarly,some defects have a sex predilection, including Hirschsprung disease and pyloric stenosis,which appear more commonly in males, and hip dislocation, which is more common amongfemales. In families in which one of these defects has occurred, the risk to future offspring isgreater if the original affected child is of the less commonly affected sex. For example, therecurrence risk for subsequent siblings of a child who has pyloric stenosis is greater if theaffected child is a girl rather than a boy. Estimation of recurrence risks also requires thatinformation be obtained about the extended family to determine if there are other similarlyaffected individuals, particularly first-degree relatives. The risk is lowest in families in whom asingle individual is affected; families that include multiple affected individuals have a higher risk.For example, the recurrence risk for subsequent siblings of a child who has clubfoot is 3%, but ifone parent also had clubfoot, the risk increases to 10%.

To provide the most accurate information about recurrence risk to families, all children whohave birth defects should be examined carefully for the presence of other abnormalities. Thepossibility that the most obvious defect actually is part of a genetic syndrome, which may carrya higher risk, must be considered in all cases. For example, cleft lip is the most obvious featureof van der Woude syndrome, which also includes lip pits and the possible absence of centralincisors. This disorder is transmitted as an autosomal dominant trait, with a 50% recurrencerisk. Thus, families that have this genetic syndrome are at much higher risk for recurrence, andthe parents should be examined if this disorder is suspected.

Alternatively, some birth defects can occur following exposure to a teratogen, in which casethe recurrence risk in future pregnancies is not increased if the teratogen is avoided (eg, cleft lipwith/without cleft palate in fetuses exposed to phenytoin).

In general, chromosome analysis should be obtained in infants and children who have two ormore major abnormalities (eg, clefting and congenital heart disease) or one major anomaly andtwo or three minor defects (eg, epicanthal folds, single palmar crease). Accordingly,chromosome analysis is not indicated for the infant described in the vignette, who has anisolated cleft lip and palate.

References:

Bacino CA. Approach to congenital malformations. UpToDate. 2006:14.1. Available at:http://www.utdol.com/utd/content/topic.do?topicKey=dis_chld/7840&type=P&selectedTitle=86~152

Jones KL. Dysmorphology. In: Behrman RE, Kliegman RM, Jenson HB, eds. Nelson Textbookof Pediatrics. 17th ed. Philadelphia Pa: Saunders; 2004:616-623

Stal S, Hollier LH Jr, Edwards M. Facial clefts and holoprosencephaly. UpToDate. 2006;14.1.Available at:

Copyright © 2007 by the American Academy of Pediatrics page 48

Page 49: AAP MCQ 2007

2007 PREP SA on CD-ROM

http://www.utdol.com/utd/content/topic.do?topicKey=neonatol/13931&type=P&selectedTitle=3~5

Copyright © 2007 by the American Academy of Pediatrics page 49

Page 50: AAP MCQ 2007

2007 PREP SA on CD-ROM

Question: 25

A 15-year-old postmenarcheal girl presents with persistent right lower quadrant discomfort of 6weeks' duration and a feeling of abdominal fullness. Her last menstrual period was 3 weeks ago.Results of a urine pregnancy test are negative, and urinalysis results are normal.Ultrasonography reveals a well-defined right ovarian cyst (Item Q25A) filled with multiple echoes.

Of the following, the BEST next step for the management of this patient is

A. laparoscopic cyst aspiration

B. measurement of serum tumor markers

C. oophorectomy

D. repeat ultrasonography in 4 to 8 weeks

E. therapy with combined oral contraceptives

Copyright © 2007 by the American Academy of Pediatrics page 50

Page 51: AAP MCQ 2007

2007 PREP SA on CD-ROM

Preferred Response: BCritique: 25

Ovarian masses in adolescent girls may result from functional cysts and benign or malignantneoplasms. Ovarian cysts are very common in adolescents. Although many ovarian cysts areasymptomatic, some patients present with pain, irregular menses, urinary frequency,constipation, or pelvic heaviness. In general, a simple, fluid-filled cyst less than 6 cm in sizedocumented on ultrasonography may be managed with conservative observation that usuallyincludes follow-up examinations or ultrasonography. Many of these cysts resolve in 4 to 8weeks.

However, solid masses and cysts that have ultrasonographic findings suspicious formalignancy require further evaluation. Suspicious cysts have complex features, such asseptations, are multiloculated, and have increased echoes or calcifications, as described for thecyst in the vignette. Such cysts subsequently are imaged with computed tomography scan ormagnetic resonance imaging to define more clearly the characteristics of a solid mass orsuspicious cyst and identify liver or lung metastases. Surgical intervention is based on theradiologic images and the identification of tumor markers. Tumor markers (eg, CA 125, alpha-fetoprotein, human chorionic gonadotropin, carcinoembryonic antigen, F9 embryoglycan) assistin the diagnosis of an ovarian neoplasm and reflect clinical response or recurrences.Accordingly, serum tumor markers should be measured in the patient in the vignette.

Patients should be counseled about the risks and benefits of surgical approaches,adjunctive chemotherapy, emerging genetic tests, and preservation of reproductive potential.Treatment of a patient who has an ovarian neoplasm should involve collaborative participation ofthe surgeon, pathologist, geneticist, and oncologist to develop an individualized plan of care.

Laparoscopic cyst aspiration is not an appropriate intervention for a cyst that has complexfeatures on ultrasonography. The aspiration of large, simple ovarian cysts has been associatedwith a high rate of recurrence. Many large, asymptomatic cysts have been followed with repeatultrasonography and spontaneously resolved.

Oophorectomy should not be the presumptive surgical intervention. Management plansshould be based on tumor markers and pathologic tissue sections. Repeat ultrasonography in 4to 6 months is inadequate to assess an ovarian mass that has complex ultrasonographicfeatures. In 4 to 8 weeks, most simple, fluid-filled ovarian cysts resolve or decrease in size,although corpus luteum cysts may be slower to regress, usually showing evidence ofregression by 3 months. Therapy with combined estrogen and progestin oral contraceptives thatsuppress the hypothalamic ovarian axis has been offered to patients who have simple, fluid-filledovarian cysts, particularly those who have von Willebrand disease or who are recipients ofanticoagulation therapy. Such therapy is an attempt to decrease the likelihood of future cystformation and the risk of hemorrhage.

References:

Laufer MR, Goldstein DP. Benign and malignant ovarian masses. In: Emans SJH, Laufer MR,Goldstein DP, eds. Pediatric and Adolescent Gynecology. 5th ed. Philadelphia, Pa: Lippincott,Williams & Wilkins; 2005:685-728

Nelson AL, Neinstein LS. Pelvic masses. In: Neinstein LS, ed. Adolescent Health Care APractical Guide. 4th ed. Philadelphia, Pa: Lippincott, Williams & Wilkins; 2002:994-1000

Stepanian M, Cohn DE. Gynecologic malignancies in adolescents. Adoles Med Clin. 2004;15:549-568

Copyright © 2007 by the American Academy of Pediatrics page 51

Page 52: AAP MCQ 2007

2007 PREP SA on CD-ROM

Question: 26

A 16-year-old boy is brought to your office by his parents because he wants medicine to helphim grow and develop muscles. He has been working out for 1 year without much effect. Onphysical examination, his height is 63 in (adjusted mid-parental height, 73 in), he weighs 106 lb,and he has testes that are 8 mL in volume. He appears healthy but very young. His father saysthat he was very small as a boy but was given three injections of growth hormone when he was16 and grew to his present height of 71 in during late high school and college. Laboratory testresults for the boy are normal, including a complete blood count, erythrocyte sedimentation rate,electrolytes, blood urea nitrogen, creatinine, prolactin, insulin-like growth factor-1, tissuetransglutaminase antibody, quantitative immunoglobulin A, free thyroxine, and thyroid-stimulatinghormone. A bone age radiograph is reported to resemble that of a 13-year-old.

Of the following, your BEST approach is to

A. counsel that this is delayed puberty and that he will grow over the next few years

B. obtain brain magnetic resonance imaging and refer the boy to an endocrinologist for growthhormone studies

C. offer an endocrinologic referral for a 1- to 2-year course of growth hormone therapy

D. offer an endocrinologic referral for a 1- to 2-year course of testosterone therapy

E. suggest that high-calorie food supplements be taken daily to enhance growth

Copyright © 2007 by the American Academy of Pediatrics page 52

Page 53: AAP MCQ 2007

2007 PREP SA on CD-ROM

Preferred Response: ACritique: 26

The normal screening laboratory results and delayed bone age described for the boy in thevignette suggest the diagnosis of constitutional delayed puberty. Because he already hasentered puberty, only counseling that his growth spurt is soon to come is necessary.

A normal insulin-like growth factor-1 concentration does not entirely rule out growth hormonedeficiency, but the boy’s regular growth rate, reasonable height, strong family history for pubertaldelay, and signs of puberty make growth hormone deficiency very unlikely. Therefore, referral toan endocrinologist and magnetic resonance imaging are unnecessary at this point. It is likely thatthe boy’s father received testosterone rather than growth hormone injections. Daily injections ofgrowth hormone induce, on average, additional height of approximately 1/3 inch for each year oftreatment. However, if treatment is stopped before adult height or target height is reached,growth usually slows, and the growth hormone-induced additional height prediction is lost. Low-dose testosterone injections sometimes are administered to induce some external signs ofpuberty (a slight increase in growth, increase in phallic size and in pubic hair) in a boy who haspubertal delay. Treatment for 3 to 6 months appears not to compromise adult height. Suchtherapy can be offered to the boy and his family, but is never required. In girls, estrogen therapyalso induces puberty. However, this treatment rarely is used in constitutional delay becauseexogenous estrogen advances bone age and decreases adult height. There is no evidence thathigh-calorie food supplements enhance growth or height significantly in otherwise normalpubertal children.

References:

Brook CG. Treatment of late puberty. Horm Res. 1999;51(suppl 3):101-103 Abstract availableat:http://www.ncbi.nlm.nih.gov/entrez/query.fcgi?cmd=Retrieve&db=pubmed&dopt=Abstract&list_uids=10592451&query_hl=50&itool=pubmed_docsum

Cuttler L. Safety and efficacy of growth hormone treatment for idiopathic short stature [editorial].J Clin Endocrinol Metab. 2005;90:5502-5504

Misra M, Park-Bennett S. Disorders of puberty. In: Burg FD, Ingelfinger JR, Polin RA, GershonAA, eds. Gellis & Kagan’s Current Pediatric Therapy. Philadelphia, Pa: WB Saunders; 2002:706-710

Nathan BM, Palmert MR. Regulation and disorders of pubertal timing. Endocrinol Metab ClinNorth Am. 2005;34:617-641. Abstract available at:http://www.ncbi.nlm.nih.gov/entrez/query.fcgi?orig_db=PubMed&db=PubMed&cmd=Search&term=%22Endocrinology+and+metabolism+clinics+of+North+America%22[Jour]+AND+617[page]+AND+2005[pdat]

Copyright © 2007 by the American Academy of Pediatrics page 53

Page 54: AAP MCQ 2007

2007 PREP SA on CD-ROM

Question: 27

You are seeing for the first time a 15-month-old boy who was born at 28 weeks' gestation. Hehad an afebrile seizure at 12 months, but takes no medications. He uses both a cup and a bottleand takes most solid foods without choking. He wakes frequently at night. He can sit alone, butdoes not crawl or walk. He uses three words other than "mama" or "dada." His growthparameters are at the 50th percentile for length and head circumference and at the 10thpercentile for weight. On physical examination, you note increased tone in his lower extremitiesand trunk.

Of the following, the MOST likely other information that you would expect is a history of

A. constipation

B. developmental regression

C. recurrent rashes

D. tachycardia

E. tendon releases

Copyright © 2007 by the American Academy of Pediatrics page 54

Page 55: AAP MCQ 2007

2007 PREP SA on CD-ROM

Preferred Response: ACritique: 27

Cerebral palsy is an umbrella term covering a group of nonprogressive, but often changing,motor impairment syndromes due to lesions or anomalies of the brain that arise in the earlystages of its development. Some 2 to 3 per 1,000 children have cerebral palsy in the UnitedStates. Although it may be difficult to determine the exact cause of the motor impairment for anindividual child, several clinical situations are associated with increased risk of cerebral palsy,including brain malformation, chromosomal abnormalities, intrauterine growth restriction,prematurity, birth hypoxia, and postnatal events such as traumatic injury and meningitis.

The motor manifestations of cerebral palsy are treated best by an interdisciplinary team ofclinicians who can address medical and surgical management of tone and joint contractures aswell as therapists who can address therapeutic issues such as range of motion and equipmentneeds.

As described for the boy in the vignette, children who have cerebral palsy frequently haveother coexisting conditions, such as cognitive disability or mental retardation, skeletalmalformations, visual impairment, communication disorders, seizures, behavioral concerns,abnormal oral-motor function, and poor growth and nutrition. Screening for and treatment ofassociated conditions can support his motor, social-adaptive, and cognitive development. Hemost likely will have difficulty with feeding and constipation, and it is helpful to provide anticipatoryguidance for the family to ensure that he receives adequate nutrition.

Cerebral palsy is a nonprogressive condition; it is not characterized by developmentalregression. Cerebral palsy also is not associated with a higher risk of skin or cardiac conditions.The boy may require tendon releases in the future, but no contractures are noted on thisphysical examination, and he is very young to develop this complication of cerebral palsy.

References:

Cooley WC; American Academy of Pediatrics Committee on Children With Disabilities. Providinga primary care medical home for children and youth with cerebral palsy. Pediatrics.2004;114:1106-1113. Available at:http://pediatrics.aappublications.org/cgi/content/full/114/4/1106

Green L, Greenberg GM, Hurwitz E. Primary care of children with cerebral palsy. Clin FamilyPract. 2003;5:467-491

Johnston MV. Cerebral palsy. In: Behrman RE, Kliegman RM, Jenson HB, eds. NelsonTextbook of Pediatrics. 17th ed. Philadelphia, Pa: WB Saunders Co; 2004:2024-2025

Copyright © 2007 by the American Academy of Pediatrics page 55

Page 56: AAP MCQ 2007

2007 PREP SA on CD-ROM

Question: 28

You are working in a refugee camp when a mother brings in her 8-day-old boy. The motherstates he started becoming irritable 2 days ago, and now any loud noise appears to cause himpain, as evidenced by muscle tightening and back arching causing his head to nearly touch hisfeet. Physical examination reveals only a dried packing on his umbilical cord, as is the localcustom. He appears normal until he is stimulated by touch or a loud noise, and then he begins tocry, stiffens, and arches his back. The stiffness continues until he calms down.

Of the following, the MOST likely diagnosis is

A. bacterial meningitis

B. botulism

C. generalized seizure

D. tetanus

E. viral encephalitis

Copyright © 2007 by the American Academy of Pediatrics page 56

Page 57: AAP MCQ 2007

2007 PREP SA on CD-ROM

Preferred Response: DCritique: 28

Tetanus is caused by a neurotoxin from the anaerobic bacterium Clostridium tetani and ischaracterized by muscle spasms (Item C28A) that often are worsened by noise or otherexternal stimuli, as described for the infant in the vignette. The muscle spasms can progressfrom mild to severe over a 1-week period. Severe spasms subside over a period of weeks if thepatient survives. The clinical manifestations of tetanus include localized tetanus (local musclespasm in areas contiguous with the infected wound), cephalic tetanus (dysfunction of the cranialnerves in association with a wound on the head or neck), and generalized tetanus (full bodyinvolvement). Neonatal tetanus remains prevalent in areas of the world where mothers have notbeen vaccinated and local umbilical cord care is poor. The diagnosis is made clinically, andtreatment includes the use of human tetanus immune globulin (TIG), oral antimicrobial agents(metronidazole or penicillin) for 10 to 14 days, and other supportive measures (eg, ventilatorysupport, decreased external stimuli such as loud noises). In addition, all infected wounds shouldbe cleaned properly and debrided.

Tetanus is prevented best through immunization with tetanus toxoid. Tetanus toxoid isavailable: 1) in combination with diphtheria toxoid and acellular pertussis vaccine (DTaP) toprovide basic immunity against tetanus, diphtheria, and pertussis; 2) as part of a double antigen(DT) for children up to 6 years of age who cannot receive the pertussis component of the DTaP;and 3) as a single antigen (tetanus toxoid) (TT) to immunize pregnant women and women ofchildbearing age to prevent tetanus in their newborns. DT must be replaced with Td or dTaP,which has a reduced diphtheria toxoid content, for patients older than 7 years of age to lessenthe adverse effects associated with the use of the higher-dose diphtheria toxin. For mostindications, TT is administered in combination with a diphtheria toxoid-containing preparation.

Patients who have bacterial meningitis and viral encephalitis usually demonstrate illnesscontinuously, not just when stimulated by loud noises. A generalized seizure typically includesloss of consciousness and a postictal state. Infection with Clostridium botulinum (botulism) is aneuroparalytic disorder that is not manifested by muscle spasms.

References:

American Academy of Pediatrics. Tetanus. In: Pickering LK, ed. Red Book: 2006 Report of theCommittee on Infectious Diseases. 27th ed. Elk Grove Village, Ill: American Academy ofPediatrics; 2006:648-654

Arnon SS. Tetanus (Clostridium tetani). In: Behrman RE, Kliegman RM, Jenson HB, eds. NelsonTextbook of Pediatrics. 17th ed. Philadelphia, Pa: WB Saunders Co; 2004:951-953

Copyright © 2007 by the American Academy of Pediatrics page 57

Page 58: AAP MCQ 2007

2007 PREP SA on CD-ROM

Question: 29

A 9-year-old girl develops daily daytime enuresis for 3 weeks. You notice her sitting in a chair inyour office waiting room with her legs crossed and squirming. About 20 minutes later, uponentering the examination room, she runs to the bathroom to urinate. You are able to obtain aurine sample, and results of analysis are normal. Her vital signs and findings on history andphysical examination are all unremarkable.

Of the following, the MOST appropriate next step in her management is to

A. consult a urologist

B. design a voiding routine

C. limit nighttime fluids

D. obtain renal ultrasonography

E. prescribe oxybutynin

Copyright © 2007 by the American Academy of Pediatrics page 58

Page 59: AAP MCQ 2007

2007 PREP SA on CD-ROM

Preferred Response: BCritique: 29

Daytime enuresis is a common disorder for children. It is usually a transient phenomenon, andthe cause is often elusive. The symptoms of daytime enuresis generally resolve regardless ofthe type of intervention.

The girl described in the vignette, who has developed daytime enuresis, sits with her legscrossed and squirming in the waiting room and runs to the bathroom to urinate. These areclassic symptoms of daytime enuresis and withholding urine in resistant children. She also hasnormal findings on urinalysis. Establishing a fixed voiding routine (eg, voiding at assigned timesthroughout the day) likely will resolve her symptoms.

It is very unlikely that the girl has a congenital urinary tract malformation because she didnot develop enuresis until the age of 9 years. Thus, consultation with a urologist or renalultrasonography is unnecessary. Limiting fluids at night would be unhelpful because she hasdaytime, not nighttime, enuresis and because limiting fluids at night is not efficacious forbedwetting. Finally, she has no symptoms of urgency incontinence (frequency, dribbling,recurrent urinary tract infections) and, therefore, therapy such as oxybutynin, which reducesthe symptoms of voiding dysfunction in many patients, is not warranted. Children who presentwith enuresis and an underlying behavioral condition (eg, environmental stress) may benefitfrom counseling or hypnosis, although the efficacy varies according to the expertise of theclinician. Bladder training exercises can help some children who do not empty completely witheach voiding.

References:

Hellerstein S, Linebarger JS. Voiding dysfunction in pediatric patients. Clin Pediatr (Phila).2003;42:43-49. Abstract available at:http://www.ncbi.nlm.nih.gov/entrez/query.fcgi?cmd=Retrieve&db=pubmed&dopt=Abstract&list_uids=12635981&query_hl=15&itool=pubmed_docsum

Kodman-Jones C, Hawkins L, Schulman SL. Behavioral characteristics of children with daytimewetting. J Urol. 2001;166:2392-2395. Abstract available at:http://www.ncbi.nlm.nih.gov/entrez/query.fcgi?db=pubmed&cmd=Retrieve&dopt=AbstractPlus&list_uids=11696795

Robson WLM, Leung AKC, Van Howe R. Primary and secondary nocturnal enuresis: similaritiesin presentation. Pediatrics. 2005;115:956-959. Available at:http://pediatrics.aappublications.org/cgi/content/full/115/4/956

Copyright © 2007 by the American Academy of Pediatrics page 59

Page 60: AAP MCQ 2007

2007 PREP SA on CD-ROM

Question: 30

A 7-month-old male infant adopted from a Somalian orphanage has been in the United States for3 weeks. Ten days after arrival, the infant is hospitalized with a fever to 104ºF (40°C), increasedirritability, and decreased oral intake and activity. Results of a blood culture are positive forHaemophilus influenzae type b. Since arriving in the United States, he has been attending thehome child care that his adoptive mother operates, which includes six other children between 12and 18 months of age, each of whom has been fully immunized.

Of the following, the MOST appropriate recommendation for chemoprophylaxis for child careattendees and staff is

A. no prophylaxis for attendees and supervisory personnel

B. prophylaxis for all attendees

C. prophylaxis for all attendees and supervisory personnel

D. prophylaxis only for attendees between 12 and 15 months of age

E. prophylaxis only for supervisory personnel

Copyright © 2007 by the American Academy of Pediatrics page 60

Page 61: AAP MCQ 2007

2007 PREP SA on CD-ROM

Preferred Response: ACritique: 30

Haemophilus influenzae type b (Hib) is an encapsulated pleomorphic gram-negativecoccobacillus that is transmitted from person to person by inhalation of respiratory droplets orby direct contact with respiratory secretions. Careful observation of exposed unimmunized orincompletely immunized household, child care, or nursery contacts is important. The risk ofinvasive Hib disease is increased among unimmunized household contacts younger than 4years of age. Nursery and child care center contacts also may be at increased risk ofsecondary disease, although the magnitude of this risk is unclear. The risk of secondarydisease in children attending child care centers seems to be lower than that seen for age-susceptible household contacts and is rare when all contacts are older than 2 years of age.

When two or more cases of invasive Hib disease have occurred within 60 days in a childcare center or nursery school where unimmunized or incompletely immunized children attend,rifampin prophylaxis for all attendees and supervisory personnel, regardless of age, should beconsidered. When a single case has occurred, as in the vignette, the use of rifampin prophylaxisin exposed child care groups that include unimmunized or incompletely immunized children iscontroversial; many experts recommend no prophylaxis for attendees or supervisory personnel.In addition to the recommendations for chemoprophylaxis, unimmunized or incompletelyimmunized children should receive a dose of conjugate Hib vaccine and should complete therecommended age-specific immunization schedule.

References:

American Academy of Pediatrics. Haemophilus influenzae infections. In: Pickering LK, ed. RedBook: 2006 Report of the Committee on Infectious Diseases. 27th ed. Elk Grove Village, Ill:American Academy of Pediatrics; 2006:310-318

Centers for Disease Control and Prevention. Progress toward elimination of Haemophilusinfluenzae type b disease among infants and children—United States, 1998-2000. MMWR MorbMortal Wkly Rep. 2002;51:234-237. Available at:http://www.cdc.gov/mmwr/preview/mmwrhtml/mm5111a4.htm

Copyright © 2007 by the American Academy of Pediatrics page 61

Page 62: AAP MCQ 2007

2007 PREP SA on CD-ROM

Question: 31

You are performing a presedation physical examination in a 3-year-old female who is scheduledto have a repeat computed tomography (CT) scan with contrast. The mother mentions that herdaughter experienced diffuse hives and facial swelling 10 minutes after the contrastadministration during her first CT.

Of the following, the BEST way to prevent future contrast reactions is to:

A. administer 1 L intravenous normal saline prior to the procedure

B. perform desensitization to contrast

C. provide pretreatment with oral antihistamines and steroids

D. use a high-osmolar contrast agent

E. use a contrast agent with low iodine content

Copyright © 2007 by the American Academy of Pediatrics page 62

Page 63: AAP MCQ 2007

2007 PREP SA on CD-ROM

Preferred Response: CCritique: 31

Reactions to radiocontrast media (RCM) are estimated to occur in 1% to 12% of all individualsreceiving standard RCM. RCM reactions are termed anaphylactoid, pseudoallergic, ornonimmunoglobulin (Ig)E-mediated. The mechanism for these reactions differs from that foranaphylaxis, which is an IgE-mediated reaction, but the clinical symptoms are indistinguishablebecause both result in mast cell degranulation and histamine release.

Initially, RCM reactions were believed to be due to the iodine content, but studies havedemonstrated that iodine is not the cause. It now generally is agreed that the osmolarity of thesolution is the primary cause. Compared with low-osmolar RCM, high-osmolar RCM isassociated with higher rates of adverse reactions during both initial and subsequent exposures.Thus, switching to a high-osmolar agent does not lessen the likelihood for a reaction. Switchingto a low-osmolar agent (eg, iohexol, ioversol, iopamidol) and pretreating with corticosteroids anda combination of H1- and H2-antihistamines lowers the risk of reaction to less than 1% inpatients who have experienced a previous reaction to a high-osmolar RCM.

Desensitization is a procedure used to allow safe administration of medications that causeIgE-mediated reactions. It does not have a role in the treatment of RCM anaphylactoid reactionsand generally is not recommended for other non-IgE-mediated reactions, such as idiosyncraticreactions, serum sickness, or erythema multiforme.

Appropriate hydration is important prior to any procedure using contrast to limit the risk ofnephrotoxicity, but there is no specific recommendation regarding administration of intravenousfluid to decrease the risk for an anaphylactoid reaction. Further, a 1-L intravenous fluid bolus in a3-year-old child who is hemodynamically stable is excessive.

References:

Boguniewicz M. Adverse reactions to drugs. In: Behrman RE, Kliegman RM, Jenson HB, eds.Nelson Textbook of Pediatrics. 17th ed. Philadelphia, Pa: WB Saunders Co 2004:783-786

Hagan JB. Anaphylactoid and adverse reactions to radiocontrast agents. Immunol Allergy ClinNorth Am. 2004;24:507-519. Abstract available at:http://www.ncbi.nlm.nih.gov/entrez/query.fcgi?orig_db=PubMed&db=PubMed&cmd=Search&term=Immunol+Allergy+Clin+North+Am[Jour]+AND+507[page]+AND+2004[pdat]

Copyright © 2007 by the American Academy of Pediatrics page 63

Page 64: AAP MCQ 2007

2007 PREP SA on CD-ROM

Question: 32

A 16-year-old girl is brought to the emergency department by ambulance after her mother founda suicide note on her bed. The girl claims that she took "20 or 30" ibuprofen tablets 4 hours agoand nothing else. Although she is tearful, her physical examination findings are normal.

Of the following, the MOST appropriate next step in the management of this patient is to

A. administer sodium bicarbonate

B. administer syrup of ipecac

C. obtain a blood level of acetaminophen

D. obtain a blood level of ibuprofen

E. perform gastric lavage

Copyright © 2007 by the American Academy of Pediatrics page 64

Page 65: AAP MCQ 2007

2007 PREP SA on CD-ROM

Preferred Response: CCritique: 32

The patient described in the vignette has attempted suicide by medication overdose. Althoughshe has stated that she took only ibuprofen, coingestants should be assumed until provenotherwise. Dangerous coingestants that can be detected with laboratory tests includeacetaminophen or aspirin and illicit drugs of abuse such as narcotics, cocaine, or ethanol. Itwould be prudent to measure serum acetaminophen and aspirin concentrations and obtain aurine toxicology screen. No currently available assays measure serum ibuprofen levels in theclinical setting. Because other toxic substances may have been ingested, administration ofactivated charcoal is appropriate for the girl described in the vignette.

Ibuprofen overdose is a common presenting complaint in the emergency department, butsignificant toxicity is exceedingly rare. In most cases, supportive care leads to an uneventfulrecovery. In a recent report, two thirds of patients were asymptomatic, and 20% experiencedmild or moderate symptoms. There have been fewer than 10 case reports of fatal overdose withibuprofen, and in each case, there were complicating factors such as coingestion of anothertoxic substance or other disease processes. Other rare reported complications associated withibuprofen overdose include duodenal perforation, renal toxicity, metabolic acidosis, somnolence,coma, and subfulminant hepatitis requiring liver transplantation.

The American Academy of Pediatrics does not recommend syrup of ipecac for themanagement of medication overdose because it is inefficient compared with gastric lavage andbecause the prolonged vomiting it can induce may interfere with therapy. Gastric lavage is onlymarginally effective and is suggested only in the setting of ingestion of a highly toxic substancein the previous 1 to 2 hours. Gastric lavage more than 2 hours after ingestion is unlikely toprovide benefit. The administration of sodium bicarbonate for metabolic acidosis rarely isindicated, except for cases of severe acidosis in which the pH is lower than 7.0. There is no rolefor bicarbonate administration in enhancing elimination of ibuprofen.

References:

Clarke SF, Arepalli N, Armstrong C, Dargan PI. Duodenal perforation after ibuprofen overdose. JToxicol Clin Toxicol. 2004;427:983-985

Easley RB, Altemeier WA 3rd. Central nervous system manifestations of an ibuprofen overdosereversed by naloxone. Pediatr Emerg Care. 2000;16:39-41 Available at:http://www.ncbi.nlm.nih.gov/entrez/query.fcgi?orig_db=PubMed&db=PubMed&cmd=Search&term=%22Pediatric+emergency+care%22[Jour]+AND+39[page]+AND+2000[pdat]

Laurent S, Rahier J, Geubel AP, Lerut J, Horsmans Y. Subfulminant hepatitis requiring livertransplantation following ibuprofen overdose. Liver. 2000;20:93-94

Volans G, Monaghan J, Colbridge M. Ibuprofen overdose. Int J Clin Pract Suppl. 2003;135:54-60. Available at:http://www.ncbi.nlm.nih.gov/entrez/query.fcgi?orig_db=PubMed&db=PubMed&cmd=Search&term=%22International+journal+of+clinical+practice.+Supplement%22[Jour]+AND+54[page]+AND+2003[pdat]

Copyright © 2007 by the American Academy of Pediatrics page 65

Page 66: AAP MCQ 2007

2007 PREP SA on CD-ROM

Question: 33

A nurse asks you to examine a 26-hour-old infant who recently developed a rash. On physicalexamination, you note erythematous macules (Item Q33A) over the trunk, face, and proximalextremities. Most of the macules have tiny central pustules. The infant is breastfeeding well, andthe remainder of the physical examination findings are normal. No lesions were present at birth.

Of the following, analysis of the pustular contents is MOST likely to reveal

A. eosinophils

B. gram-positive cocci

C. multinucleated giant cells

D. polymorphonuclear leukocytes

E. pseudohyphae and budding yeast

Copyright © 2007 by the American Academy of Pediatrics page 66

Page 67: AAP MCQ 2007

2007 PREP SA on CD-ROM

Preferred Response: ACritique: 33

Erythema toxicum neonatorum presents as blotchy, 2- to 3-cm erythematous macules (ItemC33A), each of which has a single 1- to 4-mm central papule, vesicle, or pustule. Lesions arenot present at birth but appear in the first 24 to 48 hours thereafter. The condition occurs inabout 50% of healthy term infants; preterm infants are affected less often. Individual lesionsresolve in 4 to 5 days, with new lesions appearing for up to 10 days. A Wright-stainedpreparation of the pustule contents reveals numerous eosinophils (Item C33B). If obtained, acomplete blood count may demonstrate eosinophilia. The condition is benign and self-limited, andno treatment is necessary.

A number of neonatal skin conditions are characterized by vesicles and pustules. They maybe distinguished from erythema toxicum clinically and by microscopic examination of lesionalcontents. The pustules caused by staphylococcal folliculitis (Item C33C) contain gram-positivecocci, the vesicles of herpes simplex virus infection (Item C33D) contain multinucleated giantcells (Item C33E), the pustules of transient neonatal pustular melanosis (Item C33F) containpolymorphonuclear leukocytes, and the pustules of congenital candidiasis (Item C33G) containpseudohyphae and budding yeast (Item C33H).

References:

Eichenfield L, Larralde M. Neonatal skin and skin disorders: congenital erosive and vesiculardermatosis, vesiculopustular and bullous diseases of the newborn. In: Schachner LA, HansenRC, eds. Pediatric Dermatology. 3rd ed. St. Louis, Mo: Mosby; 2003:232-239

Marchini G, Nelson A, Edner J, Lonne-Rahm S, Stavréus-Evers A, Hultenby K. Erythematoxicum neonatorum is an innate immune response to commensal microbes penetrated into theskin of the newborn infant. Pediatr Res. 2005;58:613-616. Available at:http://www.pedresearch.org/cgi/content/full/58/3/613

Weston WL, Lane AT, Morelli JG. Skin diseases in newborns: transient skin disease. In: ColorTextbook of Pediatric Dermatology. 3rd ed. St. Louis, Mo: Mosby; 2002:302-303

Copyright © 2007 by the American Academy of Pediatrics page 67

Page 68: AAP MCQ 2007

2007 PREP SA on CD-ROM

Question: 34

A 16-year-old Caucasian girl from the northeastern United States presents for a healthsupervision visit. She drinks three to four diet sodas a day and describes herself as "a couchpotato." She has a history of asthma and had been hospitalized for administration of intravenousmethylprednisolone twice in the past year. Physical examination findings are normal. She is atSexual Maturity Rating 4 and has had normal menses for 3 years. A year ago, she fell whilewalking and fractured her ulna. You are considering therapy for suspected osteopenia.

Of the following, the MOST appropriate therapy for this patient at this time is

A. intranasal calcitonin

B. oral alendronate

C. oral calcium and vitamin D supplement

D. oral estrogen

E. oral phosphorus supplement

Copyright © 2007 by the American Academy of Pediatrics page 68

Page 69: AAP MCQ 2007

2007 PREP SA on CD-ROM

Preferred Response: CCritique: 34

The adolescent described in the vignette has risk factors for osteopenia (low bone density).These include living in the northeastern United States, not being active, drinking diet soda (whichcontains phosphoric acid), having received intravenous steroids, and already having had onefracture. The initial evaluation of a teen in whom osteopenia is suspected should includedetermination of calcium, phosphorus, 25-hydroxyvitamin D, and parathyroid hormoneconcentrations as well as bone mineral density. Age-adjusted normal values for bone mineraldensity are now available. If osteopenia or hypovitaminosis D is demonstrated, the initial therapyinvolves a calcium supplement (600 to 1,200 mg/d) and vitamin D replacement (usually 800 unitsof vitamin D). Other therapies, such as alendronate, calcitonin, estrogen, and phosphorus,usually are not necessary in otherwise healthy adolescent girls.

In a patient who has suspected osteopenia, bone density of the lumbar spine and other sitescan be assessed using dual-energy x-ray absorptiometry (DEXA). The patient's bone density iscompared with age- and sex-adjusted normal values. A Z score of less than 1.0 indicates thepatient is below the 20th percentile for bone density, which most investigators define as"osteopenia." Although osteopenia correlates with hip fractures in the elderly, the clinicalsignificance of osteopenia in an adolescent is still unclear and a topic of ongoing research.Therefore, routine bone density measurements are not recommended as a screening test inchildren. However, this test should be considered in children who receive long-term steroids,have a history of recurrent fractures, or have a chronic illness (eg, rheumatoid arthritis, Crohndisease). In addition to bone density, measures of serum calcium, phosphorus, 25-hydroxyvitamin D, and parathyroid hormone are performed in the assessment of the child whohas osteopenia.

Many factors contribute to bone health in children, including intake of calcium andphosphorus, renal excretion of calcium, physical activity, and vitamin D status. According to onerecent study from the northeastern United States, the prevalence of vitamin D deficiency inotherwise healthy adolescents is approximately 25%, and vitamin D levels are lower in the winter(when there is less exposure to sunlight). In this study, vitamin D deficiency also wasassociated with African-American race, decreased milk consumption, and increased soft drinkconsumption. Adolescents all should be educated regarding the need to take in therecommended dietary allowance of calcium and vitamin D.

References:

Gordon CM, DePeter KC, Feldman HA, Grace E, Emans SJ. Prevalence of vitamin D deficiencyamong healthy adolescents. Arch Pediatr Adolesc Med. 2004;158:531-537. Abstract available at:http://www.ncbi.nlm.nih.gov/entrez/query.fcgi?cmd=Retrieve&db=pubmed&dopt=Abstract&list_uids=15184215&query_hl=13&itool=pubmed_docsum

Semeao E, Jawad AF, Stouffer NO, Zemel BS Piccoli DA, Stallings VA. Risk factors for low bonemineral density in children and young adults with Crohn's disease. J Pediatr. 1999;135:593-600.Abstract available at:http://www.ncbi.nlm.nih.gov/entrez/query.fcgi?cmd=Retrieve&db=pubmed&dopt=Abstract&list_uids=10547248&query_hl=15&itool=pubmed_docsum

Copyright © 2007 by the American Academy of Pediatrics page 69

Page 70: AAP MCQ 2007

2007 PREP SA on CD-ROM

Question: 35

A term male infant is born to a woman who has known multiple drug abuse problems. Her urinedrug screen was positive for barbiturates, benzodiazepines, and opioids upon admission to thelabor and delivery unit. The infant is delivered vaginally without complications. Apgar scores are8 and 9 at 1 and 5 minutes, respectively. His birthweight is 3,500 g. You are asked to evaluatethe infant for early discharge at 23 hours of age. Findings on physical examination are normal,with the exception of jitteriness, and the infant is not breastfeeding well.

Of the following, the BEST reason to keep this infant in the hospital is that

A. a negative urine drug screen for the infant is required

B. breastfeeding failure requires a lactation consultation

C. foster care placement must be arranged

D. jitteriness is most likely due to hypoglycemia that requires treatment

E. neonatal abstinence syndrome may not be fully evident for 5 days or more

Copyright © 2007 by the American Academy of Pediatrics page 70

Page 71: AAP MCQ 2007

2007 PREP SA on CD-ROM

Preferred Response: ECritique: 35

In the United States, 5% to 15% of all pregnancies may be complicated by perinatal drugexposure, and it is not uncommon for a pregnant woman to use more than one illicit substance.The most commonly used legal substances are tobacco and alcohol. Either, or both, maycomplicate the use and effects of other substances, such as opioids, cocaine, amphetamines,or marijuana.

The newborn may experience withdrawal from drug exposure following delivery. Suchwithdrawal effects must be distinguished from drug effects. The direct effects of some drugs ofabuse include sedation or somnolence, depression of respiratory drive, depressed neuromotortone, bradycardia, thermoregulatory problems, and pinpoint pupils (methadone, morphine,heroin, codeine, oxycodone, alcohol, barbiturates, benzodiazepines) or increased sensorystimulation with agitation, tachycardia, hypertension, and dilated pupils (amphetamines, cocaine).

Early discharge is not an option for any infant in whom perinatal drug exposure is a realconcern, such as the infant described in the vignette, because the onset of symptoms ofabstinence syndrome following delivery and postnatal cessation of drug exposure varies withthe substance to which the fetus was exposed. It is also important to consider the timing of lastknown exposure (maternal use).

No true withdrawal or abstinence syndrome for cocaine exists. Similarly, amphetamineeffects dissipate after a few postnatal days, and no real abstinence syndrome exists for thisclass of substances. However, neither drug is considered safe or free of prenatal damage to thedeveloping brain; both may have direct effects on neurotransmitters and affect brain circulation.Effects of cocaine and amphetamine on the maternal and uteroplacental circulation also may berelated to the reduced fetal body and head growth described in some exposed fetuses.

Alcohol and barbiturate withdrawal syndromes share many common physical findings,including hyperactivity, irritability, crying, hyperphagia but poorly coordinated sucking and oralfeeding, altered sleep-wake periods, tremors, diaphoresis, and seizures. Alcohol withdrawal alsomay present with hypoglycemia and excessive glucocorticoid release. In addition to a controlled,thermoneutral environment, with minimal stimulation, phenobarbital provides the bestpharmacologic management. Such treatment should be used in conjunction with an objectivescoring system for physical findings to titrate the dose to effect. A schedule for taperingphenobarbital over 4 to 6 weeks is recommended. An alternative, though less well substantiated,management approach is the use of a dilute tincture of opium and diazepam.

Opioid withdrawal is best recognized by an abstinence syndrome that is characterized bynumerous behavioral and physiologic stigmata, including hyperirritability, tremors, jitteriness,hypertonia, gastrointestinal distress (loose stools, emesis, and feeding problems), and vagueautonomic symptoms such as yawning, lip-smacking, persistent sucking, mottling, and fever.Seizures may occur. Methadone or oral morphine must be used to manage the physicalfindings, which may not become manifest for 5 days (longer if the mother was treated withmethadone). Again, the use of a neonatal abstinence scoring system is essential in managingaffected newborns.

A urine drug screen for the infant described in the vignette may be obtained within 24 hoursand would be most concerning if positive for any of the substances identified on his mother’surine drug screen. However, the more time that passes after birth and before collecting thenewborn’s urine, the less likely that the results will be positive because most drugs and theirmetabolites are cleared by 96 hours after birth. At fewer than 24 hours of postnatal age, theinfant may be demonstrating simple breastfeeding problems or the problems may reflect oral-motor coordination issues related to drug exposure. Jitteriness suggests neurologic irritabilityassociated with maternal drug use; hypoglycemia is a less likely cause in an appropriatelygrown term infant. Although social issues such as placement of an infant in a safe environmentmust be addressed, foster care placement may take a long time and is not the best reason fordelaying discharge.

References:

American Academy of Pediatrics Committee on Drugs. Neonatal drug withdrawal. Pediatrics.

Copyright © 2007 by the American Academy of Pediatrics page 71

Page 72: AAP MCQ 2007

2007 PREP SA on CD-ROM

1998;101:1079-1088. Available at:http://pediatrics.aappublications.org/cgi/content/full/101/6/1079

Godding V, Bonnier C, Fiasse L, et al. Does in utero exposure to heavy maternal smokinginduce nicotine withdrawal symptoms in neonates? Pediatr Res. 2004;55:645-651. Available at:http://www.pedresearch.org/cgi/content/full/55/4/645

Johnson K, Gerada C, Greenough A. Treatment of neonatal abstinence syndrome. Arch DisChild Fetal Neonatal Ed. 2003;8:F2-F5. Available at:http://fn.bmjjournals.com/cgi/content/full/88/1/F2

Law KL, Stroud LR, LaGasse LL, Niaura R, Liu J, Lester BM. Smoking during pregnancy andnewborn neurobehavior. Pediatrics. 2003;111:1318-1323 Available at:http://pediatrics.aappublications.org/cgi/content/full/111/6/1318

Philipp BL, Merewood A, O'Brien S. Methadone and breastfeeding: new horizons [commentary].Pediatrics. 2003;111:1429-1430. Available at:http://pediatrics.aappublications.org/cgi/content/full/111/6/1429

Wang M. Perinatal drug abuse and neonatal drug withdrawal. eMedicine Specialties: Pediatrics:Neonatology. Available at: http://www.emedicine.com/ped/topic2631.htm

Copyright © 2007 by the American Academy of Pediatrics page 72

Page 73: AAP MCQ 2007

2007 PREP SA on CD-ROM

Question: 36

A 15-month-old boy presents to the emergency department following the acute onset of nausea,vomiting, and abdominal pain. He appears agitated. His heart rate is 160 beats/min, but othervital signs are normal. His mother, who has asthma, reports finding her theophylline bottle lyingempty on the floor in her bedroom. His capillary blood gas measurement is normal, and his bloodglucose concentration is 190 mg/dL (10.5 mmol/L).

Of the following, the MOST appropriate management of this patient is

A. administration of activated charcoal

B. administration of beta-adrenergic receptor antagonists

C. administration of ipecac

D. gastric lavage

E. whole-bowel irrigation

Copyright © 2007 by the American Academy of Pediatrics page 73

Page 74: AAP MCQ 2007

2007 PREP SA on CD-ROM

Preferred Response: ACritique: 36

Although theophylline is prescribed infrequently in children, pediatricians should be aware of theclinical manifestations and management of theophylline toxicity because some adults still use thedrug for the treatment of asthma and chronic obstructive pulmonary diseases. Patients whohave acute theophylline toxicity often complain of nausea, vomiting, and abdominal pain and mayexhibit symptoms of restlessness, tremors, and seizures. Tachycardia and tachypnea arecommon. Laboratory evaluation may reveal hypokalemia, hyperglycemia, and high anion gapmetabolic acidosis. Electrocardiographic findings include sinus and supraventriculartachycardias and ventricular dysrhythmias. Chronic theophylline toxicity may occur in olderpatients who are taking medications that are metabolized by the cytochrome P450 system andhave hepatic impairment or renal impairment. Symptoms in these patients may be more subtleand milder.

As with other poisonings, the acute management of theophylline toxicity begins with theABCs (airway, breathing, and circulation), followed by intravenous catheter placement and fluidadministration. The mainstay of treatment for both acute and chronic toxicity is the administrationof multiple doses of activated charcoal, which should be continued until symptoms resolve andtheophylline levels are within therapeutic range. Electrolytes should be monitored closely, andantiemetic therapy may be used to aid with charcoal administration if vomiting is persistent. Ifseizures occur, benzodiazepines and barbiturates are effective, and cardiac dysrhythmiasshould be treated aggressively as appropriate for the type of dysrhythmia.

Beta-adrenergic receptor antagonists have been shown to be effective in treating somedysrhythmias associated with theophylline toxicity, but hypotension may be exacerbated bythese agents, and their use should be reserved for refractory cases. Administration of ipecac nolonger is recommended for any toxic ingestion. Gastric lavage may be advantageous if givenwithin 1 hour of large ingestions, but it may delay the administration of charcoal, which is thedefinitive treatment. Whole-bowel irrigation should be reserved for cases in which thetheophylline values continue to rise despite adequate charcoal administration.

References:

American Academy of Pediatrics Committee on Injury, Violence, and Poison Prevention. Policystatement: poison treatment in the home. Pediatrics. 2003;112:1182-1185 Available at:http://pediatrics.aappublications.org/cgi/content/full/112/5/1182

Stork CM, Howland MA. Theophylline. In: Ford MD, Delaney KA, Ling LJ, Erickson T, eds.Clinical Toxicology. Philadelphia, Pa: WB Saunders Co; 2001:412-416

Woolf AD. Poisoning by unknown agents. Pediatr Rev. 1999;20:166-170 Available at:http://pedsinreview.aappublications.org/cgi/content/full/20/5/166

Copyright © 2007 by the American Academy of Pediatrics page 74

Page 75: AAP MCQ 2007

2007 PREP SA on CD-ROM

Question: 37

A 2-year-old girl, who has been healthy, is scheduled for a health supervision visit today. Whenyou last saw her at her 15-month-old visit, there were no problems. You have learned that thefamily has just moved from the suburbs, purchased a home in an area of town where housesdate back to the early 20th century, and are in the midst of a major remodeling project. Youdecide to discuss with the parents the risks associated with remodeling an older home.

Of the following, it is MOST important to discuss with these parents the risks associated withtheir child's exposure to:

A. arsenic

B. asbestos

C. lead

D. mercury

E. radon

Copyright © 2007 by the American Academy of Pediatrics page 75

Page 76: AAP MCQ 2007

2007 PREP SA on CD-ROM

Preferred Response: CCritique: 37

Children are exposed to numerous potentially toxic environmental substances at home, atschool, and at play. Buildings slated for remodeling or repair in which children live, learn, or playshould be inspected for possible toxic exposures prior to such construction to minimizeexposures.

For the child described in the vignette, the single most important potential exposure isenvironmental lead. Lead poisoning adversely affects central nervous system development andcognition in the growing child. Although potential exposure to lead may be difficult to document,serum blood concentrations are inexpensive and easy to obtain, and they correlate well with therisk for developmental disability. Furthermore, lead poisoning is potentially treatable.Assessment of the risk of children and families may be determined, in part, by questionnaire.Widespread blood lead screening of children living in high-risk areas is mandated by theMedicaid program at ages 1 and 2 years. A blood lead level of greater than 10 mcg/dL (<0.48mcmol/L) deserves further evaluation of the child’s environment. Lead abatement in theenvironment has decreased widespread exposure dramatically, but children who are exposed toremodeling of older homes, impoverished children, and African-American children continue to beat increased risk.

Exposure to asbestos in an older school building or home is possible, but asbestos is not ahigh-risk exposure unless the building materials containing the substance are deteriorating, arewithin reach of children who might touch or play with them, or are being removed as part ofbuilding renovation. Asbestosis is a pulmonary disease that has a high rate of associatedmalignancy if inhalation exposure is prolonged. There is no good screening tool to measureasbestos exposure or risk. Both children and adults should be removed from schools or homesundergoing abatement of asbestos-containing tiles, insulation, and other materials. Because therisk for lung cancer associated with asbestos exposure is exponentially higher in persons whosmoke or are exposed to cigarette smoke and because smoking is a far greater risk factor forcancer, discussion of smoke avoidance and smoking cessation should accompany discussionsabout environmental risk factors.

Radon gas results from radioactive decay of radium, which is found in rocks and soil andenters homes through cracks and pores in concrete and granite floors and walls, especiallybasements. There is no clinical test for radon exposure in humans, but test kits are available toassess the radon content of homes. Long-term exposure has been associated with increasedcancer risk, as with exposure to other sources of radiation.

Although ingestion of fish containing high levels of mercury has been associated withchronic neurologic impairments, it is not a significant problem associated with home renovations.Recent withdrawal of elemental mercury-containing thermometers from the market and removalof mercury sphygmomanometers represent an effort to decrease the presence of environmentalmercury.

Arsenic has been removed as a component of most modern pesticides, but it may persist inground water, especially around areas previously used for smelting and mining. It is also apotential hazard in children’s playground equipment as a component of pressure-treated lumber(copper chromium arsenate). Arsenic may contaminate the skin by simply rubbing the handover a treated surface. Burning of such pressure-treated lumber has been reported to causeacute arsenic toxicity, with gastrointestinal and pulmonary disturbances. Chronic exposure maycause fatigue, malaise, and paresthesias.

References:

American Academy of Pediatrics Committee on Environmental Health. Arsenic. In: Etzel RA, ed.Pediatric Environmental Health. 2nd ed. Elk Grove Village, Ill: American Academy of Pediatrics;2003:87-98

American Academy of Pediatrics Committee on Environmental Health. Asbestos. In: Etzel RA,ed. Pediatric Environmental Health. 2nd ed. Elk Grove Village, Ill: American Academy ofPediatrics; 2003:99-112

Copyright © 2007 by the American Academy of Pediatrics page 76

Page 77: AAP MCQ 2007

2007 PREP SA on CD-ROM

American Academy of Pediatrics Committee on Environmental Health. Ionizing radiation(including radon). In: Etzel RA, ed. Pediatric Environmental Health. 2nd ed. Elk Grove Village, Ill:American Academy of Pediatrics; 2003:215-234

American Academy of Pediatrics Committee on Environmental Health. Lead. In: Etzel RA, ed.Pediatric Environmental Health. 2nd ed. Elk Grove Village, Ill: American Academy of Pediatrics;2003:249-266

American Academy of Pediatrics Committee on Environmental Health. Mercury. In: Etzel RA, ed.Pediatric Environmental Health. 2nd ed. Elk Grove Village, Ill: American Academy of Pediatrics;2003:267-282

American Academy of Pediatrics Committee on Environmental Health. Pesticides. In: Etzel RA,ed. Pediatric Environmental Health. 2nd ed. Elk Grove Village, Ill: American Academy ofPediatrics; 2003:323-360

American Academy of Pediatrics Committee on Environmental Health. Water pollutants. In: EtzelRA, ed. Pediatric Environmental Health. 2nd ed. Elk Grove Village, Ill: American Academy ofPediatrics; 2003:393-416

Markowitz M. Lead poisoning. Pediatr Rev. 2000;21:327-335. Available at:http://pedsinreview.aappublications.org/cgi/content/full/21/10/327

Copyright © 2007 by the American Academy of Pediatrics page 77

Page 78: AAP MCQ 2007

2007 PREP SA on CD-ROM

Question: 38

A 12-year-old girl presents with her third headache in the last 2 months. She describes the painas pounding, sharp, and severe. The pain is bifrontal and has been present for 1 hour. Pasthistory is notable for motion sickness at age 4 years. Physical examination results are normal,but the girl draws a picture of dots when asked if she sees anything prior to pain.

Of the following, the MOST appropriate treatment for this child is

A. intramuscular meperidine

B. intranasal butorphanol

C. oral ibuprofen

D. oral zolmitriptan

E. subcutaneous sumatriptan

Copyright © 2007 by the American Academy of Pediatrics page 78

Page 79: AAP MCQ 2007

2007 PREP SA on CD-ROM

Preferred Response: CCritique: 38

The 12-year-old girl described in the vignette has acute recurrent headaches that are consistentwith migraine with aura, ie, classic migraine. Her headaches are short and pounding and havelong pain-free intervals. The dots she sees during episodes are likely scintillating scotomas. Thegirl does not display the features of ominous secondary (ie, pathologic) headaches, namely,papilledema, strabismus, weakness, or ataxia.

Migraines tend to be familial and can be triggered by fatigue, glare, caffeine, fasting, lack ofsleep, menstruation, stress, or other factors. Mood changes can trigger migraine, althoughdepression tends to produce chronic daily headaches with less migrainous characteristics.

Management of migraines requires both pharmacologic and nonpharmacologic techniques.After the child and family are reassured about the benign nature of the headache, triggers shouldbe sought and removed. Daily exercise, routine meals, good hydration, and sufficient sleep areessential. Other nonpharmacologic measures should be considered, such as relaxationtechniques or biofeedback. These nonpharmacologic measures also are effective in headachesassociated with stress, particularly when chronic. Abortive medications also can be useful tostop pain, although placebo alone can be associated with cessation of pain in perhaps one thirdof children. A recent practice parameter, endorsed by the American Academy of Pediatrics,reviewed drug treatments for acute headache and found convincing data to support the use ofacetaminophen (15 mg/kg orally), ibuprofen (10 mg/kg orally), or nasal sumatriptan (20 mg ifbody mass is 30 kg or more; 5 mg if less than 30 kg). Daily prophylactic medications, such asamitriptyline, topiramate, cyproheptadine, or valproic acid, are indicated only for the child whohas several disabling migraines every month.

Evidence to support the use of subcutaneous sumatriptan or oral zolmitriptan in children islimited at this time. Intramuscular meperidine and intranasal butorphanol are both narcotics thathave addictive potential and are not indicated for treatment of childhood migraine.

References:

Fisher PG. Help for headaches: a strategy for your busy practice. Contemp Pediatr. 2005;22:34-41

Forsyth R, Farrell K. Headache in childhood. Pediatr Rev. 1999;20:39-45 Available at:http://pedsinreview.aappublications.org/cgi/content/full/20/2/39

Haslam RHA. Headaches. In: Behrman RE, Kliegman RM, Jenson HB, eds. Nelson Textbook ofPediatrics. 17th ed. Philadelphia, Pa: WB Saunders; 2004:2012-2015

Lewis D, Ashwal S, Hershey A, et al; American Academy of Neurology Quality StandardsSubcommittee; Practice Committee of the Child Neurology Society. Practice parameter:pharmacological treatment of migraine headache in children: report of the American Academy ofNeurology Quality Standards Committee and the Practice Committee of the Child NeurologySociety. Neurology. 2004;63:2215-2224. Available at:http://www.neurology.org/cgi/content/full/63/12/2215

Copyright © 2007 by the American Academy of Pediatrics page 79

Page 80: AAP MCQ 2007

2007 PREP SA on CD-ROM

Question: 39

You are seeing a 6-year-old boy for a health supervision visit. He was born with trisomy 21 andan atrioventricular septal defect for which he underwent complete surgical repair at 2 months ofage with an excellent result. His mother reports that he has not been sleeping well, that hesnores loudly, and that she believes that he fatigues easily with play. On physical examination,he appears in no distress, his respiratory rate is 24 breaths/min, his heart rate is 100 beats/min,and his blood pressure is 110/70 mm Hg. His precordial impulse is prominent, and there is a loudsecond heart sound and a 2/6 holosystolic murmur at the cardiac apex with radiation to the leftaxilla. There is mild hepatomegaly and jugular venous congestion.

Of the following, the MOST likely cause of this patient's findings is

A. dehiscence of the ventricular septal patch

B. dilated cardiomyopathy

C. residual atrial septal defect

D. right heart failure

E. supraventricular tachycardia

Copyright © 2007 by the American Academy of Pediatrics page 80

Page 81: AAP MCQ 2007

2007 PREP SA on CD-ROM

Preferred Response: DCritique: 39

Obstructive sleep apnea syndrome (OSAS) refers to a condition in which the upper airway isobstructed completely or partially during sleep and is associated with hypoxemia, hypercapnia,and disrupted normal ventilation and sleep patterns. OSAS has a prevalence of 1% to 2% inchildren. Several risk factors are associated with OSAS, including adenotonsillar hypertrophy,craniofacial abnormalities, nasal obstruction, neuromuscular disorders, and obesity. Thepathophysiology results from mechanical obstruction of the upper airway due to tonsillarhypertrophy or decreased muscular tone of the airway. The clinical manifestations are snoringduring sleep, mouth breathing, restless sleep patterns, mood swings, inattentiveness, learningdifficulties, and failure to thrive. The diagnosis is confirmed by polysomnography, whichmeasures the precise number of apneic and hypopneic episodes.

OSAS can have widespread complicating effects on the respiratory and metabolic systemsas well as a significant impact on the psychosocial well-being of the patient. Obese patients whohave OSAS have a higher incidence of systemic hypertension and stroke. Right heart failuremay develop in patients who have severe OSAS, likely due to hypoxemia and hypercapnia.These factors subsequently have an effect as pulmonary vasoconstrictors. With increasingpulmonary vasoconstriction, increased right ventricular and pulmonary artery pressure isgenerated to deliver blood into the lungs. Over time, the increased afterload on the right heartmay lead to hypertrophy, dilation, and failure of the right ventricle.

The trisomy 21 coupled with the sleep patterns and behaviors that are consistent with OSASdescribed for the patient in the vignette have led to right heart failure. Dehiscence of a ventricularseptal patch placed 6 years earlier is extremely unlikely, and if present, likely would lead topronounced signs and symptoms consistent with pulmonary overcirculation. Residual atrialseptal defect would not be expected to cause significant symptoms in a 6-year-old child andwould not explain the findings during sleep. Findings of a dilated cardiomyopathy would bereferable to the systemic circulation, and tachycardia probably would develop as acompensatory mechanism for the poor cardiac output. Supraventricular tachycardia is notpresent; the patient’s heart rate is normal for his age.

References:

Bandla P, Brooks LJ, Trimarchi T, Helfaer M. Obstructive sleep apnea syndrome in children.Anesthesiol Clin North Am. 2005;23:535-549. Abstract available at:http://www.ncbi.nlm.nih.gov/entrez/query.fcgi?orig_db=PubMed&db=PubMed&cmd=Search&term=%22Anesthesiology+clinics+of+North+America%22[Jour]+AND+2005[pdat]+AND+Bandla+P[author]

Iber C. Sleep-related breathing disorders. Neurol Clin. 2005;23:1045-1057 Abstract available at:http://www.ncbi.nlm.nih.gov/entrez/query.fcgi?cmd=Retrieve&db=pubmed&dopt=Abstract&list_uids=16243615&query_hl=18&itool=pubmed_docsum

Rosen CL. Obstructive sleep apnea syndrome in children: controversies in diagnosis andtreatment. Pediatr Clin North Am. 2004;51:153-167. Abstract available at:http://www.ncbi.nlm.nih.gov/entrez/query.fcgi?orig_db=PubMed&db=PubMed&cmd=Search&term=%22Pediatric+clinics+of+North+America%22[Jour]+AND+2004[pdat]+AND+Rosen+C[author]

Copyright © 2007 by the American Academy of Pediatrics page 81

Page 82: AAP MCQ 2007

2007 PREP SA on CD-ROM

Question: 40

You are notified by the newborn screening program that a 2-week-old infant in your practice hasan elevated phenylalanine level, which is confirmed by repeat testing. The mother reports thatthe baby is healthy and breastfeeding well.

Of the following, the MOST important first step in management of this infant is to

A. admit the baby to the hospital for further evaluation

B. consult with a metabolic geneticist or nutritionist

C. instruct the mother that she no longer should breastfeed

D. place the baby immediately on phenylalanine-free formula

E. send urine for organic acid analysis

Copyright © 2007 by the American Academy of Pediatrics page 82

Page 83: AAP MCQ 2007

2007 PREP SA on CD-ROM

Preferred Response: BCritique: 40

Phenylketonuria (PKU) is an autosomal recessive inborn error of metabolism that has anincidence of approximately 1 in 10,000 to 1 in 25,000 individuals. The gene that causes PKU iswell described and is mapped to chromosome 12 on the long arm. More than 240 mutationscausing PKU have been defined, and carrier detection and prenatal diagnosis are available usingmolecular genetic testing.

Newborn screening programs for PKU initially were established based on the discovery thatthe early diagnosis and treatment of the disease virtually eliminate morbidity and mortality. Anelevated blood phenylalanine (PHE) concentration detected by newborn screening performedafter the initiation of feeding, as described for the infant in the vignette, suggests the need for adiagnostic evaluation. However, it is important to realize that most infants in whom elevated PHEconcentrations are identified through screening programs do not have PKU; they simply havedelayed maturation of the metabolizing enzymes. Individuals who have classic PKU andcontinue to feed normally experience a rapid rise in serum PHE concentration to levels greaterthan 30 mg/dL (1,815 mcmol/L)(normal, 1 mg/dL [60.5 mcmol/L]). Some individuals have avariant termed “hyperphenylalaninemia,” which also results from defective phenylalaninehydroxylase. Yet another group of individuals has biopterin deficiency; biopterin serves as acofactor (in the form of tetrahydrobiopterin) for the formation of tyrosine from PHE.

Following the identification of an infant who has a positive newborn screening result for PKU,the most efficient next step is either to repeat the screening test or to send blood for amino acidquantitation. Because such test results are seen infrequently in general pediatric practices, it isimportant to consult with a metabolic geneticist. The results of the amino acid analysis allow thegeneticist to distinguish among true PKU, in which the PHE level is very elevated and thetyrosine level is negligible; hyperphenylalaninemia, in which the level is elevated in theintermediate range (10 to 20 mg/dL [605 to 1,200 mcmol/L]) and there may be some tyrosine;and newborn immaturity, in which the PHE level is elevated, but there is sufficient tyrosine. It isnot necessary to admit the baby described in the vignette to the hospital at this point because heis stable and is not expected to decompensate. If the mother discontinues breastfeeding and thebaby does not have PKU, the opportunity to breastfeed may be lost. Phenylalanine-free formulais expensive, and 2 to 3 days of therapy while awaiting test results has no clear advantage.Finally, urine organic acid analysis is not helpful.

Once the diagnosis of PKU is confirmed, it is critical to confer with a metabolicgeneticist/nutritionist and, if at all possible, to have the patient seen at a metabolic clinic. Therapyinvolves restriction of dietary PHE to the amount tolerated by the patient and regularmeasurement of plasma PHE concentrations. Medical foods that contain amino acid mixtures orprotein hydrolysates that lack phenylalanine are substituted for approximately 75% of the proteinrequired by the infant. It also is important to monitor plasma tyrosine concentrations becausetyrosine becomes an essential amino acid in affected children.

When carefully managed and followed, individuals who have PKU have a very brightprognosis, both for health and for cognitive function. It is important to note that in the past,affected children were taken off their special diets at about the age of 6 years. However, it nowappears best to keep affected individuals on their diets indefinitely because of measurabledecreases in the intelligence quotient and school performance of individuals who came off thediet. Additionally, because of the adverse effects of hyperphenylalaninemia on the fetus, womenwho have PKU should be on special diets and monitored closely during pregnancy.

References:

American Academy of Pediatrics Committee on Genetics. Maternal phenylketonuria. Pediatrics.2001;107:427-428. Available at: http://pediatrics.aappublications.org/cgi/content/full/107/2/427

Mitchell JJ, Scriver CR. Phenylalanine hydroxylase deficiency. GeneReviews. 2005. Availableat:http://www.geneclinics.org/servlet/access?db=geneclinics&site=gt&id=8888891&key=NOg1AvzeZtDD1&gry=&fcn=y&fw=pvPE&filename=/profiles/pku/index.html

Copyright © 2007 by the American Academy of Pediatrics page 83

Page 84: AAP MCQ 2007

2007 PREP SA on CD-ROM

Rezvani I. Phenylalanine. In: Behrman RE, Kliegman RM, Jenson HB, eds. Nelson Textbook ofPediatrics. 17th ed. Philadelphia Pa: Saunders; 2004:398-402

Wilcox WR, Cederbaum SD. Amino acid metabolism. In: Rimoin DL, Connor JM, Pyeritz RE, KorfBR, eds. Emery and Rimoin’s Principles and Practice of Medical Genetics. 4th ed. New York,NY: Churchill Livingstone; 2002:2405-2440

Copyright © 2007 by the American Academy of Pediatrics page 84

Page 85: AAP MCQ 2007

2007 PREP SA on CD-ROM

Question: 41

A 16-year-old girl complains of dysuria of 1 week's duration. She was seen 4 days ago at anurgent care facility and was prescribed trimethoprim-sulfamethoxazole because of abnormalurinalysis findings. Although she has taken the antibiotic as directed, she has had no relief fromthe dysuria. She has no nausea, vomiting, flank pain, vaginal discharge, or fever.

Of the following, the MOST appropriate course of action is to

A. add doxycycline to the treatment

B. change the antibiotic to ciprofloxacin

C. inspect her genitalia

D. order ultrasonography of the kidneys

E. treat with fluconazole

Copyright © 2007 by the American Academy of Pediatrics page 85

Page 86: AAP MCQ 2007

2007 PREP SA on CD-ROM

Preferred Response: CCritique: 41

Dysuria is a nonspecific symptom that may be caused by a variety of conditions, includingurinary tract infections (UTIs), sexually transmitted infections (STIs), vulvitis, vaginitis, urethritis,dermatitis, urolithiasis, hypercalciuria, pinworms, chemical irritation, dysfunctional voidingpatterns, and trauma. Factors that can guide appropriate laboratory tests and treatment includepatient age (eg, pinworms in a young child) and sex (UTI is very uncommon in adolescentmales), family history (eg, renal disease or stones), patient sexual history (abuse, symptoms inpartner[s]), and findings on the physical examination. History alone does not reliably differentiatethe dysuria caused by a UTI from that associated with an STI. Diagnostic clues often are foundduring inspection of the genitalia.

Adolescent females who have dysuria, pyuria, and normal findings on genitalia inspectionand have not been sexually active may be treated presumptively for a UTI with or without a urineculture. However, some clinicians prefer to obtain a pretreatment urine culture to confirm a UTIas a cause of the dysuria. A positive test for leukocyte esterase reflecting pyuria or thepresence of white blood cells on microscopy is suggestive, but not specific, for a UTI. Othercauses of pyuria include vaginitis, cervicitis, and urethritis. Pyuria and a positive urine test fornitrite indicate bacteriuria, but a concurrent STI may be present. Many adolescent females whohave UTIs (eg, caused by Staphylococcus saprophyticus) do not have a positive urine nitritetest result.

An adolescent male should not be treated empirically for a UTI; the presence of pyuria in amale most likely is due to an STI. However, males who have pyuria and a positive urine nitritetest should have a urine culture and further evaluation for associated conditions (eg, anatomicabnormalities, other foci of infection).

An STI must be considered in patients who present with dysuria and have been sexuallyactive, sexually abused, or have suspicious findings during genital inspection. Nucleic acidamplification (eg, polymerase chain reaction) testing on a urine specimen is useful to diagnoseinfection with Chlamydia trachomatis and Neisseria gonorrhoeae. A bimanual examination alsomay be necessary to evaluate adolescent females who have lower abdominal pain or abnormalvaginal discharge that might indicate pelvic inflammatory disease. A urine culture is required forthe evaluation of dysuria accompanied by symptoms suggestive of acute pyelonephritis (eg,flank pain, fever, vomiting).

Inspection of the genitalia of the girl in the vignette may reveal clues to the cause of thedysuria. The examiner may find dermatitis, evidence of trauma, vaginal discharge, or lesionsassociated with sexually transmitted diseases (eg, herpetic vesicles, Bartholin gland cyst).

Although the patient may have persistent dysuria due to a chlamydial infection or UTIunresponsive to the antibiotic, the empiric addition or change of antibiotic (eg, to doxycycline,ciprofloxacin) is not appropriate without a more detailed history, other laboratory tests, andinspection of the genitalia. Ultrasonography of the kidneys is not warranted because the patienthas no symptoms suggestive of upper tract disease. Dysuria frequently is associated with amonilial vaginitis, but empiric treatment with fluconazole without additional assessment is notappropriate.

References:

Bonny AE, Brouhard BH. Urinary tract infections among adolescents. Adolesc Med Clin.2005;16:149-161

D’Angelo LJ, Neinstein LS. Genitourinary tract infections. In: Neinstein LS, ed. Adolescent HealthCare A Practical Guide. 4th ed. Philadelphia, Pa: Lippincott, Williams & Wilkins; 2002:551-564

Peters CA. Pediatric urology in the pediatric and adolescent girl. In: Emans SJH, Laufer MR,Goldstein DP, eds. Pediatric and Adolescent Gynecology. 5th ed. Philadelphia, Pa: LippincottWilliams & Wilkins; 2005:494-524

Woods ER, Emans SJ. Vulvovaginal complaints in the adolescent. In: Emans SJH, Laufer MR,

Copyright © 2007 by the American Academy of Pediatrics page 86

Page 87: AAP MCQ 2007

2007 PREP SA on CD-ROM

Goldstein DP, eds. Pediatric and Adolescent Gynecology. 5th ed. Philadelphia, Pa: Lippincott,Williams & Wilkins; 2005:525-559

Copyright © 2007 by the American Academy of Pediatrics page 87

Page 88: AAP MCQ 2007

2007 PREP SA on CD-ROM

Question: 42

A 16-year-old girl is being seen in your clinic because of headaches for the past month thatsometimes awaken her at night. At her last visit 2 years ago, she was well, 5 ft 2 in tall, andweighed 105 lb, with Sexual Maturity Rating (SMR) 4 breast development and SMR 4 pubic hair,but she had not reached menarche. On examination at this visit, she is 5 ft 2 in tall, weighs 110lb, and still has not begun menstruating. On evaluation, you note bitemporal visual field deficits,perhaps worse on the left.

In addition to magnetic resonance imaging and an ophthalmologic evaluation, the laboratory testthat is MOST likely to be most diagnostically revealing is

A. adrenocorticotropic hormone

B. insulin-like growth factor-1

C. luteinizing hormone

D. prolactin

E. thyroid-stimulating hormone

Copyright © 2007 by the American Academy of Pediatrics page 88

Page 89: AAP MCQ 2007

2007 PREP SA on CD-ROM

Preferred Response: DCritique: 42

The 16-year-old girl described in the vignette has had arrest of pubertal development with failureof menarche for at least 2 years. The bitemporal visual field deficits imply the presence of a largesuprasellar mass. The two most common causes for such a mass lesion are pituitarymacroadenomas and craniopharyngiomas, and the most common macroadenoma is aprolactinoma. Elevations of prolactin values could be due to increased prolactin secretion from aprolactinoma or to compression of the pituitary stalk by another pituitary tumor, acraniopharyngioma, or other space-occupying lesion. Because stalk compression interfereswith the dopaminergic inhibitory influences on pituitary release of prolactin, prolactinconcentrations of up to 200 mcg/L may be due solely to stalk compression. Prolactinconcentrations in that range interfere with normal menstrual cycling and suppress normalpuberty. A large tumor also might interfere with pubertal and other pituitary hormones because ofits mass. Accordingly, measuring prolactin will be most helpful in determining whether the girl hasa prolactinoma. Prolactinomas usually respond to treatment with medication such ascabergoline; surgery rarely is required.

Elevated adrenocorticotropic hormone (ACTH) values might be seen in Cushing disease,but the girl in the vignette exhibits no signs or symptoms of Cushing disease, such as weightgain, skin striae, hypertension, cushingoid facies, and muscle weakness. Further, the pituitarytumors in affected individuals rarely are macroadenomas because the symptoms lead to anearlier diagnosis. A low concentration of insulin-like growth factor-1 might suggest growthhormone deficiency, and a high concentration suggests acromegaly. No signs or symptoms ofacromegaly, such as rapid growth, coarsening of facial features, sweating, or thickening of skin,are described for the girl. Measurement of thyroid-stimulating hormone (TSH) might identify aTSH-producing tumor or severe hypothyroidism that is primary because of thyroid gland failure.A TSH-producing tumor, which is very rare, produces hyperthyroidism, and no symptoms ofhyperthyroidism are described for the girl in the vignette. Sometimes severe primaryhypothyroidism can lead to the development of an enlarged pituitary composed of thyrotropes(TSH-secreting cells), but no symptoms or signs of severe hypothyroidism (eg, pallor, lethargy,dry skin, coarse facies, slow heart rate, myxedema) are described. Measurement of luteinizinghormone would be useful if the clinician suspected ovarian failure. However, follicle-stimulatinghormone is a better measure of ovarian failure because it rises higher than luteinizing hormone ifthere is ovarian dysfunction. Rare pituitary tumors produce luteinizing hormone, but thesetumors do not produce neuroendocrine symptoms in females unless they also produce follicle-stimulating hormone.

References:

Cannavo S, Venturino M, Curto L, et al. Clinical presentation and outcome of pituitary adenomasin teenagers. Clin Endocrinol (0xf). 2003;58:519-527 Abstract available at:http://www.ncbi.nlm.nih.gov/entrez/query.fcgi?cmd=Retrieve&db=pubmed&dopt=Abstract&list_uids=12641637&query_hl=43&itool=pubmed_docsum

Karavitaki N, Brufani C, Warner TJ, et al. Craniopharyngiomas in children and adults: systematicanalysis of 121 cases with long-term follow-up. Clin Endocrinol (Oxf). 2005;62:397-409 Abstractavailable at:http://www.ncbi.nlm.nih.gov/entrez/query.fcgi?orig_db=PubMed&db=PubMed&cmd=Search&term=%22Clinical+endocrinology%22[Jour]+AND+397[page]+AND+2005[pdat]

Molitch ME. Disorders of prolactin secretion. Endocrinol Metab Clin North Am. 2001;30:585-610

Copyright © 2007 by the American Academy of Pediatrics page 89

Page 90: AAP MCQ 2007

2007 PREP SA on CD-ROM

Question: 43

An 8-year-old boy is inattentive at home and school, has difficulty completing his homework, andis failing reading. Physical examination findings are normal, he has friends at school, and thefamily has been living in their newly built home for the past 3 years. You begin to discuss adiagnosis of attention-deficit/hyperactivity disorder, and his mother asks you what tests you willperform to try to determine the cause of the problem.

Of the following, your BEST response is that you will order

A. a lead level

B. an electroencephalogram

C. computed tomography scan of the brain

D. no tests at this time

E. thyroid studies

Copyright © 2007 by the American Academy of Pediatrics page 90

Page 91: AAP MCQ 2007

2007 PREP SA on CD-ROM

Preferred Response: DCritique: 43

Attention-deficit/hyperactivity disorder (ADHD) is a common, yet heterogeneous disorder thathas a complex etiology; genetic, environmental, and biologic factors all play roles. For example,ADHD is associated with both genetic syndromes such as fragile X and intrauterine toxicexposures such as fetal alcohol syndrome. It also has been shown in family and twin studiesthat ADHD is more common in close family members of those diagnosed with the disorder.

Research continues into the neurobiologic basis of ADHD. Evidence is increasing thatalterations in the frontal lobe and frontal subcortical connections play a significant role in thedisorder. Affected children also have difficulties with executive functioning, such as organization,impulse control, and inattention, that are common in other disorders involving impaired frontallobe function.

Although not necessary for all children, a full psychoeducational evaluation likely isnecessary before ADHD can be diagnosed in the boy in the vignette. The boy also may have alearning disability or an environmental cause of his academic difficulty, such as a chaotic homeenvironment or significant social stresses.

ADHD is diagnosed clinically using American Psychiatric Association Diagnostic andStatistical Manual of Mental Disorders-4th ed (DSM-IV) criteria; routine medical testing is notnecessary. Lead intoxication is associated with cognitive dysfunction, including inattention andhyperactivity, but it is not a significant cause of ADHD. A lead level should be obtained if anyconcerns for lead exposure are established. Thyroid dysfunction also may cause cognitivechanges, but other information from the history or physical examination should prompt therequest of these studies. Electroencephalography or computed tomography scan of the brain isunlikely to provide clinically useful information in a child who has no history of seizure, significantbrain injury, or neurologic findings on examination.

References:

American Academy of Pediatrics Committee on Quality Improvement, Subcommittee onAttention-Deficit/Hyperactivity Disorder. Clinical practice guideline: diagnosis and evaluation ofthe child with attention-deficit/hyperactivity disorder. Pediatrics. 2000;105:1158-1170. Availableat: http://pediatrics.aappublications.org/cgi/content/full/105/5/1158

Reiff MI, Tippins S, LeTourneau AA. ADHD: A Complete and Authoritative Guide. Elk GroveVillage, Ill: American Academy of Pediatrics; 2004

Sims MD. Attention-deficit/hyperactivity disorder. In: Behrman RE, Kliegman RM, Jenson HB,eds. Nelson Textbook of Pediatrics. 17th ed. Philadelphia, Pa: WB Saunders Co; 2004: 107-110

Copyright © 2007 by the American Academy of Pediatrics page 91

Page 92: AAP MCQ 2007

2007 PREP SA on CD-ROM

Question: 44

Upon entering your examination room, you find a father who is visibly upset. He has brought his1-month-old and 2-year-old daughters to see you today because his wife was recentlydiagnosed with tuberculosis. As you question him further, he tells you that his wife's physiciantold him that his wife's chest radiograph was abnormal and she was "smear-positive." He handsyou results from the health department stating that he and his daughters all had negative skintests and chest radiographs. His wife is at the health department today to start medicinesagainst tuberculosis, and he wants to know if there is anything that should be done for thechildren.

Of the following, the MOST appropriate treatment is to

A. administer the bacille Calmette-Guérin vaccine

B. begin isoniazid therapy

C. provide reassurance

D. remove the children from the home for 3 months

E. repeat the skin test in 1 month

Copyright © 2007 by the American Academy of Pediatrics page 92

Page 93: AAP MCQ 2007

2007 PREP SA on CD-ROM

Preferred Response: BCritique: 44

Cases of suspected or proven tuberculosis should be reported to the local health departmentimmediately. If a woman has pulmonary tuberculosis, it is unlikely that she will infect any fetus,but she may infect the newborn after delivery. In such cases, all contacts should have physicalexaminations, tuberculin skin tests, and chest radiographs. Besides these tests, a lumbarpuncture is indicated for any newborn who possibly may have congenital tuberculosis. Themother and infant also should be screened for the human immunodeficiency virus.

If results of the evaluation for active disease in the contacts of a person who hastuberculosis are negative, as reported for the children in the vignette, all children younger than 4years of age should begin isoniazid therapy, with follow-up skin testing in 12 weeks. If there is nosign of tuberculous disease upon follow-up skin testing and the source case has demonstrateda good clinical response and has been adherent to the treatment program, the isoniazid can bestopped. If the tuberculin skin test is positive at follow-up, the child should be reassessed with achest radiograph for tuberculous disease. If active disease is not present, he or she shouldcomplete a 9-month (total) course of isoniazid therapy.

The children in the vignette should not receive bacille Calmette-Guérin vaccine or beremoved from their home unless the mother has risk factors for or is infected with a multidrug-resistant isolate. Reassurance and repeating the skin testing in 1 month is not appropriate in thiscase because the mother is smear-positive and both children are at high risk for disseminateddisease because of their ages.

It is important to remember that most children who have active tuberculosis do not producesputum and, therefore, are not contagious and do not require special precautions. Exceptions tothis rule are children who have cavitary pulmonary lesions, positive sputum smears, laryngealdisease, extensive pulmonary disease, or suspected congenital tuberculosis. If patients havepositive sputum smears, isolation should be continued until effective therapy has been initiated,sputum smears are negative, and the cough has abated. Children who have no cough ornegative sputum smears can be hospitalized safely in an open ward.

References:

American Academy of Pediatrics. Tuberculosis. In: Pickering LK, ed. Red Book: 2006 Report ofthe Committee on Infectious Diseases. 27th ed. Elk Grove Village, Ill: American Academy ofPediatrics; 2006:678-698

Pediatric Tuberculosis Collaborative Group. Targeted tuberculin skin testing and treatment oflatent tuberculosis infection in children and adolescents. Pediatrics. 2004;114:1175-1201.Available at: http://pediatrics.aappublications.org/cgi/content/full/114/4/S2/1175

Copyright © 2007 by the American Academy of Pediatrics page 93

Page 94: AAP MCQ 2007

2007 PREP SA on CD-ROM

Question: 45

You are examining a 5-year-old girl who always has had significant daytime wetting and a historyof recurrent urinary tract infections. Findings on physical examination are normal except for thepresence of a sacral dimple above the gluteal cleft. Her urinalysis reveals a specific gravity of1.005, pH of 5.5, no blood, no protein, and no white or red blood cells. Magnetic resonanceimaging of the spine reveals spinal dysraphism.

Of the following, the MOST important next step to determine the cause of this child's primaryenuresis is to obtain

A. abdominal computed tomography scan

B. abdominal radiography

C. abdominal ultrasonography

D. renal biopsy

E. urine culture

Copyright © 2007 by the American Academy of Pediatrics page 94

Page 95: AAP MCQ 2007

2007 PREP SA on CD-ROM

Preferred Response: CCritique: 45

Primary enuresis is defined as the absence of achievement of dryness during the daytime,nighttime, or both. In contrast to secondary enuresis, in which the child experiences a period ofdryness after completing toilet training, primary enuresis generally has an anatomic cause.

The daytime wetting, history of recurrent urinary tract infections (UTIs), sacral dimple abovethe gluteal cleft, specific gravity of 1.005 and no other abnormalities on urinalysis, and spinaldysraphism on magnetic resonance imaging reported for the girl in the vignette are consistentwith probable spina bifida occulta. The spina bifida may be associated with bladder dysfunctionbecause innervation of the bladder originates from the sacral spine, although some investigatorsbelieve that the association may be only an incidental finding.

Children who have bladder dysfunction of any cause exhibit dribbling, frequency, andrecurrent UTIs due to bladder dyssynergy. This may cause some degree of urinary tractobstruction because bladder emptying is interrupted. Such obstruction can be detected easily byabdominal ultrasonography, which is far more accurate for documenting urinary tract anomaliesthan abdominal radiography. Abdominal computed tomography scan may be more precise thanultrasonography for some conditions in the urinary tract, but it is far more expensive, and theprecision for detecting urinary tract obstruction is generally similar. A renal biopsy is indicated forchildren who have findings suggestive of primary glomerular disease (eg, proteinuria), but wouldnot be helpful to assess urinary tract anomalies. Finally, although children who have bladdermalformation may develop UTIs, the child in the vignette has no symptoms or findings onurinalysis suggestive of a UTI. Moreover, a positive urine culture result will not aid inascertaining the cause of her primary enuresis.

References:

Kumar P, Aneja S, Kumar R, Taluja V. Spina bifida occulta in functional enuresis. Indian JPediatr. 2005;72:223-225. Available at: http://www.ijppediatricsindia.org/article.asp?issn=0019-5456;year=2005;volume=72;issue=3;spage=223;epage=225;aulast=Kumar

Samuel M, Boddy SA. Is spina bifida occulta associated with lower urinary tract dysfunction inchildren? J Urol. 2004;171:2664-2666. Abstract available at:http://www.ncbi.nlm.nih.gov/entrez/query.fcgi?cmd=Retrieve&db=pubmed&dopt=Abstract&list_uids=15118447&query_hl=22&itool=pubmed_docsum

Copyright © 2007 by the American Academy of Pediatrics page 95

Page 96: AAP MCQ 2007

2007 PREP SA on CD-ROM

Question: 46

A 2,700-g male infant born at 36 weeks' gestation is being treated for suspected neonatal sepsisfollowing the development of respiratory distress shortly after birth. His mother had a fever to102ºF (38.9°C) during labor and delivery but reports that she had no illnesses during pregnancy.

Of the following, the MOST appropriate antibiotic regimen for this infant is

A. ampicillin and an aminoglycoside

B. clindamycin and a third-generation cephalosporin

C. meropenem and an aminoglycoside

D. piperacillin and an aminoglycoside

E. vancomycin and a third-generation cephalosporin

Copyright © 2007 by the American Academy of Pediatrics page 96

Page 97: AAP MCQ 2007

2007 PREP SA on CD-ROM

Preferred Response: ACritique: 46

Neonatal sepsis is a clinical syndrome characterized by systemic signs of infectionaccompanied by bacteremia occurring during the first month after birth. Neonatal sepsis occursin 1 to 5 per 1,000 live births, with mortality rates ranging from 5% to 15%. It is categorized asearly-onset (infections occurring before 7 days of age) and late-onset (infections occurringbetween 7 and 30 days of age). The most common bacteria causing early-onset bacterialinfections are group B streptococci (Streptococcus agalactiae), Escherichia coli, enterococci,other enteric bacilli, alpha-hemolytic streptococci, Listeria monocytogenes, Streptococcuspneumoniae, Haemophilus influenzae, and other maternal genital flora. Empiric antibiotic therapyfor neonates suspected of having early-onset sepsis, such as the infant described in thevignette, is a combination regimen that provides coverage for the most common organisms,penetrates into the central nervous system, and has minimal toxicity. The recommended empirictherapy, which has remained unchanged for 3 decades, is ampicillin plus gentamicin.

Although some experts advocate substitution of cefotaxime or another third-generationcephalosporin for gentamicin, these agents have no better efficacy than ampicillin andgentamicin, and the routine use of these agents for empiric therapy of sepsis in neonatalintensive care units is associated with outbreaks of sepsis due to multiple-drug-resistant entericorganisms, especially Enterobacter and Serratia sp.

Clindamycin and a third-generation cephalosporin would not provide coverage forEnterococcus and Listeria sp. Meropenem plus an aminoglycoside, piperacillin plus anaminoglycoside, and vancomycin plus a third-generation cephalosporin provide much broadercoverage than needed and increase the risk for the development of antibiotic-resistant strains.

References:

Baltimore RS. Neonatal sepsis: epidemiology and management. Paediatr Drugs. 2003;5:723-740Abstract available at:http://www.ncbi.nlm.nih.gov/entrez/query.fcgi?orig_db=PubMed&db=PubMed&cmd=Search&term=%22Paediatric+drugs%22[Jour]+AND+723[page]+AND+2003[pdat]

de Man P, Verhoeven BAN, Verbrugh HA, et al. An antibiotic policy to prevent emergence ofresistant bacilli. Lancet. 2000;355:973-978 Abstract available at:http://www.ncbi.nlm.nih.gov/entrez/query.fcgi?orig_db=PubMed&db=PubMed&cmd=Search&term=%22Lancet%22[Jour]+AND+973[page]+AND+2000[pdat]

Edwards MS, Baker CJ. Bacterial infections in the neonate. In: Long SS, Pickering LK, ProberCG, eds. Principles and Practice of Pediatric Infectious Diseases. 2nd ed. New York, NY:Churchill Livingstone; 2003:536-542

Fanos V, Dall’Agnola A. Antibiotics in neonatal infections: a review. Drugs. 1999;58:405-427Abstract available at:http://www.ncbi.nlm.nih.gov/entrez/query.fcgi?orig_db=PubMed&db=PubMed&cmd=Search&term=%22Drugs%22[Jour]+AND+405[page]+AND+1999[pdat]

Gerdes JS. Diagnosis and management of bacterial infections in the neonate. Pediatr Clin NorthAm 2004;51:939-959. Abstract available at:http://www.ncbi.nlm.nih.gov/entrez/query.fcgi?orig_db=PubMed&db=PubMed&cmd=Search&term=%22Pediatric+clinics+of+North+America%22[Jour]+AND+939[page]+AND+2004[pdat]

Copyright © 2007 by the American Academy of Pediatrics page 97

Page 98: AAP MCQ 2007

2007 PREP SA on CD-ROM

Question: 47

A 10-year-old boy presents to the clinic for evaluation of an insect sting reaction. He was playingin his yard yesterday when he was stung on the right forearm by a wasp. His parentsimmediately placed ice over the area, but they are worried because when the child awoke today,the redness that initially was localized to the sting site had extended to involve his entire rightforearm. The boy denies fever, chills, nausea, vomiting, or difficulty breathing. His vital signs onphysical examination are normal. His right forearm is diffusely erythematous and warm topalpation but not tender.

Of the following, this child's insect sting reaction is BEST characterized as

A. cellulitis

B. large local reaction

C. normal reaction

D. systemic anaphylaxis

E. toxic reaction

Copyright © 2007 by the American Academy of Pediatrics page 98

Page 99: AAP MCQ 2007

2007 PREP SA on CD-ROM

Preferred Response: BCritique: 47

The reaction described for the child in the vignette is consistent with a large local reaction. Whenevaluating a child who experiences an insect sting reaction, the history should categorize thereaction type to help determine a management plan. Insect sting reactions are classified as localor normal reactions, large local reactions, or systemic reactions.

Local or normal reactions are most common and generally are of little medical consequence.Typical symptoms include erythema, warmth, pain, and swelling confined to the sting site. Oralantihistamines, cold compresses, and sometimes topical corticosteroids can be used to alleviatesymptoms.

Large local reactions generally begin with mild erythema that increases in size over the next12 to 24 hours, often exceeding 20 cm and occasionally involving the entire extremity. Suchreactions are immunoglobulin (Ig)E-mediated and may increase the risk for anaphylaxis withsubsequent stings to 5% to 10%, although long-term studies and current guidelines do notrecommend antibiotics, skin testing, or immunotherapy for individuals who only experience largelocal reactions.

Cellulitis can occur after any insect sting, but is rare compared with large local reactions.Fever, chills, tenderness, and progression of swelling beyond 48 hours are common withcellulitis but typically absent with large local reactions.

Systemic reactions include anaphylaxis and toxic reactions. Toxic reactions occur from thedirect effect of venom following multiple (usually >50) stings and have been reported to causerenal failure, rhabdomyolysis, diffuse intravascular coagulation, and even seizures.

Rapid onset of cutaneous (generalized hives, angioedema, flushing), respiratory (throattightness, chest tightness, dyspnea, wheezing), or circulatory signs (dizziness, hypotension,syncope) after only a few stings usually represents anaphylaxis, an IgE-mediated reaction. Anyindividual older than 16 years of age who experiences anaphylaxis after an insect sting shouldbe counseled about the increased risk (30% to 60%) for similar reactions with subsequentinsect stings, prescribed self-injectable epinephrine, and considered for allergy consultation.Individuals younger than 16 years of age who experience cutaneous symptoms only (urticariaor angioedema) do not appear to have the same risk with future stings and may be managedconservatively.

References:

Freeman TM. Clinical practice. Hypersensitivity to hymenoptera stings. N Engl J Med.2004;351:1978-1984

Sicherer SH, Leung DYM. Insect allergy. In: Behrman RE, Kliegman RM, Jenson HB, eds.Nelson Textbook of Pediatrics. 17th ed. Philadelphia, Pa: WB Saunders Co 2004:786-788

Copyright © 2007 by the American Academy of Pediatrics page 99

Page 100: AAP MCQ 2007

2007 PREP SA on CD-ROM

Question: 48

A 10-year-old child is brought to your office for evaluation of a 1-day history of fever, vomiting,diarrhea, and abdominal pain. His mother states that he has vomited five times, and the emesishas been clear. He has had four episodes of nonbloody diarrhea. He describes his abdominalpain as crampy but cannot localize it to any specific part of his abdomen. He denies anysymptoms of dysuria. On physical examination, the child is in no acute distress, his temperatureis 99.2°F (37.3°C), heart rate is 102 beats/min, respiratory rate is 26 breaths/min, and bloodpressure is 105/70 mm Hg. Results of examination of the head, neck, chest, and heart arenormal. His abdomen is soft, and there is no guarding. There is no rebound tenderness. Hecomplains of mild discomfort on deep palpation of his entire abdomen. He has hyperactive bowelsounds on auscultation, and he has no flank tenderness.

Of the following, the MOST appropriate next step in the management of this patient is to

A. administer intravenous fluids

B. obtain blood for a complete blood count

C. obtain serum for electrolyte analysis

D. order frontal supine and upright abdomen radiographs

E. send the patient home with instructions for supportive care

Copyright © 2007 by the American Academy of Pediatrics page 100

Page 101: AAP MCQ 2007

2007 PREP SA on CD-ROM

Preferred Response: ECritique: 48

The patient described in the vignette has signs and symptoms consistent with viralgastroenteritis. This is a clinical diagnosis that requires no further evaluation in the absence oftoxicity, rebound tenderness, distention, or evidence for dehydration, as in this child. Neither acomplete blood count nor serum electrolyte determinations are likely to alter management. Theboy should be sent home with instructions for supportive care.

Most children who have vomiting due to gastroenteritis can maintain sufficient levels ofhydration with glucose- and electrolyte-containing solutions. Intravenous hydration is notrequired unless fluid loss exceeds 10% of body weight or the patient is experiencing moremoderate 5% to 10% dehydration in conjunction with persistent vomiting and an inability orunwillingness to take oral fluids. Abdominal radiographs provide limited information for theevaluation of a child who has vomiting and diarrhea, and even in suspected appendicitis, rarelyare helpful.

Further evaluation for abdominal pain is indicated if information gained from the history andphysical examination suggest potential surgical emergencies such as appendicitis (tendernesslocalized to the right lower quadrant with or without rebound), small bowel obstruction (absentbowel sounds, abdominal distention), perforated viscus (rigid abdomen, distention, fever,toxicity), malrotation (diffuse tenderness, abdominal distention), intussusception (severe crampyabdominal pain alternating regularly with periods of pain relief), or peritonitis (fever, toxicity,diffuse abdominal tenderness).

References:

Bellemare S, Hartling L, Wiebe N, et al. Oral rehydration versus intravenous therapy for treatingdehydration due to gastroenteritis in children: a meta-analysis of randomised controlled trials.BMC Med. 2004;2:11 Available at: http://www.biomedcentral.com/1741-7015/2/11

Fleisher GR. Infectious disease emergencies. In: Fleisher GR, Ludwig S, Henretig FM, eds.Textbook of Pediatric Emergency Medicine. 5th ed. Philadelphia, Pa: Lippincott Williams &Wilkins; 2006:783-852

Kwok MY, Kim MK, Gorelick MH. Evidence-based approach to the diagnosis of appendicitis inchildren. Pediatr Emerg Care. 2004;20:690-698

Rothrock SG, Grenn SM, Harding M, et al. Plain abdominal radiography in the detection of acutemedical and surgical disease in children: a retrospective analysis. Pediatr Emerg Care.1991;7:281-285 Abstract available at:http://www.ncbi.nlm.nih.gov/entrez/query.fcgi?orig_db=PubMed&db=PubMed&cmd=Search&term=Pediatr+Emerg+Care[Jour]+AND+281[page]+AND+1991[pdat]

Rothrock SG, Pagane J. Acute appendicitis in children: emergency department diagnosis andmanagement. Ann Emerg Med. 2000;36:39-51 Abstract available at:http://www.ncbi.nlm.nih.gov/entrez/query.fcgi?orig_db=PubMed&db=PubMed&cmd=Search&term=Ann+Emerg+Med[Jour]+AND+39[page]+AND+2000[pdat]

Copyright © 2007 by the American Academy of Pediatrics page 101

Page 102: AAP MCQ 2007

2007 PREP SA on CD-ROM

Question: 49

A 1-month-old infant presents with frecklelike macules (Item Q49A) over his face andextremities. The hospital record reveals that he had multiple papules and pustules distributedover his entire body, including palms and soles, at birth. The infant appears to be very healthyand thriving.

Of the following, analysis of the pustular contents in the newborn period MOST likely would haverevealed

A. eosinophils

B. gram-positive cocci

C. multinucleated giant cells

D. polymorphonuclear leukocytes

E. pseudohyphae and budding yeast

Copyright © 2007 by the American Academy of Pediatrics page 102

Page 103: AAP MCQ 2007

2007 PREP SA on CD-ROM

Preferred Response: DCritique: 49

Transient neonatal pustular melanosis (TNPM) is a disorder of unknown cause that begins inutero. At birth, affected infants may exhibit pustules or small hyperpigmented macules (ItemC49A) surrounded by a rim of scale, the remnant of the pustule roof. Pustules resolve withinseveral days, but the hyperpigmented macules may persist for 1 to 3 months, as described forthe infant in the vignette. TNPM is more common among African-American infants and may beobserved on any skin surface, including the palms and soles. A Wright-stained preparation ofthe pustular contents reveals a predominance of neutrophils. No treatment is necessary for thisself-limited condition.

A number of neonatal skin conditions are characterized by pustules or vesicles. They maybe distinguished from transient neonatal pustular melanosis clinically and by microscopicexamination of lesional contents. The vesicles of erythema toxicum (Item C49B) containeosinophils (Item C49C), the pustules caused by staphylococcal folliculitis (Item C49D) containgram-positive cocci, the vesicles of herpes simplex virus infection (Item C49E) containmultinucleated giant cells (Item C49F), and the pustules of congenital candidiasis (Item C49G)contain pseudohyphae and budding yeast (Item C49H).

References:

Eichenfield L, Larralde M. Neonatal skin and skin disorders: congenital erosive and vesiculardermatosis, vesiculopustular and bullous diseases of the newborn. In: Schachner LA, HansenRC, eds. Pediatric Dermatology. 3rd ed. St. Louis, Mo: Mosby; 2003:232-239

Weston WL, Lane AT, Morelli JG. Skin diseases in newborns: transient skin disease. In: ColorTextbook of Pediatric Dermatology. 3rd ed. St. Louis, Mo: Mosby; 2002:299-307

Copyright © 2007 by the American Academy of Pediatrics page 103

Page 104: AAP MCQ 2007

2007 PREP SA on CD-ROM

Question: 50

A 17-year-old boy who has a 5-year history of Crohn disease comes in with a flare of his illness,characterized by fever, diarrhea, and a 15-lb weight loss. He admits he has "forgotten to takehis medicines lately." Physical examination demonstrates a very thin patient who has a perianalfistula. Abdominal computed tomography scan demonstrates thickening of the ileum (Item Q50A)and ascending colon. You are trying to decide whether to administer enteral nutrition (through anasogastric tube) or begin parenteral nutrition.

Of the following findings associated with Crohn disease, the BEST indication for institutingparenteral nutrition is

A. abdominal radiograph demonstrating air fluid levels

B. active ileitis demonstrated on colonoscopy

C. hypophosphatemia

D. institution of 6-mercaptopurine therapy

E. perianal abscess and fistula

Copyright © 2007 by the American Academy of Pediatrics page 104

Page 105: AAP MCQ 2007

2007 PREP SA on CD-ROM

Preferred Response: ACritique: 50

Crohn disease can cause many complications that require intensive nutritional therapy, includingintestinal inflammation, fistulizing disease, and small bowel strictures. In most patients, suchproblems can be managed with enteral nutrition. Typically, a polymeric or enteral formula isdelivered either by nasogastric tube, gastrostomy, or orally. The formula not only providesnutrition, but it may have a beneficial anti-inflammatory effect on the disease itself. Patients whohave hypophosphatemia, ileitis, or perianal fistula and those receiving 6-mercaptopurine therapyall could benefit from enteral feedings. Small bowel obstruction, as indicated by air fluid levels onradiography, is a contraindication to enteral feedings.

The principle for nourishing a chronically ill child is "if the gut works, use it." If tolerated,enteral nutrition is preferable to parenteral nutrition for many reasons. Enteral nutrition maintainsthe integrity of the gastrointestinal mucosa, stimulates the recovery of digestive enzymes suchas lactase, and stimulates hepatobiliary and pancreatic secretions. Parenteral nutrition, althoughlifesaving in some circumstances, is associated with an increased risk of bacterial sepsis (fromline infections), electrolyte abnormalities, hepatitis, and cholestasis. Currently availablecommercial formulas allow tube feeding in a variety of circumstances. Hydrolysates and aminoacid-based formulas are available for children who have protein allergy, and low-fat formulas canbe used for children who have fat malabsorption. Transpyloric tube placement allows enteralfeeding while minimizing the risk of aspiration for children at risk for pulmonary aspiration due toreflux. The primary use of parenteral nutrition is in children who have intestinal disorders suchas short bowel syndrome, Crohn disease, anatomic obstruction, ileus, pseudo-obstruction, orenteric fistulas.

References:

Courtney E, Grunko A, McCarthy T. Enteral nutrition. In: Hendricks KM, Duggan C, eds. Manualof Pediatric Nutrition. 4th ed. Hamilton, Ontario, Canada: BC Decker; 2005:252-316

Hyams JS. Inflammatory bowel disease. Pediatr Rev. 2005;26:314-320 Available at:http://pedsinreview.aappublications.org/cgi/content/full/26/9/314

Copyright © 2007 by the American Academy of Pediatrics page 105

Page 106: AAP MCQ 2007

2007 PREP SA on CD-ROM

Question: 51

You are asked to see a term infant in the newborn nursery at 8 hours of age and consider hertransfer to the neonatal intensive care unit. The problems and findings reported to you include:poor feeding, emesis, temperature instability (core temperature of 96°F [35.5°C]), hypoglycemia(whole blood glucose concentration of 25 mg/dL [1.4 mmol/L]), and polycythemia (hematocrit of70% [0.70]).

Of the following, the physical finding that is MOST likely to accompany these problems is

A. a tuft of hair over the sacral region

B. birthweight of 1,800 g

C. café au lait macule on the left leg

D. iris coloboma of the right eye

E. isolated cleft of the hard palate

Copyright © 2007 by the American Academy of Pediatrics page 106

Page 107: AAP MCQ 2007

2007 PREP SA on CD-ROM

Preferred Response: BCritique: 51

Infants who are small for gestational age (<2,500 g) at birth may be affected by maternal diseasestates (eg, advanced maternal diabetes mellitus, chronic hypertension, cardiovascular disease);pregnancy-related problems, including poor nutritional status with poor weight gain or pregnancy-induced hypertension (PIH); or fetal conditions such as congenital infection, chromosomeabnormality, or birth defects. The infant’s aberrant growth may be discerned prenatally(intrauterine growth restriction [IUGR]) or only noted at the time of birth. These infants have bothincreased morbidity and mortality compared with appropriately grown term infants (Item C51A).

The fetus that experiences IUGR often has a poor tolerance for labor. Consequently,perinatal asphyxia is more common, and successful transition and adaptation to extrauterine lifeis fraught with complications. Such complications include temperature instability (hypothermia)related to a large head-to-body ratio, decreased subcutaneous fat, and small energy reserves,all of which require a thermoneutral environment and an avoidance of fasting. Hypoglycemiamay be present and would be treated best with an intravenous dextrose infusion. Feedingproblems that may preclude enteral feeding for a time or require the use of a nasogastric ororogastric feeding tube are not uncommon in the small-for-gestational age infant. Polycythemiamay be present and reflects fetal adaptation to chronic hypoxia by increasing the erythrocytemass to enhance oxygen-carrying capacity. Polycythemia also may be associated with impairedfeeding, hypoglycemia, and jitteriness. When the hematocrit is greater than 70% (0.70) and thereare clinical signs of altered neurologic function, a partial exchange transfusion with normal salineshould be performed to reduce the hematocrit to 55% (0.55).

The findings reported for the infant in the vignette most likely are associated with the infantbeing small for gestational age (<2,500 g birthweight). A tuft of hair over the sacral region mayindicate occult spina bifida, but this finding would not be associated with growth impairment.Similarly, café au lait macules (often seen in neurofibromatosis), an iris coloboma (Item C51B)(seen in the CHARGE syndrome), and isolated clefting of the hard palate are not commonlyassociated with growth restriction.

References:

Hay WW Jr, Catz CS, Grave GD, Yaffe SJ. Workshop summary: fetal growth: its regulation anddisorders. Pediatrics. 1997;99:585-591. Available at:http://pediatrics.aappublications.org/cgi/content/full/99/4/585

Kliegman RM. Intrauterine growth restriction. In: Martin RJ, Fanaroff AA, Walsh MC, eds.Fanaroff and Martin’s Neonatal-Perinatal Medicine: Diseases of the Fetus and Infant. 8th ed.Philadelphia, Pa: Mosby-Elsevier; 2006:271-306

Linderkamp O. Blood viscosity of the neonate. NeoReviews. 2004;5:e406-e416. Available at:http://neoreviews.aappublications.org/cgi/content/full/5/10/e406

Thureen PJ, Anderson MS, Hay WW Jr. The small-for-gestational age infant NeoReviews.2001;2:e139-e149. Available at: http://neoreviews.aappublications.org/cgi/content/full/2/6/e139

Copyright © 2007 by the American Academy of Pediatrics page 107

Page 108: AAP MCQ 2007

2007 PREP SA on CD-ROM

Question: 52

A 12-month-old boy comes to the emergency department with a 3-day history of intractablevomiting and watery diarrhea. His mother reports decreased urine output for the past 24 hours.His heart rate is 180 beats/min, and his blood pressure is 85/40 mm Hg. He is lethargic butresponds to stimulation. His mucous membranes are very dry, his skin turgor is decreased, andhis capillary refill is 3 seconds. The remainder of his physical examination findings areunremarkable.

Of the following, the laboratory data that are MOST consistent with this patient's clinicalpresentation are

A. Serum Sodium: High; Serum Osmolality: High; Urine Sodium: Low; Urine Osmolality: Low

B. Serum Sodium: Low; Serum Osmolality: Low; Urine Sodium: Low; Urine Osmolality: High

C. Serum Sodium: Low; Serum Osmolality: Low; Urine Sodium: High; Urine Osmolality: High

D. Serum Sodium: Low; Serum Osmolality: Normal; Urine Sodium: High; Urine Osmolality: High

E. Serum Sodium: Normal; Serum Osmolality: Normal; Urine Sodium: Low; Urine Osmolality: Low

Copyright © 2007 by the American Academy of Pediatrics page 108

Page 109: AAP MCQ 2007

2007 PREP SA on CD-ROM

Preferred Response: BCritique: 52

Diarrhea due to acute gastroenteritis usually causes a proportional water and sodium loss,resulting in isonatremic dehydration. Hyponatremic dehydration also may be seen in acutegastroenteritis, especially if the only fluids tolerated are low-sodium fluids, such as water or juice.The clinical signs of both hyponatremic and isonatremic dehydration are due to decreasedextracellular fluid, primarily from the intravascular compartment. The signs include decreasedskin turgor, delayed capillary refill, decreased tear production and urine output, sunken eyes andanterior fontanelle, and tachycardia. Hypotension is a late finding, indicating decompensatedshock. Because the child described in the vignette displays these clinical signs, isonatremia orhyponatremia is likely. With hypernatremic dehydration, intravascular volume is relativelypreserved, despite an overall body water loss, so the degree of dehydration may beunderestimated.

Hyponatremia resulting from gastroenteritis is associated with a low serum osmolality. Incontrast, “factitious” hyponatremia may occur with high osmolar states such as hyperglycemiaor hyperlipidemia. In a healthy child who has gastroenteritis, renal sodium-preserving andconcentrating mechanisms remain intact, resulting in low urinary sodium concentrations andhigh urine osmolality, respectively. The findings of hyponatremia with normal-to-high urinesodium and urine osmolality values should alert the clinician to the possibility of the syndrome ofinappropriate antidiuretic hormone secretion, which is not likely in the child in the vignette.

References:

Constantinescu AR. Parenteral fluid therapy for infants and children. In: Rakel RE, Bope ET,eds. Conn’s Current Therapy. 57th ed. Philadelphia, Pa: Elsevier Saunders; 2005:716-718

Moritz ML, Ayus JC. Disorders of water metabolism in children: hyponatremia andhypernatremia. Pediatr Rev. 2002;23:371-380 Available at:http://pedsinreview.aappublications.org/cgi/content/full/23/11/371

Roberts KB. Fluid and electrolytes: parenteral fluid therapy. Pediatr Rev. 2001;22:380-387Available at: http://pedsinreview.aappublications.org/cgi/content/full/22/11/380

Copyright © 2007 by the American Academy of Pediatrics page 109

Page 110: AAP MCQ 2007

2007 PREP SA on CD-ROM

Question: 53

A 2-year-old boy presents with a history of vomiting and diarrhea for several days. Physicalexamination reveals lethargy, poor oral intake, tachycardia, dry mucous membranes, and poorskin turgor. He refuses to take oral fluids, so you decide to begin intravenous fluidadministration.

Of the following, the BEST next step in fluid management is to

A. administer a bolus of hypotonic fluids at 20 mL/kg

B. administer a bolus of isotonic fluid at 20 mL/kg

C. administer a bolus of 3% saline at 10 mL/kg

D. administer 5% dextrose and 0.25 normal saline with 20 mEq potassium chloride atmaintenance rate

E. await laboratory results before starting fluid therapy

Copyright © 2007 by the American Academy of Pediatrics page 110

Page 111: AAP MCQ 2007

2007 PREP SA on CD-ROM

Preferred Response: BCritique: 53

The child described in the vignette appears to be moderately to severely dehydrated, and hisrefusal to drink is somewhat ominous and implies depleted intravascular volume.

Initial therapy of a severely dehydrated child includes intravenous administration of isotonicfluids (generally normal saline) at 20 mL/kg, with prompt re-evaluation after 30 minutes to 1 hour.Initial therapy for mild-to-moderate dehydration may be accomplished at least as effectively, ifnot more so, with oral rehydration, but it is labor-intensive, requiring one-to-one parent or nursingpresence with the patient. After the initial intravenous fluid bolus, additional bolus fluidadministration may be required. Careful follow-up and clinical assessment is needed after eachbolus to determine if clinical improvement in hydration status has occurred and to preventvolume overload. Fluid therapy may need to be altered if there are signs of renal, cardiac, orrespiratory impairment.

Urine output, physical examination results, and improved alertness and willingness to drinkguide further fluid therapy. If the child is more awake and willing to drink after an intravenousbolus, subsequent therapy may include replacement of a calculated fluid deficit of 5% to 10% viaoral rehydration (50 to 100 mL/kg over 4 hours) or by continued administration of intravenousfluid.

In the past, oral rehydration involved isotonic fluids, and intravenous rehydration andmaintenance fluids often have been hypotonic, but some suggest that more hypotonic oral fluidsare better tolerated and that isotonic saline be continued as maintenance intravenous fluid insome cases.

Children who require intravenous rehydration should have serum electrolyte concentrationsdetermined; a contraction alkalosis may be followed by a reperfusion acidosis that transientlyworsens despite clinical improvement. Mildly dehydrated children who are managed with oralrehydration generally do not require measurement of serum electrolytes.

It is never appropriate to delay fluid resuscitation while awaiting laboratory results becausesuch results do not change initial management of volume depletion. However, interpretation ofelectrolyte values can guide further management once volume resuscitation is complete

It is never appropriate to use hypertonic saline (3% saline) as “initial” fluid resuscitation inthe dehydrated patient. Its use is limited to those who have severe hyponatremia unresponsiveto isotonic saline boluses or who need concomitant fluid restriction. Ideally, hypertonic salineshould be administered in an intensive care unit.

Because dehydrated patients may have acute renal failure, they are at risk for life-threatening hyperkalemia if potassium is included in their intravenous fluids. Therefore, it isimportant to assess renal function (urine output) before adding potassium to fluids in theseverely dehydrated individual who may have suffered a renal insult.

References:

Centers for Disease Control and Prevention. Managing acute gastroenteritis among children:oral rehydration, maintenance, and nutritional therapy. MMWR Recomm Rep. 2003;52(RR-16):1-16. Available at: http://www.cdc.gov/mmwr/preview/mmwrhtml/rr5216a1.htm

Duke T, Mathur A, Kukuruzovic R, McGuigan M. Hypotonic vs isotonic saline solutions forintravenous fluid management of acute infections. The Cochrane Database of SystematicReviews. 2003;3:CD004169. Available at:http://www.mrw.interscience.wiley.com/cochrane/clsysrev/articles/CD004169/frame.html

Finberg L. Dehydration in infancy and childhood. Pediatr Rev. 2002;23:277-282. Available at:http://pedsinreview.aappublications.org/cgi/content/full/23/8/277

Moritz ML, Ayus JC. Disorders of water metabolism: hyponatremia and hypernatremia. PediatrRev. 2002;23:371-380. Available at:http://pedsinreview.aappublications.org/cgi/content/full/23/11/371

Copyright © 2007 by the American Academy of Pediatrics page 111

Page 112: AAP MCQ 2007

2007 PREP SA on CD-ROM

Roberts KB. Fluids and electrolytes: parenteral fluid therapy. Pediatr Rev. 2001;22:380-387.Available at: http://pedsinreview.aappublications.org/cgi/content/full/22/11/380

Copyright © 2007 by the American Academy of Pediatrics page 112

Page 113: AAP MCQ 2007

2007 PREP SA on CD-ROM

Question: 54

A 4-year-old girl is brought to the emergency department by her babysitter because the childhas suddenly become clumsy, and her speech has become slurred over the last hour. Onphysical examination, the girl is afebrile and dysarthric. She has prominent vertical andhorizontal nystagmus, along with truncal and appendicular ataxia. Deep tendon reflexes arenormal, as are results of the remainder of the physical examination.

Of the following, the MOST likely diagnosis is

A. brain tumor

B. cerebellar hemorrhage

C. Guillain-Barré syndrome

D. meningoencephalitis

E. toxic ingestion

Copyright © 2007 by the American Academy of Pediatrics page 113

Page 114: AAP MCQ 2007

2007 PREP SA on CD-ROM

Preferred Response: ECritique: 54

Ataxia is a broad term implying simply dyscoordination. Although acute ataxia often is associatedwith cerebellar pathology, this disturbance also can arise from defects in proprioception,vestibular function, vision, or even weakness.

When evaluating the child who has acute ataxia, the clinician should consider categories ofdisease, including toxic ingestion (eg, alcohol, thallium used as a pesticide, and anticonvulsantssuch as phenytoin), neoplasm (eg, brainstem glioma, medulloblastoma), infection (labyrinthitis,postinfectious encephalomyelitis [also known as acute cerebellar ataxia], Guillain-Barrésyndrome), vasculopathy (cerebellar hemorrhage), trauma, metabolic conditions (eg, Hartnupdisease and abetalipoproteinemia), and conversion disorder.

Specific clues from the history and physical examination may point to a diagnosis. Forexample, fever suggests meningoencephalitis, and acute cerebellar ataxia follows a viralinfection, such as varicella or influenza. Deep tendon reflexes are depressed or absent inGuillain-Barré syndrome. Horizontal nystagmus is common with peripheral processes such aslabyrinthitis or even cerebellar hemisphere lesions or sedative-hypnotic ingestion. Verticalnystagmus can be seen with central processes such as hemorrhage or Chiari malformation, butalso is suggestive of phencyclidine, phenytoin, and lithium poisoning.

The development of clumsiness, dysarthria, and nystagmus over only hours described forthe child in the vignette suggests a very rapid process. The vertical plus horizontal nystagmus istypical of phenytoin ingestion. The lack of fever makes an infectious process unlikely. Somedegree of obtundation would be expected with cerebellar hemorrhage. The normal reflexes areinconsistent with Guillain-Barré syndrome. Ataxia with a brain tumor develops over days orweeks.

References:

Dinolfo E. Evaluation of ataxia. Pediatr Rev. 2001;22:177-178. Available at:http://pedsinreview.aappublications.org/cgi/content/full/22/5/177

Johnston MV. Movement disorders: ataxias. In: Behrman RE, Kliegman RM, Jenson HB, eds.Nelson Textbook of Pediatrics. 17th ed. Philadelphia, Pa: WB Saunders Co; 2004:2019-2023

MacDonald GP. Ataxia of childhood. In: Berg BO, ed. Child Neurology: A Clinical Manual. 2nd ed.Philadelphia, Pa: JB Lippincott Co; 1994:287-305

Copyright © 2007 by the American Academy of Pediatrics page 114

Page 115: AAP MCQ 2007

2007 PREP SA on CD-ROM

Question: 55

You are examining a 4-year-old child who has just undergone tonsillectomy and adenoidectomyfor obstructive sleep apnea syndrome. He has just been extubated. While talking with hisparents, you notice that his breathing is becoming more labored. On chest auscultation, youhear equal but poor air entry, no wheezes, and some crackles bilaterally. There are nomurmurs, but a loud and palpable second heart sound is evident. You increase the oxygen leveland reintubate the patient. After intubation, you obtain a chest radiograph.

Of the following, the MOST likely finding on the chest radiograph is

A. bilateral perihilar infiltrates

B. hemothorax

C. normal pulmonary findings

D. pulmonary edema

E. tension pneumothorax

Copyright © 2007 by the American Academy of Pediatrics page 115

Page 116: AAP MCQ 2007

2007 PREP SA on CD-ROM

Preferred Response: DCritique: 55

Although most children who have obstructive sleep apnea syndrome (OSAS) do well withsurgical treatment, respiratory compromise is a well-recognized adverse outcome of anesthesiaand surgery in affected patients. Therefore, all children who have a history of OSAS shouldreceive a comprehensive preanesthesia evaluation. Complications of tonsillectomy andadenoidectomy in children include hemorrhage, dehydration, and velopharyngeal insufficiency.Although each of these is reported to occur with a frequency of 3% or less, they are the mostcommon complications reported. Respiratory complications can include laryngospasm, apnea,pulmonary hypertension, pulmonary hypertensive crisis, and pneumonia. Children who haveOSAS are especially likely to have respiratory complications after surgery.

A subgroup of patients who have OSAS and are undergoing surgery should receiveparticularly intensive perioperative management as well as inpatient monitoring following theprocedure. These include patients younger than 3 years of age; those who have medicalcomorbidities, neuromuscular disease, bleeding diatheses, or syndromes such as trisomy 21;and those who have severe OSAS demonstrated on polysomnography. This latter group ofpatients may be at particular risk for development of the rare complication of postobstructivepulmonary edema. This entity should be suspected in any patient who has undergone surgicaltherapy for OSAS and demonstrates respiratory distress in the postoperative period, such asthe boy described in the vignette. The diagnosis is confirmed with a chest radiograph thatdemonstrates pulmonary edema. Although bilateral infiltrates and hemo- or pneumothorax mayoccur following certain procedures, the clinical findings of the child in the vignette, along with thehistory of OSAS, are most consistent with the development of acute pulmonary edema.

References:

Bandla P, Brooks LJ, Trimarchi T, Helfaer M. Obstructive sleep apnea syndrome in children.Anesthesiol Clin North Am. 2005;23:535-549. Abstract available at:http://www.ncbi.nlm.nih.gov/entrez/query.fcgi?orig_db=PubMed&db=PubMed&cmd=Search&term=%22Anesthesiology+clinics+of+North+America%22[Jour]+AND+2005[pdat]+AND+Bandla+P[author]

Chan J, Edman JC, Koltai PJ. Obstructive sleep apnea in children. Am Fam Physician.2004;69:1147-1154. Available at: http://www.aafp.org/afp/20040301/1147.html

Copyright © 2007 by the American Academy of Pediatrics page 116

Page 117: AAP MCQ 2007

2007 PREP SA on CD-ROM

Question: 56

Physical examination of a newborn female reveals meningomyelocele; dysmorphic features,including a narrow bifrontal diameter, epicanthal folds, a broad and low nasal bridge, and midfacehypoplasia; and a short systolic murmur. Results of echocardiography document coarctation ofthe aortic arch.

Of the following, the MOST likely prenatal exposure to explain these findings is

A. alcohol

B. lithium

C. retinoic acid

D. thiazide diuretic

E. valproic acid

Copyright © 2007 by the American Academy of Pediatrics page 117

Page 118: AAP MCQ 2007

2007 PREP SA on CD-ROM

Preferred Response: ECritique: 56

The use of anticonvulsants during pregnancy is associated with an increased risk for fetalanomalies. In particular, three syndromes have been delineated in association with the maternaluse of valproic acid, trimethadione, and hydantoin, respectively. Mothers should be informed ofthe possible consequences to the fetus of anticonvulsant therapy during pregnancy. However,the potential harmful consequences of uncontrolled seizures for both the mother and the fetusalso must be considered before making any specific recommendations for discontinuation ofanticonvulsant therapy during pregnancy.

Fetal valproate syndrome (Item C56A) is a pattern of malformations characterized bycraniofacial anomalies that include a narrow bifrontal diameter, high forehead, epicanthal folds,broad and low nasal bridge, midfacial hypoplasia, long philtrum, and small mouth. Exposedinfants, such as the one described in the vignette, also are at increased risk for cardiovascularanomalies, cleft lip, and meningomyelocele. The risk for meningomyelocele has been reported tobe as high as 5%, and approximately 30% of exposed infants have some features of thesyndrome.

Fetal trimethadione syndrome results from exposure to this anticonvulsant during the firsttrimester and is characterized by mental deficiency, prenatal-onset growth deficiency, andcraniofacial abnormalities, including a short upturned nose, low nasal bridge, prominentforehead, synophrys (fusion of the eyebrows), and malformed ears. Cardiovascular defects,ambiguous genitalia, and cleft lip with/without cleft palate also have been reported. Most infantsexposed to trimethadione display one or more of these features, and the prognosis is poor insome because of the combination of serious heart defects and mental retardation.

The fetal hydantoin syndrome is characterized by digit and nail hypoplasia (Item C56B), anunusual facies, and mental deficiency. Growth deficiency has been observed both prenatallyand postnatally. Approximately 10% of prenatal exposures to hydantoin result in the entiresyndrome, with another 33% producing some of the effects. Prenatal exposure to otheranticonvulsants, including carbamazepine and phenobarbital, also has been reported to result infacial features similar to those observed in hydantoin syndrome.

Prenatal exposure to alcohol may result in the fetal alcohol syndrome, which ischaracterized by prenatal onset of growth deficiency, microcephaly, a typical facies (ItemC56C), and cardiac septal defects. However, an increased incidence of meningomyelocele isnot characteristic. Although the use of diuretics in pregnancy is not encouraged, theadministration of thiazide diuretics during the first trimester is associated only minimally with anincreased incidence of birth defects, and in later trimesters, there is no strong association.

Prenatal exposure to lithium poses an increased risk for congenital heart defects, includingEbstein anomaly, but an increase in myelomeningocele and craniofacial abnormalities is notreported. Isotretinoin exposure is characterized by central nervous system defects (eg,hydrocephalus, microcephaly, structural errors of neuronal migration), but notmeningomyelocele. Other associated abnormalities include facial asymmetry, microtia or anotia,conotruncal cardiac abnormalities, and mental deficiency. The risk for retinoic acid embryopathyis 35% in women who continue to ingest the agent past the 15th day following conception.

References:

Artama M, Auvinen A, Raudaskoski T, Isojarvi I, Isojarvi J. Antiepileptic drug use of women withepilepsy and congenital malformations in offspring. Neurology. 2005;64:1874-1878. Abstractavailable at:http://www.ncbi.nlm.nih.gov/entrez/query.fcgi?orig_db=PubMed&db=PubMed&cmd=Search&term=%22Neurology%22[Jour]+AND+1874[page]+AND+2005[pdat]

Holmes LB, Harvey EA, Coull BA, et al. The teratogenicity of anticonvulsant drugs. N Engl JMed. 2001;344:1132-1138. Abstract available at:http://www.ncbi.nlm.nih.gov/entrez/query.fcgi?cmd=Retrieve&db=pubmed&dopt=Abstract&list_uids=11297704&query_hl=1&itool=pubmed_docsum

Copyright © 2007 by the American Academy of Pediatrics page 118

Page 119: AAP MCQ 2007

2007 PREP SA on CD-ROM

Question: 57

A 16-year-old girl presents with the complaint of right upper quadrant pain, right shoulder pain,and nausea. She is afebrile. She has been taking combined oral contraceptive pills for 6 months.Results of ultrasonography of the gallbladder performed after an emergency department visit 2days ago are normal.

Of the following, the most appropriate NEXT step in the evaluation of this patient's pain is

A. computed tomography scan of the liver

B. hepatobiliary scintography

C. hydrogen breath test

D. pelvic examination

E. ultrasonography of the abdomen

Copyright © 2007 by the American Academy of Pediatrics page 119

Page 120: AAP MCQ 2007

2007 PREP SA on CD-ROM

Preferred Response: DCritique: 57

Fitz-Hugh-Curtis syndrome, a perihepatitis, is a complication of pelvic inflammatory disease(PID) associated with chlamydial or, less commonly, gonococcal infections. The perihepatitisinvolves the liver capsule and surrounding peritoneum and presents as right upper quadrantpain, as described for the girl in the vignette. The diagnosis of Fitz-Hugh-Curtis syndrome initiallymay be overlooked because of the absence of associated lower abdominal symptoms. Rightupper quadrant pain coupled with a high index of suspicion, evidence of PID, and lack ofevidence for hepatitis, gallbladder disease, or pancreatitis form the basis for diagnosing Fitz-Hugh-Curtis syndrome. For the patient in the vignette, the most appropriate next step inevaluation of the pain is a pelvic examination for signs of salpingitis or PID (eg, cervical motiontenderness, adnexal tenderness, vaginal discharge).

Although rarely necessary, in challenging cases, laparoscopic visualization of the liver canprovide a definitive diagnosis of Fitz-Hugh-Curtis syndrome. Grayish granular exudates on theanterior surfaces of the liver and adjacent peritoneum are characteristic of an acuteinflammatory process, and “violin stringlike” adhesions are characteristic of a chronic responseassociated with Fitz-Hugh-Curtis syndrome. Treatment is with the same regimensrecommended for PID, and a significant improvement of the right upper quadrant pain isexpected within 24 to 48 hours.

A hydrogen breath test is particularly useful for the evaluation of lactose intolerance inadolescents who have recurrent abdominal pain. Computed tomography scan of the liver isuseful for detecting discrete lesions such as tumors, abscesses, and cysts, which are not likelycauses of the adolescent’s pain. Hepatobiliary scintography (study of radionuclide uptake in theliver and excretion in the bile) is particularly helpful for differentiating intrahepatic cholestasisfrom extrahepatic obstruction in infants. After a pelvic examination has established the absenceof signs or symptoms of PID, ultrasonography of the abdomen often is indicated to investigateother causes of right upper quadrant pain (subphrenic abscess, renal stones, and appendicitis).

References:

Banikarim C, Chacko MR. Pelvic inflammatory disease in adolescents. Adolesc Med Clin.2004;15:273-285

Bortot AT, Risser WL, Cromwell PF. Coping with pelvic inflammatory disease in the adolescent.Contemp Pediatr. 2004;21:33-48. Available at:http://www.contemporarypediatrics.com/contpeds/article/articleDetail.jsp?id=108024

Darville T. Neisseria gonorrhoeae (gonococcus). In: Behrman RE, Kleigman RM, Jenson HB,eds. Nelson Textbook of Pediatrics. 17th ed. Philadelphia, Pa: Saunders; 2004:899-904

Pletcher JR, Slap GB. Pelvic inflammatory disease. In: Neinstein LS, ed. Adolescent Health CarePractice Guide. 4th ed. Philadelphia, Pa: Lippincott, Williams & Wilkins; 2002:1161-1170

Shrier LA. Bacterial sexually transmitted infections: gonorrhea, chlamydia, pelvic inflammatorydisease, and syphilis. In: Emans SJH, Laufer MR, Goldstein DP, eds. Pediatric and AdolescentGynecology. 5th ed. Philadelphia, Pa: Lippincott, Williams & Wilkins; 2005:565-614

Copyright © 2007 by the American Academy of Pediatrics page 120

Page 121: AAP MCQ 2007

2007 PREP SA on CD-ROM

Question: 58

A normal-appearing 9-month-old boy is discovered on routine herniorrhaphy to have bilateralFallopian tubes and a rudimentary uterus. Biopsy of the gonads performed during this procedurerevealed normal testicular tissue. On examination today, his phallus is normal in size andappearance, and his descended testes are both 2 mL in volume.

Of the following, the MOST likely sex chromosome complement for this child is

A. XX

B. XX/XY

C. XXY

D. XO/XY

E. XY

Copyright © 2007 by the American Academy of Pediatrics page 121

Page 122: AAP MCQ 2007

2007 PREP SA on CD-ROM

Preferred Response: ECritique: 58

The boy described in the vignette, who has normal descended testes and a normal penis and isdiscovered to have fallopian tubes and a rudimentary uterus, has the persistent müllerian ductsyndrome. A peptide hormone (müllerian inhibiting substance [MIS] or anti-müllerian hormone[AMH]) produced by the testis induces regression of the müllerian ducts during fetaldevelopment. These ducts become the fallopian tubes and uterus as well as the upper part ofthe vagina if there is no regression. Genetic defects that lead to failure of production of MIS orlack of receptors for this hormone are responsible for failure of regression of müllerian tissues inotherwise normal males. This is an autosomal recessive disorder and is associated with an XYchromosomal pattern.

The SRY gene is the male-determining gene on the Y chromosome. Children who have anXX chromosome pattern rarely present with sex reversal (male phenotype) because of atranslocation of the SRY gene to an X chromosome or an autosome. Affected children havegonads that generally are composed of testicular tissue and should not have müllerianremnants. Children who have an XX/XY chromosomal pattern usually have external genitalia thatreflect true intersex, and gonads contain a variable mix of testicular or ovarian tissue that maydiffer in each gonad. Unilateral müllerian remnants may be present if mostly ovarian tissue isfound on that side. Individuals who have the XXY complement have Klinefelter syndrome. Theyhave normal male external and internal sexual development at birth, although testes and phallusmay be somewhat small. As they grow, they express components of Klinefelter syndrome ofvariable severity, ranging from late-onset infertility to severe behavioral and neurodevelopmentalproblems. Children who have an XO/XY chromosomal pattern usually have normal maledifferentiation but may have somewhat small phalluses. They may have infertility and shortstature. Some affected children have “mixed gonadal dysgenesis” that results in ambiguousgenitalia and some manifestations of Turner syndrome. The presence of Y chromosomalmaterial in a dysgenetic gonad increases the risk for malignancy within the gonad, although therisk is lower than previously believed.

References:

Houk CP, Levitsky LL. Evaluation of the infant with ambiguous genitalia. UptoDate. 2006;14.1.Available at:http://www.utdol.com/utd/content/topic.do?topicKey=pediendo/11554&type=P&selectedTitle=14~24

Houk CP, Levitsky LL. Management of the infant with ambiguous genitalia. UptoDate. 2006;14.1.Available at:http://www.utdol.com/utd/content/topic.do?topicKey=pediendo/12719&type=P&selectedTitle=15~24

Josso N, Belville C, di Clemente N, Picard JY. AMH and AMH receptor defects in persistentMüllerian duct syndrome. Hum Reprod Update. 2005;11:351-356. Abstract available at:http://www.ncbi.nlm.nih.gov/entrez/query.fcgi?orig_db=PubMed&db=PubMed&cmd=Search&term=%22Human+reproduction+update%22[Jour]+AND+351[page]+AND+2005[pdat]

Copyright © 2007 by the American Academy of Pediatrics page 122

Page 123: AAP MCQ 2007

2007 PREP SA on CD-ROM

Question: 59

An 8-year-old boy has difficulty with reading; he reads slowly and makes many mistakes. Hehas a history of a febrile seizure at 1 year of age and a fall at age 2 years that resulted in a briefloss of consciousness. His mother read an article about interventions for improving visionfunction in children who have reading difficulties and would like your opinion. Findings on hisphysical examination are normal, except for vision of 20/40 bilaterally.

Of the following, your BEST response is that you would like to

A. order brain magnetic resonance imaging

B. order electroencephalography

C. refer him for optometric evaluation

D. refer him for a functional vision assessment

E. request a psychoeducational evaluation

Copyright © 2007 by the American Academy of Pediatrics page 123

Page 124: AAP MCQ 2007

2007 PREP SA on CD-ROM

Preferred Response: ECritique: 59

The child described in the vignette has symptoms that are concerning for a reading learningdisability. The most helpful intervention is further evaluation of his academic difficulty byrequesting a psychoeducational evaluation. Reading is a complicated endeavor that involves theintegration of multiple neurologic pathways. Although vision is necessary for reading, complexvisual processing facilitates reading and comprehension of written language. Children who havecommon ophthalmologic disorders may have academic difficulty only when there is an inability tosee adequately what is written on the page. Mild or correctable vision deficits rarely causereading or academic problems. Of note, there is no evidence to support the value of eyeexercises or the use of special tinted lenses as therapy for reading difficulties.

Brain magnetic resonance imaging and electroencephalography are unlikely to provideuseful information in a child who has normal findings on physical examination and a single febrileseizure. He should receive routine eye evaluations, but given the near-normal visiondocumented on your office screening, he is unlikely to have a vision disorder that is causing hisreading difficulty. For children who have vision loss, a functional vision assessment, whichevaluates how the child uses vision to perform routine tasks, can be useful.

References:

Committee on Children With Learning Disabilities American Academy of Pediatrics (AAP), andAmerican Academy of Ophthalmology (AAO), and American Association for PediatricOphthalmology and Strabismus (AAPOS). Learning disabilities, dyslexia, and vision: a subjectreview. Pediatrics. 1998;102:1217-1219. Available at:http://pediatrics.aappublications.org/cgi/content/full/102/5/1217

Galaburda AM, Duchaine BC. Developmental disorders of vision. Neurol Clin. 2003;2:687-707

Olitsky SE, Nelson LB. Reading disorders in children. Pediatr Clin North Am. 2003;50:213-224

Copyright © 2007 by the American Academy of Pediatrics page 124

Page 125: AAP MCQ 2007

2007 PREP SA on CD-ROM

Question: 60

A 4-year-old boy presents to your clinic for a second opinion. He has a 3-week history ofdiarrhea, abdominal pain, and tenesmus. The parents state that he seems to be getting worse,and nobody has been able to help them despite "a bunch of tests on his poop." His stool outputhas increased from four to five per day to eight to ten per day during the past week, and he nowhas a temperature of 102°F (38.9°C). They are starting to see what appears to be blood in thetoilet after he goes to the bathroom. According to the parents, the boy was in good health until 1week after they returned from a fishing trip on the Amazon river. Physical examination reveals amoderately ill-appearing boy who has diffuse abdominal pain. During your examination, hepasses a very foul-smelling stool that appears to be a mixture of blood and pus, which you sendto the laboratory for analysis.

Of the following, the MOST appropriate next test is

A. abdominal ultrasonography

B. barium enema

C. colonic biopsy

D. gallium scan

E. liver function test

Copyright © 2007 by the American Academy of Pediatrics page 125

Page 126: AAP MCQ 2007

2007 PREP SA on CD-ROM

Preferred Response: ACritique: 60

The boy described in the vignette has a gastrointestinal infection caused by Entamoebahistolytica. The clinical presentation of amebiasis may range from asymptomatic cyst sheddingto intestinal amebiasis to extraintestinal disease. Intestinal symptoms may occur within 2 weeksof infection and present with colicky abdominal pain and diarrhea. Patients who have diarrheacomplain of tenesmus, and red and white blood cells often are present in the stool. Amebic colitisis especially common in children younger than 5 years of age and can produce toxic megacolon,fulminant colitis, ulceration of the colon, and rarely, bowel perforation. Occasionally, a chronicform of amebic colitis may occur that can mimic inflammatory bowel disease, and amebomasmay develop. This annular lesion may be mistaken for changes associated with inflammatorybowel disease, a pyogenic abscess, or a colonic carcinoma. Such lesions usually respond toantiamebic therapy. Disseminated amebiasis also may occur, and a very serious form causesliver abscess. Because the patient in the vignette is very ill and presents with diffuse abdominalpain, abdominal ultrasonography is indicated to evaluate for a liver abscess (Item C60A).

Amebiasis can be diagnosed by identifying trophozoites or cysts in the stool. Detection ofserum antibody titers also may aid in diagnosing amebic colitis or extraintestinal disease. Colonicbiopsy may demonstrate amebic organisms but is not the most appropriate test at this stage ofillness. Liver function test results often are normal in the presence of liver abscesses becauseabscess formation does not acutely destroy liver cells. A gallium scan may help find a liverabscess, but ultrasonography is an easier and more cost-effective diagnostic tool. A bariumenema is not indicated in a patient who has colitis due to the risk of developing toxic megacolonor perforation.

Patients who have amebic disease require therapy to treat both the intraluminal and tissue-invading organisms. For asymptomatic patients who excrete cysts (intraluminal disease),paromomycin, iodoquinol, or diloxanide furoate may be used; patients who have all other formsshould receive metronidazole followed by iodoquinol or paromomycin.

References:

American Academy of Pediatrics. Amebiasis. In: Pickering LK, ed. Red Book: 2006 Report of theCommittee on Infectious Diseases. 27th ed. Elk Grove Village, Ill: American Academy ofPediatrics; 2006:204-206

Yost J. In brief: amebiasis. Pediatr Rev 2002;23:293-294. Available at:http://pedsinreview.aappublications.org/cgi/content/full/23/8/293

Copyright © 2007 by the American Academy of Pediatrics page 126

Page 127: AAP MCQ 2007

2007 PREP SA on CD-ROM

Question: 61

A 12-year-old girl presents to the emergency department with nausea, vomiting, and abdominalpain of 1 month's duration. Physical examination reveals a large, smooth mass encompassingalmost the entire lower abdomen. Computed tomography scan confirms a mass, and biopsydocuments Burkitt lymphoma. She immediately begins receiving chemotherapy, and 12 hourslater she develops the classic electrolyte and urinary findings consistent with tumor lysissyndrome (TLS).

Of the following, the laboratory findings MOST consistent with TLS are

A. Serum Potassium: Elevated; Serum Phosphorous: Elevated; Serum lactate dehydrogenase:Normal; Serum sodium: Elevated

B. Serum Potassium: Elevated; Serum Phosphorous: Normal; Serum lactate dehydrogenase:Elevated; Serum sodium: Normal

C. Serum Potassium: Normal; Serum Phosphorous: Elevated; Serum lactate dehydrogenase:Elevated; Serum sodium: Elevated

D. Serum Potassium: Normal; Serum Phosphorous: Normal; Serum lactate dehydrogenase:Elevated; Serum sodium: Normal

E. Serum Potassium: Elevated; Serum Phosphorous: Elevated; Serum lactate dehydrogenase:Elevated; Serum sodium: Normal

Copyright © 2007 by the American Academy of Pediatrics page 127

Page 128: AAP MCQ 2007

2007 PREP SA on CD-ROM

Preferred Response: ECritique: 61

Tumor lysis syndrome (TLS) is caused by any oncologic condition in which the cancer cells arebeing destroyed rapidly. Various intracellular substances are released into the bloodstream (eg,potassium, lactate dehydrogenase, phosphorous), resulting in significantly elevated levels ofthese electrolytes in the serum. Because only intracellular ions are affected in TLS, serumsodium concentrations should be normal. The kidney is primarily responsible for eliminating theexcess electrolytes from the body but may become overwhelmed and unable to removedangerous quantities of various substances with sufficient speed. In addition, some substances,such as uric acid and phosphorous, may be deposited in the kidney and cause tubularobstruction and damage. If this occurs, the body is deprived of the primary organ required tomaintain electrolyte homeostasis.

The girl described in the vignette has a Burkitt lymphoma, which is a generally very largetumor that contains an abundance of cells. Once chemotherapy is initiated, the tumor cellstypically are very sensitive to the therapy and destroyed rapidly, resulting in hyperkalemia,hyperphosphatemia, and elevated serum lactate dehydrogenase. The kidney excretes massivequantities of various substances until its capacity is overwhelmed. Other oncologic conditions,such as acute lymphoblastic leukemia, especially when it is associated with exceedingly highwhite blood cell counts (>50x103/mcL [50x109/L]) also may cause TLS.

Therapy for TLS includes aggressive hydration (at least twice maintenance fluid),alkalinization of the urine (to offset the deposition of uric acid in the kidney), and diuretic therapy(to promote further urine flow). Therapy to reduce the serum concentrations of uric acid witheither allopurinol or rasburicase generally is provided early in the course of chemotherapy topatients at risk for TLS. Finally, if significant electrolyte abnormalities persist or acute renalfailure develops, continuous renal replacement therapy is indicated.

References:

Cairo MS, Bishop M. Tumour lysis syndrome: new therapeutic strategies and classification. Br JHaematol. 2004;127:3-11. Abstract available at:http://www.ncbi.nlm.nih.gov/entrez/query.fcgi?cmd=Retrieve&db=pubmed&dopt=Abstract&list_uids=15384972&query_hl=31&itool=pubmed_docsum

Lameire NH, Flombaum CD, Moreau D, Ronco C. Acute renal failure in cancer patients. AnnMed. 2005;37:13-25. Abstract available at:http://www.ncbi.nlm.nih.gov/entrez/query.fcgi?orig_db=PubMed&db=PubMed&cmd=Search&term=%22Annals+of+medicine%22[Jour]+AND+13[page]+AND+2005[pdat]

Olgar S, Yetgin S, Cetin M, Aras T, Akhan O. Electrolyte abnormalities at diagnosis of acutelymphocytic leukemia may be a clue for renal damage in long-term period. J Pediatr HematolOncol. 2005;27:202-206. Abstract available at:http://www.ncbi.nlm.nih.gov/entrez/query.fcgi?orig_db=PubMed&db=PubMed&cmd=Search&term=%22Journal+of+pediatric+hematology/oncology+:+official+journal+of+the+American+Society+of+Pediatric+Hematology/Oncology%22[Jour]+AND+202[page]+AND+2005[pdat]

Copyright © 2007 by the American Academy of Pediatrics page 128

Page 129: AAP MCQ 2007

2007 PREP SA on CD-ROM

Question: 62

The mother of a child who is infected with human immunodeficiency virus (HIV) would like toenroll her child in a local child care center.

Of the following, the circumstance that is MOST likely to exclude the child who has HIV infectionfrom attending a child care center is

A. a child who exhibits aggressive behavior such as biting and scratching

B. a child who has a history of occasional nose bleeds

C. a child who is not yet toilet trained

D. no circumstance of exclusion

E. the parent(s) or guardian who does not want to disclose the HIV status of the child

Copyright © 2007 by the American Academy of Pediatrics page 129

Page 130: AAP MCQ 2007

2007 PREP SA on CD-ROM

Preferred Response: ACritique: 62

Other than blood exposure, human immunodeficiency virus (HIV) has not been transmittedthrough the types of daily contact that usually occur in child care centers or schools, includingcontact with saliva or tears. Therefore, children who have HIV infection generally should not beexcluded from child care or school to protect personnel or other children. Some specificcircumstances, however, dictate against HIV-infected children attending child care centers orschools. These include a child who has uncovered exudative skin lesions or uncontrolledaggressive behavior (eg, biting or scratching) or the desire to minimize exposure of the HIV-infected child to contagious illnesses. A history of mild eczema or occasional nose bleeds is nota reason to exclude an infected child from child care or school. Exposure to stool or urine is nota route of HIV transmission and, therefore, lack of toilet training is not a reason for exclusion.Disclosure of HIV infection to the child care center or school is not required.

In general, to minimize possible transmission of any bloodborne disease, all child carecenters and schools should follow the recommended standard precautions to avoid contact withblood or other body fluids from any child or staff member. Such procedures include: not sharingpersonal items that might be in contact with blood (eg, toothbrush) and promptly disinfectingspills of blood or body fluids (eg, vomitus, urine, or feces) with a commercial cleaner or dilutedbleach (1:10). These protocols should be applied universally because unidentified HIV infectionsmay be present.

References:

American Academy of Pediatrics. Human immunodeficiency virus infection. In: Pickering LK, ed.Red Book: 2006 Report of the Committee on Infectious Diseases. 27th ed. Elk Grove Village, Ill:American Academy of Pediatrics; 2006:378-401

American Academy of Pediatrics Committee on Pediatric AIDS. Education of children with humanimmunodeficiency virus infection. Pediatrics. 2000;105:1358-1360. Available at:http://pediatrics.aappublications.org/cgi/content/full/105/6/1358

Copyright © 2007 by the American Academy of Pediatrics page 130

Page 131: AAP MCQ 2007

2007 PREP SA on CD-ROM

Question: 63

As part of your clinic responsibilities, you supervise allergy shots for patients who have allergicrhinitis. The nurse calls you about a 12-year-old girl who received her allergy shot 30 minutesago and now is experiencing warmth and erythema over the injection site. On physicalexamination, the patient appears healthy and in no distress. Vital signs include a temperature of97.5°F (36.4°C), heart rate of 90 beats/min, and respiratory rate of 18 breaths/min. At theinjection site, there is a 4 x 4 cm raised, erythematous, warm area (Item Q63A). There is noedema of the tongue or uvula, and findings on her pulmonary, cardiovascular, and skinexaminations are otherwise normal.

Of the following, the NEXT most appropriate action is to

A. administer an oral antibiotic

B. administer intramuscular epinephrine

C. allow the patient to leave the clinic

D. discontinue allergy shots

E. observe the patient for an additional 30 minutes

Copyright © 2007 by the American Academy of Pediatrics page 131

Page 132: AAP MCQ 2007

2007 PREP SA on CD-ROM

Preferred Response: CCritique: 63

There are three general levels of management for allergic rhinitis: avoidance, pharmacologicmanagement, and allergen immunotherapy (allergy shots). Immunotherapy can be considered inchildren ages 5 years and older who have significant allergic disease that is refractory toavoidance and pharmacologic therapy or who experience adverse effects from medications.

During immunotherapy, a number of safety recommendations should be followed, includingadministration of the immunotherapy in a clinic, a mandatory wait time of 20 to 30 minutes afteran allergy shot, prompt recognition of systemic reactions, and avoidance of allergy shots inchildren receiving beta-blockers or who have unstable asthma. The two primary complicationsfrom allergy shots, not including pain from the injection, are local reactions and anaphylaxis.

Local reactions (Item C63A), as described for the girl in the vignette, are common and occurat the injection site. These reactions are self-limited, spontaneously improve within 4 to 72 hours,and do not require any specific therapy. Patients experiencing local reactions in the absence ofother systemic symptoms should be allowed to leave the clinic. Because local reactionsrepresent localized immunoglobulin E-mediated reactions and not an infection, administration ofan oral antibiotic or epinephrine is not recommended. Some experts recommend observingpatients who have asthma or who have experienced previous anaphylaxis for 30 to 60 minutesafter an allergy injection. In the absence of systemic symptoms or previous reactions for the girlin the vignette, observation past the recommended 20 to 30 minutes is not indicated.Discontinuation of allergy shots may be warranted for patients who experience repeatedepisodes of anaphylaxis, but generally is not required for local reactions.

Systemic anaphylaxis is the primary concern during allergen immunotherapy and occurs inapproximately 1 in 200 injections. Most reactions occur during the first year of buildup, duringpeak pollen seasons, and begin within 30 minutes after an allergy shot. Common symptomsinclude generalized pruritus, hives, angioedema, sneezing fits, ocular erythema, difficultybreathing, and lightheadedness. In general, prompt administration of intramuscular epinephrineand supportive care are recommended.

References:

Atkins D, Leung DYM. Principles of treatment of allergic disease. In: Behrman RE, Kliegman RM,Jenson HB, eds. Nelson Textbook of Pediatrics. 17th ed. Philadelphia, Pa: WB Saunders Co2004:752-758

Joint Task Force on Practice Parameters. Allergen immunotherapy: a practice parameter.American Academy of Allergy, Asthma and Immunology, American College of Allergy, Asthmaand Immunology. Ann Allergy Asthma Immunol. 2003;90:1-40

Copyright © 2007 by the American Academy of Pediatrics page 132

Page 133: AAP MCQ 2007

2007 PREP SA on CD-ROM

Question: 64

A 3-year-old boy who was involved in a motor vehicle crash sustained a significant head injuryand had a Glasgow Coma Scale score of 8 on arrival in the emergency department. Anendotracheal tube was placed shortly after his arrival, and the respiratory therapist is providingventilation at 12 breaths/min with 100% oxygen via a bag-valve mask until a ventilator can bebrought to the emergency department. Ten minutes later, the child becomes more restless andagitated. His heart rate has increased from an initial 110 beats/min to about 150 beats/min, andhe appears flushed. The monitor indicates that his oxygen saturation is 96%.

Of the following, a TRUE statement about this child's ventilatory status is that

A. a patient who has acute carbon dioxide retention due to hypoventilation still can have near-normal blood oxygen saturation

B. an arterial blood gas determination likely would reveal a pH of approximately 7.45

C. his agitation is due to relative hypoxemia

D. his agitation is not related to his ventilatory status; it is due to tracheal irritation from theendotracheal tube

E. the oxygen saturation of 96% indicates that he is being ventilated adequately

Copyright © 2007 by the American Academy of Pediatrics page 133

Page 134: AAP MCQ 2007

2007 PREP SA on CD-ROM

Preferred Response: ACritique: 64

The child described in the vignette exhibits signs of acute hypercapnia, or carbon dioxideretention, likely due to the relatively low number of ventilations that are being provided perminute. The observed near-normal oxygen saturation is a reflection of the high inspired oxygenconcentration. Even minimal oxygen therapy can maintain normal oxygenation in patients whoare experiencing severe hypoventilation. For this reason, an oxygen saturation monitor is not anaccurate means to assess the adequacy of ventilation.

Hypoventilation leading to hypercapnia results in a pH well below 7.4 due to carbon dioxideretention. The oxygen saturation of 96% reported for the child in the vignette is not consistentwith hypoxemia.

Restlessness due to tracheal irritation is a possibility, especially if a patient has not receivedadequate sedation prior to beginning mechanical ventilation. However, the presence of flushing,one of the clinical manifestations of hypercapnia, is the best clue to the correct diagnosis. Theadditional manifestations of tachycardia and agitation could be observed with either condition.

References:

American Heart Association. Airway, ventilation, and management of respiratory distress andfailure. In: PALS Provider Manual. Dallas, Tex: American Heart Association; 2002:81-126

Pope J, McBride J. Consultation with the specialist: respiratory failure in children. Pediatr Rev.2004;25:160-167. Available at: http://pedsinreview.aappublications.org/cgi/content/full/25/5/160

Copyright © 2007 by the American Academy of Pediatrics page 134

Page 135: AAP MCQ 2007

2007 PREP SA on CD-ROM

Question: 65

An adolescent male presents with a hypopigmented eruption (Item Q65A) composed of multipleoval macules arrayed across the upper trunk in a yoke distribution.

Of the following, the PREFERRED management course consists of

A. administration of oral griseofulvin

B. application of psoralen followed by ultraviolet radiation

C. avoidance of the offending allergen

D. topical application of moderately potent corticosteroid

E. topical application of selenium sulfide

Copyright © 2007 by the American Academy of Pediatrics page 135

Page 136: AAP MCQ 2007

2007 PREP SA on CD-ROM

Preferred Response: ECritique: 65

Tinea versicolor typically manifests as multiple round or oval macules with superficial scalearrayed in a yoke distribution that includes the neck, chest, upper back, and shoulders, asdescribed for the boy in the vignette. Commonly, individual macules coalesce to form patches.Lesions may be hypo- (Item C65A) or hyperpigmented (Item C65B), hence, the nameversicolor. The infection occurs primarily in adolescents and young adults, perhaps because theorganism responsible, Malassezia furfur (Pityrosporum ovale) uses sebum as a nutrient. Theinfection is most prevalent in hot, humid climates. A potassium hydroxide preparation performedon scale obtained from the lesions reveals the classic “spaghetti and meatballs” appearance (ie,short hyphae and spores) (Item C65C).

Initial therapy of tinea versicolor consists of topical application of selenium sulfide 1%(present in some antiseborrheic shampoos) or 2.5% (available by prescription). Although manytreatment regimens exist, a commonly employed one advises the application of selenium sulfideto all affected areas for 10 minutes, after which it is rinsed off. An alternative is topicalketoconazole, although this drug typically is more expensive. For patients who have resistantinfection or who cannot use topical therapy, treatment with oral fluconazole or ketoconazole maybe beneficial. Regardless of the initial treatment employed, infection often returns. For thisreason, following initial treatment, topical selenium sulfide often is used prophylactically (ie, as asingle 8- to 12-hour application monthly for 3 to 4 months). Patients should be counseled thatreturn of normal skin pigmentation often requires several months.

Topical administration of psoralen compounds followed by gradually increasing exposure toultraviolet radiation may be indicated in the management of vitiligo (characterized bydepigmented [not hypopigmented] macules and patches), but it does not have a role in thetreatment of tinea versicolor. Postinflammatory hypopigmentation may follow episodes of atopicor contact dermatitis. However, the physical findings exhibited by the adolescent described inthe vignette are not consistent with these diagnoses and, therefore, treatment with a topicalcorticosteroid and epicutaneous patch testing, respectively, are not warranted. Griseofulvin isused to treat certain dermatophyte infections, particularly tinea capitis, but it is not effective intinea versicolor.

References:

Darmstadt GL, Sidbury R. Cutaneous fungal infections. In: Behrman RE, Kliegman RM, JensonHB, eds. Nelson Textbook of Pediatrics. 17th ed. Philadelphia, Pa: WB Saunders Co; 2004:2230-2234

Falabella R. Pigmentary abnormalities: nongenetic disorders of hypopigmentation. In: SchachnerLA, Hansen RC, eds. Pediatric Dermatology. 3rd ed. St. Louis, Mo: Mosby; 2003:504-513

Paller AS, Mancini AJ. Photosensitivity and photoreactions. In: Hurwitz Clinical PediatricDermatology. 3rd ed. Philadelphia, Pa: Elsevier Inc; 2006:503-5524

Paller AS, Mancini AJ. Eczematous eruptions in childhood. In: Hurwitz Clinical PediatricDermatology. 3rd ed. Philadelphia, Pa: Elsevier Inc; 2006:49-84

Paller AS, Mancini AJ. Disorders of pigmentation. In: Hurwitz Clinical Pediatric Dermatology. 3rded. Philadelphia, Pa: Elsevier Inc; 2006:265-306

Weston WL, Lane AT, Morelli JG. Fungal and year infections of the skin: yeast skin infections.In: Color Textbook of Pediatric Dermatology. 3rd ed. St. Louis, Mo: Mosby; 2002:71-76

Copyright © 2007 by the American Academy of Pediatrics page 136

Page 137: AAP MCQ 2007

2007 PREP SA on CD-ROM

Question: 66

A 3-year-old boy who recently was adopted is brought to you for the first time. The adoptivemother explains that the biologic mother had a history of intravenous drug use, and the child ishepatitis C-positive. On physical examination, the happy, active child has no evidence of anenlarged liver or spleen. You discuss with the mother the possibility of referral to a specialist forantiviral therapy.

Of the following, a TRUE statement about treatment of hepatitis C is that

A. antiviral therapy usually is administered for 12 weeks

B. coinfection with human immunodeficiency virus is a contraindication to treatment

C. patients whose alanine aminotransferase values remain persistently normal do not need to bereferred

D. the preferred treatment is oral ribavirin monotherapy

E. viral genotype determines the likelihood of response to therapy

Copyright © 2007 by the American Academy of Pediatrics page 137

Page 138: AAP MCQ 2007

2007 PREP SA on CD-ROM

Preferred Response: ECritique: 66

Hepatitis C is estimated to infect 170 million individuals worldwide and up to 200,000 children inthe United States. Infection primarily occurs after exposure to infected blood products andinfected needles, sexual contact with an infected partner, or vertical transmission from mother tochild. The incidence of transmission from infected blood has declined dramatically sincescreening was instituted in 1990. In contrast, the incidence of transmission from mother to childhas stayed stable at approximately 5%, and there is no known intervention to reduce such risk.Approximately 80% of infected adults and 50% to 80% of infected children become chroniccarriers, and up to 20% of infected individuals develop cirrhosis over time. Thus, early diagnosisand treatment is essential. Routine serologic testing for hepatitis C is recommended for injectiondrug users, recipients of blood products or organ transplants before July 1992, patients on long-term hemodialysis, people who have persistently abnormal alanine aminotransferaseconcentrations, or children born to high-risk parents. A full summary of recommendations forhepatitis C screening is included in the Red Book.

Screening of high-risk individuals is performed by hepatitis C serology, which detectsimmunoglobulin G against the viral NS5 and NS3 antigens. Antibodies usually become positivewithin 8 weeks after an exposed person is infected. Antibody testing has a sensitivity of 97%and a positive predictive value of more than 95% in patients at risk, although the predictive valueis only 25% in the general population. If a person has a positive antibody result, furtherconfirmatory testing with recombinant immunoblot antibodies and hepatitis C quantitative viralRNA is performed. In appropriate individuals, liver biopsy is useful in predicting the extent ofinflammation and hepatic cirrhosis.

Treatment of individuals infected with hepatitis C currently involves the administration ofeither interferon alone or interferon with ribavirin for 6 to 12 months. Response typically ismeasured by reductions in viral load, improvement in transaminases, and reduction ofinflammation on liver histology. Genotype determines response rate: patients who havegenotype 1 usually have a poorer response rate (<40%), while other genotypes have a betterresponse rate (50% to 70%). Although genotype status does predict response rate, therapy stillcan be used in patients who have the hepatitis C virus genotype 1. The response rate in adultsis superior among those receiving both interferon and ribavirin, although ribavirin is associatedwith anemia. Ribavirin is contraindicated in pregnancy because it may cause birth defects.Ribavirin is not effective as monotherapy. Patients who have normal transaminase values maywarrant treatment, depending on viral load and liver histology. Human immunodeficiency virus(HIV) infection is not a contraindication to therapy, although the response rate is poorer inpatients who have HIV and hepatitis C coinfection.

References:

Gonzalez-Peralta RP, Jolley CD. Hepatitis C virus. In: Walker WA, Goulet O, Kleinman RE,Sherman PM, Shneider BL, Sanderson IR, eds. Pediatric Gastrointestinal Disease. 4th ed.Hamilton, Ontario, Canada: BC Decker; 2005:1156-1169

Jonas MM. Children with hepatitis C. Hepatology. 2002;36(suppl 1):S173-S178. Available at:http://www3.interscience.wiley.com/cgi-bin/abstract/112219084/ABSTRACT

American Academy of Pediatrics. Hepatitis C. In: Pickering LK, ed. Red Book: 2006 Report ofthe Committee on Infectious Diseases. 27th ed. Elk Grove Village, Ill: American Academy ofPediatrics; 2006:358-359

Copyright © 2007 by the American Academy of Pediatrics page 138

Page 139: AAP MCQ 2007

2007 PREP SA on CD-ROM

Question: 67

An infant is delivered by cesarean section at 34 weeks' gestation because of preterm labor.There is no history of ruptured membranes, maternal fever, or abnormalities in fetal heart ratemonitoring. The infant requires assisted ventilation with a bag-mask device and 100% oxygen inthe delivery room. His Apgar scores are 5 and 7 at 1 and 5 minutes, respectively. An umbilicalcord arterial pH is 7.23 and base deficit is 3 mmol/L. He is admitted to the newborn nursery, buttransferred to the neonatal intensive care unit (NICU) within 1 hour for respiratory distressmanifested by tachypnea and grunting. His arterial blood gas results (obtained on room air) uponadmission to the NICU are: pH, 7.20; Pco2, 70 mm Hg; Po2, 50 mm Hg, and base deficit 10mmol/L.

Of the following, the MOST common cause for this infant's respiratory distress is

A. aspiration of amniotic fluid

B. hemothorax

C. pneumopericardium

D. pneumoperitoneum

E. pneumothorax

Copyright © 2007 by the American Academy of Pediatrics page 139

Page 140: AAP MCQ 2007

2007 PREP SA on CD-ROM

Preferred Response: ECritique: 67

Grunting respirations, as described for the infant in the vignette, are indicative of an infantbreathing against a partially closed glottis, allowing for a prolonged expiratory phase and anelevation of end-expiratory volume (described by some as self-induced positive end-expiratorypressure). This mechanism may be helpful to the newborn who has an effective loss offunctional residual capacity (diffuse atelectasis or impaired pulmonary compliance), acidemia(metabolic or associated with sepsis) with associated hyperpnea as an attempt to accomplishcompensatory respiratory alkalosis, and air-leak syndromes that result in grossly reducedfunctional residual capacity due to collapsed lung volume. Because the newborn in the vignettehas received positive-pressure ventilation, the most likely cause of his grunting is apneumothorax. Pneumopericardium and pneumoperitoneum are rare in newborns. Hemothoraxis seen with trauma, coagulopathy, or chest surgery. Aspiration of amniotic fluid is unlikely in thisscenario.

The blood gas values reported for the infant are indicative of a mixed, but principallyrespiratory acidemia. The normal Pco2 is 35 to 45 mm Hg, and correcting the hypercarbia willrestore the pH to greater than 7.35. The base deficit of 10 mmol/L is mildly increased and likelyreflects the mild hypoxemia (Po2 <60 mm Hg) associated with the respiratory distress,collapsed lung, and possible hemodynamic embarrassment.

References:

Durand DJ, Phillips B, Boloker J. Blood gases: technical aspects and interpretation. In:Goldsmith JP, Karotkin EH, eds. Assisted Ventilation of the Neonate. 4th ed. Philadelphia, Pa:Saunders; 2003:279-292

Flidel-Rimon O, Shinwell ES. Respiratory distress in the term and near-term infant. NeoReviews.2005;6:e289-e297. Available at: http://neoreviews.aappublications.org/cgi/content/full/6/6/e289

Rosti L, Coppa I, Perricone G, Mastretti A. Index of suspicion in the nursery. NeoReviews.2003;4:e315-e316. Available at: http://neoreviews.aappublications.org/cgi/content/full/4/11/e315

Copyright © 2007 by the American Academy of Pediatrics page 140

Page 141: AAP MCQ 2007

2007 PREP SA on CD-ROM

Question: 68

A 4 year-old-boy presents to your clinic with anal itching of 2 weeks' duration. His mother deniesitching in other family members. Tape applied to his perianal skin shows oval structures (ItemQ68A).

Of the following, the most appropriate management of this patient is

A. albendazole administered three times daily for 7 days

B. ivermectin administered in a single dose and repeated in 2 weeks

C. ketoconazole administered daily for 7 days

D. mebendazole administered in a single dose and repeated in 2 weeks

E. praziquantel administered three times in 1 day

Copyright © 2007 by the American Academy of Pediatrics page 141

Page 142: AAP MCQ 2007

2007 PREP SA on CD-ROM

Preferred Response: DCritique: 68

The child described in the vignette has an anal pinworm infection, which is caused by theroundworm Enterobius vermicularis. Transmission is via the fecal-oral route, and the primarysymptom is anal pruritus. The diagnosis may be made by documentation of adult worms (ItemC68A) in the perianal area. If worms are not grossly visible, transparent adhesive tape may beapplied to the perianal skin and then applied to a glass slide and examined under low powerthrough a microscope. Ideally, three specimens should be obtained on consecutive morningsimmediately upon awakening and before washing or stooling. Stool studies are notrecommended because the eggs (Item C68B) are found in very small quantities in the stool.

The treatment of pinworm infections is a single dose of mebendazole, pyrantel pamoate, oralbendazole. A repeat dose should be administered 2 weeks after the first dose in case ofreinfection. Ivermectin and ketoconazole are not used in the management of these infections.Praziquantel is used in the treatment of some tapeworm infections but is not indicated in thetreatment of pinworm infections. In addition to medical treatment, counseling the parent on stepsto prevent reinfection and hygiene measures is important. Washing of the infected child’spajamas, bedding, and underclothes is recommended, and measures should be taken to avoidscratching the affected area. Bathing in the morning helps to remove eggs from the area. Theentire family should be treated if multiple infections or repeated infections occur. Child careworkers should be educated on good hand washing techniques, but affected children mayattend child care.

References:

American Academy of Pediatrics. Pinworm infection. In: Pickering LK, ed. Red Book: 2006Report of the Committee on Infectious Diseases. 27th ed. Elk Grove Village, Ill: AmericanAcademy of Pediatrics; 2006:520-522

Robinson J. Infectious diseases in schools and child care facilities. Pediatr Rev. 2001;22:39-46.Available at: http://pedsinreview.aappublications.org/cgi/content/full/22/2/39

Copyright © 2007 by the American Academy of Pediatrics page 142

Page 143: AAP MCQ 2007

2007 PREP SA on CD-ROM

Question: 69

A 12-year-old child who recently emigrated from southeast Asia has beta-thalassemia for whichshe has required frequent transfusions. She presents today with polyuria and polydipsia. Onyour initial evaluation, you detect a grade II/VI systolic murmur with a gallop rhythm, palpate theliver 4 cm below the costal margin, and determine that the girl is well below the 5th percentile forheight.

Of the following, the MOST likely diagnosis is

A. acute anemia

B. congenital heart defect

C. iron overload

D. lead poisoning

E. sickle crisis

Copyright © 2007 by the American Academy of Pediatrics page 143

Page 144: AAP MCQ 2007

2007 PREP SA on CD-ROM

Preferred Response: CCritique: 69

The child described in the vignette has signs and symptoms of iron overload. Children who aredependent on transfusions for optimal hemodynamic status, maintenance of normal coagulation,or hematologic stability due to thalassemia or sickle cell disease are at risk for iron overload ortransfusion-related hemosiderosis. Hemosiderosis usually occurs in patients who have receivedmore than 100 units of transfused red cells. In beta thalassemia, transfusions must beadministered approximately every month, and each 200 mL of packed red blood cells contains200 mg of iron that cannot be excreted physiologically.

Unbound iron may damage cells via lipid peroxidation. Acute manifestations ofhemosiderosis include cardiac failure due to cardiomyopathy, arrhythmia, and liver failure.Although iron may be deposited in the ventricular myocardium, it is deposited earlier in theconduction tissues of the heart. More insidious symptoms include endocrine failure (diabetes,hypothyroidism, and pubertal delay) as well as short stature out of proportion to that expectedfrom the underlying disease. Chelation may be used to reduce the iron load by subcutaneousinjection of desferoxime, and oral chelators are being investigated.

The child described in the vignette is unlikely to have acute anemia and should be wellcompensated for her chronic anemia. A congenital heart defect cannot be excluded, but mostlikely would have been detected prior to age 12 years and would not produce polyuria andpolydipsia. Sickle beta thalassemia hemoglobinopathy would have manifested with recurrentinfection and pain crises by age 12 years, and her peripheral blood smear would reveal sickleforms. Even in this situation, however, the primary concern still would be transfusion-relatedhemosiderosis. Chronic lead poisoning would be expected to yield cognitive and behavioraldeficits and not present with signs of cardiac or liver involvement.

References:

Martin PL, Pearson HA. Hemoglobinopathies and thelassemias. In: McMillan JA, DeAngelis CD,Feigin RD, Warshaw JB, eds. Oski’s Pediatrics: Principles and Practice. 3rd ed. Philadelphia,Pa: Lippincott Williams & Wilkins; 1999:1450-1452

Orkin SH, Nathan DG. The thalassemias. In: Nathan DG, Ginsburg D, Orkin SH, Look AT, eds.Nathan and Oski’s Hematology of Infancy and Childhood. 6th ed. Philadelphia, Pa: Saunders;2003:842-920

Copyright © 2007 by the American Academy of Pediatrics page 144

Page 145: AAP MCQ 2007

2007 PREP SA on CD-ROM

Question: 70

A 2-year-old girl is rushed by ambulance to the emergency department for sudden-onset ataxia.Her parents have yet to arrive. On physical examination, the girl is afebrile, yet diaphoretic, withsome nystagmus on far lateral gaze. Her ataxia has resolved. The remainder of physicalexamination findings are normal.

Of the following, the MOST likely diagnosis for this child is

A. basilar migraine

B. benign paroxysmal vertigo

C. cerebellar hemorrhage

D. phenytoin intoxication

E. seizure

Copyright © 2007 by the American Academy of Pediatrics page 145

Page 146: AAP MCQ 2007

2007 PREP SA on CD-ROM

Preferred Response: BCritique: 70

The sudden and self-limited ataxia with nystagmus, but without vomiting or loss ofconsciousness, followed by some residual horizontal nystagmus described for the girl in thevignette is typical of benign paroxysmal vertigo. Benign paroxysmal vertigo is characterized byabrupt, brief episodes of vertigo with ataxia in children ages 1 to 3 years. The child may appearfrightened, have pallor, and may indicate feeling dizzy. Rapid eye movements or nystagmusmay be observed if the eyes are open. Benign paroxysmal vertigo is believed to be a migrainevariant.

Vertigo is defined as an illusion of movement, most often a sensation of rotation. Thesimplest approach to evaluating this complaint is to consider its temporal pattern and whetherhearing is affected. Acute vertigo without hearing loss may stem from benign paroxysmal vertigoin toddlers, vestibular neuronitis in adolescents, labyrinthine concussion, migraine, seizures(rarely), and benign paroxysmal positional vertigo (BPPV). Most of these disorders last minutesto sometimes hours, although BPPV lasts only 5 to 20 seconds and occurs with changes inhead position. BPPV can be elicited by the Hallpike-Dix maneuver. The child is moved abruptlyfrom a sitting to lying posture, with the head hanging 45 degrees below the horizontal and rotated45 degrees to one side, evoking vertigo and nystagmus.

Acute vertigo lasting hours to days accompanied by hearing loss may be due to labyrinthitis,otitis media, perilymphatic fistula, temporal bone fracture, and Ménière disease. Ménière diseaseis a recurrent disorder that involves tinnitus and progressive hearing loss.

Continuous vertigo over days to weeks may indicate a cholesteatoma; posterior fossatumor; vestibular schwannoma (associated with neurofibromatosis II); demyelinating disease; ordrug toxicity from aminoglycosides, isoniazid, furosemide, or phenytoin.

The 2-year-old child in the vignette has not displayed headache or cranial neuropathies,making basilar migraine unlikely. The spontaneous and rapid resolution of her symptomsexcludes cerebellar hemorrhage and phenytoin intoxication. No loss of consciousness has beenobserved, making seizure unlikely.

References:

MacGregor DL. Vertigo. Pediatr Rev. 2002;23:10-16. Available at:http://pedsinreview.aappublications.org/cgi/content/full/23/1/10

Copyright © 2007 by the American Academy of Pediatrics page 146

Page 147: AAP MCQ 2007

2007 PREP SA on CD-ROM

Question: 71

You are evaluating a 15-year-old boy in your office. He tells you that he experiences chest painat times while at home and at school. He describes it as sharp, located in the left chest,exacerbated by deep breathing, and resolving spontaneously. He does not feel palpitations orlightheadedness and has not had syncope. His 54-year-old father and grandfather both havehad hypertension and myocardial infarctions. At this time, he has no pain, and results of physicalexamination are normal.

Of the following, the MOST appropriate approach for this patient at this time is

A. chest radiography

B. echocardiography

C. electrocardiography

D. reassurance with clinical follow-up

E. referral to pediatric cardiologist

Copyright © 2007 by the American Academy of Pediatrics page 147

Page 148: AAP MCQ 2007

2007 PREP SA on CD-ROM

Preferred Response: DCritique: 71

Chest pain in children and adolescents is a common problem in the office, clinic, urgent carepractice, and emergency department. Societal attention has focused on the importance of earlyrecognition of chest pain in adults as an important presenting sign of cardiovascular disease andpotential emergency. Some of this attention accounts for the high level of concern that manyparents and children have when chest pain presents in the pediatric population. In fact, thelikelihood that chest pain in an otherwise well child represents cardiovascular disease isextremely low. Overall, the incidence of chest pain attributable to a cardiac cause is less than1% in children.

Several organ systems have the potential to cause chest pain in children. The history andphysical examination are essential to determine which system likely is associated and, asimportantly, which is not. Information about pain quality, intensity, location, and temporalrelationships is essential. Pain that radiates to the back, neck, shoulders, or left arm should raiseconcern about a cardiac cause. Pain that is constant or frequent, dull, pressurelike, orassociated with exercise is more likely to be associated with cardiac causes than is pain that isbrief, infrequent, and sharp, as described for the boy in the vignette. Pain that worsens withinspiration is generally reassuring, suggesting a musculoskeletal or pulmonary rather thancardiac cause. It is not uncommon to identify a history of a recent cardiac problem in an oldermember of the patient’s family, a reminder of the importance of and the potential forpsychological impact on somatic findings. In addition to the physical examination that includespalpation of the pulses in the right arm and the femoral region and auscultation of the chest andheart, the chest wall and the costochondral margin should be palpated. If the physicalexamination results are normal and the history is consistent with a noncardiac source of thepain, as described for the boy in the vignette, reassurance with clinical follow-up usually is themost appropriate management. Because the pain is pleuritic, transient, and spontaneouslyresolving in this well adolescent, no referral or laboratory testing is indicated.

References:

Cava JR, Sayger PL. Chest pain in children and adolescents. Pediatr Clin North Am.2004;51:1553-1568. Abstract available at:http://www.ncbi.nlm.nih.gov/entrez/query.fcgi?orig_db=PubMed&db=PubMed&cmd=Search&term=%22Pediatric+clinics+of+North+America%22[Jour]+AND+1553[page]+AND+2004[pdat]

Fahey J. Chest pain. In: Rudolph CD, Rudolph AM, Hostetter MK, Lister G, Siegel NJ, eds.Rudolph’s Pediatrics. 21st ed. New York, NY: McGraw Hill; 2003:1894-1897

Copyright © 2007 by the American Academy of Pediatrics page 148

Page 149: AAP MCQ 2007

2007 PREP SA on CD-ROM

Question: 72

A 10-year-old boy (individual III,1 in the pedigree) (Item Q72A) presents for evaluation ofligamentous laxity and multiple joint dislocations. The family history is notable for a father whohas scoliosis and ligamentous laxity, a paternal aunt who has had retinal detachments and mitralvalve prolapse, and a paternal grandmother who had joint dislocations and now hasosteoarthritis.

Based on the family history, the MOST likely pattern of inheritance of this connective tissuedisorder is

A. autosomal dominant

B. autosomal recessive

C. mitochondrial

D. X-linked dominant

E. X-linked recessive

Copyright © 2007 by the American Academy of Pediatrics page 149

Page 150: AAP MCQ 2007

2007 PREP SA on CD-ROM

Preferred Response: ACritique: 72

To determine the pattern of inheritance for a given condition, it is helpful to draw a pedigree. Inthe pedigree of the child described in the vignette (Item C72A), there is vertical transmission of aconnective tissue disorder through three generations, which rules out autosomal recessiveinheritance if there is no consanguinity. X-linked inheritance is ruled out by the male-to-maletransmission (from the father to the son). Male-to-male transmission also excludes mitochondrialinheritance because mitochondria are transmitted almost exclusively through our mothers.Therefore, the mode of transmission for the disorder described in the vignette is autosomaldominant.

Hallmarks of autosomal dominant inheritance include father-to-son transmission, variableexpressivity, and the potential for reduced penetrance. Many dominant conditions have a highrate of spontaneous mutation (eg, neurofibromatosis type 1 [50%] and achondroplasia [80%]).The recurrence risk is 50% for all individuals who have autosomal dominant conditions.

References:

Hoyme HE. Patterns of inheritance. In: Behrman RE, Kliegman RM, Jenson HB, eds. NelsonTextbook of Pediatrics. 17th ed. Philadelphia Pa: Saunders; 2004:376-382

Nussbaum RL, McInnes RR, Willard HF. Patterns of single-gene inheritance. In: Thompson &Thompson Genetics in Medicine. 6th ed, revised reprint. Philadelphia, Pa: Saunders; 2004:51-78

Pinsky L. Overview of genetic assessment. UpToDate. 2006; 14.1. Available at:http://www.utdol.com/utd/content/topic.do?topicKey=genr_med/27636&type=A&selectedTitle=1~85

Copyright © 2007 by the American Academy of Pediatrics page 150

Page 151: AAP MCQ 2007

2007 PREP SA on CD-ROM

Question: 73

A 5-year-old girl presents with a foul-smelling vaginal discharge of 2 weeks' duration. She has aprevious history of one urinary tract infection at age 3 years.

Of the following, the MOST appropriate next step is

A. culture of the discharge for respiratory pathogens

B. genital examination using the knee-chest position

C. perianal adhesive tape test

D. sitz baths and application of an estrogen cream

E. urine for culture and sensitivity

Copyright © 2007 by the American Academy of Pediatrics page 151

Page 152: AAP MCQ 2007

2007 PREP SA on CD-ROM

Preferred Response: BCritique: 73

A vaginal discharge in a prepubertal girl frequently is accompanied by vulvitis. The evaluation ofvaginal discharge or vulvovaginitis in a prepubertal girl is guided by clues from the history andfindings on physical examination. The clinician must inquire about a history of sexual molestationor activity, including instrumentation, in all girls who have a vaginal discharge with or withoutgenital pain, regardless of their age or pubertal status. Although less commonly encountered,clinicians evaluating males must remember that instrumentation of the penis may cause genitalpain or urethral discharge.

Documentation of a recent history of oral broad-spectrum antibiotic use is importantbecause it increases the risk of candidal vulvovaginitis. The character of the discharge maysuggest a specific cause: a green discharge with Neisseria gonorrhoeae and group A beta-streptococcal infection; a malodorous discharge with a retained foreign body; and a bloodydischarge with trauma, foreign body, and Shigella or group A beta-hemolytic streptococcalinfections. Other important historical information includes recent infections, hygiene habits,masturbation, use of skin products, previous occurrences of vulvovaginitis, and priortreatments.

Often no specific cause of vulvovaginitis is found for prepubertal girls, and the condition isattributed to poor hygiene. Causes of a specific vulvovaginitis in prepubertal girls includeinfections (respiratory and enteric pathogens, Candida, sexually transmitted diseases, andpinworms), foreign body, vulvar skin diseases (eg, lichen sclerosus, seborrhea, psoriasis,atopic dermatitis, contact dermatitis), and trauma (eg, excessive cleaning or masturbation).Less common causes include systemic illnesses such as varicella and genitourinarymalformations such as an ectopic ureter that may cause perineal wetness.

The physical examination of prepubertal girls who have vulvovaginitis should include aninspection of the external genitalia, including the perineum and anus. A speculum examinationusually is not necessary; samples of vaginal fluid can be obtained with a saline-moistened swabthrough the hymenal opening, avoiding contact with the sensitive hymenal edges, or a vaginalwash for a wet preparation, potassium hydroxide preparation, and cultures. The knee-chestposition in a cooperative child often allows adequate visualization of the vaginal canal for thediscovery of retained foreign bodies.

If a foreign body is found, removal followed by sitz baths to clear residual symptoms isadequate treatment. Culture of the discharge for respiratory pathogens is not necessary. Aperianal adhesive tape test is performed if a pinworm infestation is suspected as cause ofvulvovaginitis associated with pruritus or excoriations. Sitz baths and application of an estrogencream may be prescribed for persistent nonspecific vulvovaginitis, but would not be appropriatefor a patient who has a foul-smelling discharge that is suggestive of a foreign body withoutinspection of the vaginal canal. Vulvovaginitis that is not associated with other signs orsymptoms to implicate the urinary system is an unlikely presentation of a urinary tract infection,so obtaining a urine specimen for culture and sensitivity is not appropriate.

References:

Emans SJ. Vulvovaginal problems in the prepubertal child. In: Emans SJH, Laufer MR, GoldsteinDP, eds. Pediatric and Adolescent Gynecology. 5th ed. Philadelphia, Pa: Lippincott, Williams &Wilkins; 2005:83-119

Sanfilippo JS. Vulvovaginitis. In: Behrman RE, Kleigman RM, Jenson HB, eds. Nelson Textbookof Pediatrics. 17th ed. Philadelphia, Pa: Saunders; 2004:1828-1832

Copyright © 2007 by the American Academy of Pediatrics page 152

Page 153: AAP MCQ 2007

2007 PREP SA on CD-ROM

Question: 74

You are asked to evaluate a 1-day-old infant who has mild clitoromegaly (Item Q74A) andpalpable masses in the labial folds bilaterally. Chromosome studies reveal that the infant has anXY karyotype.

Of the following, additional history is MOST likely to reveal that the infant's

A. father was exposed to pesticides

B. maternal aunts are infertile

C. mother took androgens during the pregnancy

D. mother took progestogens during the pregnancy

E. paternal aunts and uncles are infertile

Copyright © 2007 by the American Academy of Pediatrics page 153

Page 154: AAP MCQ 2007

2007 PREP SA on CD-ROM

Preferred Response: BCritique: 74

The baby described in the vignette, who has mild masculinization and palpable gonadal masseswith an XY karyotype, is an undervirilized XY infant (male pseudohermaphroditism). Becausegonads are present below the inguinal ligament, there is testicular tissue. Even with carefulevaluation, an underlying cause cannot presently be determined in up to 40% of undervirilizedXY infants. However, one of the most common findings is androgen insensitivity (AIS), whichmay be complete (CAIS) or partial (PAIS). The finding usually represents a defect in theandrogen receptor and is inherited in an X-linked pattern. Therefore, some maternal aunts arelikely to have the same disorder, have an XY chromosomal pattern, and be infertile. Becausethis disorder is not inherited through the father, a history of infertility in paternal relatives wouldnot be expected. Failure of masculinization of an XY fetus cannot be attributed to maternalandrogen or progesterone treatment or to paternal exposure to pesticides, even if they haveantiandrogenic effects.

Physicians faced with the diagnostic management and care of XY infants should consult withexperienced endocrinologists, geneticists, and psychological support personnel. Additionaldiagnostic studies should rule out other genetic disorders, such as 5 alpha-reductase deficiencyand 17-ketosteroid reductase defects. The difficult decision of determining the sex of rearing ofthe affected child must involve the parents. A trial of exogenous testosterone can help to definethe likely adult response to androgen and assist in this decision.

References:

Ahmed SF, Cheng A, Dovey L, et al. Phenotypic features, androgen receptor binding, andmutational analysis in 278 clinical cases reported as androgen insensitivity syndrome. J ClinEndocrinol Metab. 2000;85:658-665. Available at:http://jcem.endojournals.org/cgi/content/full/85/2/658

Androgen Insensitivity Syndrome Support Group Web site. Available at:http://www.medhelp.org/www/ais/

Boehmer AL, Brinkmann O, Bruggenwirth H, et al. Genotype versus phenotype in families withandrogen insensitivity syndrome. J Clin Endocrinol Metab. 2001;86:4151-4160. Available at:http://jcem.endojournals.org/cgi/content/full/86/9/4151

Gottlieb B, Beitel LK, Trifiro MA. Androgen insensitivity syndrome. Gene Reviews. 2004.Available at: http://www.geneclinics.org/profiles/androgen/details.html

Houk CP, Lee PA. Intersexed states: diagnosis and management. Endocrinol Metab Clin NorthAm. 2005;34:791-810

Houk CP, Levitsky LL. Evaluation of the infant with ambiguous genitalia. UptoDate. 2006;14.1.Available at:http://www.utdol.com/utd/content/topic.do?topicKey=pediendo/11554&type=P&selectedTitle=14~24

Wilson BE. Androgen insensitivity syndrome. eMedicine Specialties: Pediatrics: Endocrinology.2003. Available at: http://www.emedicine.com/ped/topic2222.htm

Copyright © 2007 by the American Academy of Pediatrics page 154

Page 155: AAP MCQ 2007

2007 PREP SA on CD-ROM

Question: 75

You diagnose attention-deficit/hyperactivity disorder in a 10-year-old girl. She is growing well,and physical examination findings are normal.

Of the following, the MOST appropriate test to confirm the diagnosis is

A. brain computed tomography

B. brain magnetic resonance imaging

C. electroencephalography

D. no test at this time

E. positron emission testing

Copyright © 2007 by the American Academy of Pediatrics page 155

Page 156: AAP MCQ 2007

2007 PREP SA on CD-ROM

Preferred Response: DCritique: 75

Attention-deficit/hyperactivity disorder (ADHD) is a common, yet heterogeneous disorder thathas a complex etiology; genetic, environmental, and biologic factors all play roles. For example,ADHD is associated with both genetic syndromes such as fragile X and intrauterine toxicexposures such as fetal alcohol syndrome. It also has been shown in family and twin studiesthat ADHD is more common in close family members of those diagnosed with the disorder.

Research continues into the neurobiologic basis of ADHD. Evidence is increasing thatalterations in the frontal lobe and frontal subcortical connections play significant roles in thedisorder. Affected children have difficulties with executive functioning, such as organization,impulse control, and inattention, that are common in other disorders involving impaired frontallobe function.

Neurodiagnostic studies such as magnetic resonance imaging, electroencephalography,computed tomography scan, and positron emission testing may be used in research settings,but they are not used routinely in the evaluation of children who have ADHD. Such studies maybe appropriate if there are specific concerns regarding risk for seizure, brain malformation, orbrain injury.

References:

American Academy of Pediatrics Committee on Quality Improvement, Subcommittee onAttention-Deficit/Hyperactivity Disorder. Clinical practice guideline: diagnosis and evaluation ofthe child with attention-deficit/hyperactivity disorder. Pediatrics. 2000;105:1158-1170. Availableat: http://pediatrics.aappublications.org/cgi/content/full/105/5/1158

American Academy of Pediatrics Subcommittee on Attention-Deficit/Hyperactivity Disorder andCommittee on Quality Improvement. Clinical practice guideline: treatment of the school-agedchild with attention-deficit/hyperactivity disorder. Pediatrics. 2001;108:1033-1044. Available at:http://pediatrics.aappublications.org/cgi/content/full/108/4/1033

Reiff MI, Tippins S, LeTourneau AA. ADHD: A Complete and Authoritative Guide. Elk GroveVillage, Ill: American Academy of Pediatrics; 2004

Sims MD. Attention-deficit/hyperactivity disorder. In: Behrman RE, Kliegman RM, Jenson HB,eds. Nelson Textbook of Pediatrics. 17th ed. Philadelphia, Pa: WB Saunders Co; 2004:107-110

Copyright © 2007 by the American Academy of Pediatrics page 156

Page 157: AAP MCQ 2007

2007 PREP SA on CD-ROM

Question: 76

You have been treating a 2-year-old girl for pneumococcal meningitis for the past 5 days.

Of the following, the MOST likely complication of her disease is

A. brain abscess

B. cerebral infarct

C. cranial nerve palsy

D. hearing impairment

E. sagittal sinus thrombosis

Copyright © 2007 by the American Academy of Pediatrics page 157

Page 158: AAP MCQ 2007

2007 PREP SA on CD-ROM

Preferred Response: DCritique: 76

Sensorineural hearing loss is the most common long-term sequela of bacterial meningitis. It hasbeen described in up to 30% of patients who have pneumococcal disease and 10% of thosewho have meningococcal meningitis. The hearing loss is due to either a labyrinthitis or directinflammation of the auditory nerve. Patients who have significant hearing loss often demonstratesignificant truncal ataxia during their recovery because of middle ear inflammation. Other long-term sequelae include developmental or learning delays, behavioral problems, seizures,hydrocephalus, and cerebral palsy.

To decrease the risk of long-term neurologic sequelae, the American Academy of Pediatricsrecommends the administration of adjunctive therapy with dexamethasone for the treatment ofbacterial meningitis due to Haemophilus influenzae type B and consideration of treatment forpneumococcal meningitis in infants and children older than 6 weeks of age. If dexamethasone isused, it should be given regardless of the clinical severity of the illness and administered assoon as possible in the course of treatment; best results occur when the first dose isadministered prior to any antimicrobial therapy.

Other acute complications of bacterial meningitis include the syndrome of inappropriateantidiuretic hormone secretion, seizures, and increased intracranial pressure. Fever may bepresent for 5 to 7 days during the illness or may disappear if dexamethasone therapy is used,only to return after the corticosteroids are stopped. Less common but more serious acutecomplications include infarction of the brain or spinal cord, cranial nerve palsy, cerebralherniation, thrombosis of the sagittal sinus, subdural effusions, and brain abscess formation.

References:

Prober CG. Central nervous system infections. In: Behrman RE, Kliegman RM, Jenson HB, eds.Nelson Textbook of Pediatrics. 17th ed. Philadelphia, Pa: WB Saunders Co; 2004:2038-2047

Koomen I, Grobbee DE, Roord JJ, Donders R, Jennekens-Schinkel A, Van Furth AM. Hearingloss at school age in survivors of bacterial meningitis: assessment, incidence, and prediction.Pediatrics. 2003;112:1049-1053. Available at:http://pediatrics.aappublications.org/cgi/content/full/112/5/1049

Feigin RD, Watson JT, Gerber SI. Use of corticosteroids in bacterial meningitis. Pediatr InfectDis J. 2004;23:355-357

Copyright © 2007 by the American Academy of Pediatrics page 158

Page 159: AAP MCQ 2007

2007 PREP SA on CD-ROM

Question: 77

You are evaluating a 7-year-old boy for hematuria and proteinuria. As part of the evaluation, youmeasure serum electrolytes. The serum creatinine is 1.1 mg/dL (97.2 mcmol/L).

Of the following, the MOST accurate serum creatinine measurements for children of normalphysical development are

(All measurements mg/dL [mcmol/L])

A. 3 months old: 0.3 (26.5); 2 years old: 0.4 (35.4); 7 years old: 1.0 (88.4); 17 years old: 1.0(88.4)

B. 3 months old: 0.6 (53.0); 2 years old: 0.8 (70.7); 7 years old: 1.0 (88.4); 17 years old: 1.2(106.1)

C. 3 months old: 0.3 (26.5); 2 years old: 0.4 (35.4); 7 years old: 0.6 (53.1); 17 years old: 0.9(79.6)

D. 3 months old: 0.6 (53.0); 2 years old: 0.4 (35.4); 7 years old: 0.7 (61.9); 17 years old: 0.7(61.9)

E. 3 months old: 0.7 (61.9); 2 years old: 0.8 (70.7); 7 years old: 0.7 (61.9); 17 years old: 0.7(61.9)

Copyright © 2007 by the American Academy of Pediatrics page 159

Page 160: AAP MCQ 2007

2007 PREP SA on CD-ROM

Preferred Response: CCritique: 77

The kidneys have a full complement of glomeruli and tubules at birth, but the nephrons growduring childhood, with most of the development occurring during the first 2 years. Thus, renalfunction may require up to 2 years to mature completely. Development of glomerular function isgenerally complete by age 2 years. Tubular function matures rapidly during the first year afterbirth, resulting in improved ability to reabsorb sodium, potassium, and water. In turn, theadaptability of the kidney to perturbations of normal body homeostasis (eg, dehydration) is morepronounced after the first postnatal year than during infancy.

The level of the mother’s serum creatinine generally determines the serum creatinine of thenewborn in the first 7 to 10 postnatal days. The serum creatinine is relatively low during infancyand early childhood due to lower muscle mass (Item C77A). Additionally, blood flow to thekidneys is lower in early infancy due to increased peripheral resistance, thus reducing effectiverenal plasma flow. Taken together, these factors contribute to a lower serum creatinine and arelatively lower glomerular filtration rate (GFR) during early childhood. GFR is determinedroutinely from the serum creatinine, but there are several other methods to determine the GFR,including inulin clearance and measurement of serum cystatin A.

References:

Atiyeh BA, Dabbagh SS, Gruskin AB. Evaluation of renal function during childhood. Pediatr Rev.1996;17:175-180

Bökenkamp A, Domanetzki M, Zinck R, Schumann G, Byrd D, Brodehl J. Cystatin C—a newmarker of glomerular filtration rate in children independent of age and height. Pediatrics.1998;101:875-881. Available at: http://pediatrics.aappublications.org/cgi/content/full/101/5/875

Schwartz GJ, Haycock GB, Edelmann CM, Spitzer A. A simple estimate of glomerular filtrationrate in children derived from body length and plasma creatinine. Pediatrics. 1976;58:259-263.Abstract available at: http://pediatrics.aappublications.org/cgi/content/abstract/58/2/259

Copyright © 2007 by the American Academy of Pediatrics page 160

Page 161: AAP MCQ 2007

2007 PREP SA on CD-ROM

Question: 78

A 6-month-old infant has been receiving high-dose amoxicillin therapy for bilateral otitis media.After 48 hours of therapy, she continues to be febrile, with a temperature of 102ºF (38.9ºC), andis irritable. Physical examination reveals erythematous, dull, and bulging tympanic membranes,with no movement on insufflation.

Of the following, the MOST appropriate antibiotic to change this patient to is

A. azithromycin

B. cefdinir

C. clindamycin

D. doxycycline

E. trimethoprim-sulfamethoxazole

Copyright © 2007 by the American Academy of Pediatrics page 161

Page 162: AAP MCQ 2007

2007 PREP SA on CD-ROM

Preferred Response: BCritique: 78

The infant described in the vignette has otitis media that is not responding to high-doseamoxicillin therapy, suggesting that the causative pathogen is beta-lactamase-producingnontypeable Haemophilus sp or Moraxella catarrhalis. Accordingly, the child requires a beta-lactamase-resistant antibiotic, such as cefdinir, for treatment of her infection.

Beta-lactamases are bacterial enzymes that hydrolyze the beta-lactam ring of beta-lactamantibiotics, inhibiting the ability of such antibiotics to interfere with cell wall polymerization of thepenicillin-binding proteins, thereby decreasing the organisms’ susceptibility to the drugs. Beta-lactamase production is one of the most frequent mechanisms of antibiotic resistance used bygram-negative organisms such as Haemophilus sp, Moraxella catarrhalis, strains of Klebsiellapneumoniae, Escherichia coli, Salmonella sp, Pseudomonas aeruginosa, Enterobacter sp,Citrobacter sp, and Neisseria gonorrhoeae. Some of these organisms can produce extended-spectrum beta-lactamases (ESBLs) that can confer resistance to all penicillins, allcephalosporins, and aztreonam, making treatment of infections due to these organisms muchmore difficult. Testing for beta-lactamase production or ESBL-producing organisms is conductedin most clinical laboratories. The second- and third-generation oral cephalosporins are notaffected by beta-lactamases produced by the most common otitis media pathogens.

Clindamycin is effective only against gram-positive organisms and, therefore, is not effectiveagainst nontypeable Haemophilus sp or M catarrhalis. Doxycycline, although effective againstthese pathogens, should not be used in children younger than 8 years of age if other efficaciousdrugs are available. Azithromycin has activity against both nontypeable Haemophilus sp and Mcatarrhalis, but its activity is greatly decreased against beta-lactamase-producing isolates. Theprevalence of resistance among bacteria causing otitis media and potential adverse effectsmake trimethoprim-sulfamethoxazole a less preferred agent.

References:

Christenson JC, Korgenski EK. Laboratory diagnosis of infection due to bacteria, fungi, parasitesand rickettsiae. In: Long SS, Pickering LK, Prober CG, eds. Principles and Practice of PediatricInfectious Diseases. 2nd ed. New York, NY: Churchill Livingstone, 2003:1380-1391

Doern GV, Jones RN. Antimicrobial susceptibility testing of Haemophilus influenzae,Branhamella catarrhalis, and Neisseria gonorrhoeae. Antimicrob Agents Chemother.1988;32:1747-1753. Available at:http://www.pubmedcentral.gov/articlerender.fcgi?tool=pubmed&pubmedid=3149883

Samaha-Kfoury JN, Araj GF. Recent developments in beta lactamases and extended spectrumbeta lactamases. BMJ. 2003;327:1209-1213. Available at:http://bmj.bmjjournals.com/cgi/content/full/327/7425/1209

Copyright © 2007 by the American Academy of Pediatrics page 162

Page 163: AAP MCQ 2007

2007 PREP SA on CD-ROM

Question: 79

A medical student who is rotating in your clinic has just evaluated a 12-month-old girl whopresented with a history of recurrent bacterial and viral infections. As part of your discussionwith the medical student, you review the different aspects of the immune system and theevaluation of the infant's host defense.

Of the following, the test that is the BEST measure of cell-mediated immunity is

A. Candida skin test

B. complement 50 assay

C. dihydrorhodamine flow cytometry

D. isohemmaglutinins

E. serum immunoglobulins (Ig) A, M, and G

Copyright © 2007 by the American Academy of Pediatrics page 163

Page 164: AAP MCQ 2007

2007 PREP SA on CD-ROM

Preferred Response: ACritique: 79

Primary care physicians evaluating children who have recurrent bacterial or viral infections needto maintain a high index of suspicion for underlying defects in the child’s immune system. Hostdefense beyond the skin and gastrointestinal tract includes humoral, cellular, phagocytic, andcomplement. Investigation into one or more of these areas may be warranted, depending on thechild’s age, type and location of infections, and family medical history.

A simple qualitative assessment of cell-mediated immunity that can be employed to assessthe girl in the vignette is the Candida delayed-type hypersensitivity (DTH) intradermal skin test.Induration of 10 mm or greater at 48 hours virtually excludes all primary T-cell defects. Otherantigens used for DTH testing include tetanus, mumps, and trichophyton. If DTH testing resultsare negative, other measurements of the cellular immune system, including lymphocyte counts,enumeration of T-cell subpopulations by flow cytometry, and lymphocyte stimulation testing, maybe considered.

Total complement assay (eg, complement 50) measures the intactness of the classiccomplement pathway. Low levels can result from a delay in specimen processing, early (C1through C4) complement deficiencies, or late (C5 through C9) complement deficiencies.

The dihydrorhodamine fluorescence (DHR 123) test is a flow cytometry-based oxidationassay that can measure neutrophil respiratory burst, a phagocytic function. In many centers,the DHR 123 test is replacing the nitroblue tetrazolium dye test for evaluation of chronicgranulomatous disease.

Isohemagglutinins and serum immunoglobulins are qualitative and quantitativemeasurements of the humoral immune system, respectively. Isohemagglutinins are antibodies totype A and B red blood cell polysaccharide antigens and normally are present in children ages 1year and older who have blood type A, B, or O. Serum immunoglobulins (Igs) include IgG, IgA,and IgM.

References:

Ballow M, O’Neil KM. Approach to the patient with recurrent infections. In: Adkinson NF Jr,Yunginger JW, Busse WW, Bochner BS, Holgate ST, Simons FER, eds. Middleton’s AllergyPrinciples and Practice. 6th ed. Philadelphia, Pa: Mosby; 2003:1043-1072

Buckley RH. Evaluation of suspected immunodeficiency. In: Behrman RE, Kliegman RM, JensonHB, eds. Nelson Textbook of Pediatrics. 17th ed. Philadelphia, Pa: WB Saunders Co 2004:681-682

Copyright © 2007 by the American Academy of Pediatrics page 164

Page 165: AAP MCQ 2007

2007 PREP SA on CD-ROM

Question: 80

A frail 6-year-old child who has cystic fibrosis is transported by ambulance to the emergencydepartment. She has had hemoptysis for the past 4 hours, yielding approximately 10 mL ofbright red blood. She has had increased cough over the past 3 days. Physical examinationfindings include a respiratory rate of 38 breaths/min, heart rate of 90 beats/min, oxygensaturation of 92% on room air, blood pressure of 100/70 mm Hg, and temperature of 98.6ºF(37ºC). She is awake and alert but seems breathless when she tries to speak. On auscultation,you note diffuse crackles throughout her lung fields.

Of the following, the BEST next step in the management of this patient is to

A. administer methylprednisolone

B. begin therapy with ceftriaxone

C. insert an endotracheal tube and begin positive pressure ventilation

D. obtain blood for determination of prothrombin and partial thromboplastin time

E. transfuse with O-negative blood

Copyright © 2007 by the American Academy of Pediatrics page 165

Page 166: AAP MCQ 2007

2007 PREP SA on CD-ROM

Preferred Response: DCritique: 80

Acute pulmonary hemorrhage, or hemoptysis, is an uncommon problem in pediatrics. Acutelower respiratory tract infection is the leading cause today, accounting for 40% or more ofcases. Other causes include cystic fibrosis and congenital heart disease, both of which canmanifest as recurrent bleeding. In children younger than 4 years of age, foreign body aspirationshould be considered. Other, less common causes include bronchiectasis, clotting disorders,and trauma. Unlike in adults, neoplasm is an uncommon cause of hemoptysis in children.Massive hemoptysis is defined as acute bleeding of more than 240 mL in 24 hours.

The first step in the evaluation of a child who has hemoptysis is to determine the source ofthe bleeding. Bleeding from the upper airway (epistaxis) and bleeding from the uppergastrointestinal tract (hematemesis) sometimes can be mistaken for hemoptysis. Bloodproduced via hemoptysis usually is bright red and frothy, in contrast to the dark or “coffeeground” material produced in hematemesis. The pH of the material also can be helpful: analkaline pH is found in hemoptysis, and an acidic pH is found in hematemesis. Epistaxisgenerally can be established after careful examination of the oropharynx and nasopharynx.

Initial laboratory tests should include a complete blood count to assess for blood loss.Coagulation studies, such as prothrombin and partial thromboplastin time, to rule out acoagulation defect also are helpful, particularly because patients who have cystic fibrosis, suchas the girl in the vignette, may have derangements in their ability to clot due to vitamin Kdeficiency or liver disease. Other initial laboratory tests to consider include serum chemistrydeterminations to assess for renal pathology, urinalysis, and a sputum sample for Gram stain. Achest radiograph also should be ordered to aid in localizing the bleeding, although films appearnormal in up to one third of patients.

The source of the bleeding for the child in the vignette likely is either pulmonary infection orerosion of a blood vessel due to bronchiectasis. In neither case would methylprednisolone be ofbenefit. Initial therapy with antibiotics is appropriate only after collection of blood and sputumsamples if pneumonia is suspected. However, ceftriaxone is not effective therapy against theexpected pathogens. Most hemoptysis in children resolves spontaneously without the need forinvasive measures. The girl in the vignette has not had massive hemoptysis and has adequateoxygen saturation on room air, so endotracheal intubation is not indicated. Transfusion with O-negative blood is indicated only for hypotension due to acute massive blood loss.

Patients whose hemoptysis does not resolve spontaneously or who experience markedblood loss may require bronchoscopy to determine the source of the bleeding. Additionalmanagement options include bronchial washings with 1:10,000 epinephrine, endoscopictamponade with a balloon catheter, or partial lung resection.

References:

Flume PA, Yankaskas JR, Ebeling M, Hulsey T, Clark LL. Massive hemoptysis in cystic fibrosis.Chest. 2005;128:729-738. Abstract available at:http://www.ncbi.nlm.nih.gov/entrez/query.fcgi?orig_db=PubMed&db=PubMed&cmd=Search&term=Chest[Jour]+AND+729[page]+AND+2005[pdat]

Pianosi P, Al-sadoon H. Hemoptysis in children. Pediatr Rev. 1996;17:344-348

Sidman JD, Wheeler WB, Cabalka AK, Soumekh B, Brown CA, Wright GB. Management of acutepulmonary hemorrhage in children. Laryngoscope. 2001; 111:33-35. Abstract available at:http://www.ncbi.nlm.nih.gov/entrez/query.fcgi?orig_db=PubMed&db=PubMed&cmd=Search&term=Laryngoscope[Jour]+AND+33[page]+AND+2001[pdat]

Sloniewsky D, Green TP. Pulmonary hemorrhage, embolism and infarction. In: Behrman RE,Kliegman RM, Jenson HB, eds. Nelson Textbook of Pediatrics. 17th ed. Philadelphia, Pa:Saunders; 2004:1457-1458

Copyright © 2007 by the American Academy of Pediatrics page 166

Page 167: AAP MCQ 2007

2007 PREP SA on CD-ROM

Question: 81

A 1-month-old infant was born with an extensive erythematous patch (Item Q81A) on the rightside of his face that has grown at a rate matching his somatic growth.

Of the following, the child is at GREATEST risk for

A. congestive heart failure

B. consumptive coagulopathy

C. glaucoma

D. ocular axis occlusion

E. tethered spinal cord

Copyright © 2007 by the American Academy of Pediatrics page 167

Page 168: AAP MCQ 2007

2007 PREP SA on CD-ROM

Preferred Response: CCritique: 81

Port-wine stains (PWSs) are permanent capillary vascular malformations that are present frombirth, as reported for the infant in the vignette. Unlike hemangiomas, PWSs do not proliferate, butgrow proportionally with the child’s somatic growth. Although a PWS may occur anywhere on thebody, facial lesions should raise concern about possible Sturge-Weber syndrome (SWS). SWSis the association of a facial PWS in the V1 distribution with central nervous systemleptomeningeal angiomatosis (that may cause seizures) and/or glaucoma. One study found thatonly 8% of individuals who have a facial PWS have SWS; all of those affected had a PWS in thedistribution of V1 (upper lid) and/or V2 (lower lid). The risk of SWS was increased if the facialPWS was bilateral or if unilateral, involved the distribution of all three branches of the trigeminalnerve.

Congestive heart failure is not associated with SWS; it is an occasional complication inchildren who have multiple cutaneous hemangiomas and liver involvement or in those who havearteriovenous malformations. Consumptive coagulopathy (along with thrombocytopenia andanemia) is a feature of Kasabach-Merritt syndrome (Item C81A), in which patients have anatypical-appearing hemangioma (actually a hemangioepithelioma or tufted angioma). Ocular axisocclusion that prevents stimulation of the visual cortex is a concern with enlarging hemangiomaslocated near the eye (Item C81B). A tethered spinal cord results when a thickened, ropelike filumterminale remains firmly attached, reducing cord mobility. Tension on the cord and compromisedblood supply produce neurologic findings, such as abnormal gait and secondary enuresis. Amidline lumbosacral PWS might herald the existence of this or another occult spinal lesion.

References:

Frieden I, Enjolras O, Esterly N. Vascular birthmarks, other abnormalities of blood vessels andlymphatics. In: Schachner LA, Hansen RC, eds. Pediatric Dermatology. 3rd ed. St. Louis, Mo:Mosby, 2003:833-862

Halsam RHA. Tethered cord. In: Behrman RE, Kliegman RM, Jenson HB, eds. Nelson Textbookof Pediatrics. 17th ed. Philadelphia, Pa: WB Saunders Co; 2004:2050

Weston WL, Lane AT, Morelli JG. Vascular lesions: vascular tumors and malformations. In:Color Textbook of Pediatric Dermatology. 3rd ed. St. Louis, Mo: Mosby; 2002:187-199

Copyright © 2007 by the American Academy of Pediatrics page 168

Page 169: AAP MCQ 2007

2007 PREP SA on CD-ROM

Question: 82

A 14-year-old overweight girl developed an episode of abdominal pain and jaundice 1 week ago.At that time, total bilirubin measured 8 mg/dL (136.8 mcmol/L) and direct bilirubin measured 5mg/dL (85.5 mcmol/L). She was admitted to the hospital, where ultrasonography demonstrated adilated common bile duct and gallbladder stones. She underwent endoscopic retrogradecholangiopancreatography (ERCP), and a gallstone was extracted from her common bile duct.She presents to your office today for follow-up and is asymptomatic. Physical examinationdemonstrates a normal-size liver and spleen without tenderness. Repeat laboratory studiesreveal a total bilirubin of 2.5 mg/dL (42.8 mcmol/L) and direct bilirubin of 1.5 mg/dL (25.7mcmol/L). Transaminase measurements are normal.

Of the following, the MOST appropriate next step is to

A. evaluate for autoimmune hepatitis

B. initiate treatment with cholic acid

C. obtain hepatitis B serologic studies

D. repeat the ERCP

E. repeat the bilirubin measurement in 2 weeks

Copyright © 2007 by the American Academy of Pediatrics page 169

Page 170: AAP MCQ 2007

2007 PREP SA on CD-ROM

Preferred Response: ECritique: 82

Hepatobiliary obstruction is characterized by elevation of three laboratory parameters:conjugated (direct) bilirubin, alkaline phosphatase, and gamma-glutamyl transpeptidase. None ofthese markers is completely specific for hepatobiliary obstruction. For example, direct bilirubincan be elevated by intrahepatic cholestasis (as seen in Alagille syndrome and chlorpromazinetoxicity), alkaline phosphatase can be raised by bone disease, and gamma-glutamyltranspeptidase can be increased by anticonvulsants and alcohol consumption. Therefore, ifbiliary obstruction is suspected, hepatobiliary ultrasonography should be performed to evaluatefor bile duct dilation, gallstones, and hepatobiliary sludge. Gallstones composed primarily ofcalcium bilirubinate are referred to as pigment stones; those composed primarily of cholesterolare termed cholesterol stones. "Biliary sludge" is an amorphous material composed ofcholesterol, mucin, and bilirubinate that is thicker than regular bile but not as hard as a stone.Sludge in the common bile duct can contribute to both obstructive jaundice and pancreatitis.

The patient described in the vignette had a recent gallstone removed endoscopically.Generally, such patients can be followed clinically, and repeat laboratory studies are notnecessary if the patient is asymptomatic. This girl’s bilirubin concentration is mildly elevated,which is not uncommon after a recent episode of obstructive jaundice; repeat measurement in 2weeks can determine whether the hyperbilirubinemia has resolved. Repeat endoscopicretrograde cholangiopancreatography is not necessary at this time, although it should beconsidered if there is evidence of another retained stone. Transaminase values are normal,making both autoimmune and viral hepatitis unlikely. For a patient who has jaundice due topersistent biliary sludge, oral administration of the synthetic bile acid ursodeoxycholic acid mayhelp facilitate biliary flow and promote sludge dissolution. Cholic acid generally is not used in thechemical dissolution of biliary sludge and gallstones.

References:

Dourakis SP, Mayroyannis C, Alexopoulou A, Hadziyannis SJ. Prolonged cholestatic jaundiceafter endoscopic retrograde cholangiography. Hepatogastroenterology. 1997;44:677-680.Abstract available at:http://www.ncbi.nlm.nih.gov/entrez/query.fcgi?cmd=Retrieve&db=pubmed&dopt=Abstract&list_uids=9222670&query_hl=21&itool=pubmed_docsum

Lee SP, Nicholls JF, Park HZ. Biliary sludge as a cause of acute pancreatitis. N Engl J Med.1992;326:589-593. Abstract available at:http://www.ncbi.nlm.nih.gov/entrez/query.fcgi?cmd=Retrieve&db=pubmed&dopt=Abstract&list_uids=1734248&query_hl=24&itool=pubmed_docsum

Pratt DS, Kaplan MM. Evaluation of abnormal liver-enzyme results in asymptomatic patients. NEngl J Med. 2000;342:1266-1271. Available at:http://content.nejm.org/cgi/content/extract/342/17/1266

Copyright © 2007 by the American Academy of Pediatrics page 170

Page 171: AAP MCQ 2007

2007 PREP SA on CD-ROM

Question: 83

A 30 weeks' gestation infant weighing 1,500 g experiences respiratory distress that requiresassisted ventilation in the first 4 hours after birth. Some tachypnea with mild subcostal andintercostal retractions is evident on physical examination. Current ventilator parameters are:positive end-expiratory pressure of 4 cm H2O, peak inspiratory pressure of 22 cm H2O,synchronized ventilator rate of 40 breaths/min, and fraction of inspired oxygen of 0.8 to maintaina Pao2 of 60 mm Hg. A chest radiograph reveals low lung volumes, diffuse microatelectasis, anddense ground-glass opacities. You order surfactant for intratracheal administration.

Of the following, the MOST expected event in the 4 hours following surfactant administration is

A. decreased oxygen requirement

B. decreased pulmonary compliance

C. hypotension

D. increased inspiratory pressure

E. pulmonary hemorrhage

Copyright © 2007 by the American Academy of Pediatrics page 171

Page 172: AAP MCQ 2007

2007 PREP SA on CD-ROM

Preferred Response: ACritique: 83

Exogenous surfactant therapy for neonatal respiratory distress syndrome (RDS) is perhaps thegreatest example of bench-to-bedside research that has benefited newborns. Since the late1980s, when clinical trials demonstrated the efficacy and safety of intratracheally administeredsurfactant, it has become the standard therapy, in conjunction with assisted ventilation andjudicious oxygen therapy, in treating millions of newborns who have RDS. Upon diagnosing RDSin a preterm infant by clinical, radiographic, and blood gas parameters, early administration ofsurfactant results in obvious improvement in each of these broad categories.

Clinically, treated infants demonstrate less work of breathing, improved oxygenation andventilation, a reduction in assisted ventilation (apparent by diminished Fio2), decreased meanairway pressure (most readily accomplished by a reduction in inspiratory pressure), andimproved pulmonary compliance (reflected in improved changes in lung volume per unit ofinspiratory pressure administered). Radiographically, the degree of lung aeration improves asmicroatelectasis is overcome, and lung volumes typically improve. Blood gas measurementsindicate improved ventilation and oxygenation and resolution of acidemia.

Hypotension is a complication of RDS in the critically ill newborn and may reflect impairedcirculatory volume, venous return to the heart (preload), and cardiac output. With significanthypoxemia and acidemia, myocardial function may be impaired. Additionally, the ductusarteriosus may remain patent as a result of poor neonatal oxygenation. These conditions are notdirectly corrected by surfactant administration, but they are ameliorated as lung diseaseimproves.

Pulmonary hemorrhage is an infrequent adverse effect of surfactant therapy that is seenmost commonly in the extremely low-birthweight infant (<1,000 g) and is associated with a patentductus arteriosus.

References:

American Academy of Pediatrics Committee on Fetus and Newborn. Surfactant replacementtherapy for respiratory distress syndrome. Pediatrics. 1999;103: 684-685. Available at:http://pediatrics.aappublications.org/cgi/content/full/103/3/684

Bancalari E, Claure N, Gonzalez A. Patent ductus arteriosus and respiratory outcome inpremature infants. Biol Neonate. 2005;88:192-201. Abstract available at:http://www.ncbi.nlm.nih.gov/entrez/query.fcgi?cmd=Retrieve&db=pubmed&dopt=Abstract&list_uids=16210841&query_hl=30&itool=pubmed_docsum

Halliday HL. History of surfactant from 1980. Biol Neonate. 2005;87:317-322. Available at:http://content.karger.com/produktedb/produkte.asp?typ=fulltext&file=BON2005087004317

Philip AGS, Avery ME. Historical perspectives: surfactant deficiency to surfactant use.NeoReviews. 2002;3:e239-e242. Available at:http://neoreviews.aappublications.org/cgi/content/full/3/12/e239

Rodriguez RJ, Martin RJ, Fanaroff AA. Respiratory distress syndrome and its management. In:Martin RJ, Fanaroff AA, Walsh MC, eds. Fanaroff and Martin’s Neonatal-Perinatal Medicine:Diseases of the Fetus and Infant. 8th ed. Philadelphia, Pa: Mosby-Elsevier; 2006:1097-1107

Copyright © 2007 by the American Academy of Pediatrics page 172

Page 173: AAP MCQ 2007

2007 PREP SA on CD-ROM

Question: 84

You are examining a 2-day-old term newborn prior to discharge. On physical examination, younote a 5 x 5 cm mass in the right abdomen. The remainder of her physical examination findingsare normal.

Of the following, the MOST likely diagnosis is

A. hydrometrocolpos

B. lymphoma

C. multicystic dysplastic kidney

D. neuroblastoma

E. Wilms tumor

Copyright © 2007 by the American Academy of Pediatrics page 173

Page 174: AAP MCQ 2007

2007 PREP SA on CD-ROM

Preferred Response: CCritique: 84

The differential diagnosis for a child who has an abdominal mass differs according to age. In theneonate, the most common causes of abdominal or flank mass are hydronephrosis andmulticystic dysplastic kidney. Hydronephrosis may be unilateral, as in ureteropelvic orureterovesicular junction obstruction, or bilateral, as in posterior urethral valves. Multicysticdysplastic kidney presents with a unilateral flank mass, as reported for the newborn in thevignette. The diagnosis can be supported with renal ultrasonography, which shows multiplecysts and little renal parenchyma.

Hydrometrocolpos may result from imperforate hymen. Affected infants may present with amidline lower abdominal mass that represents accumulation of mucus and other fluid in thevagina. Other, less common causes of abdominal mass in the neonate include solid renaltumors, adrenal hemorrhage, intestinal duplication cyst, and renal vein thrombosis.

Lymphoma is very rare in infants younger than 1 year of age. Neuroblastoma may presentwith an abdominal mass in the neonatal period, but it is far more common later in infancy andtoddlerhood. Wilms tumor presents with a unilateral flank mass, but it is rarely diagnosed before6 months of age.

References:

Chandler JC, Gauderer MW. The neonate with an abdominal mass. Pediatr Clin North Am.2004;51:979-997. Abstract available at:http://www.ncbi.nlm.nih.gov/entrez/query.fcgi?orig_db=PubMed&db=PubMed&cmd=Search&term=%22Pediatric+clinics+of+North+America%22[Jour]+AND+2004[pdat]+AND+Chandler[author]

Jaffe N, Huff V. Neoplasms of the kidney. In: Behrman RE, Kliegman RM, Jenson HB, eds:Nelson Textbook of Pediatrics. 17th ed. Philadelphia, Pa: WB Saunders Co; 2004:1711-1714

Copyright © 2007 by the American Academy of Pediatrics page 174

Page 175: AAP MCQ 2007

2007 PREP SA on CD-ROM

Question: 85

The parents of a child who was diagnosed at birth with Beckwith-Wiedemann syndrome bring inthe baby for his 2-month evaluation. They ask about future health problems and his prognosisnow that his omphalocele has been repaired.

Of the following, the child is MOST at risk for

A. acute lymphocytic leukemia

B. astrocytoma

C. Hodgkin disease

D. rhabdomyosarcoma

E. Wilms tumor

Copyright © 2007 by the American Academy of Pediatrics page 175

Page 176: AAP MCQ 2007

2007 PREP SA on CD-ROM

Preferred Response: ECritique: 85

Children who have hemihypertrophy and other somatic overgrowth syndromes are at higher riskof developing Wilms tumor and hepatoblastomas than the general population. Beckwith-Wiedemann syndrome, which involves a chromosome 11p15.5 gene deletion, is characterizedby somatic overgrowth and macroglossia (Item C85A). Affected children have a 3% to 5% riskof developing Wilms tumor compared with an incidence of about 8.5 per 1 million children in thegeneral population. Wilms tumor occurs in hereditary and nonhereditary patterns. It presents asan asymptomatic abdominal mass in two thirds of cases. Surveillance for the development ofthis tumor should include measurement of serum alpha-fetoprotein, urinalysis for microscopichematuria, and abdominal ultrasonography every 3 months for at least the first 5 years afterbirth.

The most classic association with Wilms tumor is the WAGR syndrome (Wilms, aniridia,genitourinary anomalies, and mental retardation), which is related to chromosome 11p13deletion.

Although acute lymphocytic leukemia, astrocytoma, Hodgkin disease, andrhabdomyosarcoma are relatively common neoplasms in children, none is chromosomallyrelated to the Beckwith-Wiedemann syndrome.

References:

Chintagumpala MM, Steuber CP. Wilms tumor. In: McMillan JA, DeAngelis CD, Feigin RD,Warshaw JB, eds. Oski’s Pediatrics: Principles and Practice. 3rd ed. Philadelphia, Pa: LippincottWilliams & Wilkins; 1999:1515-1516

Copyright © 2007 by the American Academy of Pediatrics page 176

Page 177: AAP MCQ 2007

2007 PREP SA on CD-ROM

Question: 86

A 4-year-old girl presents to the emergency department with a 3-day history of left-sidedweakness. On physical examination, she has a temperature of 99°F (37.2°C), pulse of 50beats/min, respiratory rate of 24 breaths/min, and blood pressure of 118/78 mm Hg. She cannotabduct her right eye, but has normal funduscopic examination results. Strength on the left is 4/5in the upper and lower extremity and on the right is normal.

Of the following, the MOST appropriate next step in the evaluation of this child is

A. cerebral angiography

B. computed tomography scan of the brain

C. electroencephalography

D. positron emission tomography scan of the head

E. single-photon emission computed tomography scan of the brain

Copyright © 2007 by the American Academy of Pediatrics page 177

Page 178: AAP MCQ 2007

2007 PREP SA on CD-ROM

Preferred Response: BCritique: 86

Increased intracranial pressure (ICP) is an emergency that must be recognized quickly bypediatricians. Elevated ICP often is insidious and nonspecific initially. Among affected school-agechildren, declining academic performance, fatigue, behavioral changes, and vague intermittentheadaches are common. Over time, morning headaches, especially pain at the occipital orfrontal region, along with vomiting and lethargy ensue. Horizontal diplopia becomes apparentonce the abducens nerve becomes compressed by the tentorium. Papilledema may develop ifthe pressure is longstanding, but the optic fundus can appear normal early in the course ofincreased ICP. Hypertension with bradycardia and variable respiratory pattern (Cushing triad)may indicate critically elevated ICP that is nearing cerebral herniation.

Irritability, anorexia, failure to thrive, and even developmental regression can be early signsof increased ICP in infants. Increased ICP may lead to macrocephaly, splitting of the cranialsutures, or a bulging anterior fontanelle. The “setting sun” sign, a downward deviation of theeyes, may be seen with attendant hydrocephalus.

Once there is concern about elevated ICP, immediate neuroimaging is required withcomputed tomography (CT) scan of the head or magnetic resonance imaging (MRI) of the brain.CT scan without contrast can be performed in just minutes, without conscious sedation.Although MRI provides better definition of the underlying pathology, such neuroimaging takesabout 30 or more minutes and may require sedation. Either modality can determine whetherthere is increased ICP from a mass lesion.

The hypertension, bradycardia, and right abducens paresis reported for the child in thevignette are worrisome for increased ICP, and she should receive a CT scan of the brain forevaluation. Cerebral angiography only delineates the intracranial vasculature. Positron emissiontomography and single-photon CT scan determine whether a lesion is metabolically active (eg,tumor or infection). Single-photon CT scan may be useful in identifying the structural origin ofepilepsy immediately following a seizure. Electroencephalography is used to evaluate seizuresand level of consciousness, but has limited, if any, utility in the delineation of mass lesions of thebrain.

References:

DiCarlo JV, Frankel LR. Stabilization of the critically ill child: neurologic stabilization. In: BehrmanRE, Kliegman RM, Jenson HB, eds. Nelson Textbook of Pediatrics. 17th ed. Philadelphia, Pa:WB Saunders Co; 2004:308-311

Larsen GY, Goldstein B. Consultation with the specialist: increased intracranial pressure. PediatrRev. 1999;20:234-239. Available at:http://pedsinreview.aappublications.org/cgi/content/full/20/7/234

Copyright © 2007 by the American Academy of Pediatrics page 178

Page 179: AAP MCQ 2007

2007 PREP SA on CD-ROM

Question: 87

A 12-year-old girl presents to your office for the first time with a swollen, painful, erythematousright knee joint. She tells you that her left knee felt and looked similar yesterday, but now feelsnormal. She also is easily fatigued and has had fever. On physical examination, she has atemperature of 101.7°F (38.7°C), a heart rate of 125 beats/min, a respiratory rate of 24breaths/min, and a blood pressure of 120/78 mm Hg. Her lungs are clear. On auscultation, younote a 3/6 holosystolic murmur (Item Q87A) at the cardiac apex with radiation to the axilla.

Of the following, the BEST plan for management of this patient's joint swelling includes

A. antibiotic therapy with doxycycline

B. anti-inflammatory therapy with aspirin

C. aspiration of the right knee joint

D. heat, elevation, and splinting of the right knee

E. immunotherapy with azathioprine

Copyright © 2007 by the American Academy of Pediatrics page 179

Page 180: AAP MCQ 2007

2007 PREP SA on CD-ROM

Preferred Response: BCritique: 87

Rheumatic fever follows infections caused by certain strains of group A streptococci. Thisinflammatory disease affects the heart, central nervous system, joints, subcutaneous tissue,and skin. Rheumatic fever is a significant cause of cardiovascular morbidity in developingcountries and continues to be seen in the United States, despite decades of declining incidence.The disease can be viewed as a nonsuppurative complication of group A streptococcalinfections of the upper respiratory tract that typically occurs in children between the ages of 4and 15 years. Patients who have had rheumatic fever in the past have an increased incidence ofrecurrent disease. The pathognomonic cardiac lesion in rheumatic fever is the Aschoff body,which is a perivascular infiltrate of large cells that have polymorphous nuclei and basophiliccytoplasm and are arranged in a rosette pattern.

In 1992, the American Heart Association presented guidelines for the diagnosis of rheumaticfever that included an update of the original Jones criteria. The clinical manifestations ofrheumatic fever can be divided into major criteria and minor criteria. The major manifestations ofthe disease are:

•Carditis

•Polyarthritis

•Subcutaneous nodules

•Erythema marginatum•Chorea

Minor manifestations are:

•Arthralgia (not considered a criterion if polyarthritis is present)

•Fever

•Increased acute-phase reactants (erythrocyte sedimentation rate, C-reactive protein)•Prolonged P-R interval

The diagnosis requires two of the major manifestations or one of the major manifestationsplus two of the minor manifestations.

The carditis of rheumatic fever may involve any of the three layers of the heart: theendocardium, myocardium, or pericardium. If all three layers are involved, pancarditis exists.Patients who have carditis may not have symptoms, but they likely have signs on cardiacexamination. When signs or symptoms are present, they most often are the result ofendocarditis, which is caused by hyaline degeneration of the affected valve. The mitral valve isthe most commonly affected valve, followed by the aortic valve. It is rare for the tricuspid orpulmonary valve to be affected. Therefore, the clinician should auscultate for the apical,holosystolic murmur that radiates to the left axilla and represents mitral regurgitation (ItemC87A). If the mitral regurgitation is severe, there may be a low-frequency mid- to late diastolicmurmur of relative mitral stenosis, which is referred to as the Carey Coombs murmur. Theclinician also should listen for the early diastolic decrescendo murmur of aortic regurgitation(Item C87B), which may be exaggerated when the patient is in the sitting position, leaningforward. Signs and symptoms of congestive heart failure occur in about 5% of patients whohave rheumatic carditis.

The patient described in the vignette has fever and two major Jones criteria: migratoryarthritis and carditis represented by mitral regurgitation. Therefore, she meets the diagnosticcriteria for rheumatic fever. The acute management of rheumatic fever encompasses threeprimary strategies: treat the infection causing the disease, control and alleviate the symptoms,and provide supportive care as indicated. Because rheumatic fever is caused by group A beta-hemolytic streptococci, antibiotic therapy should be directed at eradication of this pathogen.

Copyright © 2007 by the American Academy of Pediatrics page 180

Page 181: AAP MCQ 2007

2007 PREP SA on CD-ROM

Typically, a 10-day course of oral penicillin V is adequate. Alternatively, intramuscular orintravenous penicillin preparations may be used. For those who have a penicillin allergy, analternate antibiotic such as erythromycin may be used. Doxycycline, a treatment for Lymearthritis, is not the best choice for management of arthritis resulting from acute rheumatic fever.

Anti-inflammatory agents are important for symptomatic relief and to combat theinflammatory process. Salicylates are particularly effective for the migratory arthritis, withimprovement often occurring in the first 12 to 24 hours of therapy. In addition, these medicationsfrequently relieve the accompanying fever. Aspirin typically is administered in high doses (80 to100 mg/kg per day to a serum level of 25 mg/dL) for several weeks before a gradual tapering isbegun. For patients who cannot tolerate aspirin, the use of other nonsteroidal anti-inflammatorydrugs may be considered. Steroids generally are reserved for patients who demonstratemoderate-to-severe carditis with congestive heart failure. Supportive care has historicallyincluded the use of bed rest, especially during the acute phase of the illness.

Joint aspiration, immunotherapy, and elevation and immobilization of the right knee do notaddress the underlying problem for the patient in the vignette.

References:

Ayoub EM. Rheumatic fever. In: Moller JH, Hoffman JIE, eds. Pediatric CardiovascularMedicine. Philadelphia, Pa: Churchill Livingstone; 2000:740-751

Dajani A, Taubert K, Ferrieri P, Peter G, Shulman S. Treatment of acute streptococcalpharyngitis and prevention of rheumatic fever: a statement for health professionals. Committeeon Rheumatic Fever, Endocarditis, and Kawasaki Disease of the Council on CardiovascularDisease in the Young, American Heart Association. Pediatrics. 1995;96:758-764. Abstractavailable at: http://pediatrics.aappublications.org/cgi/content/abstract/96/4/758

Ferrieri P. Proceedings of the Jones Criteria workshop. Circulation. 2002;106:2521-2523.Available at: http://circ.ahajournals.org/cgi/content/full/106/19/2521

Hoffman J. Rhuematic fever. In: Rudolph CD, Rudolph AM, Hostetter MK, Lister G, Siegel NJ,eds. Rudolph’s Pediatrics. 21st ed. New York, NY: McGraw Hill; 2003:1901-1904

Special Writing Group of the Committee on Rheumatic Fever, Endocarditis, and KawasakiDisease of the Council of Cardiovascular Disease in the Young of the American HeartAssociation. Guidelines for the diagnosis of rheumatic fever. Jones criteria, 1992 update. JAMA.1992;268;2069-2073. Abstract available at:http://www.ncbi.nlm.nih.gov/entrez/query.fcgi?orig_db=PubMed&db=PubMed&cmd=Search&term=%22JAMA+:+the+journal+of+the+American+Medical+Association%22[Jour]+AND+2069[page]+AND+1992[pdat]

Copyright © 2007 by the American Academy of Pediatrics page 181

Page 182: AAP MCQ 2007

2007 PREP SA on CD-ROM

Question: 88

You are discussing the common indications for chromosome analysis with a group of third-yearmedical students.

Of the following, the MOST appropriate statement to include in your discussion is that

A. a blood karyotype should be obtained in any newborn who has multiple congenital anomaliesand growth retardation

B. a blood karyotype should be obtained only for a girl who has short stature if a buccal smear isnegative

C. chromosome analysis is not necessary to confirm the diagnosis of Down syndrome if themajor clinical features are present

D. chromosome analysis must be obtained to assess the reproductive risk for a woman who hasa sibling who has trisomy 21

E. routine chromosome analysis is adequate for the diagnosis of microdeletion syndromes, suchas DiGeorge syndrome

Copyright © 2007 by the American Academy of Pediatrics page 182

Page 183: AAP MCQ 2007

2007 PREP SA on CD-ROM

Preferred Response: ACritique: 88

Blood chromosome analysis can detect both abnormalities in chromosome number and grossstructural defects of individual chromosomes. The numerical abnormalities can involve extrasets of all the chromosomes (eg, triploidy), the presence of a single extra chromosome (eg,trisomy), or the absence of a chromosome (eg, monosomy). Structural changes includedeletions, duplications, translocations, and inversions. In general, chromosomal defects areassociated with a combination of dysmorphic features; multiple congenital anomalies; growthdeficiency; and variable degrees of developmental delay, learning problems, and mentalretardation. Therefore, it is appropriate to obtain a chromosome analysis in any newborn whohas multiple congenital anomalies and growth retardation.

Turner syndrome (TS) results from complete absence or structural abnormality of an Xchromosome. Growth retardation is one feature of the syndrome. Although features of TS ofteninclude a webbed neck, congenital heart disease, and ovarian dysgenesis, affected girls mayhave short stature alone. Because a buccal smear is not a reliable screening test for TS, allpatients in whom TS is suspected should have a blood karyotype.

Chromosome analysis should be obtained in every infant suspected of having Downsyndrome, which can result from trisomy 21 or structural rearrangements involvingchromosome 21. In particular, it is important to determine if there is a translocation involvingchromosome 21 because one of the parents also may carry the structural rearrangement in abalanced form. In this case, the risk to future pregnancies is increased markedly. The sibling ofan individual who has trisomy 21 is not at any additional risk for having a child who has achromosome abnormality.

Microdeletion syndromes such as 22q11 syndrome usually cannot be detected on routinechromosome analysis. Therefore, it is recommended that fluorescent in situ hybridizationstudies be carried out using a molecular probe for the precise area of interest to determine if amicrodeletion is present.

References:

Hoyme HE. Patterns of inheritance. In: Behrman RE, Kliegman RM, Jenson HB, eds. NelsonTextbook of Pediatrics. 17th ed. Philadelphia Pa: Saunders; 2004:376-382

Hsu LYF. Cytogenetic abnormalities in the embryo, fetus, and infant. UpToDate. 2006;14.1.Available at:http://www.utdol.com/utd/content/topic.do?topicKey=antenatl/5719&type=P&selectedTitle=66~81

Pinsky L. Overview of genetic assessment. UpToDate. 2006; 14.1. Available at:http://www.utdol.com/utd/content/topic.do?topicKey=genr_med/27636&type=A&selectedTitle=1~85

Roizen NJ, Stark AR. Epidemiology and genetics of Down syndrome. UpToDate. 2006:14.1.Available at:http://www.utdol.com/utd/content/topic.do?topicKey=dis_chld/2412&type=P&selectedTitle=27~54

Copyright © 2007 by the American Academy of Pediatrics page 183

Page 184: AAP MCQ 2007

2007 PREP SA on CD-ROM

Question: 89

During a sports physical, you note that a 14-year-old boy has thoracic kyphosis (Item Q89A).He cannot pull his shoulders back to correct the curvature in the standing position. Thekyphosis persists in the prone position. A standing lateral radiograph reveals a kyphotic curve of50 degrees between T3 and T12, with anterior wedging of vertebrae at T7 and T8 and T9. Thepatient is at Risser stage 4.

Of the following, the MOST appropriate management plan for this patient is to

A. follow up the patient in 6 months

B. obtain pulmonary function tests

C. obtain renal ultrasonography

D. obtain tissue typing for human leukocyte antigen B27

E. recommend rigorous hyperextension exercises

Copyright © 2007 by the American Academy of Pediatrics page 184

Page 185: AAP MCQ 2007

2007 PREP SA on CD-ROM

Preferred Response: ACritique: 89

Kyphosis refers to the convex alignment in the sagittal plane of the thoracic spine. The normalrange of convexity is 20 to 40 degrees. Those who have an increased convex alignment have aclinical sign of round back that may be flexible (postural) or structural. A structural, nonflexible,round back is called idiopathic kyphosis or Scheuermann disease.

A round back commonly is noticed during observations of adolescent posture. During thephysical examination, the round back is viewed best from the side with the patient in the forwardbending position. Affected patients who have a flexible kyphosis can correct the posture activelywith hyperextension in both the standing and prone positions. Although the posture often is aconcern of parents, such flexible kyphosis has no associated adverse health consequences.

Scheuermann disease, as exhibited by the patient described in the vignette, is a fixedkyphosis (Item C89A) that develops around the time of puberty, most commonly in the thoracicregion, and affects 7% to 8% of the population. Affected patients cannot correct the round backappearance with active hyperextension. Other findings on physical examination include anaccentuated dorsal kyphosis with a sharp contour (or steep apex), a compensatory lumbarlordosis with a protuberant abdomen, and less frequently, a compensatory lordosis of thecervical spine. Neurologic examination findings generally are normal. A definitive diagnosis ofScheuermann disease of the thoracic spine is based on an increased kyphotic angle and thepathognomonic finding on the standing lateral radiograph of anterior wedging (Item C89B) of atleast 5 degrees in three or more adjacent vertebrae. The standard radiographic evaluationshould include both standing anteroposterior and lateral radiographs of the entire spine. Patientswho have Scheuermann kyphosis also may have spondylolisthesis and scoliosis, which aretreated as separate conditions.

Treatment of Scheuermann disease depends on several factors, including the age of thepatient, degree of skeletal maturity, degree of deformity, progression of the deformity, pain in theapical region of the kyphosis, and cosmetic and psychosocial factors. Neurologic complicationsand cardiopulmonary complaints are rare. Symptoms, including pain, usually are associated withmore severe, often short-segment deformities that are greater than 90 degrees. Fortunately,Scheuermann disease is primarily a cosmetic problem for most patients. Treatment decisionsare individualized in consultation with an orthopedic surgeon; there are no absolute guidelines.

The patient described in the vignette is asymptomatic and skeletally mature (Risser stage 4[indicating near-complete fusion of the iliac crest apophysis]) (Item C89C) and has a curve thatis modest and cosmetically acceptable. Therefore, he can be managed with education andguidance and followed without treatment. The use of the Milwaukee brace early in the growthspurt may be recommended in adolescents who have kyphotic deformities of at least 45degrees and often greater than 60 degrees. The bracing is continued until skeletal maturity isattained. However, bracing therapy remains controversial; long-term benefits are debated byexperts. Detractors believe that the gains from bracing are slowly lost with subsequent long-term progression.

A physical therapy program probably will not alter the natural progression of the disease, buta program that incorporates thoracic extension exercises and emphasizes general conditioningmay aid in reducing symptoms. At present, surgical intervention usually is reserved for patientswho have a mature skeleton and a severe, painful deformity. More options may becomeavailable as advances are made in microsurgical techniques, instruments, and materials for thetreatment of spinal conditions. Pulmonary function tests are not indicated for a patient who hasan asymptomatic thoracic kyphosis of 50 degrees. Restrictive lung disease is uncommon evenin patients who have severe curves of 90 degrees or more.

Scheuermann disease is not associated with renal abnormalities and, therefore,ultrasonography is not indicated. Human leukocyte antigen B27 is associated withspondyloarthritis and Reiter syndrome, not Scheuermann disease.

References:

Lowe TG. Scheuermann’s disease. Orthop Clin North Am. 1999;30:475-487. Abstract availableat:

Copyright © 2007 by the American Academy of Pediatrics page 185

Page 186: AAP MCQ 2007

2007 PREP SA on CD-ROM

http://www.ncbi.nlm.nih.gov/entrez/query.fcgi?orig_db=PubMed&db=PubMed&cmd=Search&term=%22The+Orthopedic+clinics+of+North+America%22[Jour]+AND+475[page]+AND+1999[pdat]

Murray PM, Weinstein SL, Spratt KF. The natural history and long-term follow-up ofScheuermann kyphosis. J Bone Joint Surg Am. 1993;75:236-248

Thompson GH. The spine. In: Behrman RE, Kliegman RM, Jenson HB, eds. Nelson Textbook ofPediatrics. 17th ed. Philadelphia, Pa: Saunders; 2004:2280-2288

Copyright © 2007 by the American Academy of Pediatrics page 186

Page 187: AAP MCQ 2007

2007 PREP SA on CD-ROM

Question: 90

A 16-year-old girl who was treated successfully for leukemia that presented as a mediastinalmass at 2 years of age comes to your office complaining of a "lump" in her neck (Item Q90A) of3 weeks' duration. On physical examination, you note a firm, hard, fixed mass that seems to bepart of the left lobe of the thyroid and scattered shotty lymphadenopathy on the left.

Of the following, the MOST informative laboratory study for this patient is

A. fine-needle aspiration thyroid biopsy

B. serum calcitonin concentration

C. technetium thyroid scan

D. thyroid ultrasonography

E. 131-iodine thyroid scan

Copyright © 2007 by the American Academy of Pediatrics page 187

Page 188: AAP MCQ 2007

2007 PREP SA on CD-ROM

Preferred Response: ACritique: 90

All children who have discrete thyroid masses should be referred to an endocrinologist. Theprevious chemotherapy and irradiation to the chest probably received by the girl in the vignettefor treatment of leukemia places her at risk for a thyroid malignancy. Thyroid cancer accountsfor about 10% of the second malignancies among cancer survivors and is especially commonamong survivors of Hodgkin lymphoma. A thyroid mass associated with lymphadenopathy thatis firm, hard, and fixed is much more likely to be malignant than a soft, mobile mass withoutassociated adenopathy. A fine-needle aspiration biopsy can provide pathologic confirmation ofmalignancy in most cases, leading to definitive surgery and other necessary therapy. The mostcommon thyroid malignancies in children are papillary and follicular carcinomas.

Serum calcitonin concentration should be measured if there is suspicion of medullarycarcinoma of the thyroid, but this malignancy is not found frequently after irradiation. A thyroidscan (Item C90A), whether technetium or 131-iodine, probably can help to identify the nodule,but the differentiation between “hot” and “cold” nodules, once so important in diagnosis, nolonger is made. Thyroid malignancies may be found in lesions that take up iodine or technetium.Thyroid ultrasonography may be very useful for choosing the area to biopsy and assessing theextent of the thyroid lesion. Some suggest that biopsy performed with ultrasonography is morelikely to be informative, but in this case, the pathologic reading of the aspiration biopsy of thelarge mass should be definitive.

References:

Halac I, Zimmerman D. Thyroid nodules and cancers in children. Endocrinol Metab Clin NorthAm. 2005;34:725-744. Abstract available at:http://www.ncbi.nlm.nih.gov/entrez/query.fcgi?orig_db=PubMed&db=PubMed&cmd=Search&term=%22Endocrinology+and+metabolism+clinics+of+North+America%22[Jour]+AND+725[page]+AND+2005[pdat]

Amrikachi M, Ponder TB, Wheeler TM, Smith D, Ramzy I. Thyroid fine-needle aspiration biopsyin children and adolescents: experience with 218 aspirates. Diagn Cytopathol. 2005;32:189-192.Abstract available at:http://www.ncbi.nlm.nih.gov/entrez/query.fcgi?orig_db=PubMed&db=PubMed&cmd=Search&term=%22Diagnostic+cytopathology%22[Jour]+AND+189[page]+AND+2005[pdat]

Corrias A, Einaudi S, Chiorboli E, et al. Accuracy of fine needle aspiration biopsy of thyroidnodules in detecting malignancy in childhood: comparison with conventional clinical, laboratory,and imaging approaches. J Clin Endocrinol Metab. 2001;86:4644-4648. Available at:http://jcem.endojournals.org/cgi/content/full/86/10/4644

Copyright © 2007 by the American Academy of Pediatrics page 188

Page 189: AAP MCQ 2007

2007 PREP SA on CD-ROM

Question: 91

You and your colleagues are discussing implementation of routine developmental screening inyour office.

In your research, you have found that

A. early identification is effective in improving educational outcome

B. most developmental screening tests have a sensitivity of approximately 90%

C. screening for behavioral and developmental concerns requires separate questionnaires

D. subsequent screening is not necessary after children pass two screening tests

E. the use of developmental screening tools requires extensive staff training

Copyright © 2007 by the American Academy of Pediatrics page 189

Page 190: AAP MCQ 2007

2007 PREP SA on CD-ROM

Preferred Response: ACritique: 91

Developmental or behavioral disabilities are seen in approximately 15% to 18% of children in theUnited States. The American Academy of Pediatrics recommends that pediatricians usevalidated screening tools at each health supervision visit, but time constraints and otherpressing issues can make it difficult to comply with this recommendation. However, earlyintervention has been shown to be effective in improving long-term educational and vocationaloutcomes for children who have developmental or behavioral disabilities as well as preventingteen pregnancy and criminality.

The hundreds of assessment measures available in the United States can add topractitioners’ confusion when choosing a screening test. Different tools are used in varying agegroups and have different screening focuses (behavioral versus developmental versuspsychosocial). However, children who have disabilities and are not identified on an initialscreening are likely to be identified at subsequent screenings, and children who areoveridentified likely have unique needs even though they might not qualify for special services.

Although separate questionnaires are available for behavioral and developmental screening,tools also are available to screen for both areas in the same questionnaire. Screening should beperformed at all health supervision visits, even if the child has passed several screening tests.Developmental screening is simple and can be completed by parent questionnaire oradministered by staff who have minimal training (Item C91A).

References:

American Academy of Pediatrics Committee on Children with Disabilities. Developmentalsurveillance and screening of infants and young children. Pediatrics. 2001;108:192-196.Available at: http://pediatrics.aappublications.org/cgi/content/full/108/1/192

Glascoe FP. Early detection of developmental and behavioral problems. Pediatr Rev.2000;21:272-280. Available at: http://pedsinreview.aappublications.org/cgi/content/full/21/8/272

Copyright © 2007 by the American Academy of Pediatrics page 190

Page 191: AAP MCQ 2007

2007 PREP SA on CD-ROM

Question: 92

An 18-year-old pregnant girl presents to your clinic with complaints of discomfort in her kneesand hands. She denies any swelling or erythema of these areas or recent trauma. She takes noregular medications, and she has been healthy until 6 days ago, when she developed thesecomplaints. She did receive a rubella vaccination about 1 month ago because on her first visit toan obstetrician she was found to have a negative serum titer to rubella. Her physicalexamination findings are normal.

Of the following, the MOST likely diagnosis is

A. adverse effect of the rubella vaccine

B. anicteric hepatitis B infection

C. poststreptococcal arthritis

D. reactive arthritis due to Salmonella sp

E. recent infection with parvovirus

Copyright © 2007 by the American Academy of Pediatrics page 191

Page 192: AAP MCQ 2007

2007 PREP SA on CD-ROM

Preferred Response: ACritique: 92

The patient described in the vignette has joint complaints due to the rubella vaccine. Rubella-associated joint complaints can follow natural infection or immunization. Complications ofimmunization or natural disease usually occur in women and are uncommon in preadolescentchildren and males. Arthralgias of the knees and hands usually begin within 7 days of naturalillness and 10 to 28 days after immunization.

Postinfectious arthritis/arthralgia usually follows a viral infection and is short-lived comparedwith reactive arthritis/arthralgia, which follows a gastrointestinal or genitourinary infection, has avariable course, and may progress to a chronic spondyloarthropathy. Postinfectious arthropathyfollowing viral infections appears to involve the deposition of immune complexes containing viralantigens into the joints. Reactive arthropathy may represent an autoimmune response involvingT lymphocytes that cross-react to antigens present in the joints.

Postinfectious or reactive joint disease can be diagnosed once the symptoms haveresolved. Acute joint complaints involving only one joint should raise the suspicion of anotherdiagnosis (eg, pyogenic infection of a joint). It is important to remember that some otherautoimmune diseases can present with joint complaints (eg, inflammatory bowel disease,juvenile rheumatoid arthritis). No specific treatment is necessary for postinfectious or reactivejoint complaints except for relief of pain or discomfort with nonsteroidal anti-inflammatorymedications.

Anicteric hepatitis B infection, poststreptococcal arthritis, reactive arthritis due to Salmonellasp, and a recent infection with parvovirus all can produce joint complaints, but the history ofrubella immunization described for the girl in the vignette makes these diagnoses less likely.

References:

Keat A. Reactive arthritis or post-infective arthritis? Best Pract Res Clin Rheumatol.2002;16:507-522. Abstract available at:http://www.ncbi.nlm.nih.gov/entrez/query.fcgi?db=pubmed&cmd=Retrieve&dopt=Abstract&list_uids=12406424&query_hl=8&itool=pubmed_DocSum

Miller ML, Cassidy JT. Postinfectious arthritis and related conditions. In: Behrman RE, KliegmanRM, Jenson HB, eds. Nelson Textbook of Pediatrics. 17th ed. Philadelphia, Pa: WB SaundersCo; 2004:808-809

Copyright © 2007 by the American Academy of Pediatrics page 192

Page 193: AAP MCQ 2007

2007 PREP SA on CD-ROM

Question: 93

You have been the primary practitioner for a 15-year-old girl since she was 3 years old. Shenever has been hospitalized, and her history, vital signs, and physical examination findings onthis health supervision visit are normal. At each of the past two yearly visits, urinalysis revealedno abnormalities except 2+ protein. Her urinalysis today again reveals 2+ protein with no otherabnormalities. Her complete blood count, measurements of serum electrolytes and serumcomplements, and antinuclear antibody test results are all normal. Two successive 24-hoururine collections reveal 321 and 387 mg protein. You now refer the girl to a pediatricnephrologist.

Of the following, the MOST important next step for the pediatric nephrologist is to

A. obtain a first morning urine specimen

B. obtain renal ultrasonography

C. perform a renal biopsy

D. repeat the 24-hour urine collection

E. repeat the urinalysis

Copyright © 2007 by the American Academy of Pediatrics page 193

Page 194: AAP MCQ 2007

2007 PREP SA on CD-ROM

Preferred Response: ACritique: 93

Detection of proteinuria on routine urine dipstick evaluation is an uncommon but perplexingdilemma for pediatricians because proteinuria may suggest underlying renal disease or mayrepresent a benign condition. It is incumbent on the pediatrician to determine the appropriatesteps to be undertaken in a child who has a urine dipstick positive for protein.

Among the many benign conditions causing a positive dipstick result for urinary protein areone of the following: a very concentrated urine (specific gravity =1.020), alkaline urine (pH =7.5),or the presence of mucoproteins. In addition, acute illness may result in a small degree ofproteinuria. The most important feature of these conditions is that the urinary dipstick resultalmost never exceeds a reading of 1+.

In the absence of these conditions, it is vital to determine if the proteinuria is transient(orthostatic) or fixed. Orthostatic proteinuria (OP) occurs during the daytime but disappearswhen the person is supine (eg, asleep) for at least 2 hours. Although the exact cause of OPremains controversial, it is a benign condition that requires no follow-up or treatment. Childrenwho have OP cannot have a history of renal disease, hematuria, or edema. In contrast, fixedproteinuria is present at all times of the day, regardless of body position.

The teenager in the vignette is asymptomatic and has had several urinalyses (UAs)revealing 2+ protein but no other abnormalities. A negative first morning UA, with a urine protein-to-creatinine ratio less than 0.2, would establish the diagnosis of OP. Prior to referral to thepediatric nephrologist, obtaining a repeat UA within 2 weeks and subsequently obtaining a firstmorning UA likely would have established the diagnosis of OP, obviating the need for severalother unnecessary tests (eg, electrolyte panel, complete blood count, serum complements).

Renal ultrasonography may be helpful in detecting a potential urinary tract malformation insome patients who may have proteinuria, but it is not the most appropriate first step. If thepatient has a positive (1+ or more) first morning UA result, renal biopsy may be indicated. A 24-hour urine collection (instead of a random protein-to-creatinine ratio) rarely is necessary anddoes not aid in the diagnosis of OP. Proteinuria has been established firmly for this child, andwaiting for results from another daytime UA would only delay the diagnosis.

References:

Chandar J, Gomez-Marin O, del Pozo R, et al. Role of routine urinalysis in asymptomaticpediatric patients. Clin Pediatr (Phila). 2005;44:43-48. Abstract available at:http://www.ncbi.nlm.nih.gov/entrez/query.fcgi?orig_db=PubMed&db=PubMed&cmd=Search&term=%22Clinical+pediatrics%22[Jour]+AND+43[page]+AND+2005[pdat]

Hogg RJ, Portman RJ, Milliner D, Lemley KV, Eddy A, Ingelfinger J. Evaluation and managementof proteinuria and nephrotic syndrome in children: recommendations from a pediatric nephrologypanel established at the National Kidney Foundation Conference on Proteinuria, Albuminuria,Risk, Assessment, Detection, and Elimination (PARADE). Pediatrics. 2000;105:1242-1249.Available at: http://pediatrics.aappublications.org/cgi/content/full/105/6/1242

Wingo C, Clapp WL. Proteinuria: potential causes and approach to evaluation. Am J Med Sci.2000;320:188-194. Abstract available at:http://www.ncbi.nlm.nih.gov/entrez/query.fcgi?orig_db=PubMed&db=PubMed&cmd=Search&term=%22The+American+journal+of+the+medical+sciences%22[Jour]+AND+188[page]+AND+2000[pdat]

Copyright © 2007 by the American Academy of Pediatrics page 194

Page 195: AAP MCQ 2007

2007 PREP SA on CD-ROM

Question: 94

A 10-year-old boy presents with a 2-day history of a temperature to 102.5ºF (39.2ºC) and amaculopapular rash that began on his trunk and now has spread to his extremities and back.Several of the lesions appear to be vesicles (Item Q94A), and he is starting to develop newlesions on his face.

Of the following, the MOST rapid method to confirm the cause of this patient's infection is

A. complement fixation

B. direct fluorescent antibody

C. enzyme immunoassay

D. polymerase chain reaction

E. viral culture

Copyright © 2007 by the American Academy of Pediatrics page 195

Page 196: AAP MCQ 2007

2007 PREP SA on CD-ROM

Preferred Response: BCritique: 94

The availability of rapid and reliable viral diagnostic tests facilitates decision making in theprevention and treatment of viral infections and the institution of effective infection controlmeasures. The available rapid testing methods detect viral antigen by immunologic methods,such as fluorescent antibody (FA) testing or enzyme immunoassay (EIA), and viral nucleic acidby labeled complementary molecular probes, either by direct hybridization or after some form ofamplification such as polymerase chain reaction.

Antigen detection tests are performed directly on specimens from the patient. Kits to performEIA or the FA test are available for the detection of rotavirus and enteric adenovirus in stoolspecimens; respiratory syncytial virus, influenza A and B viruses, parainfluenza viruses, andadenoviruses in respiratory specimens; hepatitis B surface antigen and humanimmunodeficiency virus (HIV) p24 antigen in serum; herpes simplex virus (HSV) and varicella-zoster virus in vesicle swab specimens; and cytomegalovirus (CMV) in bronchoalveolar lavageand blood specimens. Results of these tests usually are available within hours and are preferredwhen sensitive and specific test kits and reagents are commercially available. Thedisadvantages of antigen detection tests include lack of available test kits for some clinicallyimportant viruses, inferior sensitivity compared with culture for most viruses that can becultured, and inferior specificity for all viruses.

Molecular probes for nucleic acid are most useful for detecting and typing viruses for whichreliable culture methods are not available, such as human papillomavirus (HPV). Molecularprobes are available in commercial kits for the detection of HIV in plasma or white blood cells;HSV and enteroviruses in cerebrospinal fluid; CMV, hepatitis B virus, and hepatitis C virus inserum; HPV in cervical cells; and parvovirus in serum. However, for some viruses, theconcentration of viral genomes in direct patient specimens may be too low to allow detection withadequate sensitivity. Molecular amplification technologies have been developed for manyviruses to increase the sensitivity for detection of viral genomes. The use of nucleic aciddetection techniques is appropriate when the virus cannot be detected by rapid isolation or whenantigen detection methods are not available or are insensitive.

The patient described in the vignette has varicella-zoster infection, and the most rapidmethod available to identify this virus is direct fluorescent antibody testing on a sample ofvesicular fluid. Complement fixation and EIA require acute and convalescent serum specimens,making them less rapid approaches. Further, complement fixation has poor sensitivity.Polymerase chain reaction is very sensitive, but its availability is limited, and highly trainedpersonnel and specific equipment are required to perform the test. Viral culture is sensitive andspecific, but it is expensive and has limited availability. Results from both polymerase chainreaction testing and viral cultures take much longer to obtain than direct FA.

References:

American Academy of Pediatrics. Varicella-zoster infections. In: Pickering LK, ed. Red Book:2006 Report of the Committee on Infectious Diseases. 27th ed. Elk Grove Village, Ill: AmericanAcademy of Pediatrics; 2006:711-725

Mazzulli T. Laboratory diagnosis of infection due to viruses, Chlamydia, and Mycoplasma. In:Long SS, Pickering LK, Prober CG, eds. Principles and Practice of Pediatric InfectiousDiseases. 2nd ed. New York, NY: Churchill Livingstone; 2003:1392-1408

Naber SP. Molecular pathology—diagnosis of infectious disease. N Engl J Med. 1994;331:1212-1215

Copyright © 2007 by the American Academy of Pediatrics page 196

Page 197: AAP MCQ 2007

2007 PREP SA on CD-ROM

Question: 95

A 12-month-old boy presents for evaluation of severe eczema. His parents state that their son'seczema started soon after birth and has not responded to topical corticosteroids. The parentsalso are concerned that the infant has had frequent otitis media, sinus infections, and oneepisode of Streptococcus pneumoniae pneumonia requiring hospitalization last month. Duringyour review of the laboratory studies performed, you notice that the infant has thrombocytopenia(20x103/mcL [20x109/L]) and small platelets.

Of the following, the MOST likely diagnosis for this infant is

A. Bruton (X-linked) agammaglobulinemia

B. chronic granulomatous disease

C. DiGeorge syndrome

D. Wiskott-Aldrich syndrome

E. X-linked severe combined immunodeficiency

Copyright © 2007 by the American Academy of Pediatrics page 197

Page 198: AAP MCQ 2007

2007 PREP SA on CD-ROM

Preferred Response: DCritique: 95

The clinical constellation of recurrent infections, severe eczema, and small platelets describedfor the patient in the vignette is pathognomonic for Wiskott-Aldrich syndrome (WAS). WAS is aprimary immune deficiency that has been mapped to the Xp11.22-11.23 chromosome. Patientscan present initially with prolonged bleeding after circumcision or bloody diarrhea during infancy.Recurrent infections and eczema develop prior to age 1 year. Treatment options includeintravenous or subcutaneous immunoglobulin, prophylactic antibiotics, and ideally a humanleukocyte antigen-identical bone marrow transplant.

Bruton (X-linked) agammaglobulinemia is another X-linked primary immune deficiencyresulting from defects in the B-cell tyrosine kinase protein. Patients have a profound defect in B-cell production, severe hypogammaglobulinemia, small-to-absent tonsils, and recurrentsinopulmonary infections. Symptoms typically occur after 6 months, coinciding with waning ofpassive maternal antibody transfer. Treatment consists of lifelong immunoglobulin infusions,evaluation and aggressive treatment of infections, and monitoring for development oflymphoreticular malignancies.

Chronic granulomatous disease (CGD) is a disorder of the phagocytic system resultingfrom an inability to kill catalase-positive organisms (eg, Staphylococcus aureus, Aspergillus sp,Serratia marcescens, Burkholderia cepacia, Candida albicans, and Salmonella sp). Patientspresent with recurrent lymphadenitis, hepatic abscesses, recurrent skin infections, andosteomyelitis. Bone marrow transplant is the only known cure. Aggressive supportive care,interferon-gamma injections (50 mcg/m2, three times per week), and prophylactic antibiotics arethe mainstay therapy in lieu of transplant.

DiGeorge syndrome is a genetic disorder linked to chromosome 22q11.2 and thedysmorphogenesis of the third and fourth pharyngeal pouches that usually presents in infancy.Clinical manifestations vary, but patients who have “complete” DiGeorge syndrome haverecurrent viral, fungal, and bacterial infections similar to children who have severe combinedimmunodeficiency (SCID). Treatment consists of either bone marrow or thymic tissuetransplant.

X-linked SCID is the most common form of SCID and results from a defect in the commongamma chain that encodes several interleukin receptors. Affected infants present within the firstfew postnatal months with frequent episodes of diarrhea, sinopulmonary infections, and skininfections. Prompt recognition and referral to an immunologist is vital because replacementimmunoglobulin fails to halt the progressive worsening of this immunodeficiency. Unless bonemarrow transplantation is performed, death usually occurs before the child’s second birthday.

References:

Buckley RH. Primary defects of cellular immunity. In: Behrman RE, Kliegman RM, Jenson HB,eds. Nelson Textbook of Pediatrics. 17th ed. Philadelphia, Pa: WB Saunders Co; 2004:693-694

Thampakkul S, Ballow M. Replacement intravenous immune serum globulin therapy in patientswith antibody immune deficiency. Immunol Allergy Clin North Am. 2001;21:165-184

Copyright © 2007 by the American Academy of Pediatrics page 198

Page 199: AAP MCQ 2007

2007 PREP SA on CD-ROM

Question: 96

You inserted an endotracheal tube into a 2-year-old child who had severe head trauma after amotor vehicle crash. Initially, she had good chest rise, with oxygen saturations of 99% on 100%inspired oxygen delivered via bag-valve mask, but 10 minutes later, she suddenly deteriorates.Her oxygen saturation now is 60%, and her heart rate is 50 beats/min and falling rapidly. Younote poor breath sounds on both sides of her chest and poor chest rise.

Of the following, the BEST initial management of this patient is to

A. consult a surgeon for urgent placement of a chest tube

B. insert an 18-gauge angiocatheter into the upper right chest to evacuate a possiblepneumothorax

C. order an immediate chest radiograph to determine if she has a pneumothorax

D. perform deep suction through the endotracheal tube using an 8-French flexible catheter

E. remove the endotracheal tube and resume ventilation with bag-valve mask

Copyright © 2007 by the American Academy of Pediatrics page 199

Page 200: AAP MCQ 2007

2007 PREP SA on CD-ROM

Preferred Response: ECritique: 96

The child in the vignette has had a sudden deterioration in pulmonary status shortly afterendotracheal intubation. Rapid identification and correction of the problem is essential forsurvival. In this situation, the authors of the Pediatric Advanced Life Support course suggest theuse of a mnemonic to help determine the cause. “DOPE” refers to Displacement of theendotracheal tube either into the right mainstem bronchus or completely out of the trachea,Obstruction of the tube, Pneumothorax, and Equipment failure such as interruption of theoxygen supply.

This child is experiencing hypoxia and bradycardia and likely will develop full cardiac arrestwithin seconds unless rapid action is taken. Removal of the endotracheal tube and ventilation viabag-valve mask addresses all of the elements in the “DOPE” pneumonic except pneumothorax.If her oxygen saturation does not improve with this intervention, a rapid assessment forpneumothorax should be performed.

If an intubated patient has a more gradual deterioration in pulmonary status, rapidassessment for all the elements of the “DOPE” mnemonic may be performed rather thanimmediately removing the endotracheal tube. Direct laryngoscopy can be performed to view theendotracheal tube passing through the vocal cords. A suction catheter can be passed down thetube to check for obstruction. Inspection of the equipment and circuit can identify obviousproblems.

If breath sounds are diminished on only one side of the chest, especially in the presence offalling oxygen saturation, hyperresonance to percussion, and decreasing blood pressure, asuspected tension pneumothorax should be decompressed immediately using an angiocatheterattached to a syringe. The catheter should be inserted through the second interspace in the mid-clavicular line. A rush of air with immediate improvement in the patient’s status helps to confirmthe diagnosis. At that point, a chest tube is inserted for long-term management of thepneumothorax.

A chest radiograph may be helpful in a patient who is experiencing marginal deterioration inpulmonary status, especially if quick assessment using the “DOPE” mnemonic does not identifythe source of the problem. Radiographic confirmation of a suspected tension pneumothoraxprior to decompression is neither required nor suggested in the patient who has had acutesevere deterioration.

References:

American Heart Association. Airway, ventilation, and management of respiratory distress andfailure. In: PALS Provider Manual. Dallas, Tex: American Heart Association; 2002:81-126

Kelly RE Jr, Isaacman DJ. Thoracic emergencies. In: Fleisher GR, Ludwig S, Henretig FM, eds.Textbook of Pediatric Emergency Medicine. 5th ed. Philadelphia, Pa: Lippincott Williams &Wilkins; 2006:1631-1654

Silbergleit R, Lee DC, Blank-Reid C, McNamara RM. Sudden severe barotrauma from self-inflating bag-valve devices. J Trauma. 1996;40:320-322. Abstract available at:http://www.ncbi.nlm.nih.gov/entrez/query.fcgi?orig_db=PubMed&db=PubMed&cmd=Search&term=%22The+Journal+of+trauma%22[Jour]+AND+1996[pdat]+AND+Silbergleit+R[author]

Copyright © 2007 by the American Academy of Pediatrics page 200

Page 201: AAP MCQ 2007

2007 PREP SA on CD-ROM

Question: 97

A 10-year-old girl presents with progressive fatigue without pain. Physical examination revealsedema and a pink-to-violet discoloration of the upper eyelids and malar areas (Item Q97A).Scaly, red papules (Item Q97B) are observed over the knuckles.

Of the following, the MOST appropriate laboratory investigation to support the diagnosis is

A. antinuclear antibody assay

B. creatine phosphokinase concentration

C. epicutaneous patch testing

D. erythrocyte sedimentation rate

E. skin biopsy

Copyright © 2007 by the American Academy of Pediatrics page 201

Page 202: AAP MCQ 2007

2007 PREP SA on CD-ROM

Preferred Response: BCritique: 97

The girl described in the vignette has findings consistent with the diagnosis of juveniledermatomyositis (JDM). The most common of the pediatric inflammatory myopathies, JDM is asystemic vasculopathy that has characteristic cutaneous findings and focal areas of myositisresulting in progressive proximal muscle weakness that is generally responsive to the promptinstitution of immunosuppressive therapy. Disease presentation is variable. Approximately 30%of affected children experience the rapid onset of rash, weakness, and pain. Another 50% haveinsidious development of muscle pain and progressive weakness. The remaining 20% have asubacute onset in which a rash appears long before muscular or constitutional symptoms. Asmall proportion of children have typical cutaneous findings but no evidence of muscle disease(amyopathic dermatomyositis).

Classic clinical findings include pink-to-violet color of the eyelids or cheeks (heliotrope) (ItemC97A) and scaly, erythematous papules symmetrically arrayed over extensor surfaces of theelbows, knees, or metacarpophalangeal or interphalangeal joints (Gottron papules) (Item C97B).JDM usually is diagnosed clinically by the pathognomonic rash and proximal muscle findings.Elevated serum concentrations of muscle-derived enzymes (eg, creatine phosphokinase,aldolase, aspartate aminotransferase, and lactic acid dehydrogenase) reflect the leaky musclemembranes.

Elevations of antinuclear antibody assay (in 60% of cases) and the erythrocytesedimentation rate may be present but are not specific for JDM. Because the appearance andlocation of the child's skin lesions are not consistent with a diagnosis of contact dermatitis,epicutaneous patch testing to identify an offending allergen is not indicated. Skin biopsy in JDMis not diagnostic.

References:

Barnett NK, Wright D, Kawasaki T, Treadwell PA. Collagen vascular, connective tissuediseases, and selected systemic diseases with skin manifestations: toxic oil syndrome andeosinophilia myalgia syndrome, juvenile dermatomyositis. In: Schachner LA, Hansen RC, eds.Pediatric Dermatology. 3rd ed. St. Louis, Mo: Mosby; 2003:955-960

Pachman LM. Juvenile dermatomyositis. In: Behrman RE, Kliegman RM, Jenson HB, eds.Nelson Textbook of Pediatrics. 17th ed. Philadelphia, Pa: WB Saunders Co; 2004:813-816

Weston WL, Lane AT, Morelli JG. Papulosquamous disorders: dermatomyositis In: ColorTextbook of Pediatric Dermatology. 3rd ed. St. Louis, Mo: Mosby; 2002:136-138

Copyright © 2007 by the American Academy of Pediatrics page 202

Page 203: AAP MCQ 2007

2007 PREP SA on CD-ROM

Question: 98

A 2-year-old who has a history of repaired biliary atresia presents to your office with fatigue andintermittent dark stools. On physical examination, he is afebrile and pale but active. His heartrate is 110 beats/min, liver and spleen are both enlarged, and abdomen is distended, withprominent abdominal veins. The hematocrit is 22% (0.22).

Of the following, the MOST appropriate next step is to

A. arrange for outpatient endoscopy

B. arrange for hospital-based care

C. begin oral iron supplementation

D. obtain abdominal ultrasonography

E. refer the boy for therapeutic paracentesis

Copyright © 2007 by the American Academy of Pediatrics page 203

Page 204: AAP MCQ 2007

2007 PREP SA on CD-ROM

Preferred Response: BCritique: 98

The patient described in the vignette has a history of biliary atresia palliated with a Kasaiprocedure and now has melena. On physical examination, there is evidence of portalhypertension, pallor, and tachycardia, and the hematocrit demonstrates severe anemia. Themost likely diagnosis is an upper gastrointestinal hemorrhage due to esophageal varicealbleeding, portal gastropathy, or an ulcer. The patient should be hospitalized, have hishemodynamic status stabilized through transfusion, and undergo endoscopy to determine thecause of the bleeding.

The presence of melena in a child typically suggests gastrointestinal bleeding from a lesioncephalad to the ligament of Treitz. The black color of the stool probably is caused by thecompound hematin, which is generated when heme is oxidized by intestinal bacteria. Thedifferential diagnosis of melena in the infant and child varies with age. The most common lesionscausing upper gastrointestinal bleeding are duodenal ulcer, gastric ulcer, gastritis, arteriovenousmalformations, erosive esophagitis, or esophageal varices. Although large ulcers andesophageal varices typically present with hematemesis, low-grade bleeding may present withmelena and no hematemesis.

The patient's clinical status and underlying disease history determine the need for acutecare. Patients who have hemodynamic instability, anemia, or serious comorbidities need urgentintervention. Initial care should include intravenous catheter placement and laboratoryassessment, including a complete blood count, coagulation studies, and blood typing and cross-matching. Endoscopy can help characterize the precise lesion and may allow for therapeuticintervention. For example, a patient who has bleeding varices can have rubber bands placedaround them to stop the bleeding (band ligation), and a patient who has a bleeding vessel withinan ulcer can be treated by endoscopic hemostasis.

Because of his tenuous status and anemia, elective endoscopy or oral outpatient therapywith iron are not appropriate as first-line treatments for the boy described in the vignette.Abdominal ultrasonography can demonstrate portal hypertension and may show varices, but itcannot identify the site of bleeding or treat the bleeding. Paracentesis is useful if spontaneousbacterial peritonitis is suspected, but it will not identify the bleeding lesion, which is in the lumen ofthe gastrointestinal tract.

References:

Gilger MA. Upper gastrointestinal bleeding. In: Walker WA, Goulet O, Kleinman RE, ShermanPM, Shneider BL, Sanderson IR, eds. Pediatric Gastrointestinal Disease. 4th ed. Hamilton,Ontario, Canada: BC Decker; 2005:258-265

Squires RH Jr. Gastrointestinal bleeding. Pediatr Rev. 1999;20:95-101. Available at:http://pedsinreview.aappublications.org/cgi/content/full/20/3/95

Copyright © 2007 by the American Academy of Pediatrics page 204

Page 205: AAP MCQ 2007

2007 PREP SA on CD-ROM

Question: 99

A term infant is delivered vaginally following a pregnancy complicated by diabetes mellitus. Hisoral and nasal airways are suctioned and found to be patent and free of meconium. He hascyanosis and respiratory distress immediately following birth that requires intubation andassisted ventilation with 100% oxygen. Because no improvement is apparent in the next 5minutes, he is admitted to the neonatal intensive care unit. His birthweight is 4,500 g. A chestradiograph reveals findings consistent with decreased pulmonary blood flow.

Of the following, the MOST likely cause of respiratory distress in this infant is

A. anemia

B. choanal atresia

C. hyperglycemia

D. hypermagnesemia

E. persistent pulmonary hypertension

Copyright © 2007 by the American Academy of Pediatrics page 205

Page 206: AAP MCQ 2007

2007 PREP SA on CD-ROM

Preferred Response: ECritique: 99

An infant of a diabetic mother (IDM), such as the infant described in the vignette, is predisposedto some increased risk of respiratory distress syndrome associated with an immaturepulmonary surfactant profile. However, findings on this infant’s radiograph are not consistentwith this diagnosis, and the combination of cyanosis and respiratory distress that does notimprove with intubation and assisted ventilation raises concerns for cyanotic heart disease anddecreased pulmonary blood flow. Transposition of the great arteries is the most commoncyanotic heart lesion in the IDM.

The possibility of persistent pulmonary hypertension of the newborn (PPHN) must beevaluated. The findings on chest radiograph vary with the cause of PPHN. Diffuse ground-glassdensities and low lung volumes are observed with congenital pneumonia. Patchy infiltrates andhyperinflation are seen in meconium aspiration syndrome. In idiopathic cases of PPHN, the lungfields are clear, with evidence for decreased pulmonary blood flow.

Echocardiography is vital to discerning this infant’s diagnosis. A hyperoxia test (assistedventilation with 100% oxygen and blood gas analysis from preductal and postductal sites) todetermine the presence of a fixed right-to-left extrapulmonary shunt is also an importantevaluative tool.

The IDM who has macrosomia is at risk for hypoglycemia, which also may present withcyanosis. Choanal atresia is characterized by cyanosis that resolves when an infant cries andan inability to pass a suction catheter through the nares. Anemia is associated with pallor, and inthe IDM, polycythemia and plethora are more likely. Additionally, hypocalcemia andhypomagnesemia are common in the IDM.

References:

Bernstein D. Epidemiology and genetic basis of congenital heart disease. In: Behrman RE,Kliegman RM, Jenson HB, eds. Nelson Textbook of Pediatrics. 17th ed. Philadelphia, Pa:Saunders 2004;1499-1501

Bernstein D. Evaluation of the infant or child with congenital heart disease. In: Behrman RE,Kliegman RM, Jenson HB, eds. Nelson Textbook of Pediatrics. 17th ed. Philadelphia, Pa:Saunders 2004;1502-1503

Cowett R. The infant of the diabetic mother. Neoreviews. 2002;3:e173-e189. Available at:http://neoreviews.aappublications.org/cgi/content/full/3/9/e173

Flidel-Rimon O, Shinwell ES. Respiratory distress in the term and near-term infant. NeoReviews.2005;6: e289-e297. Available at: http://neoreviews.aappublications.org/cgi/content/full/6/6/e289

Nold JL, Georgieff MK. Infants of diabetic mothers. Pediatr Clin North Am. 2004;51:619-637.Abstract available at:http://www.ncbi.nlm.nih.gov/entrez/query.fcgi?cmd=Retrieve&db=pubmed&dopt=Abstract&list_uids=15157588&query_hl=32&itool=pubmed_docsum

Stoll BJ, Kliegman RM. Respiratory tract disorders. In: Behrman RE, Kliegman RM, Jenson HB,eds. Nelson Textbook of Pediatrics. 17th ed. Philadelphia, Pa: Saunders; 2004:573-588

Stoll BJ, Kliegman RM. The endocrine system. In: Behrman RE, Kliegman RM, Jenson HB, eds.Nelson Textbook of Pediatrics. 17th ed. Philadelphia, Pa: Saunders; 2004:613-615

Copyright © 2007 by the American Academy of Pediatrics page 206

Page 207: AAP MCQ 2007

2007 PREP SA on CD-ROM

Question: 100

A 4-week-old boy has been vomiting all of his feedings for the last 2 days. His mother reportsdecreased urine output but no fever. On physical examination, he is alert, appears hungry, andis moderately dehydrated. Abdominal examination reveals a small mass in the epigastrium.When he is offered a bottle, he takes it vigorously and then vomits forcefully. You orderlaboratory tests.

Of the following, the MOST likely abnormality to expect is

A. hyperchloremia

B. hyperkalemia

C. hypernatremia

D. metabolic acidosis

E. metabolic alkalosis

Copyright © 2007 by the American Academy of Pediatrics page 207

Page 208: AAP MCQ 2007

2007 PREP SA on CD-ROM

Preferred Response: ECritique: 100

The infant described in the vignette has forceful, nonbilious vomiting consistent with a gastricoutlet obstruction. Pyloric stenosis is the most common cause of gastric obstruction inneonates. It results from hypertrophy of the pylorus and is most common in males older than 2to 3 weeks of age. Affected infants generally appear well, with the exception of dehydration, andthey often are very hungry after vomiting. Abdominal examination may reveal a gastric peristalticwave after eating and a small olive-shaped mass in the epigastrium, which may be appreciatedmore easily with the infant in a prone position. Normal findings on abdominal examination are notunusual and do not rule out the diagnosis.

The forceful vomiting of all stomach contents results in massive fluid loss and dehydrationover time. Hypochloremic metabolic alkalosis develops from the progressive loss of hydrogenion and chloride. The serum potassium concentration may be low or normal, andhyperbilirubinemia may be seen. Hypernatremia, hyperkalemia, hyperchloremia, and metabolicacidosis are not typical electrolyte abnormalities in pyloric stenosis. The diagnostic examinationof choice is pyloric ultrasonography, which reveals thickening and elongation of the pylorus.Care should be taken to rehydrate the infant and correct any electrolyte abnormalities beforeundertaking surgical correction. The prognosis after surgical correction is excellent.

References:

Wyllie R. Pyloric stenosis and congenital anomalies of the stomach. In: Behrman RE, KliegmanRM, Jenson HB, eds. Nelson Textbook of Pediatrics. 17th ed. Philadelphia, Pa: WB SaundersCo; 2004:1229-1231

Dinkevich E, Ozuah PO. In brief: pyloric stenosis. Pediatr Rev. 2001;21:249-250. Available at:http://pedsinreview.aappublications.org/cgi/content/full/21/7/249

Copyright © 2007 by the American Academy of Pediatrics page 208

Page 209: AAP MCQ 2007

2007 PREP SA on CD-ROM

Question: 101

A 2-year-old girl presents for evaluation of fussiness, low-grade fever, and what her parentsdescribe as "growing pains." On physical examination, you palpate a nontender mass deep inthe right periumbilical area and note mild purple discoloration of the eyelids.

Of the following, the MOST likely diagnosis is

A. hepatoblastoma

B. Hirschsprung disease

C. intussusception

D. neuroblastoma

E. Wilms tumor

Copyright © 2007 by the American Academy of Pediatrics page 209

Page 210: AAP MCQ 2007

2007 PREP SA on CD-ROM

Preferred Response: DCritique: 101

Neuroblastoma is the most common extracranial solid tumor of childhood. It occurs mostfrequently in children younger than the age of 4 years and has a slight male predilection. It mayarise anywhere in the sympathetic nervous system, but most commonly presents in theabdomen. Although it may present as an abdominal mass, as for the child in the vignette, it isdiagnosed most commonly as widely disseminated disease. The discoloration of the periorbitaltissues reported for the child in the vignette results from mass effects of involved lymphoidtissue in the periorbital area.

Paraneoplastic manifestations of neuroblastoma, such as opsoclonus or “dancing eyes,”may result from a postulated immune mechanism and may persist despite surgical orchemotherapeutic cure of the underlying malignancy.

Surgical staging was used for prognostic purposes until the late 1980s, but recent advancesin tumor molecular biology have changed the method by which neuroblastoma is treated andprognosed. Tumors with amplification of the N-myc protooncogene or with multidrug resistanceprotein expression have a poorer prognosis.

Wilms tumor often presents as a painless abdominal mass with few systemic symptoms andfrequently is associated with somatic overgrowth symptoms or other genetic syndromes.Intussusception usually has an abrupt onset but may involve a palpable intussusceptum andtypically is characterized by bloody stools and vomiting. Hirschsprung disease may present witha palpable toxic megacolon and fever, but there is often a history of constipation (diarrhea maybe common in neuroblastoma due to catecholamine excess). Hepatoblastoma is rare but shouldbe considered in children who have chronic hepatitis B infection or a somatic overgrowthsyndrome.

References:

Strother DR, Dreyer ZE. Neuroblastoma. In: McMillan JA, DeAngelis CD, Feigin RD, WarshawJB, eds. Oski’s Pediatrics: Principles and Practice. 3rd ed. Philadelphia, Pa: Lippincott, Williams& Wilkins; 1999:1517-1519

Copyright © 2007 by the American Academy of Pediatrics page 210

Page 211: AAP MCQ 2007

2007 PREP SA on CD-ROM

Question: 102

A 7-year-old girl presents with a 4-week history of an erythematous rash that involves theeyelids and malar areas and now has occurred at the elbows and knees. Her mother reportsthat the girl has had a poor appetite, low-grade fevers, muscle aches, and weakness during thepast week. Physical examination confirms the rash (Item Q102A). She has difficulty rising from achair. Other findings on her physical examination are normal. Serum creatine kinase is 677 U/L.

Of the following, the test MOST likely to lead to this child's diagnosis is

A. erythrocyte sedimentation rate

B. molecular testing for fascioscapulohumeral dystrophy

C. muscle biopsy

D. polymerase chain reaction testing for dystrophin

E. serum antinuclear antibody

Copyright © 2007 by the American Academy of Pediatrics page 211

Page 212: AAP MCQ 2007

2007 PREP SA on CD-ROM

Preferred Response: CCritique: 102

Weakness can be a broad complaint in a child, but identifying the neurologic origin of theweakness, its temporal pattern, and associated findings can point to appropriate neurodiagnostictests and help establish the diagnosis. In general, muscular weakness tends to be greaterproximally than distally, affecting most the hips and shoulders. Creatine kinase is often elevatedin myopathy. Neuropathy also produces symmetric weakness, but the deficit is greater distallythan proximally and associated with hyporeflexia and sometimes, sensory loss. Nerveconduction velocities in neuropathy demonstrate diminished velocity or amplitude.Neuromuscular junction problems tend to worsen throughout the day, and in children are morenotable rostrally than caudally. Myelopathy produces weakness inferior to a spinal lesion, oftenwith a sensory deficit inferior to a specific dermatome. Weakness associated with brainstemprocesses often is asymmetric and characterized by cranial neuropathies. Cerebral processeswith weakness produce hemiparesis and unilateral hyperreflexia and may cause visual field lossor speech problems.

Acute causes of weakness include stroke (cerebrum or brainstem); acute intermittentporphyria, diphtheria, poliomyelitis, West Nile virus infection, or tick paralysis (neuropathy);organophosphate poisoning or botulism (neuromuscular junction); and periodic paralysis andacute infectious myositis (muscle). Subacute processes include tumor or abscess (cerebrum orspine), transverse myelitis (spine), myasthenia gravis (neuromuscular junction), Guillain-Barrésyndrome (nerve), and polymyositis and dermatomyositis (muscle). Muscular dystrophies tendto present chronically.

The 7-year-old girl described in the vignette displays weakness that is proximal more thandistal, as well as elevated creatine kinase concentrations, suggesting a myopathy. The 4-weekduration plus the erythematous malar rash raise suspicion about dermatomyositis. Musclebiopsy can demonstrate the fiber type variability, central nuclei that are vesicular, infiltration withinflammatory cells, and necrosis that characterize the condition.

The erythrocyte sedimentation rate is a nonspecific test; it is elevated in myopathy,infections, and multiple other disorders. This child does not have the chronic course of the threemost common dystrophies (Duchenne, Becker, and fascioscapulohumeral). Therefore,polymerase chain reaction testing for dystrophin or genetic testing for fascioscapulohumeraldystrophy is not warranted. Serum antinuclear antibody may be positive in patients who havedermatomyositis, but it is a nonspecific finding. It is most useful in the evaluation of patients whomay have systemic lupus erythematosus, but lupus will not produce the myopathic weaknessreported for this girl.

References:

Jacobson RD. Approach to the child with weakness or clumsiness. Pediatr Clin North Am.1998;45:145-168. Abstract available at:http://www.ncbi.nlm.nih.gov/entrez/query.fcgi?cmd=Retrieve&db=pubmed&dopt=Abstract&list_uids=9491091&query_hl=14&itool=pubmed_docsum

Roland EH. Muscular dystrophy. Pediatr Rev. 2000;21:233-238. Available at:http://pedsinreview.aappublications.org/cgi/content/full/21/7/233

Sarnat HB. Neuromuscular disorders. In: Behrman RE, Kliegman RM, Jenson HB, eds. NelsonTextbook of Pediatrics. 17th ed. Philadelphia, Pa: WB Saunders Co; 2004:2053-2082

Turner TL, Boom JA. Index of suspicion. Case 1. Pediatr Rev. 2000;21:389-392. Available at:http://pedsinreview.aappublications.org/cgi/content/full/21/11/389

Copyright © 2007 by the American Academy of Pediatrics page 212

Page 213: AAP MCQ 2007

2007 PREP SA on CD-ROM

Question: 103

During a health supervision visit for a 16-year-old boy, you learn that he has experienced chestpain twice with intense exercise during practice for his high school varsity soccer team. Eachtime the pain felt like pressure, radiated to his left shoulder, and was associated withlightheadedness. He did not seek medical attention after either episode. His father, who is age49 years, has hypertension and uses lipid-lowering medication. Results of the boy's physicalexamination are normal, including his blood pressure and cardiovascular examination.

Of the following, the BEST management plan is

A. cardiology referral, with only light exercise pending evaluation

B. cardiology referral, with restriction of all exercise pending evaluation

C. echocardiography, with clearance for exercise if results are normal

D. electrocardiography, with clearance for exercise if results are normal

E. trial of bronchodilator therapy for exercise-induced asthma

Copyright © 2007 by the American Academy of Pediatrics page 213

Page 214: AAP MCQ 2007

2007 PREP SA on CD-ROM

Preferred Response: BCritique: 103

Chest pain in children and adolescents is a common problem. The causes of chest pain in thepediatric population are varied and can be considered by organ systems: musculoskeletal,respiratory, gastrointestinal, psychological, and cardiac. Among the musculoskeletal causes arechest wall strain, trauma, costochondritis, and the precordial catch syndrome. Respiratorycauses include asthma, pneumonia, pneumothorax, pneumomediastinum, and chronic cough.Chest pain may be due to gastritis, esophagitis, or indigestion. Chest pain also may result fromor be exacerbated by psychogenic processes, including anxiety, fear, and attention-seekingbehaviors. Perhaps the most common causes of chest pain in pediatrics are those referred toas idiopathic. This diagnosis often is given to the patient who presents with a 1- to 2-weekhistory of intermittent, brief, sharp, or stabbing pain that is not associated with exercise orexertion.

The cardiac causes of chest pain are important to recognize because they can beassociated with significant morbidity and mortality. Chest pain can result from pericardial pain,angina and myocardial ischemia, arrhythmias, and aortic dissection. Pericardial pain results frominflammation and often is associated with pericarditis. The pain typically is substernal, positional,and can be severe. Affected patients frequently prefer to sit and may refuse to lie down. Painthat results from angina and myocardial ischemia is rare in children but can occur in those whohave anomalies of the coronary arteries. The pain often is referred to as a pressure sensationwith burning, and there may be radiation to the neck, shoulder, or arm. As for the boy describedin the vignette, the pain typically occurs during or following exercise or activity, and it mayimprove with rest. The clinician should consider myocardial ischemia strongly in the patient whohas had Kawasaki disease or cardiac surgery. It also is important to consider use of illicit drugs,such as cocaine and other adrenergic stimulators, as potential causes of coronary vasospasm.Some of the tachyarrhythmias, such as supraventricular tachycardia, may present with chestpain, although this usually is described as a discomfort associated with palpitation and othersymptoms. Although it may be associated with exercise, this is not the typical presentation. Painfrom aortic dissection is usually acute and sharp and may present in the anterior chest or theback, depending on the area of the aorta that is affected. A history of Marfan syndrome orEhlers-Danlos syndrome should be pursued in affected patients and their families.

The pain experienced by the patient in the vignette has several qualities that suggest anginaand possible myocardial ischemia. The best management plan is to seek cardiology referral,with a complete restriction of all activity until his evaluation is complete. Often, light activity andsports practice are as strenuous as games and competition. Imaging studies andelectrocardiography, although important to the evaluation, should not be used as a surrogate fora complete history, physical examination, and specialized diagnostic testing from those trained inthe evaluation of cardiac disease in children and adolescents. Similarly, although exercise-induced bronchospasm is common, the concerning history of pressure-like pain referred to theleft arm in this patient mandates a complete cardiac evaluation prior to initiation of othertherapies.

References:

Cava JR, Sayger PL. Chest pain in children and adolescents. Pediatr Clin North Am.2004;51:1553-1568. Abstract available at:http://www.ncbi.nlm.nih.gov/entrez/query.fcgi?orig_db=PubMed&db=PubMed&cmd=Search&term=%22Pediatric+clinics+of+North+America%22[Jour]+AND+1553[page]+AND+2004[pdat]

Fahey J. Chest pain. In: Rudolph CD, Rudolph AM, Hostetter MK, Lister G, Siegel NJ, eds.Rudolph’s Pediatrics. 21st ed. New York, NY: McGraw Hill; 2003:1894-1897

Copyright © 2007 by the American Academy of Pediatrics page 214

Page 215: AAP MCQ 2007

2007 PREP SA on CD-ROM

Question: 104

The pregnant mother of a 3-year-old girl in your practice is concerned because her obstetricianjust told her that her serum alpha-fetoprotein value is markedly increased.

Of the following, the MOST appropriate advice to give to this woman is that she should consider

A. having an ultrasonographic examination to date her pregnancy and to search for fetalanomalies

B. having another blood sample drawn to repeat the test

C. having chorionic villus sampling as soon as possible to determine the chromosomecomplement of the fetus

D. obtaining further testing only if she is older than age 35

E. being concerned only if there is a history of open neural tube defects in her family

Copyright © 2007 by the American Academy of Pediatrics page 215

Page 216: AAP MCQ 2007

2007 PREP SA on CD-ROM

Preferred Response: ACritique: 104

Alpha-fetoprotein (AFP) is produced by the fetal liver and crosses the placenta to enter thematernal circulation. Any defect that causes a breech in fetal skin can result in increased levelsof AFP in the maternal circulation due to leaking of the protein. These fetal defects include openneural tube defects, anencephaly, gastroschisis, and omphalocele. Maternal serum AFP(MSAFP) concentrations also can be increased in twin pregnancies, those in which there is fetaldemise or impending fetal demise, and in pregnancies in which the fetus has congenitalnephrosis. Because the measurement of MSAFP is simple and inexpensive, the AmericanCollege of Obstetricians and Gynecologists, the American Society of Human Genetics, and theAmerican Academy of Pediatrics recommend offering MSAFP screening to all pregnant womenat 16 to 18 weeks of gestation. However, such screening should be undertaken only if there isadequate counseling, access to high-quality laboratory services, and appropriate facilities forfollow-up testing (ie, qualified diagnostic centers that offer conventional and high-resolutionultrasonography and amniocentesis). More recently, other biochemical markers have beenadded to this screening test to permit the identification of pregnancies at increased risk forchromosomal abnormalities, most notably trisomy 21.

Depending on the cutoff used by the laboratory to define an elevated MSAFP concentration,which is usually 2 to 2.5 times the median value for gestational age, MSAFP screening detectsmost fetuses that have open neural tube defects. However, because it is a screening test,MSAFP results are abnormal in approximately 1% to 5% of pregnant women. Because theincidence of open neural tube defects is generally 1 in 1,000 or less, most findings of elevatedMSAFP are due to other reasons (eg, incorrect dating of the pregnancy, other congenitalanomalies, intrauterine growth restriction, multiple gestations, fetal demise). Incorrect dating isthe most common reason for a false-positive MSAFP result, and ultrasonography should beused to date the pregnancy. If the dating of the pregnancy is changed by ultrasonography, theMSAFP value should be recalculated to reassess the risk. If the dating is correct, the patientshould be offered high-resolution fetal ultrasonography to search for anomalies. Ifultrasonography does not provide an explanation for the abnormal result (as occurs in about50% of cases), amniocentesis should be offered to measure amniotic fluid AFP.

Some laboratories request a second sample after an initial elevated MSAFP finding; othersproceed immediately to follow-up ultrasonography. In general, second samples should beobtained only if the initial MSAFP concentration is minimally elevated and there is sufficient timefor processing a second specimen.

Clinical trials in pregnant women who have had a prior pregnancy affected by a neural tubedefect have demonstrated that folic acid supplements substantially reduce the risk of recurrentneural tube defects. In one such trial, administration of 4 mg of folic acid daily, beginning at least1 month before conception through the first trimester, reduced the recurrence risk of neural tubedefects from 3.5% to 1.0%. Trials in women who have not had a prior affected pregnancy alsohave shown a beneficial effect. It is now recommended that all women of childbearing age takefolic acid supplements to reduce the risk for neural tube defects. All women still should undergomaternal serum screening because folic acid supplements do not prevent all neural tubedefects.

Because elevated MSAFP concentrations are associated primarily with open neural tubedefects, which usually do not result from chromosome abnormalities, chorionic villus sampling(performed at 10 to 12 weeks’ gestation), which is used to determine fetal karyotype, is notindicated for the woman described in the vignette. In addition, AFP cannot be measured inchorionic villi. Open neural tube defects are among the most common birth defects, occurring inapproximately 1 in 1,000 pregnancies. In most cases, there is no family history, although thosewho do have a positive family history may be at increased risk over the general population.There is no association between the occurrence of neural tube defects and maternal age.

References:

American Academy of Pediatrics Committee on Genetics. Maternal serum alpha-fetoproteinscreening. Pediatrics. 1991;88:1282-1283

Copyright © 2007 by the American Academy of Pediatrics page 216

Page 217: AAP MCQ 2007

2007 PREP SA on CD-ROM

Canick JA, Messerlian GM, Farina A. General principles of second trimester maternal serumscreening for Down syndrome. UpToDate. 2006;14.1. Available at:http://www.utdol.com/utd/content/topic.do?topicKey=antenatl/12590&type=A&selectedTitle=3~54

Cunniff C, Committee on Genetics. Prenatal screening and diagnosis for pediatricians.Pediatrics. 2004;114:889-894. Available at:http://pediatrics.aappublications.org/cgi/content/full/114/3/889

Copyright © 2007 by the American Academy of Pediatrics page 217

Page 218: AAP MCQ 2007

2007 PREP SA on CD-ROM

Question: 105

A 12-year-old basketball player has complained of right knee pain immediately after games for 2weeks. Although he had complained of knee pain intermittently earlier in the season, there is nohistory of swelling or injury of the knee. Physical examination findings of the knee areremarkable for full range of motion no effusion, and swelling over the proximal right tibia (ItemQ105A) that is tender to palpation.

Of the following, the MOST likely diagnosis is

A. jumper's knee

B. Osgood-Schlatter disease

C. patellofemoral syndrome

D. prepatellar bursitis

E. Sinding-Larsen Johansson disease

Copyright © 2007 by the American Academy of Pediatrics page 218

Page 219: AAP MCQ 2007

2007 PREP SA on CD-ROM

Preferred Response: BCritique: 105

The chronic knee pain and swelling and tenderness to palpation over the area of the tibialtuberosity described for the young adolescent in the vignette are consistent with Osgood-Schlatter disease, an overuse injury that is a common cause of chronic anterior knee pain inyoung athletes. During adolescence, the tibial tuberosity develops as a specialized growthcenter called an apophysis. The tuberosity is the site of attachment of the quadriceps muscle viathe patellar tendon. Repetitive traction stress applied to the apophysis results in inflammation orsmall fractures, with resultant pain and swelling (Item C105A). Pain is aggravated by activitiesthat involve quadriceps contraction (eg, running and jumping) or by kneeling on the tuberosity.The disease may be bilateral, although unilateral presentations or predominance of pain aremore common. The onset of Osgood-Schlatter disease often coincides with the period of rapidadolescent growth and poses a chronic, intermittent problem for several months or occasionallyyears before abating. After symptoms resolve, permanent prominence of the tuberosity mayremain. The diagnosis generally is made clinically, although a radiograph may be obtained whensymptoms or history are strikingly atypical or unilateral to rule out a bony neoplasm orcomplication (eg, avulsion of the tibial tubercle or a loose ossicle in the patellar tendon).

Prepatellar bursitis, a cause of anterior knee pain, presents with swelling overlying thepatella that is visible and palpable. The clinician usually can demonstrate full extension of theknee. The bursitis commonly is the result of direct trauma and friction or repetitive flexion andextension overuse that presents as an abrupt and rapid swelling. The mainstay of therapy isrest and the use of anti-inflammatory medication. Protective pads are recommended for use insports that are associated with knee friction or trauma (eg, volleyball, wrestling). Patients whohave very large swelling or recurrent episodes may need to be referred to an orthopedist forevaluation that may include aspiration, analysis, and culture of the fluid. The differential diagnosisincludes septic bursitis that is commonly caused by Staphylococcus aureus.

The term “jumper’s knee” is given to knee extensor tendonitis (patellar and quadricepstendonitis), which affects older and more skeletally mature adolescents. The associated pain islocalized to the superior or inferior pole of the patella. Pain at the inferior pole may be caused bya partial tear of deep layers of the patellar tendon. Sinding-Larsen-Johansson disease also isassociated with tenderness at the inferior pole of the patella rather than over the anterior tibialtuberosity, as in Osgood-Schlatter disease. Similar to Osgood-Schlatter disease, it is believed tobe an apophysitis caused by repetitive stress injuries at the junction of the patella and thepatellar tendon during early adolescent growth.

Patellofemoral syndrome is another common cause of chronic anterior knee pain. Physicalexamination findings consistent with this diagnosis are tenderness to palpation of the undersideof the patella medially and laterally, crepitus with flexion and extension, and pain with patellarcompression.

References:

Fein FD, Ching D, Kim E. Septic bursitis: experience in a community practice. Orthopedics.1991;14:981-984. Abstract available at:http://www.ncbi.nlm.nih.gov/entrez/query.fcgi?orig_db=PubMed&db=PubMed&cmd=Search&term=%22Orthopedics%22[Jour]+AND+981[page]+AND+1991[pdat]

Hogan KA, Gross RH. Overuse injuries in pediatric athletes. Orthop Clin North Am. 2003;34:405-415. Abstract available at:http://www.ncbi.nlm.nih.gov/entrez/query.fcgi?orig_db=PubMed&db=PubMed&cmd=Search&term=%22The+Orthopedic+clinics+of+North+America%22[Jour]+AND+405[page]+AND+2003[pdat]

Pasque CB, McGinnis DW. Knee. In: Sullivan JA, Anderson SJ, eds. Care of the Young Athlete.Elk Grove Village, Ill: American Academy of Orthopaedic Surgeons and American Academy ofPediatrics; 2000:377-404

Copyright © 2007 by the American Academy of Pediatrics page 219

Page 220: AAP MCQ 2007

2007 PREP SA on CD-ROM

Question: 106

You see a 7-day-old baby boy the day that his newborn screening test is reported to show a lowthyroxine concentration of 7.8 mcg/dL (100.4 nmol/L) and a thyroid-stimulating hormoneconcentration of 25 mcU/mL. The baby had a birthweight of 3,000 g and now weighs 3,100 g andis 48.3 cm long. He looks healthy, and the thyroid is not palpable. The mother reports that she isbreastfeeding, and the baby seems to be feeding well. He is her first child. The mother tells youthat she takes thyroid hormone for an underactive thyroid and has needed to take calcium andvitamin D since she was a small child "to keep her calcium up." You look at her more closely andrealize that she is plump, quite short (perhaps 4 ft 10 in), has a round face, and has short stubbyfingers. She says that her mother also had short stature and similar problems. She has onebrother who has similar problems and a sister who is 5 ft 6 in and does not have a problem withher calcium.

Of the following, this baby and his mother MOST likely have

A. McCune Albright syndrome

B. multiple endocrine autoimmune syndrome

C. pseudohypoparathyroidism

D. Noonan syndrome

E. vitamin D resistance

Copyright © 2007 by the American Academy of Pediatrics page 220

Page 221: AAP MCQ 2007

2007 PREP SA on CD-ROM

Preferred Response: CCritique: 106

Pseudohypoparathyroidism is an autosomal dominant disorder usually associated withmutations in the stimulatory G protein alpha - subunit. As a result, the body does not respond toparathyroid hormone and other hormone effects mediated through this G protein mechanism.Individuals who have this disorder tend to be short and stocky, with short fourth metacarpalsand metatarsals (brachydactyly) (Item C106A) and relatively slow mentation or mild mentalretardation. Because the disorder is imprinted, children born to women who have this disorderhave the full clinical expression with hypocalcemia and hyperphosphatemia, but children born tomen who have the disorder have the phenotype without the signs and symptoms of hormoneresistance (sometimes termed pseudopseudohypoparathyroidism).

Genomic imprinting is the mechanism for sex-linked transmission of some specific geneticinformation. It is the result of zygotic alterations in the methylation pattern of genes that leads toinactivation of some genes derived from the maternal or paternal lines. Examples of otherimprinted disorders include Beckwith-Wiedemann syndrome, Prader-Willi syndrome, andAngelman syndrome.

Although hypocalcemia and subcutaneous ossifications are the most commonmanifestations of pseudohypoparathyroidism, resistance to thyroid-stimulating hormone (TSH),gonadotropins, and vasopressin also is common. This leads to elevated TSH concentrations ina newborn as a marker of hypothyroidism. McCune Albright syndrome is a disorder associatedwith polyostotic fibrous dysplasia, irregular café au lait skin macules, and sexual precocity.Other endocrine and somatic abnormalities, including hyperthyroidism, Cushing syndrome, andgigantism (growth hormone excess), have been reported. Multiple endocrine autoimmunedisorders do not present with the same physical appearance as reported for the mother in thevignette, although hypothyroidism and hypoparathyroidism may be prominent endocrinedeficiency disorders as a result of autoimmune destruction of these glands. Noonan syndromeusually is inherited as an autosomal dominant disorder and is associated with short stature,delayed puberty, classic facial appearance (Item C106B), and mild mental retardation in manycases. The disorder is not associated with hypocalcemia or hypothyroidism. Vitamin D-resistantrickets is not associated with hypothyroidism and the physical findings reported for this family.

References:

Abraham MR, Khardori R. Pseudohypoparathyroidism. eMedicine Specialities: Medicine,Ob/Gyn, Psychiatry, and Surgery: Endocrinology. 2005. Available at:http://www.emedicine.com/med/topic1940.htm

Bastepe M, Juppner H. Pseudohypoparathyroidism and mechanisms of resistance towardmultiple hormones: molecular evidence to clinical presentation [editorial]. J Clin EndocrinolMetab. 2003;88:4055-4058. Available at: http://jcem.endojournals.org/cgi/content/full/88/9/4055

Weinstein LS, Yu S, Warner DR, Liu J. Endocrine manifestations of stimulatory G protein alpha-subunit mutations and the role of genomic imprinting. Endocr Rev. 2001;22:675-705. Available at:http://edrv.endojournals.org/cgi/content/full/22/5/675

Copyright © 2007 by the American Academy of Pediatrics page 221

Page 222: AAP MCQ 2007

2007 PREP SA on CD-ROM

Question: 107

A 10-year-old child recently had cognitive testing that showed a full scale intelligence quotient of105. On achievement testing, he performed at the 3rd percentile in reading and at the 50thpercentile in math.

Of the following, these findings MOST likely represent

A. attention-deficit/hyperactivity disorder

B. mental retardation

C. poor testing conditions for the achievement test

D. specific learning disability

E. vision impairment

Copyright © 2007 by the American Academy of Pediatrics page 222

Page 223: AAP MCQ 2007

2007 PREP SA on CD-ROM

Preferred Response: DCritique: 107

Learning disabilities are characterized by an unexpected difficulty in one academic area inchildren who otherwise have the intelligence, motivation, and educational opportunity to learn.The child described in the vignette has normal cognition, with a full scale intelligence quotient of105 and a normal math performance, but subnormal reading performance on achievementtesting, a pattern that is concerning for a specific learning disability. For the boy in the vignette,the disability appears to be in reading.

Children who have learning disabilities may appear inattentive in academic situations inwhich they do not understand the material, and attention-deficit/hyperactivity disorder (ADHD)may be comorbid with learning disability. However, ADHD is unlikely to be the primary diagnosisfor the boy in the vignette. Children who have mental retardation have intelligence quotients ofless than 70 plus associated limitations in adaptive functioning. Testing conditions for the boy inthe vignette are unlikely to have been poor, given his performance on cognitive testing and themath portion of the achievement testing. Only severe vision impairment would affect the ability toread and would have been obvious through history or physical examination.

References:

Lyon GR, Shaywitz SE, Shaywitz BA. Specific reading disability (dyslexia). In: Behrman RE,Kliegman RM, Jenson HB, eds. Nelson Textbook of Pediatrics. 17th ed. Philadelphia, Pa: WBSaunders Co; 2004:110-112

Silver L. Developmental learning disorders. In: Lewis M, ed. Child and Adolescent Psychiatry: AComprehensive Textbook. 3rd ed. Philadelphia, Pa: Lippincott Williams & Wilkins; 2002:621-629

Copyright © 2007 by the American Academy of Pediatrics page 223

Page 224: AAP MCQ 2007

2007 PREP SA on CD-ROM

Question: 108

You are seeing a 14-year-old boy in your office who complains of fever, facial pain, and cough atnight. His mother states that he has had problems with sinusitis for the past few months and hasrequired repeated antimicrobial therapy during this period. Physical examination reveals atemperature of 101°F (38.3°C), yellow-green nasal discharge, and tenderness to palpation of themaxillary and frontal sinus regions.

Of the following, the MOST likely pathogen associated with chronic sinusitis is

A. Moraxella catarrhalis

B. nontypeable Haemophilus influenzae

C. Staphylococcus aureus

D. Streptococcus pneumoniae

E. Streptococcus pyogenes

Copyright © 2007 by the American Academy of Pediatrics page 224

Page 225: AAP MCQ 2007

2007 PREP SA on CD-ROM

Preferred Response: CCritique: 108

The signs and symptoms of chronic sinusitis are similar to those of an acute infection except forpersistent (>90 days) nasal discharge, congestion, and cough (day or night), as reported for theboy in the vignette. Other symptoms include headache, sore throat, halitosis, intermittent fever,malaise, and poor appetite. The pathogens associated with chronic sinusitis are similar to thoseof acute disease (eg, Streptococcus pneumoniae, nontypeable Haemophilus influenzae,Moraxella catarrhalis), but Staphylococcus aureus, other streptococci, fungi, and anaerobesplay a more prominent role. Complications of chronic sinusitis include orbital cellulitis and centralnervous system extension of infection into the cranium (eg, brain abscess, subdural abscess).

Due to the lack of data, treatment of patients who have chronic sinusitis is largely empiric.Antibiotic treatment is problematic because of the large amount of beta-lactamase-producingbacteria in the sinuses, the prevalence of methicillin-resistant S aureus, and the unprovenefficacy of therapy. Most experts suggest starting a regimen that would treat acute sinusitis forpatients who have chronic sinusitis and are being considered for antibiotic therapy. High-doseamoxicillin (75 to 90 mg/kg per day) or amoxicillin-clavulanate is appropriate. Alternativetreatments include cephalosporins (eg, cefprozil, cefuroxime axetil, cefpodoxime), clindamycin,and trimethoprim-sulfamethoxazole. The use of decongestants, antihistamines, mucolytics, andintranasal corticosteroids is controversial; most experts suggest that they have no role in thetreatment of chronic sinusitis in children.

References:

Pappas DE, Hendley JO. Sinusitis. In: Behrman RE, Kliegman RM, Jenson HB, eds. NelsonTextbook of Pediatrics. 17th ed. Philadelphia, Pa: WB Saunders Co; 2004:1391-1393

Steele RW. Chronic sinusitis in children. Clin Pediatr (Phila). 2005;44:465-471. Abstract availableat:http://www.ncbi.nlm.nih.gov/entrez/query.fcgi?orig_db=PubMed&db=PubMed&cmd=Search&term=%22Clinical+pediatrics%22[Jour]+AND+465[page]+AND+2005[pdat]

Copyright © 2007 by the American Academy of Pediatrics page 225

Page 226: AAP MCQ 2007

2007 PREP SA on CD-ROM

Question: 109

An 11-year-old Caucasian boy who has no significant past medical history presents to theemergency department with a 3-day history of brown urine. He reports no dysuria, urgency,frequency, or abdominal or flank pain. His vital signs reveal: temperature, 99°F (37.2°C); bloodpressure, 141/84 mm Hg; heart rate, 92 beats/min; and respiratory rate, 24 breaths/min.Significant findings on physical examination include moderate periorbital and leg edema. Hisurinalysis reveals moderate blood and 4+ protein. The serum complement 3 (C3) and C4concentrations are both low.

Of the following, the MOST likely cause of his hematuria is

A. immunoglobulin A nephropathy

B. membranoproliferative glomerulonephritis

C. postinfectious acute glomerulonephritis

D. urinary tract infection

E. urolithiasis

Copyright © 2007 by the American Academy of Pediatrics page 226

Page 227: AAP MCQ 2007

2007 PREP SA on CD-ROM

Preferred Response: BCritique: 109

Gross hematuria is defined as discolored urine. The precise color may aid in establishing anorigin of the “blood.” It is important to recognize that not all patients suspected of having grosshematuria have red blood cells (RBCs) in the urine. Indeed, the urinalysis in some patients whohave presumed gross hematuria may lack RBCs, suggesting the presence of myoglobin,hemoglobin, or porphyrins, all of which may discolor the urine. Additionally, some foods (eg,beets), drugs, or additives (red dye) may discolor the urine.

A history should be obtained to ascertain a history of renal disease or urinary tractmalformation; the duration of gross hematuria; associated symptoms such abdominal, flank, orsuprapubic pain (suggesting an infection or renal malformation); fever (suggesting an infection);or the passage of sand, gravel, or stones. The next step in determining the cause of grosshematuria is to obtain a urinalysis to assess for blood, protein, and RBCs.

The association of blood and protein in the urine with an elevated blood pressure reportedfor the boy in the vignette strongly indicates new-onset glomerular disease. The low serumcomplement values are consistent with acute glomerulonephritis (AGN). Of the options listed,only membranoproliferative GN (MPGN) and postinfectious AGN (PIAGN) result in reducedserum complements, and of these two conditions, the C3 and C4 are reduced only in patientswho have MPGN. Although PIAGN is much more common than MPGN, only C3 is low in PIAGN.Patients who have immunoglobulin A nephropathy typically develop recurrent episodes of grosshematuria, but the serum complement values are normal. Patients who have infections maydevelop gross hematuria, but this is rare and usually is limited to viral cystitis. Finally, grosshematuria is common in patients who have nephrolithiasis, but they usually experience pain anddo not develop significant proteinuria or hypocomplementemia.

One of the most common causes of gross hematuria in children is hypercalciuria, with orwithout renal stones. Hypercalciuria usually is idiopathic, but it may be due to causes ofhypercalcemia (eg, hyperparathyroidism, malignancy, Addison disease, sarcoidosis, vitamin Dtherapy) or result from increased urinary excretion of calcium (eg, furosemide therapy, Barttersyndrome, Dent disease [inherited hypercalciuria], distal renal tubular acidosis). Grosshematuria may occur in patients who have sickle cell disease or trait due to intrarenal sicklingand sludging.

References:

Bergstein J, Leiser J, Andreoli S. The clinical significance of asymptomatic gross andmicroscopic hematuria in children. Arch Pediatr Adolesc Med. 2005;159:353-355. Abstractavailable at:http://www.ncbi.nlm.nih.gov/entrez/query.fcgi?orig_db=PubMed&db=PubMed&cmd=Search&term=%22Archives+of+pediatrics+%26+adolescent+medicine%22[Jour]+AND+353[page]+AND+2005[pdat]

Feld LG, Waz WR, Perez LM, Joseph DB. Hematuria. An integrated medical and surgicalapproach. Pediatr Clin North Am. 1997;44:1191-1210. Abstract available at:http://www.ncbi.nlm.nih.gov/entrez/query.fcgi?cmd=Retrieve&db=pubmed&dopt=Abstract&list_uids=9326958&query_hl=39&itool=pubmed_docsum

Patel HP, Bissler JJ. Hematuria in children. Pediatr Clin North Am. 2001;48:1519-1537. Abstractavailable at:http://www.ncbi.nlm.nih.gov/entrez/query.fcgi?orig_db=PubMed&db=PubMed&cmd=Search&term=%22Pediatric+clinics+of+North+America%22[Jour]+AND+1519[page]+AND+2001[pdat]

Copyright © 2007 by the American Academy of Pediatrics page 227

Page 228: AAP MCQ 2007

2007 PREP SA on CD-ROM

Question: 110

You are evaluating a 2-year-old boy in your office for recurrent cellulitis of his right thigh. Thepatient has had three episodes in the last 4 months. According to his mother, all the episodesstart with a "red bump" that progressively enlarges and, in most cases, drains spontaneously.He has received two courses of cephalexin in the past, but there was no clinical improvementuntil the abscess spontaneously drained. On one occasion, an incision and drainage procedurehad to be performed. Except for pain with walking, the patient has been afebrile and experiencedno other systemic symptoms. On physical examination, you note a 6x6 cm area of indurationand erythema on the lateral right thigh that is warm, firm, and tender to palpation. There is noactive drainage from the site.

Of the following, the MOST appropriate antibiotic for treatment of this patient is

A. amoxicillin

B. amoxicillin/clavulanic acid

C. clindamycin

D. cefdinir

E. cephalexin

Copyright © 2007 by the American Academy of Pediatrics page 228

Page 229: AAP MCQ 2007

2007 PREP SA on CD-ROM

Preferred Response: CCritique: 110

The most common etiologic agents associated with skin and soft-tissue infections areStaphylococcus aureus and group A streptococci. The patient described in the vignette has hadrecurrent episodes of cellulitis and abscess that did not respond to cephalexin, a beta-lactamantibiotic. He most likely has an infection that is caused by a strain of community-acquiredmethicillin-resistant S aureus. Over the last 5 years, this organism has emerged as thepredominant pathogen causing skin and soft-tissue infections nationwide. The pathogen is notsusceptible to any of the beta-lactam antibiotics, including amoxicillin, amoxicillin/clavulanic acid,cefdinir, and cephalexin. Clindamycin, trimethoprim/sulfamethoxazole, vancomycin, thetetracyclines, linezolid, and levofloxacin are the drugs of choice for the treatment of infectionscaused by these organisms. However, because of growing resistance to these drugs,susceptibility testing should be performed, especially when treating staphylococcal infections.Staphylococci that are susceptible to clindamycin but resistant to erythromycin may exhibitinducible clindamycin resistance (resistance develops while the patient is receiving clindamycintherapy), which should be confirmed using a double-disk diffusion test. Regardless of theantibiotic selected, appropriate management of an abscess includes incision and drainage.

Clindamycin is a semisynthetic antibiotic that is a member of the lincosamide family ofantibiotics whose mechanism of action is binding to the bacterial 50S ribosomal subunit, therebyinterfering with protein synthesis. It is considered a bacteriostatic agent but has bactericidalactivity against some strains of staphylococci, streptococci, and anaerobes, includingBacteroides fragilis, depending on drug concentration and bacterial growth conditions. It also hasdemonstrated the ability to inhibit production of staphylococcal exotoxin associated with toxicshock syndromes and to facilitate opsonization and phagocytosis of bacteria, even at low drugconcentrations. The drug is well absorbed following oral administration, reaching peak serumconcentrations within 1 hour, with intravenous administration resulting in peak concentrationswithin 20 to 45 minutes. Clindamycin reaches therapeutic concentrations in most body fluids andtissues, including bones and joints; exceptions include cerebrospinal fluid and bile. The drug ismetabolized primarily in the liver, with most of the parent drug and its metabolites eliminated inthe bile.

Clindamycin is active against most gram-positive cocci, most anaerobes, and certainprotozoa. Of the aerobic gram-positive cocci, it is active against streptococci (groups A, B, Cand G; S bovis; microaerophilic streptococci; and most strains of pneumococci and viridansstreptococci), staphylococci (methicillin-sensitive S aureus, certain strains of methicillin-resistantS aureus, and S epidermidis), and Corynebacterium diphtheriae. However, the drug has noactivity against enterococci. Among the anaerobes, it is active against most gram-positive cocci(Peptostreptococcus sp), gram-positive nonspore-forming bacilli (Actinomyces sp,Propionibacterium sp), clostridia (excluding C difficile and a significant percentage of somenonperfringens clostridial species), and gram-negative bacilli (Bacteroides, Prevotella,Fusobacterium sp). It also has activity against Chlamydia trachomatis and certain protozoalpathogens (when used in combination with other agents), such as Plasmodium sp,Pneumocystis carinii, Toxoplasma gondii, and Babesia sp.

References:

Bradley J. Newer antistaphylococcal agents. Curr Opin Pediatr. 2005;17:71-77. Abstractavailable at:http://www.ncbi.nlm.nih.gov/entrez/query.fcgi?orig_db=PubMed&db=PubMed&cmd=Search&term=%22Current+opinion+in+pediatrics%22[Jour]+AND+71[page]+AND+2005[pdat]

Falagas ME, Gorbach SL. Clindamycin and metronidazole. Med Clin North Am. 1995;79:845-867. Available at:http://www.ncbi.nlm.nih.gov/entrez/query.fcgi?orig_db=PubMed&db=PubMed&cmd=Search&term=%22The+Medical+clinics+of+North+America%22[Jour]+AND+845[page]+AND+1995[pdat]

Kasten MJ. Clindamycin, metronidazole, and chloramphenicol. Mayo Clin Proc. 1999;74:825-

Copyright © 2007 by the American Academy of Pediatrics page 229

Page 230: AAP MCQ 2007

2007 PREP SA on CD-ROM

833. Available at:http://www.ncbi.nlm.nih.gov/entrez/query.fcgi?db=pubmed&cmd=Retrieve&dopt=Abstract&list_uids=10473362&query_hl=11&itool=pubmed_docsum

Marcinak JF, Frank AL. Treatment of community-acquired methicillin-resistant Staphylococcusaureus in children. Curr Opin Infect Dis. 2003;16:265-269. Abstract available at:http://www.ncbi.nlm.nih.gov/entrez/query.fcgi?orig_db=PubMed&db=PubMed&cmd=Search&term=%22Current+opinion+in+infectious+diseases%22[Jour]+AND+265[page]+AND+2003[pdat]

Weingarten-Arams J, Adam HM. In brief: clindamycin. Pediatr Rev. 2002;23:149-150. Availableat: http://pedsinreview.aappublications.org/cgi/content/full/23/4/149

Copyright © 2007 by the American Academy of Pediatrics page 230

Page 231: AAP MCQ 2007

2007 PREP SA on CD-ROM

Question: 111

You are seeing a 2-month-old male infant who has trisomy 21 for a health supervision visit. Theboy's mother expresses concern that the infant has been having "noisy breathing" during thepast 2 to 3 weeks. The infant has been exclusively formula-fed and has had no choking ordifficulty feeding. According to the mother, the noise, which occurs on inspiration, is louder whenthe infant is supine and when crying. She has not noticed any rhinorrhea, cough, or other upperrespiratory viral illness symptoms. The infant was born via an uneventful vaginal delivery that didnot require forceps. Apgar scores were 8 and 9 at 1 and 5 minutes, respectively. On physicalexamination, the infant, whose physical appearance is consistent with trisomy 21, is restingcomfortably. His vital signs are appropriate for age, but you hear an audible noise duringinspiration.

Of the following, the MOST likely explanation for the infant's respiratory symptoms is

A. laryngomalacia

B. subglottic tracheal web

C. tracheomalacia

D. vascular ring

E. vocal cord paralysis

Copyright © 2007 by the American Academy of Pediatrics page 231

Page 232: AAP MCQ 2007

2007 PREP SA on CD-ROM

Preferred Response: ACritique: 111

When evaluating an infant who presents with “noisy breathing,” it is important to obtain athorough birth history and current medical history, observe breathing patterns in differentpositions, and auscultate the airway. Secondary evaluation tools include radiography,spirometry, and direct airway visualization.

In general, breathing noises can be classified as inspiratory, expiratory, or biphasic.Disorders at the level of and superior to the vocal cords result in a harsh, inspiratory noisecalled stridor (Item C111A). Lesions inferior to the vocal cords typically result in wheezing, ahigh-pitched expiratory noise.

Laryngomalacia is the most common congenital laryngeal abnormality resulting in stridor.Symptoms may begin shortly after birth, although they typically occur between 1 and 2 monthsafter birth. As with the child described in the vignette, infants are happy, thriving, and not havingdifficulty during feedings, but stridor usually worsens during supine positioning, increased cryingor agitation, or a viral illness. Direct visualization of an omega-shaped epiglottis that prolapses(Item C111B) during inspiration is a hallmark of the condition. Severe cases may require surgicalcorrection, although laryngomalacia usually improves spontaneously by 2 years of age.

A laryngeal web results from failure of the embryonic airway to recanalize. Most laryngealwebs occur at the level of the vocal cords and present at birth with stridor. The delayed onset ofstridor in the infant in the vignette makes a laryngeal web unlikely.

Tracheomalacia (Item C111C) results from defective cartilaginous rings causing flaccidity ofthe tracheal wall. Both tracheomalacia and laryngomalacia present at similar ages and areexacerbated by crying, viral respiratory infections, and supine positioning. However, infants whohave tracheomalacia present with wheezing instead of stridor.

Extrinsic compression of the trachea by vascular anomalies such as a vascular ring (ItemC111D) can result in recurrent wheezing that is worsened with crying, feeding, or neck flexion.The right-sided aortic arch with left ligamentum arteriosum and the double aortic arch accountfor most cases.

Unilateral or bilateral vocal cord paralysis (Item C111E) can be associated with inspiratorystridor, but often results in a weak cry, coughing, and choking. When vocal cord paralysis issuspected, a thorough search for underlying causes (eg, congenital central lesions, atrialenlargement, birth trauma, traumatic forceps delivery, prior surgical procedures) should beundertaken.

References:

Bertrand P, Navarro H, Caussade S, Holmgren N, Sanchez I. Airway anomalies in children withDown syndrome: endoscopic findings. Pediatr Pulmonol. 2003;36:137-141. Abstract available at:http://www.ncbi.nlm.nih.gov/entrez/query.fcgi?orig_db=PubMed&db=PubMed&cmd=Search&term=Pediatr+Pulmonol[Jour]+AND+137[page]+AND+2003[pdat]

Boat TF. Chronic or recurrent respiratory symptoms. In: Behrman RE, Kliegman RM, JensonHB, eds. Nelson Textbook of Pediatrics. 17th ed. Philadelphia, Pa: WB Saunders Co; 2004:1401-1404

Paston F, Bye M. In brief: tracheomalacia. Pediatr Rev. 1996;17:328

Vicencio AG, Parikh S. Laryngomalacia and tracheomalacia: common dynamic airway lesions.Pediatr Rev. 2006; 27:e33-35. Available athttp://pedsinreview.aappublications.org/cgi/content/full/27/4/e33

Copyright © 2007 by the American Academy of Pediatrics page 232

Page 233: AAP MCQ 2007

2007 PREP SA on CD-ROM

Question: 112

An 11-year-old child is brought to the emergency department with massive trauma after a motorvehicle crash. His Glasgow Coma Scale score is 3. Rapid assessment reveals marked bruisingof the face and blood from the mouth, nose, and ears. In addition, the child has large areas ofbruising and abrasions across the chest, abdomen, and pelvis as well as bright red blood fromthe urethral meatus and a deformity of the left femur. Among the initial major interventions areinsertion of an endotracheal tube and establishment of two large-bore intravenous lines.

Of the following, the MOST appropriate next step is to

A. administer broad-spectrum antibiotics to treat his contaminated wounds

B. administer tetanus toxoid

C. obtain upright abdominal films to assess for ruptured viscus

D. order contrast-enhanced magnetic resonance imaging of his head

E. place a Foley catheter into his bladder to monitor urine output

Copyright © 2007 by the American Academy of Pediatrics page 233

Page 234: AAP MCQ 2007

2007 PREP SA on CD-ROM

Preferred Response: BCritique: 112

Tetanus toxoid should be administered to all victims of massive trauma who have contaminatedwounds and have not received this vaccine within the past 5 years or whose immunizationstatus is not known. For the patient described in the vignette, the widespread areas of abrasionput him at particular risk for tetanus, much like a burn victim.

A patient who has blood at the urethral meatus, as described in the vignette, should beassumed to have a urethral injury. Blind placement of a Foley catheter is discouraged becausesuch action may convert a partial urethral tear into a complete transection. Retrogradeurethrography should be performed to evaluate for this injury.

Upright abdominal films have no role in the evaluation of a patient who has had massivetrauma. The cervical, thoracic, and lumbosacral spine must remain immobilized during the initialmanagement of such patients. Optimal radiologic evaluation of the abdomen in major traumavictims is accomplished by computed tomography. Ideally, this should be performed using bothintravenous and oral contrast.

Computed tomography of the head, not magnetic resonance imaging, is an important part ofthis patient’s evaluation. Contrast adds no information to the scan, so it should not beadministered.

Prophylactic antibiotics are not warranted for the patient in the vignette. Even in the case ofsevere burns, prophylactic antibiotics do not significantly decrease the risk of secondaryinfections, and they may select for resistant organisms.

References:

American Academy of Pediatrics. Tetanus. In: Pickering LK, ed. Red Book: 2003 Report of theCommittee on Infectious Diseases. 26th ed. Elk Grove Village, Ill: American Academy ofPediatrics; 2003:611-616

Garcia CT. Genitourinary trauma. In: Fleisher GR, Ludwig S, Henretig FM, eds. Textbook ofPediatric Emergency Medicine. Philadelphia, Pa: Lippincott Williams & Wilkins; 2006:1463-1474

American College of Surgeons. Abdominal trauma. In: Advanced Trauma Life Support® Programfor Doctors. 6th ed. Chicago, Ill: American College of Surgeons; 1997:157-175

Copyright © 2007 by the American Academy of Pediatrics page 234

Page 235: AAP MCQ 2007

2007 PREP SA on CD-ROM

Question: 113

A 6-week-old infant presents with excessive tearing of the right eye, as evidenced by overflowof tears onto the eyelid and cheek, and frequent appearance of mucoid material. The childexhibits no systemic symptoms.

Of the following, the MOST appropriate primary management is

A. administration of systemic antibiotics

B. fluorescein staining of the cornea

C. instillation of ophthalmic corticosteroid drops

D. nasolacrimal massage and cleansing of the lids

E. referral to ophthalmology for nasolacrimal duct probing

Copyright © 2007 by the American Academy of Pediatrics page 235

Page 236: AAP MCQ 2007

2007 PREP SA on CD-ROM

Preferred Response: DCritique: 113

Tears enter either the upper or lower punctum at the medial aspect of the eye and drain into thenasolacrimal duct (NLD). Among neonates, obstruction of the NLD frequently occurs at the sitewhere the duct passes through the maxilla into the nose. Some 6% of newborns exhibitobstruction (Item C113A) when the embryonic structures fail to fully canalize. NLD is observedwith increased frequency in certain populations, including preterm infants and children who haveDown syndrome. Signs of obstruction include pooling or overflowing of tears, which may not beobserved until normal tear production develops. Often, there is accumulation of mucoid materialat the medial canthus. Stasis of tears in the obstructed NLD results in conditions favorable forbacterial infection. Progression of infection can lead to cellulitis of the overlying soft tissue.

Preferred management of the infant described in the vignette is to recommend massageoverlying the NLD two to three times daily and cleansing of the lids as necessary. Wheninfection complicates the obstruction of the drainage system, instillation of a topical ophthalmicantibiotic is justified, but systemic antibiotics are not indicated. NLD obstruction typically resolvesby 9 to 12 months of age. Persistence of symptoms beyond this time justifies referral to anophthalmologist for possible probing of the NLD.

For the patient in the vignette, the cornea is not involved, and fluorescein staining would notbe beneficial. Instillation of a topical ophthalmic corticosteroid is not indicated in the managementof this disorder and is associated with significant risks.

References:

Olitsky SE, Nelson LB. Disorders of the lacrimal system. In: Behrman RE, Kliegman RM, JensonHB, eds. Nelson Textbook of Pediatrics. 17th ed. Philadelphia, Pa: WB Saunders Co; 2004:2099

Prasad S. Congenital nasolacrimal duct obstruction. Pediatr Rev. 1994:15:88.

Copyright © 2007 by the American Academy of Pediatrics page 236

Page 237: AAP MCQ 2007

2007 PREP SA on CD-ROM

Question: 114

A 16-year-old girl presents to your office with a recent diagnosis of peptic ulcer. Thegastroenterologist has prescribed sucralfate.

Of the following, the condition that is a relative contraindication to sucralfate treatment is

A. congenital heart disease

B. congenital hepatic fibrosis

C. diabetes mellitus

D. end-stage renal disease

E. steroid-dependent asthma

Copyright © 2007 by the American Academy of Pediatrics page 237

Page 238: AAP MCQ 2007

2007 PREP SA on CD-ROM

Preferred Response: DCritique: 114

In 2005, the primary therapy of a gastric or duodenal ulcer involves treatment with a protonpump inhibitor such as omeprazole, lansoprazole, or pantoprazole. If Helicobacter pylori also ispresent, the pathogen should be eradicated through combination therapy with a proton pumpinhibitor (eg, omeprazole, lansoprazole, pantoprazole) and two antibiotics (eg, tetracycline +clarithromycin, amoxicillin + metronidazole, amoxicillin + clarithromycin). At times, othermedications, including histamine-2 receptor blockers, antacids, and sucralfate, are used asadjunctive therapy. Histamine-2 blockers reduce acid secretion, antacids neutralize gastric acid,and sucralfate coats the inflamed area and forms a protective barrier.

Sucralfate is an aluminum hydroxide complex of sucrose that is effective both for treatmentand prevention of peptic ulcers; it is generally less effective as monotherapy compared witheither histamine blockers or proton pump inhibitors. It also has been used in a wide variety ofother gastrointestinal conditions, including radiation-induced mucositis and proctitis, nonulcerdyspepsia, and bile reflux gastritis. Although the precise mechanism of action of sucralfate isunclear, it is believed to coat the ulcer region, leaving it less susceptible to acid-induced injury.Sucralfate generally is well tolerated, but it may cause constipation and decrease thebioavailability of certain medications (eg, fluoroquinolones, ketoconazole, warfarin). In addition,because it contains aluminum and has been reported to increase plasma aluminumconcentrations in patients who have renal failure, it is not recommended for use in such patients.Congenital heart disease, congenital hepatic fibrosis, diabetes mellitus, and steroid-dependentasthma are not contraindications to sucralfate treatment.

References:

Jacobson K, Chiba N, Chen Y, et al. Gastric acid secretory response in Helicobacter pylori-positive patients with duodenal ulcer disease. Can J Gastroenterol. 2001;15:29-39. Abstractavailable at:http://www.ncbi.nlm.nih.gov/entrez/query.fcgi?cmd=Retrieve&db=pubmed&dopt=Abstract&list_uids=11173324&query_hl=26&itool=pubmed_DocSum

Sherrard DJ. Aluminum—much ado about nothing [comment]. N Engl J Med. 1991;324:558-559

Copyright © 2007 by the American Academy of Pediatrics page 238

Page 239: AAP MCQ 2007

2007 PREP SA on CD-ROM

Question: 115

You are making rounds in the neonatal intensive care unit and discussing the complications ofnecrotizing enterocolitis (NEC) with the residents.

Of the following, the MOST likely long-term complication following NEC is

A. encopresis

B. failure to thrive

C. gastroesophageal reflux disease

D. intestinal stricture

E. pneumatosis intestinalis

Copyright © 2007 by the American Academy of Pediatrics page 239

Page 240: AAP MCQ 2007

2007 PREP SA on CD-ROM

Preferred Response: DCritique: 115

The diagnosis of neonatal necrotizing enterocolitis (NEC) is based on clinical manifestations(poor feeding, vomiting, gastrointestinal tract bleeding, septic shock); laboratory data andphysical findings (thrombocytopenia, abdominal distention); and radiologic evidence of an ileus(paucity of bowel gas), intramural air (intestinal pneumatosis) (Item C115A), portal venous air, orfree air in the abdomen indicative of bowel perforation (pneumoperitoneum) (Item C115B). Earlyradiographic findings may be nonspecific and include dilated loops of intestine and bowel wallthickening consistent with edema. One third of cases lack radiographic findings, and in these,NEC is diagnosed only at surgery or autopsy. Very low-birthweight (<1,500 g) infants areaffected most frequently by NEC, with an incidence of approximately 10% in this population inmodern neonatal intensive care units (NICUs). The overall mortality for NEC ranges from 10%to 30%. It is a multifactorial disorder, the precise causes of which have not been completelyelucidated. Risk factors include prematurity (most frequent), congenital heart disease, early andrapid feeding, hyperosmolar formulas, maternal cocaine use, respiratory distress syndrome,and perinatal asphyxia.

The clinical staging of NEC is given in Item C115C.Initial management of NEC includes making the infant nil per os, passing a nasogastric tube

and applying low intermittent suction, providing intravenous fluids and nutrition, and administeringappropriate antibiotic therapy. Additional monitoring of abdominal radiographic findings andlaboratory parameters (complete blood count, platelet count, coagulation profile, and serumelectrolyte concentrations) is essential. Surgical intervention may be necessary in up to 50% ofinfants and includes placing a peritoneal drain, performing an exploratory laparotomy, or bowelresection.

Complications from NEC can be separated into acute or short-term problems encounteredin the NICU or longer-term problems encountered after weeks or months. Among the short-termconcerns are surgical site, anastomotic, or stoma problems; dehiscence or fistula development;infection; and recurrent NEC. Longer-term complications are either related to nutrition andgrowth (associated with degree of short bowel syndrome, ability to feed enterally, ordependance on parenteral nutrition) and late-occurring surgical complications such asstrictures.

Strictures (Item C115D) form at the site of either old medically managed NEC or at asurgical anastomotic site. More than 10% of infants who have had NEC develop intestinalstrictures. Pneumatosis intestinalis is an acute finding with active NEC. Gastroesophageal refluxdisease is no more common in infants who have NEC than in other preterm infants. Theincidence of encopresis is not increased in children who have a history of NEC. Failure to thrivemay occur in infants who have a history of NEC and short bowel syndrome. Although shortbowel syndrome may occur in up to 25% of infants who have NEC, management with balancedenteral and parenteral nutrition largely avoids overt failure to thrive. Recent evidence attests tothe negative impact that prematurity combined with NEC and associated lengthy hospitalizationhas on overall neurodevelopment in extremely low-birthweight infants (<1,000 g). Up to 50% ofthose who survive NEC experience some neurodevelopmental delay.

References:

Blakely ML, Lally KP, McDonald S, et al. Postoperative outcomes of extremely low birth-weightinfants with necrotizing enterocolitis or isolated intestinal perforation: a prospective cohort studyby the NICHD Neonatal Research Network. Ann Surg. 2005;241:984-989. Available at:http://www.annalsofsurgery.com/pt/re/annos/abstract.00000658-200506000-00016.htm;jsessionid=Ej4fiXcDG59KsIFpGCZWXdkTOzFTsC7BXVazr8Wl2NPl0lsrxBzx!-1861031787!-949856144!9001!-1

Caplan MS, Jilling T. The pathophysiology of necrotizing enterocolitis. NeoReviews. 2001;2:e103-e109. Available at: http://neoreviews.aappublications.org/cgi/content/full/2/5/e103

Dimmitt RA, Moss RL. Clinical management of necrotizing enterocolitis. NeoReviews.

Copyright © 2007 by the American Academy of Pediatrics page 240

Page 241: AAP MCQ 2007

2007 PREP SA on CD-ROM

2001;2:e110-e117. Available at: http://neoreviews.aappublications.org/cgi/content/full/2/5/e110

Hintz SR, Kendrick DE, Stoll BJ, et al for the NICHD Neonatal Research Network.Neurodevelopmental and growth outcomes of extremely low birth weight infants after necrotizingenterocolitis. Pediatrics. 2005;115:696-703. Available at:http://pediatrics.aappublications.org/cgi/content/full/115/3/696

Springer SC, Annibale DJ. Necrotizing enterocolitis. eMedicine Specialties: Pediatrics:Neonatology. 2002. Available at: http://www.emedicine.com/ped/topic2601.htm

Stoll BJ, Kliegman RM. Digestive system disorders: neonatal necrotizing enterocolitis. In:Behrman RE, Kliegman RM, Jenson HB, eds. Nelson Textbook of Pediatrics. 17th ed.Philadelphia, Pa: Saunders; 2004:590-592

Copyright © 2007 by the American Academy of Pediatrics page 241

Page 242: AAP MCQ 2007

2007 PREP SA on CD-ROM

Question: 116

A 2-year-old boy is brought to your clinic because he has a nighttime cough. According to hismother, several times over the past few months he has awakened with a barking, nonproductivecough that improves by the next morning. She denies fever and rhinorrhea with the episodes.Evaluation of his lungs yields normal results.

Of the following, the clinical feature that is MOST suggestive of spasmodic croup rather thanrecurrent laryngotracheobronchitis in this boy is

A. age of the patient

B. barking nature of the cough

C. lack of rhinorrhea and fever

D. nonproductive nature of the cough

E. normal findings on physical examination

Copyright © 2007 by the American Academy of Pediatrics page 242

Page 243: AAP MCQ 2007

2007 PREP SA on CD-ROM

Preferred Response: CCritique: 116

Acute laryngotracheobronchitis, or croup, is the most common cause of acute upper airwayobstruction in children. It usually is caused by a virus (typically parainfluenza), and it may affectinfants, toddlers, and school-age children. Common features include low-grade fever,rhinorrhea, and mild cough, followed 1 to 3 days later by a harsh, nonproductive, “barking”cough (Item C116A) and inspiratory stridor. Laryngoscopy rarely is indicated, but if performed,reveals erythema and edema of the tracheal epithelium. Symptoms usually abate within 1 week.Recurrent infections may occur, especially in children who attend child care and are repeatedlyexposed to viral illnesses.

Spasmodic croup is similar to acute laryngotracheobronchitis in that the cough is often“barking” and nonproductive. It also may affect young children. Allergic and psychologicalfactors are believed to be primarily responsible for spasmodic croup. Therefore, the classicinfectious signs of rhinorrhea and fever are not seen. Symptoms may last for only a few hoursand generally not as long as with infectious croup. If performed, laryngoscopy reveals boggy,pale edema of the epithelium, with little to no inflammatory changes.

References:

Cressman WR, Myer CM 3rd. Diagnosis and management of croup and epiglottitis. Pediatr ClinNorth Am. 1994;41:265-276. Abstract available at:http://www.ncbi.nlm.nih.gov/entrez/query.fcgi?cmd=Retrieve&db=pubmed&dopt=Abstract&list_uids=8139876&query_hl=3&itool=pubmed_docsum

Malhotra A, Krilov LR. Viral croup. Pediatr Rev. 2001;22:5-12. Available at:http://pedsinreview.aappublications.org/cgi/content/full/22/1/5

Roosevelt GE. Acute inflammatory upper airway obstruction. In: Behrman RE, Kliegman RM,Jenson HB, eds: Nelson Textbook of Pediatrics. 17th ed. Philadelphia, Pa: WB Saunders Co;2004:1405-1409

Copyright © 2007 by the American Academy of Pediatrics page 243

Page 244: AAP MCQ 2007

2007 PREP SA on CD-ROM

Question: 117

A 4-year-old boy presents for a health supervision visit, and in the course of the visit, his motherdiscloses that ever since the birth of his 2-month-old sister, the boy has resumed bedwetting,which had ceased to be a regular occurrence. He is dry during the day and has no stoolincontinence. Past medical history reveals that the child has never been hospitalized, had anegative urine culture at age 9 months associated with a fever, and was toilet trained completelyat age 3, with only episodic bedwetting about once every month. Physical examination reveals ahappy, playful child whose growth and developmental parameters are normal for age and whohas a circumcised penis and normal findings on scrotal examination. Urinalysis of a specimenobtained via clean catch urination shows normal results.

Of the following, the next BEST step in the management of this boy is to

A. begin imipramine therapy at bedtime

B. obtain urine for culture and sensitivity

C. perform renal ultrasonography

D. reassure the mother that this is most likely a temporary regression

E. refer the boy for voiding cystourethrography

Copyright © 2007 by the American Academy of Pediatrics page 244

Page 245: AAP MCQ 2007

2007 PREP SA on CD-ROM

Preferred Response: DCritique: 117

Bedwetting (nocturnal enuresis) is primary if the child has never been dry at night. It issecondary when a child has previously sustained dryness at night and subsequently resumesbedwetting. Only 2% to 3% of children who have nocturnal enuresis have a contributing physicalproblem; most have either diminished bladder capacity (also evidenced by daytime wetting ordribbling) or diminished nocturnal arousal (also evidenced by failure to awaken with wet clothes).This is a maturational problem, and parents should be reassured that the problem generallyresolves over time. Essentially, a child who has enuresis is unable to awaken to the sensation ofa full bladder or unable to awaken to urinate in the toilet. Secondary enuresis usually representsa “relapse” of physiologic primary enuresis. True regression due to psychological stressorsoften results in diurnal enuresis or other behavioral regressions.

No specific evaluation with laboratory tests or imaging is required for either primary orsecondary enuresis other than a clean voided specimen for urinalysis to exclude diabetes andinfection.

Enuresis may be treated with expectant management. Other treatment modalities includealarms and medications. Alarms may have a better long-term success rate than medicationssuch as desmopressin, imipramine, or oxybutynin. These medications have good short-termsuccess rates but also are associated with more adverse effects.

References:

Glazener CMA, Evans JHC, Peto R. Tricyclic and related drugs for nocturnal enuresis inchildren. The Cochrane Database of Systematic Reviews. 2003;3:CD002117. Available at:http://www.mrw.interscience.wiley.com/cochrane/clsysrev/articles/CD002117/frame.html

Lawless MR, McElderry DH. Nocturnal enuresis: current concepts. Pediatr Rev. 2001;22:399-407. Available at: http://pedsinreview.aappublications.org/cgi/content/full/22/12/399

Schmitt BD. Nocturnal enuresis. Pediatr Rev. 1997;18:183-191. Available at:http://pedsinreview.aappublications.org/cgi/content/full/18/6/183

Copyright © 2007 by the American Academy of Pediatrics page 245

Page 246: AAP MCQ 2007

2007 PREP SA on CD-ROM

Question: 118

The parents of a 4-month-old bring the boy to you because of failure to thrive. The baby wasborn after a term gestation, and there were no complications at delivery. The mother reports thathe has a weak suck, and his body seems limp. On physical examination, the child's weight is 4.1kg, length is 52 cm, and occipitofrontal circumference is 40.5 cm. His penis appears small forage (Item Q118A), and the child exhibits diffuse hypotonia. Deep tendon reflexes are normal.Other findings on the physical examination are normal.

Of the following, the test MOST likely to establish this child's diagnosis is

A. arylsulfatase A measurement

B. electromyography with nerve conduction velocities

C. fluorescent in situ hybridization testing

D. magnetic resonance imaging of the brain

E. thyroid-stimulating hormone measurement

Copyright © 2007 by the American Academy of Pediatrics page 246

Page 247: AAP MCQ 2007

2007 PREP SA on CD-ROM

Preferred Response: CCritique: 118

The limpness or floppiness reported for the infant in the vignette points to hypotonia, which oftenis disproportionate to weakness. The trunk and extremities of the infant who has hypotonia hanglimply when the child is suspended horizontally. When the clinician suspends the child verticallyby holding the infant’s axillae, the baby will “slip through” the examiner’s hands, rather thanadducting his or her shoulders.

Hypotonia is found in a wide variety of pathophysiologic processes arising throughout themotor system. Cerebral, or central, causes of hypotonia include chromosomal abnormalities(Down syndrome, Turner syndrome, Prader-Willi syndrome), metabolic abnormalities(hypothyroidism, leukodystrophies), sepsis, and perinatal trauma (hypoxia-ischemia orintracranial hemorrhage). The most worrisome cause of hypotonia localizing to the brainstemand spine is spinal muscular atrophy. Neuropathy is rarely a cause of infant hypotonia(congenital polyneuropathy, familial dysautonomia), but peripheral hypotonia commonly stemsfrom the neuromuscular junction (botulism, myasthenia gravis, or myasthenic syndrome) ormuscle (muscular dystrophy and congenital myopathies).

Clues on physical examination often help lead to the diagnosis. Depressed or absent deeptendon reflexes are seen with spinal muscular atrophy, neuropathies, and even botulism.Tongue fasciculations are common with spinal muscular atrophy. Botulism produces rostralgreater than caudal weakness. Low birthweight, almond-shaped palpebral fissures, andhypogonadism are typical of Prader-Willi syndrome.

Laboratory investigations are targeted toward the disorders suggested by physicalexamination findings. The child in the vignette has physical findings typical for Prader-Willisyndrome; fluorescent in situ hybridization testing for the gene locus at the 15q11-13chromosomal region is indicated. Arylsulfatase A testing assesses for metachromaticleukodystrophy, which develops later in the first postnatal year. Measuring thyroid-stimulatinghormone is not likely to be useful because the boy lacks signs of hypothyroidism.Electromyography and nerve conduction studies are used to assess a congenital myopathy orspinal muscular atrophy. Magnetic resonance imaging of the brain would be useful if there werea history of perinatal trauma or concern about cerebral dysgenesis.

References:

Berg BO. The hypotonic infant. In: Child Neurology: A Clinical Manual. 2nd ed. Philadelphia, Pa:JB Lippincott Co; 1994:279-286

Stiefel L. In brief: hypotonia in infants. Pediatr Rev. 1996;17:104-105

Copyright © 2007 by the American Academy of Pediatrics page 247

Page 248: AAP MCQ 2007

2007 PREP SA on CD-ROM

Question: 119

The mother of a 6-month-old girl calls you because the infant has not been feeding well all dayand appears pale. You tell her to meet you at the emergency department. Upon arrival, you findan awake, alert, but irritable child who wants to take her bottle but fatigues quickly thereafter. Onphysical examination, she has no fever, a respiratory rate of 50 breaths/min, heart rate of 250beats/min, and blood pressure of 80/50 mm Hg. Her oxygen saturation is 98% in room air. Herlungs are clear, with good aeration. Her rapid pulse is palpable in all extremities, and she has acapillary refill time of 2 seconds. You administer oxygen and place the child on a cardiac monitor,which reveals a rapid heart rate with a narrow QRS complex (Item Q119A).

Of the following, the BEST management plan for this infant is

A. carotid massage

B. electrical cardioversion

C. intravenous administration of adenosine

D. intravenous administration of verapamil

E. oral administration of digoxin

Copyright © 2007 by the American Academy of Pediatrics page 248

Page 249: AAP MCQ 2007

2007 PREP SA on CD-ROM

Preferred Response: CCritique: 119

Any sustained or frequently recurring tachyarrhythmia should prompt medical attention toevaluate the mechanism and cause of the arrhythmia and initiate treatment. The medicaltreatment of a tachyarrhythmia can be approached in terms of acute and chronic therapy. Acutetherapy depends on the patient’s clinical presentation. The evaluation should begin withassessment of the airway, breathing, and the circulation (ABCs). For patients who have alteredconsciousness, diminished perfusion, and shock, immediate and emergent therapy to addressthe ABCs is undertaken. Following this, emergent treatment of the arrhythmia may include theuse of electric cardioversion or medications.

A more deliberate approach to diagnosis and management can be undertaken for patientswho are hemodynamically stable. For arrhythmias that use the atrioventricular node (AVN) aspart of the re-entrant circuit, blocking conduction through the AVN can be therapeutic.Increasing vagal tone affects the AVN by slowing conduction through it. Although vagalmaneuvers such as ocular pressure and carotid massage are NOT recommended in children,the resulting changes in vagal input on the heart are likely the mechanism by which thesemaneuvers may be effective. The fastest and safest therapy to block the AVN is the use ofintravenous adenosine, which works by interrupting conduction through the AVN. Its half-life isextremely short (seconds), requiring intravenous push administration followed by a bolus of 5 to10 mL of normal saline, so that the medication quickly reaches the central circulation. Adverseeffects of adenosine are generally minor and short-lived and may include flushing and apressure sensation or feeling of tightness in the chest.

The patient described in the vignette has a narrow QRS complex tachycardia that isconsistent with a supraventricular tachycardia or paroxysmal atrial tachycardia. Sucharrhythmias invariably use the AVN to complete the circuit and, therefore, typically respond wellto the use of medications that block conduction through the AVN, such as adenosine. Verapamil,a calcium channel blocker, also affects the AVN, but its use is contraindicated in childrenyounger than 1 year of age because of reports that it can cause hypotension, bradycardia, andeven sudden death in infants who have supraventricular tachycardia. Carotid massage is notperformed in children for several reasons, including the risk of soft-tissue injury. Oral digoxinwould not be used in this case because its onset of action is significantly longer than that ofadenosine.

References:

Balaji S. Supraventricular tachycardia in children. Curr Treat Options Cardiovasc Med.2000;2:521-528. Abstract available at:http://www.ncbi.nlm.nih.gov/entrez/query.fcgi?orig_db=PubMed&db=PubMed&cmd=Search&term=%22Current+treatment+options+in+cardiovascular+medicine%22[Jour]+AND+2000[pdat]+AND+Balaji+S[author]

Chun TU, Van Hare GF. Advances in the approach to treatment of supraventricular tachycardiain the pediatric population. Curr Cardiol Rep. 2004;6:322-326. Abstract available at:http://www.ncbi.nlm.nih.gov/entrez/query.fcgi?orig_db=PubMed&db=PubMed&cmd=Search&term=%22Current+cardiology+reports%22[Jour]+AND+2004[pdat]+AND+Chun[author]

Dixon J, Foster K, Wyllie J, Wren C. Guidelines and adenosine dosing in supraventriculartachycardia. Arch Dis Child. 2005;90:1190-1191. Abstract available at:http://www.ncbi.nlm.nih.gov/entrez/query.fcgi?cmd=Retrieve&db=pubmed&dopt=Abstract&list_uids=16243875&query_hl=29&itool=pubmed_docsum

Copyright © 2007 by the American Academy of Pediatrics page 249

Page 250: AAP MCQ 2007

2007 PREP SA on CD-ROM

Question: 120

A 42-year-old primigravida is in her 16th week of pregnancy. She is concerned about thepossibility of her child having myelomeningocele.

Of the following, the MOST useful diagnostic evaluation is

A. amniocentesis

B. chorionic villus sampling

C. cordocentesis

D. fetal ultrasonography

E. maternal alpha-fetoprotein screening

Copyright © 2007 by the American Academy of Pediatrics page 250

Page 251: AAP MCQ 2007

2007 PREP SA on CD-ROM

Preferred Response: DCritique: 120

Fetal ultrasonography performed between 12 and 24 weeks of pregnancy is most useful forearly detection of structural anomalies such as a myelomeningocele. Its sensitivity andspecificity as a screening test for fetal anomalies can be improved substantially if the screeningis performed by trained, experienced ultrasonographers. It is noninvasive, safe, and accurate inexperienced hands. Advantages of early fetal ultrasonography include improved diagnosis ofmultiple gestation and accurate dating of the pregnancy. This imaging technique may betechnically difficult in certain conditions, such as maternal obesity, and the uncertain outcome ofsome malformations and the late appearance of others may complicate interpretation of findings.

Although concentrations of alpha-fetoprotein (AFP) in amniotic fluid are increased in thefetus that has open defects of the spinal cord, this test has limited sensitivity and specificity forthe diagnosis of a myelomeningocele. Early amniocentesis, performed between 12 and 14weeks of pregnancy, is most useful for detecting fetal chromosomal abnormalities such astrisomy 21 and genetic defects such as cystic fibrosis, hemoglobinopathy, and certain inbornerrors of metabolism. The desquamated fetal cells (amniocytes) from the amnion provide asource of mitotically active cells that may be used for cytogenetic evaluation or DNA analysis.Midtrimester amniocentesis, performed between 15 and 25 weeks of pregnancy, is similarlyuseful. The most common indication for amniocentesis performed in the third trimester ofpregnancy is assessment of fetal lung maturity. This invasive procedure has potential risks ofbleeding, leakage of fluid, cramping, infection, and fetal loss.

Chorionic villus sampling, which entails a biopsy of the placenta between 10 and 12 weeksof pregnancy, is not useful for diagnosing a myelomeningocele. It is most helpful for detectingcytogenetic abnormalities in the fetus early in gestation, but there may be an increased risk forcomplications over amniocentesis (risk is operator-dependent).

Cordocentesis, which entails aspiration of fetal blood by direct puncture of the umbilical cordunder ultrasonographic guidance, generally is limited to pregnancies of more than 19 weeks’gestation and is not useful for the diagnosis of a myelomeningocele. It is most suitable forassessing the fetus that is at risk for acidosis, cytogenetic abnormalities, or hemolytic anemia.Cordocentesis also can aid in diagnosing other conditions, including coagulation disorders, bloodcell abnormalities, and congenital infections such as toxoplasmosis. This invasive procedurehas potential risks of bleeding, thromboembolism, placental abruption, fetal-maternalhemorrhage, infection, and fetal loss.

Although maternal serum AFP screening performed between 15 and 20 weeks of pregnancyis safe and useful for identification of myelomeningocele in the fetus, it does not delineate theextent and location of the lesion. Maternal serum AFP is increased in other fetal conditions, suchas abdominal wall defects and urogenital malformations, and it is decreased in Down syndrome.

References:

ACOG Practice Bulletin No. 58. Ultrasonography in pregnancy. Obstet Gynecol. 2004;104:1449-1458

Cunniff C, Committee on Genetics. Prenatal screening and diagnosis for pediatricians.Pediatrics. 2004;114:889-894. Available at:http://pediatrics.aappublications.org/cgi/content/full/114/3/889

Ghidini A. Fetal blood sampling: indications and invasive fetal therapy. UpToDate. 2006;14.1.Available at:http://www.utdol.com/utd/content/topic.do?topicKey=antenatl/6605&type=A&selectedTitle=1~12

Ghidini A, McLaren R. Chorionic villus sampling: risks, complications, and techniques.UpToDate. 2006;134.1. Available at:http://www.utdol.com/utd/content/topic.do?topicKey=antenatl/4910&type=A&selectedTitle=1~16

Copyright © 2007 by the American Academy of Pediatrics page 251

Page 252: AAP MCQ 2007

2007 PREP SA on CD-ROM

Question: 121

A 16-year-old boy presents with a very swollen, painful right knee (Item Q121A). He is a soccerplayer, but there is no history of recent injury. During the interview, you notice the boy hasinjected conjunctivae (Item Q121B).

Of the following, further evaluation MOST likely will reveal

A. alopecia areata

B. Gottron papules

C. Kayser-Fleischer rings

D. malar rash

E. urethritis

Copyright © 2007 by the American Academy of Pediatrics page 252

Page 253: AAP MCQ 2007

2007 PREP SA on CD-ROM

Preferred Response: ECritique: 121

The teenage boy described in the vignette displays two physical findings, arthritis andconjunctivitis, that are typical and suggestive of Reiter syndrome (RS). The third clinical findingof the classic triad for RS is urethritis. A finding of leukocyte esterase on a urine dipstick from amale patient suggests the presence of urethritis and is confirmed by the presence of 10 or morepolymorphonuclear leukocytes per high-power field in the centrifuged sediment of a first morningvoid. (Although RS most often is recognized in males, it also may occur in females who haveurethritis or cervicitis.) Dermatologic findings of RS are common and include balanitis; painlessulcers on the tongue, palate, pharynx, and buccal mucosa; onycholysis; and vesicles andpapules that mimic psoriasis (keratoderma blennorrhagicum). The arthritis associated with RS isoligo/monoarticular, often affecting the knees, ankles, feet, or wrists. RS is believed to resultfrom an autoimmune response to an infectious disease, most commonly the sexually transmittedChlamydia trachomatis infection. Individuals who have human leukocyte antigen B-27 are atincreased risk of developing RS.

Although the cause of alopecia areata (Item C121A) is unknown, there is an infrequentassociation with autoimmune diseases such as Hashimoto thyroiditis, Addison disease, vitiligo,and collagen vascular disease. However, these diseases are unlikely to present withmonoarticular arthritis and conjunctivitis. Gottron papules (erythematous papules located overthe metacarpal, phalangeal, and interphalangeal joints) (Item C121B) and violaceousdiscoloration of the upper eyelids (Item C121C) are associated with juvenile dermatomyositis(JDM). Other common clinical findings of JDM include arthralgia, fatigue, and proximal muscleweakness. Kayser-Fleischer rings are found in the corneas of patients who have Wilsondisease, a rare genetic disorder of copper metabolism. Monoarticular arthritis and conjunctivitisare not part of the usual clinical picture for JDM or Wilson disease. The finding of a malar rash(Item C121D) is suggestive of systemic lupus erythematosus (SLE), an unlikely diagnosis forthe patient described in the vignette. Although musculoskeletal findings of SLE include arthralgia,arthritis, tendonitis, and myositis, the clinical presentation of monoarticular arthritis is not typical,and conjunctivitis is not an associated finding. In addition, SLE presenting after puberty has afemale predominance of 8 to 1.

References:

Amor B. Reiter’s syndrome: diagnosis and clinical features. Rheum Dis Clin North Am.1998;24:677-695. Abstract available at:http://www.ncbi.nlm.nih.gov/entrez/query.fcgi?orig_db=PubMed&db=PubMed&cmd=Search&term=%22Rheumatic+diseases+clinics+of+North+America%22[Jour]+AND+677[page]+AND+1998[pdat]

Mazziotta JM, Ahmed N. Conjunctivitis and cervicitis. J Fam Prac. 2004;53:121-123. Availableat: http://www.jfponline.com/Pages.asp?AID=1648

Shrier LA. Bacterial sexually transmitted infections: gonorrhea, chlamydia, pelvic inflammatorydisease, and syphilis. In: Emans SJH, Laufer MR, Goldstein DP, eds. Pediatric and AdolescentGynecology. 5th ed. Philadelphia, Pa: Lippincott, Williams & Wilkins; 2005:565-614

Simpson T, Oh K. Urethritis and cervicitis in adolescents. Adolesc Med Clin. 2004;15:253-271

Copyright © 2007 by the American Academy of Pediatrics page 253

Page 254: AAP MCQ 2007

2007 PREP SA on CD-ROM

Question: 122

A mother brings her 14-year-old son to your office complaining that she can never get him toclean his neck and asks you to talk with him. You note that he is overweight, with a body massindex of 32 mg/m2, and he has a Sexual Maturity Rating 3 for pubic hair, with testes that areabout 10 mL. He has velvety dark pigmentation (Item Q122A) on the back of his neck and underhis arms.

On laboratory testing, the boy is most likely to have

A. elevated cortisol concentration

B. elevated fasting glucose value

C. elevated hemoglobin A1c measurement

D. low high-density lipoprotein cholesterol value

E. low triglyceride concentrations

Copyright © 2007 by the American Academy of Pediatrics page 254

Page 255: AAP MCQ 2007

2007 PREP SA on CD-ROM

Preferred Response: DCritique: 122

The dark velvety skin pigmentation described for the boy in the vignette is acanthosis nigricans(Item C122A) and is a result of insulin stimulation of skin insulin-like growth factor-1 receptors.Hyperinsulinism in obese children is due to insulin resistance induced by adipose tissue andinactivity. Puberty also intensifies insulin resistance so that findings frequently are more obviousafter the beginning of sexual maturation. Insulin resistance is associated with a constellation ofphysical findings, including obesity, increased abdominal girth, hypertension, and acanthosisnigricans. Laboratory findings include low levels of high-density lipoprotein cholesterol, hightriglyceride concentrations, and glucose intolerance or frank diabetes.

Although Cushing syndrome, which is characterized by elevated cortisol concentrations,could be associated with insulin resistance, acanthosis nigricans, and obesity, it is a relativelyrare disorder compared with exogenous obesity and insulin resistance. Frank type 2 diabeteswith fasting hyperglycemia or elevated hemoglobin A1c as a marker for hyperglycemia is muchless common than insulin resistance with compensatory hyperinsulinism and normal glucosetolerance.

References:

Burke JP, Hale DE, Hazuda HP, Stern MP. A quantitative scale of acanthosis nigricans.Diabetes Care. 1999;22:1655-1659. Available at:http://care.diabetesjournals.org/cgi/reprint/22/10/1655

Hirschler V, Aranda C, Calcagno Mde L, Maccalini G, Jadzinsky M. Can waist circumferenceidentify children with the metabolic syndrome? Arch Pediatr Adolesc Med. 2005;159:740-744.Abstract available at:http://www.ncbi.nlm.nih.gov/entrez/query.fcgi?orig_db=PubMed&db=PubMed&cmd=Search&term=%22Archives+of+pediatrics+%26+adolescent+medicine%22[Jour]+AND+740[page]+AND+2005[pdat]

Ten S, Maclaren N. Insulin resistance syndrome in children. J Clin Endocrinol Metab.2004;89:2526-2539. Available at: http://jcem.endojournals.org/cgi/content/full/89/6/2526

Copyright © 2007 by the American Academy of Pediatrics page 255

Page 256: AAP MCQ 2007

2007 PREP SA on CD-ROM

Question: 123

You diagnose attention-deficit/hyperactivity disorder in a 10-year-old boy and recommendtreatment with methylphenidate. His mother asks about potential adverse effects of thetreatment.

Of the following, the MOST common adverse effect of treatment is

A. delayed sleep onset

B. depression

C. hallucinations

D. tics

E. weight gain

Copyright © 2007 by the American Academy of Pediatrics page 256

Page 257: AAP MCQ 2007

2007 PREP SA on CD-ROM

Preferred Response: ACritique: 123

Most children who receive medication for treatment of attention-deficit/hyperactivity disorder(ADHD) are prescribed stimulant medications, such as methylphenidate or dextroamphetamine.Stimulants generally are considered safe, with generally mild and short-lived adverse effects.The most common adverse effects are decreased appetite, irritability, and delayed sleep onset.Approximately 15% to 30% of children experience motor tics, most of which are transient and donot represent an absolute contraindication to continuing with the medication. Although stimulantsdo not cause Tourette syndrome, they may unmask the tic symptoms.

The safety profile of stimulants is being reviewed, and in February 2006, the United StatesFood and Drug Administration placed a black box warning for cardiovascular risk, includingsudden death and stroke, especially in children who have underlying heart disease.

Other medications for ADHD include atomoxetine and tricyclic antidepressants. Atomoxetineis a norepinephrine reuptake inhibitor that had a similar adverse effect profile to stimulants inclinical trials, but has been associated with nausea, hepatic injury, and risk of suicidal ideation.Tricyclic antidepressants typically cause sedation and related anticholinergic adverse effectssuch as dry mouth and constipation.

If the child in the vignette experiences delayed sleep onset, the condition can be managedby changing the timing of the medication dose or decreasing an afternoon dose. Depression andhallucinations are rare adverse effects of stimulant medication. Tics are less common thanchanges in sleep onset. A child who receives stimulant medication for treatment of ADHD ismore likely to have weight loss or lack of weight gain related to decreased appetite than weightgain.

References:

American Academy of Pediatrics Committee on Quality Improvement, Subcommittee onAttention-Deficit/Hyperactivity Disorder. Clinical practice guideline: diagnosis and evaluation ofthe child with attention-deficit/hyperactivity disorder. Pediatrics. 2000;105:1158-1170. Availableat: http://pediatrics.aappublications.org/cgi/content/full/105/5/1158

American Academy of Pediatrics Subcommittee on Attention-Deficit/Hyperactivity Disorder andCommittee on Quality Improvement. Clinical practice guideline: treatment of the school-agedchild with attention-deficit/hyperactivity disorder. Pediatrics. 2001;108:1033-1044. Available at:http://pediatrics.aappublications.org/cgi/content/full/108/4/1033

MTA Cooperative Group. National Institute of Mental Health multimodal treatment study of ADHDfollow-up: 24-month outcomes of treatment strategies for attention-deficit/hyperactivity disorder.Pediatrics. 2004;113:754-761. Available at:http://pediatrics.aappublications.org/cgi/content/full/113/4/754

Reiff MI, Tippins S, LeTourneau AA. ADHD: A Complete and Authoritative Guide. Elk GroveVillage, Ill: American Academy of Pediatrics; 2004

Sims MD. Attention-deficit/hyperactivity disorder. In: Behrman RE, Kliegman RM, Jenson HB,eds. Nelson Textbook of Pediatrics. 17th ed. Philadelphia, Pa: WB Saunders Co; 2004:107-110

Copyright © 2007 by the American Academy of Pediatrics page 257

Page 258: AAP MCQ 2007

2007 PREP SA on CD-ROM

Question: 124

A 2-year-old boy presents to the emergency department and requires intubation due to apnea.Subsequent tests reveal a diagnosis of meningococcal meningitis. Because of the emergentnature of the intubation, you were not able to put on a mask prior to performing this task.Therefore, you will need to take prophylactic medications to prevent transmission of theorganism to you.

Of the following, the MOST appropriate antimicrobial agent for prophylaxis is

A. azithromycin

B. cefdinir

C. doxycycline

D. penicillin

E. rifampin

Copyright © 2007 by the American Academy of Pediatrics page 258

Page 259: AAP MCQ 2007

2007 PREP SA on CD-ROM

Preferred Response: ECritique: 124

Postexposure prophylaxis with rifampin, ceftriaxone, or ciprofloxacin should be given to high-riskcontacts of a patient who has meningococcal meningitis. All persons who have had contact withthe patient’s oral secretions or those who have examined the throat are considered high risk. Inaddition, those living in the same house as the patient or those having significant contact with thepatient (eg, best friends, child care, school) also should be considered for prophylaxis. The drugof choice is rifampin (10 mg/kg [maximum, 600 mg] orally every 12 hours for 2 days).Ceftriaxone in a single dose of 125 mg for those younger than 15 years of age and 250 mg forthose 15 years of age and older can be used, but it is expensive and should be reserved forpatients who are unable to take rifampin or ciprofloxacin (eg, pregnant women). A single dose ofciprofloxacin (500 mg) can be administered to patients older than 18 years of age.Meningococcal vaccine can be used as an adjunct to chemoprophylaxis when outbreaks arecaused by a serogroup contained within available vaccines (eg, A, C, Y, W-135). Azithromycin,cefdinir, doxycycline, and penicillin are not indicated for the prevention of meningococcal diseasebecause they do not eradicate nasopharyngeal colonization.

Other uses for rifampin include: 1) in combination with antistaphylococcal regimens to treatinvasive or recurrent staphylococcal disease (eg, endocarditis, osteomyelitis); 2) in combinationwith other medications (eg, isoniazid, pyrazinamide) to treat disease due to Mycobacteriumtuberculosis; and 3) to eradicate the nasal carriage of other bacteria (eg, Haemophilusinfluenzae type b, Staphylococcus aureus, Streptococcus pyogenes).

References:

Adam HM. Rifampin. Pediatr Rev. 2004;25:216-217. Available at:http://pedsinreview.aappublications.org/cgi/content/full/25/6/216

American Academy of Pediatrics. Meningococcal infections. In: Pickering LK, ed. Red Book:2006 Report of the Committee on Infectious Diseases. 27th ed. Elk Grove Village, Ill: AmericanAcademy of Pediatrics; 2006:452-460

Kaye D. Current use for old antibacterial agents: polymyxins, rifampin, and aminoglycosides.Infect Dis Clin North Am. 2004;18:669-689

Copyright © 2007 by the American Academy of Pediatrics page 259

Page 260: AAP MCQ 2007

2007 PREP SA on CD-ROM

Question: 125

You are asked to evaluate a 15 year-old boy who presented to the emergency department withgross hematuria that began the day before. He ran his first marathon yesterday. He reports nodysuria, urgency, frequency, or abdominal or flank pain. His vital signs and physical examinationfindings are normal. Urinalysis reveals: specific gravity, 1.010; pH, 6.0; large blood; no protein; 0to 2 red blood cells; and 0 to 2 white blood cells. A serum creatinine is 2.7 mg/dL (238.7mcmol/L). His complete blood count is normal.

Of the following, the MOST likely cause of his hematuria and renal failure is

A. hemoglobinuria

B. immunoglobulin A nephropathy

C. myoglobinuria

D. urinary tract obstruction

E. urolithiasis

Copyright © 2007 by the American Academy of Pediatrics page 260

Page 261: AAP MCQ 2007

2007 PREP SA on CD-ROM

Preferred Response: CCritique: 125

Because routine screening of urine has become a common practice, many children who areotherwise asymptomatic are being discovered to have microscopic hematuria. For most ofthese children who have no other urinary findings and are asymptomatic, it is unlikely that anunderlying cause will be ascertained. In contrast, determining a specific cause of grosshematuria in children is far more likely.

History and physical examination are essential for assessing the child who has grosshematuria. Pertinent information includes the presence of prior renal disease and symptomssuch as fever, pain, or edema. The physical examination should focus on probing for edema,bruising, joint swelling, or rashes. Regardless of the findings, a urinalysis is mandatory. Bloodand significant (greater than 1+) protein are strongly suggestive of glomerular or interstitialdisease. Microscopic analysis of a centrifuged urine specimen is necessary to determine if redblood cells (RBCs) are present. In the absence of RBCs, other causes of a dipstick positive for“blood” include hemoglobin, myoglobin, or porphyrins.

The likelihood of myoglobinuria is strong for the boy described in the vignette because of theabsence of RBCs in the urine and the antecedent strenuous exercise. He probably hasdeveloped acute renal failure due to rhabdomyolysis, myoglobin deposition in the kidney, andurinary tract obstruction. Hemoglobinuria is rare in the absence of hemolysis. Immunoglobulin Anephropathy may present with gross hematuria, but RBCs should be detected on microscopicanalysis. Although patients who have urinary tract infections often have microscopic hematuria,gross hematuria is unusual. Finally, patients who have urolithiasis may develop gross hematuria,but concomitant colicky pain is usually present.

Myoglobinuria is caused by any process that involves excessive destruction of muscle orrhabdomyolysis. This may occur after excessive, strenuous exercise in hot weather. Othercauses of myoglobinuria include crush injury to muscles from trauma, mitochondrial disorders,other inherited muscle diseases (eg, carnitine palmitoyltransferase II deficiency, McArdledisease), malignant hyperthermia, or rarely after systemic viral illness or due to medicationusage (eg, HMG-CoA reductase inhibitors).

References:

Allison RC, Bedsole DL. The other medical causes of rhabdomyolysis. Am J Med Sci.2003;326:79-88. Abstract available at:http://www.ncbi.nlm.nih.gov/entrez/query.fcgi?db=pubmed&cmd=Retrieve&dopt=AbstractPlus&list_uids=12920439

Malinoski DJ, Slater MS, Mullins RJ. Crush injury and rhabdomyolysis. Crit Care Clin.2004;20:171-192. Abstract available at:http://www.ncbi.nlm.nih.gov/entrez/query.fcgi?db=pubmed&cmd=Retrieve&dopt=AbstractPlus&list_uids=14979336

Melli G, Chaudhry V, Cornblath DR. Rhabdomyolysis: an evaluation of 475 hospitalized patients.Medicine. 2005;84:377-385. Abstract available at:http://www.ncbi.nlm.nih.gov/entrez/query.fcgi?cmd=Retrieve&db=pubmed&dopt=Abstract&list_uids=16267412&query_hl=41&itool=pubmed_docsum

Copyright © 2007 by the American Academy of Pediatrics page 261

Page 262: AAP MCQ 2007

2007 PREP SA on CD-ROM

Question: 126

You are evaluating a 7-year-old boy who has sickle cell disease for a 5-day history of fever,rhinorrhea, headache, and worsening fatigue. Physical examination reveals a tired-appearing,pale boy who has a temperature of 101.5ºF (38.6ºC); heart rate of 150 beats/min; very paleconjunctivae and mucous membranes; and a faint diffuse, erythematous, lacy rash that is mostprominent on his cheeks and trunk. Laboratory tests show a white blood cell count of12x103/mcL (12x109/L), with 50% neutrophils, 45% lymphocytes, and 5% monocytes;hemoglobin of 4 g/dL (40 g/L); hematocrit of 16% (0.16); and reticulocyte count of less than 1%(0.01).

Of the following, the MOST likely cause for this patient's present illness is infection with

A. coxsackievirus

B. cytomegalovirus

C. Epstein-Barr virus

D. parainfluenza virus

E. parvovirus B19

Copyright © 2007 by the American Academy of Pediatrics page 262

Page 263: AAP MCQ 2007

2007 PREP SA on CD-ROM

Preferred Response: ECritique: 126

Erythema infectiosum (fifth disease) is caused by human parvovirus B19, a nonenveloped,single-stranded DNA virus that infects only humans and replicates primarily in humanerythrocyte precursors. The receptor for the virus is the erythrocyte P antigen, which is found inthe red blood cell precursor cell line and in a variety of other tissues, including myocardium,endothelium, placenta, megakaryocyte, and fetal liver.

The modes of transmission include contact with respiratory tract secretions, percutaneousexposure to blood or blood products, and vertical transmission from mother to fetus. Mostinfections are asymptomatic or mild. The most common findings in symptomatic disease arefever in 15% to 30% of patients and a distinctive rash. The facial rash is intensely erythematous,giving the patient a “slapped cheek” appearance (Item C126A), and often is accompanied bycircumoral pallor. A symmetric, maculopapular, lacy rash (Item C126B) also may develop on thetrunk, as described for the boy in the vignette, and subsequently spread to involve the buttocksand extremities. The rash can be exacerbated by environmental temperature, exercise, hotbaths, stress, or sunlight and can persist for weeks to months. As reported for the boy in thevignette, a brief, mild, nonspecific prodromal illness consisting of fever, malaise, myalgias, andheadache often precedes the development of the rash by 7 to 10 days. Arthralgia and arthritis,or both, of the small joints of the hands and occasionally the ankles, knees, and wrists canoccur, especially in infected adult women.

Among patients who have chronic hemolytic anemias (eg, sickle cell disease, glucose-6-phosphate dehydrogenase deficiency, and autoimmune hemolytic anemia) and other conditionsassociated with low hemoglobin concentrations, including hemorrhage, severe anemia, andthalassemia, parvovirus B19 infection can cause a transient aplastic crisis lasting for up to 10days. Parvovirus infects and destroys red blood cell precursors preferentially, causingreticulocytopenia and decreased red blood cell synthesis. Although infection withcoxsackievirus, cytomegalovirus, Epstein-Barr virus, and parainfluenza virus may present withfever, rhinorrhea, headache, and worsening fatigue, they would not cause severe anemia or therash described.

Parvovirus B19 infection occurring during pregnancy can damage the placenta and leads toinfection in approximately 30% of fetuses. This may cause fetal hydrops and death, with the riskof fetal death following infection being between 2% and 6%; the greatest risk for death is wheninfection occurs during the first half of pregnancy. Parvovirus B19 infection is not believed to beteratogenic, and children who have had infections in utero usually are healthy at follow-up.

References:

American Academy of Pediatrics. Parvovirus B19 (erythema infectiosum, fifth disease). In:Pickering LK, ed. Red Book: 2006 Report of the Committee on Infectious Diseases. 27th ed. ElkGrove Village, Ill: American Academy of Pediatrics; 2006:484-487

Brown KE. Human parvovirus B19. In: Long SS, Pickering LK, Prober CG, eds. Principles andPractice of Pediatric Infectious Diseases. 2nd ed. New York, NY: Churchill Livingstone;2003:1101-1104

Chorba T, Coccia P, Holman RC, et al. The role of parvovirus B19 in aplastic crisis anderythema infectiosum (fifth disease). J Infect Dis. 1986;154:383-393. Abstract available at:http://www.ncbi.nlm.nih.gov/entrez/query.fcgi?orig_db=PubMed&db=PubMed&cmd=Search&term=%22The+Journal+of+infectious+diseases%22[Jour]+AND+383[page]+AND+1986[pdat]

Miller E, Fairley CK, Cohen BJ, Seng C. Immediate and long term outcome of human parvovirusB19 infection in pregnancy. Br J Obstet Gynaecol. 1998;105:174-178. Abstract available at:http://www.ncbi.nlm.nih.gov/entrez/query.fcgi?db=pubmed&cmd=Retrieve&dopt=AbstractPlus&list_uids=9501782

Young NS, Brown KE. Parvovirus B19. N Engl J Med. 2004;350:586-597

Copyright © 2007 by the American Academy of Pediatrics page 263

Page 264: AAP MCQ 2007

2007 PREP SA on CD-ROM

Question: 127

A 3-year-old boy presents to the emergency department following the abrupt onset of coughingand wheezing. You order a chest radiograph for evaluation of a suspected foreign bodyaspiration.

Of the following, the MOST appropriate statement regarding foreign body aspiration is that

A. most foreign body aspirations present within 24 hours

B. nonfood items (eg, coins, pins, pencaps) are the most common items aspirated by infants andtoddlers

C. the classic triad of cough, wheeze, and decreased breath sounds is present in most cases

D. the majority of aspirated foreign bodies are located in the larynx or trachea

E. toy balloons are a common cause of foreign body aspirations

Copyright © 2007 by the American Academy of Pediatrics page 264

Page 265: AAP MCQ 2007

2007 PREP SA on CD-ROM

Preferred Response: ACritique: 127

The presentation of a foreign body aspiration depends on whether the event was witnessed, theage of the child, the type of object aspirated, the elapsed time since the event, and the location ofthe object. Most foreign body aspirations (50% to 75%) present and are diagnosed within 24hours. Although an acute choking or coughing episode accompanied by wheezing is highlysuspicious for a foreign body aspiration, some infants and children present with few or nosymptoms. The classic triad of coughing, wheezing, and decreased breath sounds is present inless than 50% of cases.

Food is the most common item aspirated by infants and toddlers (eg, sunflower seeds, nuts,beans, carrots, corn); nonfood items (eg, coins, paper clips, pins, pen caps) more commonlyare aspirated by older children. Although a rare cause of foreign body aspiration, toy balloonaspiration can be fatal.

Most aspirated foreign bodies lodge in the right lung rather than the larynx or trachea (ItemC127A). Long-term complications of undiagnosed or retained foreign bodies include recurrentpneumonia, atelectasis, and bronchiectasis.

References:

Holinger LD. Foreign bodies of the airway. In: Behrman RE, Kliegman RM, Jenson HB, eds.Nelson Textbook of Pediatrics. 17th ed. Philadelphia, Pa: WB Saunders Co; 2004:1410-1411

Karakoc F, Karadag B, Akbenlioglu C, et al. Foreign body aspiration: what is the outcome?Pediatr Pulmonol. 2002;34:30-36. Abstract available at:http://www.ncbi.nlm.nih.gov/entrez/query.fcgi?cmd=Retrieve&db=pubmed&dopt=Abstract&list_uids=12112794&query_hl=111&itool=pubmed_docsum

Copyright © 2007 by the American Academy of Pediatrics page 265

Page 266: AAP MCQ 2007

2007 PREP SA on CD-ROM

Question: 128

A 14-year-old girl presents to your office with complaints of a red and "irritated" eye for the past12 hours. She wears contact lenses, but has not worn them since the previous evening. Herpain and inflammation have continued to worsen despite removing the contact lenses. Onphysical examination, you note diffuse moderate injection of the bulbar conjunctiva of the lefteye. Her extraocular motions and pupillary reflexes are normal. Results of funduscopicexamination are normal. There is no discharge. Fluorescein examination reveals diffuse uptakeof stain but no evidence of corneal abrasion.

Of the following, the MOST appropriate next step in the management of this child is to

A. apply a soft patch to the eye and refer to an ophthalmologist within 2 to 3 days

B. arrange for urgent consultation with an ophthalmologist

C. prescribe an ophthalmic antibiotic and ask her to return for re-examination in 2 to 3 days

D. prescribe no antibiotics but ask her to return for re-examination in 2 to 3 days

E. prescribe ophthalmic antihistamine drops

Copyright © 2007 by the American Academy of Pediatrics page 266

Page 267: AAP MCQ 2007

2007 PREP SA on CD-ROM

Preferred Response: BCritique: 128

A red eye in a patient who wears contact lenses may represent dangerous corneal infectionsdue to gram-negative bacteria or fungi or to breakdown and ulceration of corneal epithelium.Such patients should have their contact lenses removed and be evaluated by an ophthalmologistwithin 12 hours. Empiric antibiotics are not recommended; antibiotic therapy should be directedby the ophthalmologist, depending on the problem identified. For those who do not wear contactlenses and who have either corneal abrasion or bacterial conjunctivitis, empiric therapy with atopical antibiotic is appropriate.

Common causes for red eye in contact lens wearers include microbial keratitis, contact lens-induced peripheral ulcer (CLPU), and contact lens-induced acute red eye (CLARE). Microbialkeratitis is an infection of the cornea characterized by excavation and necrosis of tissue.Inappropriate treatment or a significant delay in treatment can result in visual impairment.Removal of contact lenses and frequent instillation of sterile saline drops generally result incomplete recovery from CLPU and CLARE.

Fluorescein stain is taken up by damaged ocular epithelial cells. In a corneal abrasion,fluorescein uptake is focal at the site of injury. With microbial keratitis and CLARE, the uptake isboth multifocal and diffuse.

Prescribing ophthalmic antihistamine drops and asking the patient to return for re-examination in 2 to 3 days are unnecessary delays to further evaluation and treatment by anophthalmologist.

References:

Hume EBH, Stapleton F, Willcox MDP. Evasion of cellular ocular defenses by contact lensisolates of Serratia marcescens. Eye Contact Lens. 2003;29:108-112. Abstract available at:http://www.ncbi.nlm.nih.gov/entrez/query.fcgi?orig_db=PubMed&db=PubMed&cmd=Search&term=Eye+Contact+Lens[Jour]+AND+108[page]+AND+2003[pdat]

Levin AV. Eye—Red. In: Fleisher GR, Ludwig S, Henretig FM, eds. Textbook of PediatricEmergency Medicine. 5th ed. Philadelphia, Pa: Lippincott Williams & Wilkins; 2006:267-271

Kim J. The use of vital dyes in corneal disease. Curr Opin Ophthalmol. 2000;11:241-247.Abstract available at:http://www.ncbi.nlm.nih.gov/entrez/query.fcgi?cmd=Retrieve&db=pubmed&dopt=Abstract&list_uids=10977768&query_hl=11&itool=pubmed_docsum

Sweeney DF, Jalbert I, Covey M, et al. Clinical characterization of corneal infiltrative eventsobserved with soft contact lens wear. Cornea. 2003;22:435-442. Abstract available at:http://www.ncbi.nlm.nih.gov/entrez/query.fcgi?orig_db=PubMed&db=PubMed&cmd=Search&term=Cornea[Jour]+AND+435[page]+AND+2003[pdat]

Copyright © 2007 by the American Academy of Pediatrics page 267

Page 268: AAP MCQ 2007

2007 PREP SA on CD-ROM

Question: 129

The mother of a 4-year-old otherwise healthy-appearing boy brings him in for evaluationbecause he had profound generalized body odor (bromhidrosis) for the past several days. Uponcareful questioning, the mother recalls an episode of sneezing followed by a 3- to 4-day historyof a purulent, blood-tinged nasal discharge.

Of the following, the most likely cause of the body odor is

A. allergic rhinitis

B. epidermolytic hyperkeratosis

C. nasal foreign body

D. pachyonychia congenita

E. premature adrenarche

Copyright © 2007 by the American Academy of Pediatrics page 268

Page 269: AAP MCQ 2007

2007 PREP SA on CD-ROM

Preferred Response: CCritique: 129

Insertion of small objects such as beads, berries, seeds, and paper into the nares is a commonpractice of young children. Undetected nasal foreign bodies can lead to local irritation, mucosaledema, and possible infection, producing a bloody, mucous discharge. This sequence of eventsfrequently results in a noticeably unpleasant odor that may be perceived as generalized bodyodor (bromhidrosis), as described for the boy in the vignette.

Allergic rhinitis generally produces thin, watery secretions that occur seasonally or year-round. Bromhidrosis is not present. Epidermolytic hyperkeratosis (EHK), an autosomal dominantdisorder, is characterized by extensive skin scaling at birth, erythroderma, and recurrent bullaeformation. By school age, bromhidrosis may result, as thick, dirty-yellow scales (Item C129A)develop on palms, soles, elbows, knees and become malodorous due to bacterial colonization.Purulent nasal secretions are not characteristic of EHK. Pachyonychia congenita is a raregenodermatosis in which combinations of ectodermal defects produce thickened, tented nails(Item C129B); palmoplantar hyperkeratosis; and hyperhidrosis that leads to marked foot odor.Premature adrenarche is an early maturational event of adrenal androgen production in whichsexual hair appears before the age of 9 years in boys who have no other evidence of pubertaldevelopment. Axillary odor is common.

References:

Garibaldi L. Incomplete (partial) precocious development. In: Behrman RE, Kliegman RM,Jenson HB, eds. Nelson Textbook of Pediatrics. 17th ed. Philadelphia, Pa: WB Saunders Co;2004:1868

Haddad J Jr. Acquired disorders of the nose. In: Behrman RE, Kliegman RM, Jenson HB, eds.Nelson Textbook of Pediatrics. 17th ed. Philadelphia, Pa: WB Saunders Co; 2004:1387

Weston WL, Lane AT, Morelli JG. Genodermatoses: ichthyosis. In: Color Textbook of PediatricDermatology. 3rd ed. St. Louis, Mo: Mosby; 2002:270-274

Copyright © 2007 by the American Academy of Pediatrics page 269

Page 270: AAP MCQ 2007

2007 PREP SA on CD-ROM

Question: 130

A 15-year-old girl who has juvenile rheumatoid arthritis has been treated with ibuprofen 30 mg/kgper day for 3 months. She has had epigastric abdominal pain for 1 month that has beenunresponsive to empiric therapy with omeprazole 20 mg/day. You are considering addingmisoprostol 100 mcg four times daily to her current treatment regimen.

Of the following, a TRUE statement regarding misoprostol is that the drug

A. frequently causes abdominal pain

B. frequently causes constipation

C. is a cyclooxygenase-2 inhibitor

D. is contraindicated in pregnancy

E. is effective treatment for bleeding ulcers

Copyright © 2007 by the American Academy of Pediatrics page 270

Page 271: AAP MCQ 2007

2007 PREP SA on CD-ROM

Preferred Response: DCritique: 130

The gastric epithelium is exposed constantly to both gastric acid (secreted by the parietal cells)and the proteolytic enzyme pepsin (secreted by the chief cells). To prevent cell damage andgastric ulcer formation, the gastric epithelium is coated by a protective layer primarily composedof mucin and bicarbonate. Prostaglandin E stimulates mucus production and is important inmaintaining the integrity of the protective mucin glycoprotein-bicarbonate layer. The protectivelayer can be damaged by Helicobacter pylori organisms and nonsteroidal anti-inflammatorydrugs (NSAIDs) such as ibuprofen and aspirin, leading to peptic ulcer disease.

Misoprostol is a synthetic prostaglandin used to prevent peptic ulcer disease in high-riskpatients. It is employed most commonly in patients receiving long-term NSAID therapy, such asthe girl described in the vignette, who is taking aspirin for treatment of arthritis. Althoughmisoprostol is effective as a preventive agent, it is not useful in the acute treatment of bleedingpeptic ulcers. Common adverse events include nausea and diarrhea; abdominal pain andconstipation are not frequent adverse effects. In addition, because it is a synthetic prostaglandin,it may stimulate uterine contractions. Thus, misoprostol can cause spontaneous abortion and iscontraindicated in pregnancy. Because ulcer prophylaxis with proton pump inhibitors is moreeffective and better tolerated than misoprostol, gastroenterologists rarely use misoprostol today.

References:

Lanza FL. A guideline for the treatment and prevention of NSAID-induced ulcers. Members ofthe Ad Hoc Committee on Practice Parameters of the American College of Gastroenterology.Am J Gastroenterol. 1998;93:2037-2046

Stupnicki T, Dietrich K, González-Carro P, et al. Efficacy and tolerability of pantoprazolecompared with misoprostol for the prevention of NSAID-related gastrointestinal lesions andsymptoms in rheumatic patients. Digestion. 2003;68:198-208. Available at:http://content.karger.com/produktedb/produkte.asp?typ=fulltext&file=DIG2003068004198

Copyright © 2007 by the American Academy of Pediatrics page 271

Page 272: AAP MCQ 2007

2007 PREP SA on CD-ROM

Question: 131

The mother of an infant born at 34 weeks' gestation asks you whether any of the medicationsshe was given prior to delivery will have any effect on her daughter's growth and development.

Of the following, the MOST correct statement concerning drugs commonly used in labor is that

A. beta-adrenergic tocolytic agent safety for the fetus is inversely related to the dose and durationof maternal treatment

B. hypoglycemia may complicate the neonatal course of infants whose mothers were treated withbeta-adrenergic tocolytic agents

C. neonatal hypermagnesemia is an asymptomatic incidental electrolyte problem that rarelyfollows maternal treatment with magnesium sulfate

D. opioids used for analgesia are safest when administered within 4 hours of delivery

E. tocolysis with indomethacin is preferred over beta-adrenergic agents because of its lack ofadverse effects for the fetus and newborn

Copyright © 2007 by the American Academy of Pediatrics page 272

Page 273: AAP MCQ 2007

2007 PREP SA on CD-ROM

Preferred Response: BCritique: 131

In addition to antibiotics, numerous medications may be administered to a woman in labor (ItemC131A). These may be prescribed for analgesia, management of pregnancy-inducedhypertension, seizure prophylaxis, tocolysis, or for fetal indications (eg, to enhance fetal lungmaturity).

Beta-adrenergic tocolytics (eg, terbutaline) may be administered subcutaneously for short-term tocolysis or orally for longer-term effects. Beta-adrenergic effects are related directly to thedose and duration of treatment. One of the adverse effects is fetal hyperinsulinemia andsubsequent neonatal hypoglycemia. Indomethacin is only recommended for short-term tocolysis(<48 h) and only before 32 weeks’ completed gestation due to its hemodynamic risks to thefetus. Neonatal hypermagnesemia can follow maternal treatment with magnesium sulfate and isassociated with a number of adverse effects. Opioid analgesic risks to the newborn are reducedif more than 4 hours have passed since the mother received her last dose, but, if necessary,can be reversed with naloxone.

References:

Benitz WE, Druzin ML. Drugs that affect neonatal resuscitation. NeoReviews. 2005;6:e189-e195. Available at: http://neoreviews.aappublications.org/cgi/content/full/6/4/e189

Slattery MM, Morrison JJ. Preterm delivery. Lancet. 2002;360:1489-1497. Abstract available at:http://www.ncbi.nlm.nih.gov/entrez/query.fcgi?cmd=Retrieve&db=pubmed&dopt=Abstract&list_uids=12433531&query_hl=36&itool=pubmed_docsum

Copyright © 2007 by the American Academy of Pediatrics page 273

Page 274: AAP MCQ 2007

2007 PREP SA on CD-ROM

Question: 132

A 3-month-boy who has been previously healthy is brought to the emergency department with a3-day history of rhinorrhea, mild cough, and wheezing. He has been afebrile and has had somedifficulty feeding. His pulse oximetry reading is 90% on room air, respiratory rate is 60breaths/min, and heart rate is 130 beats/min. Chest examination reveals mild subcostalretractions, scattered wheezes, and coarse crackles bilaterally. The rest of the physicalexamination findings are normal.

Of the following, the pathogen that is MOST likely responsible for his symptoms is

A. adenovirus

B. Chlamydia trachomatis

C. Haemophilus influenzae

D. respiratory syncytial virus

E. Streptococcus pneumoniae

Copyright © 2007 by the American Academy of Pediatrics page 274

Page 275: AAP MCQ 2007

2007 PREP SA on CD-ROM

Preferred Response: DCritique: 132

The acute onset of wheezing, shortness of breath, and rhinorrhea plus nonfocal wheezes andbilateral crackles described for the infant in the vignette are most consistent with acute viralbronchiolitis. Bronchiolitis is a lower respiratory tract illness in which the bronchioles areobstructed with mucus and edema. Clinical features include a mild upper respiratory tractinfection with rhinorrhea, occasional low-grade fever, and congestion, followed within 1 to 3 daysby wheezing, worsening cough, and dyspnea. Focal findings are not typical, but coarsecrackles, wheezing, and transmitted upper airway sounds are prominent. Tachypnea may bemarked and may interfere with feeding, and hypoxia is common. Radiographic findings includehyperinflation (Item C132A) and patchy atelectasis. More than 50% of cases of bronchiolitis arecaused by respiratory syncytial virus; the remainder are caused by adenovirus, parainfluenza,influenza, and other pathogens.

Infants born to mothers infected with Chlamydia trachomatis may present with tachypneaand a persistent, “staccato” cough at 1 to 3 weeks of age. These infants are typically afebrile,crackles are present on physical examination, and wheezing is rare. Radiographic findingsinclude hyperinflation and interstitial infiltrates. Children who have bacterial pneumonia caused byHaemophilus influenzae or Streptococcus pneumoniae usually present with fever, dyspnea, andfocal findings on chest examination, such as decreased breath sounds and rales. Chestradiography reveals a focal infiltrate with normal inflation.

References:

Goodman D. Inflammatory disorders of the small airway. In: Behrman RE, Kliegman RM, JensonHB, eds: Nelson Textbook of Pediatrics. 17th ed. Philadelphia, Pa: WB Saunders Co; 2004:1414-1417

McCarthy CA, Hall CB. Respiratory syncytial virus: concerns and control. Pediatr Rev.2003;24:301-309. Available at: http://pedsinreview.aappublications.org/cgi/content/full/24/9/301

Copyright © 2007 by the American Academy of Pediatrics page 275

Page 276: AAP MCQ 2007

2007 PREP SA on CD-ROM

Question: 133

A 2-week-old boy whose birthweight was 5 kg and whom you saw in the nursery presents forevaluation. He is exclusively breastfed. The father is concerned that the child's penis is toosmall. The testes are descended bilaterally. The stretched penile length is 3.8 cm.

Of the following, the MOST appropriate step in the evaluation of the child is to

A. obtain thyroid studies

B. reassure the father that the penile length is normal

C. obtain a genetics consultation

D. obtain a karyotype

E. obtain a pediatric urology consultation

Copyright © 2007 by the American Academy of Pediatrics page 276

Page 277: AAP MCQ 2007

2007 PREP SA on CD-ROM

Preferred Response: BCritique: 133

Assessment of penile length in a child who has a prominent pubic fat pad requires gentlestretching of the penis to its full length and measuring from the symphysis pubis. For an infantborn at term, the mean penile length during the first postnatal 5 months is 3.9 cm. Accordingly,the father in the vignette can be reassured that his son’s penile length is normal; no tests orconsultations are necessary.

If the penile length is less than 2.5 cm (less than 2 standard deviations from the mean),endocrine evaluation for hypothalamic hypopituitarism, androgen insensitivity, or chromosomaldisorder is appropriate.

For older boys who are overweight, the penis may “disappear” into the pubic fat pad andmust be measured in the manner described previously. Tables for penile length should beconsulted for age-appropriate norms.

References:

Jones KL. Smith’s Recognizable Patterns of Human Malformations. 6th ed. Philadelphia, Pa:Elsevier Saunders; 2006:863

Weiner DE. Micropenis. Pediatr Rev. 1994;15:288

Copyright © 2007 by the American Academy of Pediatrics page 277

Page 278: AAP MCQ 2007

2007 PREP SA on CD-ROM

Question: 134

A father brings in his 6-year-old boy who has chronic constipation because the child now hasdeveloped enuresis during both night and day. On physical examination, you discern a small,firm sacral dimple just to the right of the gluteal cleft. The remainder of the physical examinationfindings are normal.

Of the following, the MOST appropriate next step in the evaluation of this child is

A. magnetic resonance imaging of the spine

B. radiographs of the abdomen and pelvis

C. ultrasonography of the kidneys, ureters, and bladder

D. urine culture

E. voiding cystourethrography

Copyright © 2007 by the American Academy of Pediatrics page 278

Page 279: AAP MCQ 2007

2007 PREP SA on CD-ROM

Preferred Response: ACritique: 134

Occult spinal dysraphism (spina bifida occulta) can be insidious and nonspecific upon initialpresentation. The pathology can include syringomyelia, diastematomyelia, a spinal cord tetheredto the inferior thecal sac, sacral agenesis, or dermoid sinus. Dermoid sinuses pass from theskin through the dura, acting as conduits for the entry of bacteria into the nervous system.

Clinical manifestations of occult spinal dysraphism can be subtle. In some instances, alipoma, patch of hair, hemangioma, or discoloration of skin lies over the lumbosacral spine. Asacral dimple superior to the gluteal cleft or lateral to the cleft with displacement of the glutealfold, as described for the boy in the vignette, should raise suspicion for dysraphism. Childrenlater may develop high-arched feet, discrepancy in strength between lower extremities, or a gaitabnormality. Back pain or absent perineal sensation, urinary incontinence, lower urinary tractinfection, or fecal soiling also may occur.

Children younger than age 6 months, whose vertebral bones have not ossified fully, can beevaluated for spina bifida by ultrasonography. Older children require initial evaluation withmagnetic resonance imaging of the spine. Radiographs might be helpful to evaluateaccompanying constipation, but will not evaluate dysraphism adequately. Urinary tractabnormalities should be managed after the dysraphism is identified, and the managementapproach is similar to that in other children who have myelodysplasia. Ultrasonography of thekidneys, ureter, and bladder might be helpful, as can urine culture or voidingcystourethrography, in evaluation of the urinary tract.

References:

Drolet BA. Cutaneous signs of neural tube dysraphism. Pediatr Clin North Am. 2000;47:813-823.Abstract available at:http://www.ncbi.nlm.nih.gov/entrez/query.fcgi?cmd=Retrieve&db=pubmed&dopt=Abstract&list_uids=10943258&query_hl=16&itool=pubmed_docsum

Johnston MV, Kinsman S. Congenital anomalies of the central nervous system: neural tubedefects (dysraphism). In: Behrman RE, Kliegman RM, Jenson HB, eds. Nelson Textbook ofPediatrics. 17th ed. Philadelphia, Pa: WB Saunders Co; 2004:1983

Copyright © 2007 by the American Academy of Pediatrics page 279

Page 280: AAP MCQ 2007

2007 PREP SA on CD-ROM

Question: 135

A 15-year-old previously healthy girl had an episode of syncope in gym class. She awoke after 1minute and was taken to the emergency department where physical examination findings werenormal. Results of electrocardiography are shown in Item Q135A.

Of the following, the BEST assessment of the electrocardiographic result is

A. atrioventricular block

B. Brugada syndrome

C. prolonged QT syndrome

D. sinus rhythm with premature contractions

E. Wolff-Parkinson-White syndrome

Copyright © 2007 by the American Academy of Pediatrics page 280

Page 281: AAP MCQ 2007

2007 PREP SA on CD-ROM

Preferred Response: CCritique: 135

Syncope or presyncope during exercise is abnormal and should prompt a thoroughcardiovascular evaluation because such symptoms could be caused by disorders that havebeen associated with sudden cardiac death in young athletes. Among these conditions arehypertrophic cardiomyopathy (HCM), anomalies of the coronary arteries, ruptured aorticaneurysm (as can be seen in Marfan syndrome), and arrhythmias (eg, long QT syndrome). It iscritical to obtain a detailed history, including family history for sudden death, arrhythmia, anddeafness, because several of these entities (HCM, Marfan syndrome, and long QT syndrome)can have autosomal dominant inheritance patterns. Familial syndromes such as the Jervell andLange-Nielsen syndrome, an autosomal recessive condition of prolonged QT syndromeassociated with deafness, may become apparent with a detailed family history.

As part of the initial evaluation of any patient who has syncope during exercise, a complete(at least 12 leads) electrocardiogram (ECG) should be obtained. The timing of the voltageintervals is particularly important in terms of looking for evidence of heart block, QRSabnormalities, prolonged QT interval corrected for heart rate, and sinus node dysfunction. TheECG also can be used to evaluate for pre-excitation syndromes such as the Wolff-Parkinson-White syndrome and to diagnose such entities as the Brugada syndrome.

Because the patient in the vignette has had syncope with exercise, she should be removedfrom all gym classes and participation in sports until her evaluation is complete. Her ECG tracingshows clear evidence of a prolonged QT interval, even when the interval is corrected for theheart rate (dividing the QT interval by the square root of the R-R interval) (Item C135A).Abnormal corrected QT intervals (QTc) are greater than 450 msec. There is no evidence ofatrioventricular block (Item C135B) in the patient because the PR interval is normal and there isa direct relationship between every P wave and every QRS complex. Brugada syndrome (ItemC135C), often a familial syndrome, is the combination of a right bundle branch block pattern andST segment elevation on ECG, with the potential for sudden death. Neither of these ECGfindings is seen on this tracing. Wolff-Parkinson-White syndrome requires ventricular pre-excitation (early depolarization of the QRS complex) (Item C135D), which is not present on theECG tracing reported for the patient. Although the ECG tracing demonstrates a sinus rhythm,there is no evidence of premature contractions.

References:

Berger S, Utech L, Fran Hazinski M. Sudden death in children and adolescents. Pediatr ClinNorth Am. 2004;51:1653-1677. Abstract available at:http://www.ncbi.nlm.nih.gov/entrez/query.fcgi?cmd=Retrieve&db=pubmed&dopt=Abstract&list_uids=15561179&query_hl=2&itool=pubmed_docsum

Chiang CE. Congenital and acquired long QT syndrome. Current concepts and management.Cardiol Rev. 2004;12:222-234. Available at:http://www.cardiologyinreview.com/pt/re/cardiorev/abstract.00045415-200407000-00007.htm;jsessionid=EGmnHdsk2bBzGB8lmmKJ2CksqGuCo0Oai31wLs7zmc3bNCcedP3e!-469394890!-949856145!9001!-1

Maron BJ. Sudden cardiac death in the young athlete and the preparticipation cardiovascularevaluation. In: Moller JH, Hoffman JIE, eds. Pediatric Cardiovascular Medicine. Philadelphia Pa:Churchill Livingstone; 2000:891-900

Rhodes LA. Syncope. In Moller JH, Hoffman JIE, eds. Pediatric Cardiovascular Medicine.Philadelphia, Pa: Churchill Livingstone; 2000:885-890

Copyright © 2007 by the American Academy of Pediatrics page 281

Page 282: AAP MCQ 2007

2007 PREP SA on CD-ROM

Question: 136

A newborn has microcephaly, dysmorphic features (including a prominent forehead, protuberantears, and micrognathia), bilateral hip dislocation, clinodactyly, and a single transverse palmarcrease bilaterally. Peripheral blood chromosome analysis reveals an unbalanced translocationthat results in partial trisomy 9.

Of the following, the MOST appropriate statement to include when discussing the diagnosis withthe parents of this infant is that

A. chromosome analysis should be obtained on both parents to determine if they are balancedtranslocation carriers

B. chromosome analysis of their other children is unnecessary

C. mental development should be normal because their infant does not have a full trisomy

D. their risk for a similarly affected child in future pregnancies is very small

E. they are at increased risk for other trisomies (eg, Down syndrome) in future pregnancies

Copyright © 2007 by the American Academy of Pediatrics page 282

Page 283: AAP MCQ 2007

2007 PREP SA on CD-ROM

Preferred Response: ACritique: 136

The finding of an unbalanced translocation in an infant always should prompt examination of theparental blood karyotypes to determine if the translocation is familial. Translocations usuallyinvolve the exchange of genetic material between two chromosomes. Robertsoniantranslocations result from fusion of the centromere of two acrocentric chromosomes(chromosomes 13, 14, 15, 21, and 22); reciprocal translocations result from the breakage of twochromosomes away from the centromere, with exchange of the chromosomal material distal tothe breakpoints. During meiosis, distribution of the translocated chromosomes can give rise tonormal gametes, gametes that carry the balanced rearrangement, or gametes that have aduplication or deficiency of some chromosomal material. Unless the translocation has disrupteda gene, carriers of balanced translocations usually are phenotypically normal. However, carriersare at risk for having abnormal offspring if unbalanced gametes are formed. Thus, it is importantto study the chromosomes of the parents of any infant who has a translocation to assess theirrisk for recurrence in future pregnancies. Some couples in whom one member carries abalanced translocation may have a history of multiple miscarriages, which is explained by theloss of embryos/fetuses that have unbalanced translocations incompatible with survival.

Trisomy for the short arm of chromosome 9 is the most frequent partial trisomy and resultsin the phenotypic features noted in the infant described in the vignette. Intellectual impairmentalways is present, although to varying degrees. The phenotypically normal child of a parent whocarries a balanced translocation would be expected to have either a normal karyotype or to be abalanced translocation carrier. Such children should be offered chromosomal analysis andgenetic counseling by the time they are of reproductive age. Balanced translocation carriers areprimarily at risk for having children who have an unbalanced chromosome complement, nottrisomies.

References:

Jones KL. Trisomy 9 mosaic syndrome. In: Smith's Recognizable Patterns of HumanMalformation. 6th ed. Philadelphia, Pa: Elsevier Saunders Co; 2006:26-27

Pinsky L. Overview of genetic assessment. UpToDate. 2006; 14.1. Available at:http://www.utdol.com/utd/content/topic.do?topicKey=genr_med/27636&type=A&selectedTitle=1~85

Copyright © 2007 by the American Academy of Pediatrics page 283

Page 284: AAP MCQ 2007

2007 PREP SA on CD-ROM

Question: 137

A 17-year-old girl presents with amenorrhea of 6 months' duration. One year ago she joined thecross country team at school. At that time, her periods had been regular, about every 26 days,and remained so for the 3-month running season. At the end of the season, she continued to run5 miles a day to be more competitive for the subsequent season. She noticed that her menstrualflow was lighter for a few months preceding the amenorrhea. You suspect exercise-inducedamenorrhea and recommend a decrease in exercise.

Of the following, the factor MOST likely to be associated with a low bone density and stressfractures in this patient is

A. cigarette smoking

B. early onset of puberty

C. high body mass index

D. use of antidepressant medication

E. use of oral contraceptives

Copyright © 2007 by the American Academy of Pediatrics page 284

Page 285: AAP MCQ 2007

2007 PREP SA on CD-ROM

Preferred Response: ACritique: 137

Polycystic ovary syndrome (PCOS) and exercise-induced amenorrhea are two commoncauses of secondary amenorrhea during the adolescent years. The patient described in thevignette is a runner and, like other athletes who have intensive training schedules, is at risk forexercise-induced amenorrhea. Such serious athletes also have higher rates of disorderedeating. The female athlete triad includes disordered eating, amenorrhea, and decreased bonedensity. Included in the group of athletes who have secondary amenorrhea are patients whohave anorexia nervosa. A history and physical examination can reveal possible causes of theamenorrhea and guide the choice of laboratory tests, which should include a pregnancy test.Low levels of estradiol (E2) have been implicated in the loss of bone mineralization andincreased risk of stress fractures found in patients who have exercise-induced amenorrhea. (Anegative result to a progesterone withdrawal test is expected with low E2 concentrations.) Otherfactors associated with low bone density in female athletes are low weight, low percentage ofbody fat, delayed puberty, longer duration of amenorrhea, low calcium and protein intake, and afamily history of osteoporosis. Adolescents who have exercise-induced amenorrhea should beadvised to reduce the intensity of their training and increase their caloric intake. Resumption ofmenses is necessary to prevent further bone demineralization. The benefits of hormonalreplacement have not been established. Patients also are advised to take 1,500 mg/d of calciumand encouraged to abstain from smoking tobacco because it increases the risk for stressfractures.

Although obesity frequently is associated with PCOS, it is not always present. The diagnosisshould be suspected in any adolescent female who has amenorrhea and clinical features ofhyperandrogenism (eg, hirsutism, acne).

Delayed puberty, rather than early puberty, is associated with a low bone density. A lowpercentage of body fat, reflected in a low body mass index, also is associated with a low bonedensity. The use of antidepressants has not been associated with a low bone density orincreased risk of stress fractures. There is some evidence that stress fractures occur lessfrequently in athletes who have used oral contraceptives.

References:

Emans SJ. Amenorrhea in the adolescent. In: Emans SJH, Laufer MR, Goldstein DP, eds.Pediatric and Adolescent Gynecology. 5th ed. Philadelphia, Pa: Lippincott, Williams & Wilkins;2005:214-269

Hergenroeder AC, Chorley JN. Female athletes: menstrual problems and the risk forosteopenia. In: Behrman RE, Kliegman RM, Jenson HB, eds. Nelson Textbook of Pediatrics.17th ed. Philadelphia, Pa: Saunders; 2004:2315-2316

Copyright © 2007 by the American Academy of Pediatrics page 285

Page 286: AAP MCQ 2007

2007 PREP SA on CD-ROM

Question: 138

You are asked to see a 14-year-old girl who developed pubic hair at age 11 years and breastbuds at age 12 years, but has not reached menarche. She is a gymnast who practices 2 hoursa day. Breast tissue is Sexual Maturity Rating (SMR) 2 and pubic hair is SMR 4. She is 57 in talland weighs 86 lb. The results of gonadotropin laboratory studies are a luteinizing hormoneconcentration of 18 mIU/mL (18 IU/L) (normal adult female, 2 to 70 mIU/mL [2 to 70 IU/L]) and afollicle-stimulating hormone concentration of 40 mIU/mL (40 IU/L) (normal adult female, 1 to 30mIU/mL [1 to 30 IU/L]).

Of the following, the MOST likely cause of the primary amenorrhea in this patient is

A. autoimmune ovarian failure

B. excessive exercise

C. imperforate hymen

D. prolactinoma

E. Turner syndrome

Copyright © 2007 by the American Academy of Pediatrics page 286

Page 287: AAP MCQ 2007

2007 PREP SA on CD-ROM

Preferred Response: ECritique: 138

Most girls who have Turner syndrome are identified at a relatively young age because of shortstature or growth attenuation. Thelarche (breast development) does not occur in most affectedgirls because streak ovaries present at birth never develop follicles. However, a substantialminority of girls who have X chromosome abnormalities at least begin puberty before developingcomplete ovarian failure. Adrenarche (adrenal puberty) usually is not delayed in girls who haveTurner syndrome, so they have normal development of pubic and axillary hair. Any girl who hashad a failure or delay of pubertal maturation (failure of menarche within 3 years of developmentof initial age-appropriate thelarche) should have gonadotropins (luteinizing hormone [LH] andfollicle-stimulating hormone [FSH]) measured. An LH elevation can be seen during the midcyclesurge of this hormone, but elevations of FSH are pathognomonic for ovarian failure. Girls whohave ovarian failure and elevated FSH values are said to have hypergonadotropichypogonadism, and the two most common causes of this finding are Turner syndrome andautoimmune ovarian failure. Such findings in a girl who is short and has had pubertal arrest,such as the girl described in the vignette, strongly suggest Turner syndrome. Autoimmuneovarian failure usually is not associated with short stature. Chromosome studies are mandatedin any girl who has ovarian failure.

Excessive exercise can cause pubertal delay, but this is associated with low levels of LHand FSH because the excessive exercise leads to hypogonadotropic hypogonadism.Hyperprolactinemia, which can be caused by prolactinoma, also suppresses puberty, butovaries are intact, and affected girls also have low concentrations of LH and FSH. Girls whohave imperforate hymens complete puberty but do not have normal menses. Levels of LH andFSH are normal.

References:

Misra M, Park-Bennett S. Disorders of puberty. In: Burg FD, Ingelfinger JR, Polin RA, GershonAA, eds. Gellis & Kagan’s Current Pediatric Therapy. Philadelphia, Pa: WB Saunders; 2002:706-710

Saenger P. Clinical manifestations and diagnosis of Turner syndrome (gonadal dysgenesis).UptoDate. 2006;14;1. Available at:http://uptodateonline.com/utd/content/topic.do?file=r_endo_f/13147&type=A&selectedTitle=1~36

Sedlmeyer IL, Palmert MR. Delayed puberty: analysis of a large case series from an academiccenter. J Clin Endocrinol Metab. 2002;87:1613-1620. Available at:http://jcem.endojournals.org/cgi/content/full/87/4/1613

Sybert VP, McCauley E. Turner's syndrome. N Engl J Med. 2004;351:1227-1238

Copyright © 2007 by the American Academy of Pediatrics page 287

Page 288: AAP MCQ 2007

2007 PREP SA on CD-ROM

Question: 139

You diagnose attention-deficit/hyperactivity disorder in an 8-year-old girl and initiate therapy witha daily morning dose of long-acting methylphenidate. Her mother asks about using stimulantmedication after school, on weekends, and during the summer.

Of the following, your BEST response is that

A. an after-school dose is not necessary with a long-acting form of methylphenidate

B. dosing outside school hours allows parents to monitor medication effect

C. immediate-release medication should be used during holidays

D. summer dosing increases the risk of tolerance to medication

E. weekend dosing should be decreased by 50%

Copyright © 2007 by the American Academy of Pediatrics page 288

Page 289: AAP MCQ 2007

2007 PREP SA on CD-ROM

Preferred Response: BCritique: 139

Children who have attention-deficit/hyperactivity disorder (ADHD) commonly have difficultyattending to and completing tasks both at home and school. Families may resist givingmedication outside of school hours because they assume that the same degree of attentionneeded in a school setting is not necessary in the child’s other settings. However, best resultsfrom stimulant medication therapy have been noted when children receive medication in all oftheir typical settings. Additionally, parents will be more aware of medication adverse effects andadequacy of dosing if they observe the child when he or she is taking the medication.

An after-school dose of medication may be necessary for the child to perform successfullyin after-school and home activities if the medication effects have worn off at the end of theschool day. There is no need to change the type or the dosing of medication used duringweekends and holidays. Continuous dosing throughout the year does not make a child moretolerant to stimulant medications, and drug holidays are not necessary.

References:

American Academy of Pediatrics Subcommittee on Attention-Deficit/Hyperactivity Disorder andCommittee on Quality Improvement. Clinical practice guideline: treatment of the school-agedchild with attention-deficit/hyperactivity disorder. Pediatrics. 2001;108:1033-1044. Available at:http://pediatrics.aappublications.org/cgi/content/full/108/4/1033

Reiff MI, Tippins S, LeTourneau AA. ADHD: A Complete and Authoritative Guide. Elk GroveVillage, Ill: American Academy of Pediatrics; 2004

Wender EH. Managing stimulant medication for attention-deficit/hyperactivity disorder. PediatrRev. 2001;22:183-190. Available at:http://pedsinreview.aappublications.org/cgi/content/full/22/6/183

Wender EH. Managing stimulant medication for attention-deficit/hyperactivity disorder: anupdate. Pediatr Rev. 2002;23:234-236. Available at:http://pedsinreview.aappublications.org/cgi/content/full/23/7/234

Copyright © 2007 by the American Academy of Pediatrics page 289

Page 290: AAP MCQ 2007

2007 PREP SA on CD-ROM

Question: 140

A 4-year-old boy presents to your clinic with a 4-day history of fever. Over the past 24 hours, hedeveloped swelling over the left side of his face, and his left eye is starting to close. On physicalexamination, his temperature is 102°F (38.9°C), and his left cheek and lower eyelid are swollen(Item Q140A) but not red or warm. On oral examination, you note a severe decay of the secondmaxillary molar and elicit pain when you tap on this tooth.

Of the following, the MOST appropriate therapy is

A. azithromycin

B. cefdinir

C. cephalexin

D. penicillin

E. trimethoprim-sulfamethoxazole

Copyright © 2007 by the American Academy of Pediatrics page 290

Page 291: AAP MCQ 2007

2007 PREP SA on CD-ROM

Preferred Response: DCritique: 140

Antimicrobial therapy is indicated for patients who have dental infections if there is associatedcellulitis or facial swelling, as described for the patient in the vignette. Many dentists want to tryto treat the infection with antimicrobial agents prior to repairing or removing the involved tooth.Penicillin remains the antibiotic of choice for dental infections. If the patient is allergic to penicillin,clindamycin or erythromycin is an acceptable alternative. Azithromycin, cefdinir, cephalexin, andtrimethoprim-sulfamethoxazole are not indicated for dental infections.

Dental caries caused by Streptococcus mutans and other streptococci destroy most of thetooth and invade the dental pulp. Pulpitis (inflammation of the pulp) can lead to extension of theinfection beyond the tooth to the periapical tissues of the periodontal ligament, possibly resultingin an abscess. Depending on the location of the tooth involved, the patient may develop swellingon the face or below the jaw. Fever, facial swelling, and tooth pain that is exaggerated when thetooth is touched usually indicate an abscess. Abscesses can have a polymicrobial etiology,including anaerobes (eg, Actinobacillus actinomycetemcomitans), especially when periodontaldisease also is present. Careful attention to dental hygiene, especially in high-risk patients (eg,those who have complex congenital heart disease) is important to prevent systemic illness.

References:

Tinanoff N. Dental caries. In: Behrman RE, Kliegman RM, Jenson HB, eds. Nelson Textbook ofPediatrics. 17th ed. Philadelphia, Pa: WB Saunders Co; 2004:1209-1212

Wayne DB, Trajtenberg CP, Hyman DJ. Tooth and periodontal disease: a review for the primary-care physician. South Med J. 2001;94:925-932. Abstract available at:http://www.ncbi.nlm.nih.gov/entrez/query.fcgi?cmd=Retrieve&db=pubmed&dopt=Abstract&list_uids=11592756&query_hl=7&itool=pubmed_docsum

Copyright © 2007 by the American Academy of Pediatrics page 291

Page 292: AAP MCQ 2007

2007 PREP SA on CD-ROM

Question: 141

You are seeing a newborn boy for the first time. Several prenatal ultrasonographic examinationsrevealed bilateral hydronephrosis. The boy's weight and height are appropriate for gestation, andhis physical examination findings are unremarkable. Postnatal ultrasonography reveals severebilateral hydronephrosis.

Of the following, the MOST likely cause of the hydronephrosis is

A. polycystic kidney disease

B. posterior urethral valves

C. ureteropelvic junction obstruction

D. vesicoureteral reflux

E. Wilms tumor

Copyright © 2007 by the American Academy of Pediatrics page 292

Page 293: AAP MCQ 2007

2007 PREP SA on CD-ROM

Preferred Response: BCritique: 141

Urinary tract obstruction (UTO) may indicate an abnormality in the kidney(s), ureter(s), bladder,or urethra. A logical and stepwise approach to determine the cause usually results in reachingan accurate diagnosis.

UTO generally is found in utero or early in infancy. The advent of routine fetalultrasonography has dramatically increased the detection of UTO prior to birth. In most othercases of UTO, a urinary tract infection in infancy stimulates evaluation of the system. Thepractitioner’s goal is to ascertain the site of obstruction.

Renal ultrasonography is the primary imaging technique used to determine if UTO ispresent, and if so, the degree of obstruction. In general, hydronephrosis is described as mild,moderate, or severe. Most studies show that renal function is better preserved in mild comparedwith moderate-to-severe hydronephrosis. Another invaluable piece of information is whether thehydronephrosis is unilateral or bilateral. Obviously, bilateral hydronephrosis increases thelikelihood of developing renal insufficiency, but it also may provide an indication of the origin ofobstruction. Generally, the origin of obstruction is lower with bilateral versus unilateralhydronephrosis. It is essential to determine the cause of any obstruction; if left untreated, it mayresult in recurrent urinary tract infections and renal scarring and damage.

The bilateral hydronephrosis reported for the boy in the vignette strongly suggests thepresence of UTO, although the hydronephrosis may be nonobstructive. Voidingcystourethrography is necessary to determine if there is vesicoureteral reflux (VUR) (ItemC141A), a ureteral abnormality such as a duplicated ureter, or posterior urethral valves (PUV)(Item C141B). PUV must be considered a likely cause of bilateral hydronephrosis in a male. Themost common presentation of autosomal recessive or dominant polycystic kidney disease (ItemC141C) is enlarged kidneys in the newborn period, with or without small cysts; these kidneys donot display hydronephrosis. Although ureteropelvic junction obstruction (UPJO) and VURcommonly cause hydronephrosis, the sex of the child and bilateral hydronephrosis makes PUVmore common. However, it should be emphasized that children who have PUV may haveconcomitant VUR or UPJO. Wilms tumor (Item C141D), the most common abdominal tumor ininfants, presents most commonly as a solid mass, occasionally with concomitanthydronephrosis.

References:

Aksu N, Yavascan O, Kangin M, et al. Postnatal management of infants with antenatally detectedhydronephrosis. Pediatr Nephrol. 2005;20:1253-1259. Abstract available at:http://www.ncbi.nlm.nih.gov/entrez/query.fcgi?orig_db=PubMed&db=PubMed&cmd=Search&term=%22Pediatric+nephrology+(Berlin,+Germany)%22[Jour]+AND+1253[page]+AND+2005[pdat]

Chandrasekharam VV, Shah MA. Outcome of patients with antenatally detected pelviuretericjunction obstruction. Pediatr Nephrol. 2005;20:547

Onen A, Jayanthi VR, Koff SA. Long-term followup of prenatally detected severe bilateralnewborn hydronephrosis initially managed nonoperatively. J Urol. 2002;168:1118-1120. Abstractavailable at:http://www.ncbi.nlm.nih.gov/entrez/query.fcgi?orig_db=PubMed&db=PubMed&cmd=Search&term=%22The+Journal+of+urology%22[Jour]+AND+1118[page]+AND+2002[pdat]

Copyright © 2007 by the American Academy of Pediatrics page 293

Page 294: AAP MCQ 2007

2007 PREP SA on CD-ROM

Question: 142

Over the past week, you and your colleagues have seen a number of previously healthychildren younger than 2 years of age who present with fever, coryza, cough, vomiting, anddiarrhea. Several of the children have required hospitalization for respiratory distress andpneumonia. You suspect that influenza virus infection is the cause of these illnesses.

Of the following, the MOST rapid test to confirm the diagnosis in these patients is

A. enzyme immunoassay antigen detection

B. immunoglobulin M (IgM) titers on acute sera

C. IgM titers on acute and convalescent sera

D. polymerase chain reaction

E. viral culture

Copyright © 2007 by the American Academy of Pediatrics page 294

Page 295: AAP MCQ 2007

2007 PREP SA on CD-ROM

Preferred Response: ACritique: 142

Influenza is a common respiratory disease that is caused by the influenza A and B viruses andis associated with annual epidemics. It is spread person-to-person by respiratory droplets viacoughing or sneezing or through direct contact with infected nasopharyngeal secretions. Theclinical manifestations of the disease differ, depending on the age of the person who is infected.In young infants, the presentation can range from fever and a crouplike cough to a pictureresembling sepsis. In children, the most common presenting signs and symptoms are fever,malaise, cough, rhinitis, pharyngitis, vomiting, and diarrhea, although in some children, thedisease manifests as a simple upper respiratory tract infection or a febrile illness with few othersymptoms. In adolescents, the illness has a similar presentation to that seen in adults and ischaracterized by the sudden onset of high fever, malaise, myalgia, headache, chills, rigors,pharyngitis, and a nonproductive cough. In uncomplicated cases, symptoms usually resolve in 3to 7 days, although cough and malaise may persist for 2 weeks or more.

The treatment of influenza is supportive, but specific antiviral therapy should be consideredin high-risk patients or those who have severe disease (Item C142A). Amantadine andrimantadine are in the M2 class of antiviral drugs and work by inhibiting viral replication. Theseagents are only licensed for the treatment of influenza A disease. Oseltamivir and zanamivir areneuraminidase inhibitors that are effective in the treatment of both influenza A and B infection.Amantadine and oseltamivir may be used in children 1 year of age and older; rimantadine islicensed only for use in children 13 years of age or older. Zanamivir is approved for use inchildren who are 7 years of age and older and is administered by inhalation. A 2-week course ofchemoprophylaxis should be considered in patients in high-risk groups who have been exposedto influenza. Amantadine and rimantadine are licensed for prophylaxis in children 1 year of ageand older, oseltamivir can be used for prophylaxis in children 13 years of age and older, andzanamivir is not currently licensed for prophylaxis.

The accurate and early diagnosis of influenza in patients who have respiratory illnesses canreduce inappropriate use of antibiotics and provide the option of using antiviral therapy. Anumber of diagnostic tests are available, but the most rapid is enzyme immunoassay antigendetection. Paired acute and convalescent serology can aid in the diagnosis of a recent influenzainfection, although results are not available for 2 or more weeks. Similarly, polymerase chainreaction and viral culture do not provide rapid results. A single acute serum specimen is notvaluable in diagnosis.

Annual immunization against influenza is the best approach to prevention. Two types ofinfluenza vaccines may be used, both of which contain the same three influenza strains (twoinfluenza A strains and one influenza B strain) that are believed to be circulating in the UnitedStates in a given year. Vaccination is strongly recommended for all healthy children 6 to 23months of age, all children and adults who have chronic underlying diseases (eg, asthma,chronic cardiac and pulmonary conditions, immunocompromise), persons living in chronic carefacilities or nursing homes, women who are pregnant (>14 weeks’ gestation), householdmembers of high-risk patients, and health-care workers. The inactivated intramuscular influenzavaccine may be used in children 6 months of age and older. Children younger than 9 years ofage should receive two doses 1 month apart for their initial vaccination, then a single dose eachsubsequent fall. The live, attenuated, intranasal vaccine is licensed only for use in healthyindividuals ages 5 to 49 years. Children 5 to 9 years of age should receive two doses 6 weeksapart for their initial vaccination, then a single dose each year subsequently. Efficacy in thepediatric population is similar for both vaccines, ranging from 70% to 90%.

References:

Abramson JS, Katz SL, Offit PA, McMillan JA. Reducing the risk of pediatric influenza:prevention strategies help both the young and the old. Contemp Pediatr. 2005;22(suppl 1):1-8

Advanced Studies in Medicine. Prevention of influenza virus infection among children. ASIMCME2002;2(8):286-323. Available at:http://www.jhasim.com/template.cfm?TEMPLATE=include_program.cfm&ID=38&ZoneID=8&pag

Copyright © 2007 by the American Academy of Pediatrics page 295

Page 296: AAP MCQ 2007

2007 PREP SA on CD-ROM

eName=Volume%202,%20(8)

American Academy of Pediatrics. Influenza. In: Pickering LK, ed. Red Book: 2006 Report of theCommittee on Infectious Diseases. 27th ed. Elk Grove Village, Ill: American Academy ofPediatrics; 2006:401-411

Harper SA, Fukuda K, Uyeki TM, Cox NJ, Bridges CB. Prevention and control of influenza.Recommendations of the Advisory Committee on Immunization Practices (ACIP). MMWRRecomm Rep. 2005;54:1-40. Available at:www.cdc.gov/mmwr/preview/mmwrhtml/rr54e713a1.htm

Copyright © 2007 by the American Academy of Pediatrics page 296

Page 297: AAP MCQ 2007

2007 PREP SA on CD-ROM

Question: 143

A 4-year-old boy presents with a history of chronic upper and lower respiratory tract infections.His weight is 15 kg (25th percentile), height is 97 cm (10th percentile), temperature is 98.1°F(36.8°C), and pulse oximetry is 96% on room air. On physical examination, he coughsintermittently and has mild clubbing. On nasal examination, you note purulent rhinorrhea andnasal polyps (Item Q143A). Auscultation of the heart reveals a regular rate and rhythm, with thepoint of maximal impulse displaced to the right.

Of the following, the MOST likely diagnosis is

A. cystic fibrosis

B. human immunodeficiency virus infection

C. primary ciliary dyskinesia (Kartagener syndrome)

D. severe combined immunodeficiency

E. X-linked (Bruton) agammaglobulinemia

Copyright © 2007 by the American Academy of Pediatrics page 297

Page 298: AAP MCQ 2007

2007 PREP SA on CD-ROM

Preferred Response: CCritique: 143

The chronic sinopulmonary infections, nasal polyps, and right-sided heart (dextrocardia)described for the boy in the vignette are virtually diagnostic for primary ciliary dyskinesia (PCD).Approximately 50% of patients who have PCD have situs inversus, a reversal of the visceralorgans (Kartagener syndrome). The diagnosis can be confirmed by identification of abnormalcilia orientation or missing dynein arms by electron microscopy.

Cystic fibrosis can present similarly to PCD, with chronic sinopulmonary infections,bronchiectasis, nasal polyps, and clubbing, but neither dextrocardia nor situs inversus is atypical finding.

Primary immunodeficiencies such as X-linked agammaglobulinemia and severe combinedimmunodeficiency present with recurrent bacterial and viral infections. Nasal polyps may resultfrom chronic sinusitis, although digital clubbing and dextrocardia are not observed.

Clinical manifestations of human immunodeficiency virus infection vary widely. Initialsymptoms can include lymphadenopathy, hepatosplenomegaly, failure to thrive, diarrhea, andthrush. Recurrent otitis media, sinusitis, and lymphocytic interstitial pneumonia are common, butbronchiectasis and nasal polyps are rare.

References:

Chilvers MA, Rutman A, O’Callaghan C. Ciliary beat pattern is associated with specificultrastructural defects in primary ciliary dyskinesia. J Allergy Clin Immunol. 2003;112:518-524

Haddad GG. Primary ciliary dyskinesia (immotile cilia syndrome). In: Behrman RE, KliegmanRM, Jenson HB, eds. Nelson Textbook of Pediatrics. 17th ed. Philadelphia, Pa: WB SaundersCo; 2004:1450-1451

Welch JE, Hogan MB, Wilson NW. Ten-year experience using a plastic, disposable curette forthe diagnosis of primary ciliary dyskinesia. Ann Allergy Asthma Immunol. 2004;93:189-192

Copyright © 2007 by the American Academy of Pediatrics page 298

Page 299: AAP MCQ 2007

2007 PREP SA on CD-ROM

Question: 144

A 12-year-old child is struck in the right eye by a baseball. He now complains of double visionwhen he looks to the left. On physical examination, you note marked swelling and bruisingcircumferentially around the boy's right eye. His pupillary reflexes are normal, he has no blood inthe anterior chamber of the eye, and his optic disc margins are sharp. He has dysconjugategaze to the left but not to the right. His vision is 20/20 out of the left eye and 20/30 out of the righteye. Other findings on his neurologic examination are normal.

Of the following, the MOST likely diagnosis is

A. blow-out fracture

B. corneal abrasion

C. detached retina

D. hyphema

E. traumatic iritis

Copyright © 2007 by the American Academy of Pediatrics page 299

Page 300: AAP MCQ 2007

2007 PREP SA on CD-ROM

Preferred Response: ACritique: 144

The findings described for the boy in the vignette are indicative of an internal orbital or “blow-out”fracture, which is defined as a fracture of the orbital floor or orbital wall with an intact orbital rim.This is an uncommon injury before 8 years of age, possibly due to poorly pneumatized sinusesin very young children.

Rupture of the orbital wall often is accompanied by extrusion of orbital soft tissue, includingperiosteum, fat, or muscle. Entrapment of the inferior rectus muscle and, less commonly, themedial rectus muscle is manifested by impaired eye movement. Impaired eye movement resultsin intermittent diplopia, depending on the direction of eye movement. Surgical reduction isrequired within 4 to 10 days to prevent necrosis and subsequent fibrosis of the entrappedtissues.

A child who has a suspected blow-out fracture should be evaluated by orbital computedtomography (CT) scan; skull films are of limited value. CT scan is especially valuable because itallows for evaluation of both the bony orbit and the globe. Associated injuries include rupturedglobe, retrobulbar hemorrhage, and traumatic optic neuropathy, the first two of which can bedemonstrated on CT scan. A child in whom a blow-out fracture is confirmed should be examinedimmediately by an ophthalmologist to determine the need for surgical reduction, assess forassociated injuries, and monitor for complications.

Corneal abrasion, detached retina, hyphema, and traumatic iritis can occur in conjunctionwith a blow-out fracture. Corneal abrasion causes severe pain and tearing but does not causedysconjugate gaze or diplopia. Hyphema (Item C144A) refers to blood in the anterior chamber,which may cause visual impairment but generally not diplopia. Traumatic iritis is characterizedby pain and severe photophobia but not diplopia. Detached retina can cause a “curtainlike”deficit in the peripheral vision but does not cause dysconjugate gaze. A detached retina is anophthalmologic emergency.

References:

American College of Surgeons. Ocular trauma. In: Advanced Trauma Life Support® Program forDoctors. 6th ed. Chicago, Ill: American College of Surgeons; 1997:411-418

Cruz AA, Eichenberger GCD. Epidemiology and management of orbital fractures. Curr OpinOphthalmol. 2004;15:416-421. Abstract available at:http://www.ncbi.nlm.nih.gov/entrez/query.fcgi?orig_db=PubMed&db=PubMed&cmd=Search&term=Curr+Opin+Ophthalmol[Jour]+AND+416[page]+AND+2004[pdat]

Klenk G, Kovacs A. Blow-out fracture of the orbital floor in early childhood. J Craniofac Surg.2003;14:666-671. Abstract available at:http://www.ncbi.nlm.nih.gov/entrez/query.fcgi?orig_db=PubMed&db=PubMed&cmd=Search&term=J+Craniofac+Surg[Jour]+AND+666[page]+AND+2003[pdat]

Neuman MI, Eriksson E. Facial trauma. In: Fleisher GR, Ludwig S, Henretig FM, eds. Textbookof Pediatric Emergency Medicine. 5th ed. Philadelphia, Pa: Lippincott Williams & Wilkins;2006:1475-1484

Copyright © 2007 by the American Academy of Pediatrics page 300

Page 301: AAP MCQ 2007

2007 PREP SA on CD-ROM

Question: 145

A 14-month-old infant who is breastfed with bottle supplementation presents with early evidenceof dental caries (Item Q145A).

Of the following, the MOST appropriate practice to disrupt this process is to

A. discontinue bottle feeding

B. discontinue bottle feeding except for juices

C. encourage brushing teeth with fluoride-containing toothpaste

D. switch from formula to whole milk for bottle feeding

E. use bottled water to prepare formula

Copyright © 2007 by the American Academy of Pediatrics page 301

Page 302: AAP MCQ 2007

2007 PREP SA on CD-ROM

Preferred Response: ACritique: 145

Nursing infants with erupted dentition and toddlers who are allowed to fall asleep while taking abottle containing high sugar content liquid such as juices and formula are at risk for thedevelopment of nursing bottle caries (Item C145A). Recently, prolonged nocturnal breastfeedingalso has been recognized as a contributing factor. Characteristically, the sequence of cariesdevelopment progresses from maxillary incisors to maxillary molars and subsequentlymandibular dentition. Brushing before bedtime and elimination of feeding in bed should greatlydecrease the appearance of nursing caries.

Bottle feeding should be discontinued for the 14-month-old infant described in the vignette.Continuing juices by bottle would compound the problem. Preparing formula with bottled waterwould have no impact on the disease process. Changing from formula to whole milk isappropriate for the infant, but nocturnal bottle feeding of whole milk would continue to place thetoddler at risk for caries. Brushing teeth is a good practice, but discontinuation of bottle feeding ismost important.

References:

American Academy of Pediatrics. Policy statement: oral health risk assessment timing andestablishment of the dental home. Pediatrics. 2003;111:1113-1116. Available at:http://pediatrics.aappublications.org/cgi/content/full/111/5/1113

Tinanoff N. Dental caries. In: Behrman RE, Kliegman RM, Jenson HB, eds. Nelson Textbook ofPediatrics. 17th ed. Philadelphia, Pa: WB Saunders Co; 2004:1209-1211

Zitelli BJ, Davis HW. Caries. In: Atlas of Pediatric Physical Diagnosis. 3rd ed. St. Louis, Mo:Mosby-Wolfe; 1997:613

Copyright © 2007 by the American Academy of Pediatrics page 302

Page 303: AAP MCQ 2007

2007 PREP SA on CD-ROM

Question: 146

During the health supervision visit for a 1-month-old infant, you note mild jaundice, but no feverand clear lungs. Cardiac examination demonstrates a 2/6 systolic murmur at the left sternalborder. Abdominal examination demonstrates a firm liver 3 cm below the costal margin andextending past the midline. The alanine aminotransferase is 140 U/L, aspartateaminotransferase is 130 U/L, total bilirubin is 5.5 mg/dL (94.1 mcmol/L), and direct bilirubin is 3.0mg/dL (51.3 mcmol/L).

Of the following, the finding that MOST strongly suggests the need for referral to a surgeon forintraoperative cholangiography is

A. abdominal ultrasonography demonstrating a prominent gallbladder

B. brain computed tomography scan demonstrating cerebral calcifications

C. echocardiography demonstrating pulmonary artery stenosis

D. dimethyl iminodiacetic acid scan demonstrating no tracer in the bowel

E. liver biopsy demonstrating periodic acid-Schiff stain-positive globules in the hepatocytes

Copyright © 2007 by the American Academy of Pediatrics page 303

Page 304: AAP MCQ 2007

2007 PREP SA on CD-ROM

Preferred Response: DCritique: 146

In a 1-month-old infant, the liver edge can be palpated below the costal margin. As the infantgrows, more of the liver is covered by the ribcage, making it unusual to feel a liver edge inchildren older than 1 year of age. A preferred approach to assessing liver size in all childreninvolves percussion of the liver span; the dull sound of the liver can be differentiated easily fromthe more hollow sound of the bowel and chest. Because chest hyperinflation can push the liverdown, thus mimicking "hepatomegaly," determination of liver span can help differentiate trueenlargement from liver displacement.

Hepatomegaly should be suspected when the liver edge extends more than 1 cm below thecostal margin, if the liver is hard to palpation, or if there is a prominent left lobe in the epigastrium.The causes of hepatomegaly in the infant and child are diverse and include primary liverdisease, infection, malignancy, and storage disorders.

For an infant younger than 2 months of age who has direct hyperbilirubinemia andhepatomegaly, such as the infant described in the vignette, the primary responsibility of thephysician is to diagnose or rule out extrahepatic biliary atresia. This condition must bedifferentiated from other anatomic, infectious, and metabolic causes of neonatal cholestasis.Biliary atresia occurs in as many as 30% of infants who have conjugated hyperbilirubinemia. Theinitial evaluation of an infant who has suspected biliary atresia typically involves exclusion ofinfectious causes by appropriate cultures and serologies, hepatobiliary ultrasonography toexclude gallstones, and radionuclide scintigraphy. The scintigraphic examination involvesinjecting an infant with a radionuclide tracer such as dimethyl iminodiacetic acid. A liver that isaffected by biliary atresia demonstrates tracer uptake into the liver but no excretion into thebowel (Item C146A). In contrast, a liver affected by cholestasis from other causes but that has apatent biliary system demonstrates delayed uptake in the liver but excretion into the bowel. As acaveat, however, infants who have severe cholestasis may have false-positive test results, inwhich tracer uptake is so slow that there is no significant excretion into the bowel. If this is thecase, additional testing with liver biopsy and intraoperative cholangiography is necessary toexclude biliary atresia.

Abdominal ultrasonography demonstrating a prominent gallbladder makes biliary atresia lesslikely because the gallbladder in biliary atresia is typically shrunken or not visible. Braincalcifications are seen in congenital toxoplasmosis and cytomegalovirus infection, two infectiouscauses of liver disease. Peripheral pulmonary artery stenosis is characteristic of Alagillesyndrome, a genetic cause of intrahepatic bile duct paucity. A liver biopsy with periodic acid-Schiff stain-positive granules suggests a storage disorder (glycogen storage disease, alpha-1-antitrypsin deficiency) as a cause of the child's hepatomegaly.

References:

Campbell KM, Bezerra JA. Biliary atresia. In: Walker WA, Goulet O, Kleinman RE, Sherman PM,Shneider BL, Sanderson IR, eds. Pediatric Gastrointestinal Disease. 4th ed. Hamilton, Ontario,Canada: BC Decker; 2005:1122-1138

Wolf AD, Lavine JR. Hepatomegaly in neonates and children. Pediatr Rev. 2000;21:303-310.Available at: http://pedsinreview.aappublications.org/cgi/content/full/21/9/303

Copyright © 2007 by the American Academy of Pediatrics page 304

Page 305: AAP MCQ 2007

2007 PREP SA on CD-ROM

Question: 147

You are evaluating a very low-birthweight (VLBW) preterm infant who experienced polyuria in thefirst 72 hours after birth. No diuretics have been prescribed, and there is no glycosuria,hematuria, or obvious anasarca on examination. You collect a urine sample to measureelectrolytes and creatinine and simultaneously obtain a blood sample to measure serumelectrolytes and creatinine.

Of the following, the MOST correct statement regarding sodium handling in the VLBW infant isthat

A. fractional excretion of sodium is lower than in term infants

B. intravenous sodium supplementation is necessary from birth

C. phototherapy increases sodium requirements

D. sodium excretion increases with gestational age

E. water losses generally exceed sodium losses

Copyright © 2007 by the American Academy of Pediatrics page 305

Page 306: AAP MCQ 2007

2007 PREP SA on CD-ROM

Preferred Response: ECritique: 147

The fractional excretion of sodium (FENa) is a measure of renal handling of solute. The equationfor calculating the FENa is:

FENa = [(Urine Na x Plasma Cr)/(Plasma Na x Urine Cr)] x 100

Example: [(90 x 0.9)/(145 x 120)] x 100 = 0.5

Sodium is reabsorbed in the proximal renal tubule. In the preterm kidney, a number offactors affect the renal handling of sodium. First, the renal blood flow increases throughoutgestation in the fetus and in the first week of postnatal life. Second, the glomerular filtration rateincreases throughout the first postnatal week. Third, the extracellular fluid compartment, wheremost of the total body content of sodium is located, is greatest in the most preterm infants.Depending on the degree of prematurity, the functional number of nephrons may be reducedsignificantly (nephrogenesis is not complete until after 36 weeks’ gestation), and renal efficiencyin handling solute load is reduced accordingly because sodium transporter activity in the renaltubules is immature. Additionally, the premature kidney has poor concentrating ability, withmaximal urinary concentration of 600 to 800 mOsm/L in the first 2 weeks of postnatal life. Thisresults in a high risk of hypervolemia and hyponatremia (due to dilution) for the preterm infantwho is given too much water. However, because of postnatal adjustments in fluid compartments(Item C147A), the addition of sodium to intravenous fluids for the preterm neonate is largelyunnecessary in the first 24 to 72 hours after birth.

The very low-birthweight (VLBW) newborn described in the vignette has polyuria, generallydefined as a urine output of greater than 6 mL/kg per hour. Causes typically include iatrogenicoverhydration, diabetes insipidus (genetic or sometimes related to intracranial pathology),hyperglycemia with a corresponding osmotic drag of water and the occurrence of glycosuria, oranatomic renal problems such as an obstruction that has been relieved. Water losses in theVLBW newborn generally exceed solute (sodium) losses, especially in cases of polyuria, anddehydration with elevated serum sodium concentrations may result.

As noted previously, sodium excretion (FENa) decreases with increasing gestational ageand postnatal age. Accordingly, the FENa is higher in a preterm newborn than in a term infant.Phototherapy increases fluid (water) requirements for the VLBW newborn, but not sodiumrequirements. Intravenous sodium supplementation is not necessary in most VLBW newborns inthe first 24 to 72 hours of postnatal life.

References:

Chua AN, Sarwal MM. Acute renal failure management in the neonate. NeoReviews.2005;6:e369-e376. Available at: http://neoreviews.aappublications.org/cgi/content/full/6/8/e369

Dell KM, Davis ID. Fluid and electrolyte management. In: Martin RJ, Fanaroff AA, Walch MC,eds. Fanaroff and Martin’s Neonatal-Perinatal Medicine: Diseases of the Fetus and Infant. 8thed. Philadelphia, Pa: Mosby-Elsevier; 2006:695-702

Copyright © 2007 by the American Academy of Pediatrics page 306

Page 307: AAP MCQ 2007

2007 PREP SA on CD-ROM

Question: 148

A young mother brings in her toddler immediately after seeing him drink an unknown amount ofmineral spirits. He appears tired and has a persistent cough. His respiratory rate is 40breaths/min, and his lungs are clear bilaterally. There are no other abnormal findings on physicalexamination. A chest radiograph shows no abnormality.

Of the following, the MOST appropriate management option is

A. administration of activated charcoal

B. administration of intravenous corticosteroids

C. discharge home with follow-up the next day

D. gastric lavage

E. hospitalization for observation and supportive care

Copyright © 2007 by the American Academy of Pediatrics page 307

Page 308: AAP MCQ 2007

2007 PREP SA on CD-ROM

Preferred Response: ECritique: 148

Hydrocarbons, such as mineral spirits, kerosene, gasoline, turpentine, and pine oil, are amongthe most common ingestions in young children. The hydrocarbons that have the lowestviscosity, such as furniture polish, are associated with the highest risk for aspiration. Childrenwho ingest hydrocarbons are at potentially very high risk for complications and should beevaluated carefully.

Children who have ingested hydrocarbons may have a slight cough initially, which does notnecessarily indicate an aspiration. However, those who exhibit persistent cough, tachypnea, ordyspnea within a few hours are at high risk of aspiration. Some children, including the boydescribed in the vignette, develop signs of central nervous system involvement, such asirritability or lethargy, which is largely due to hypoxia, not direct toxic effects to the brain. Fevermay be present initially but is not an indicator of aspiration pneumonitis. Intestinal symptoms,such as vomiting and abdominal pain, may occur because of mucosal irritation.

Children who have confirmed or suspected hydrocarbon ingestion should be evaluated witha chest radiograph, but early radiographs (within 1 hour of ingestion) may appear normal, evenin the presence of aspiration. Findings indicative of aspiration include bilateral, patchy opacities(Item C148A), but these findings correlate poorly with clinical manifestations. Blood gas analysismay reveal hypoxemia and should be considered in children who have low pulse oximetryreadings.

Even children who do not have respiratory symptoms should be observed in the emergencydepartment for 4 to 6 hours before discharge. Hospitalization is indicated for those who haverespiratory symptoms, altered mental status, abnormal chest radiographs, or inadequate follow-up, such as the boy in the vignette. Management is largely supportive. Oxygen should beadministered, and intubation with ventilatory support should be performed for those who haveimpending respiratory failure. Gastric decontamination with lavage is only indicated in caseswhen the ingested hydrocarbon solution contains other harmful elements, such as camphor,aromatics, heavy metals, or pesticides. Activated charcoal is not effective and should not beadministered because it may increase the risk of emesis, thereby further increasing the risk ofaspiration. Corticosteroids and antibiotics have not been shown to have a role in themanagement of hydrocarbon ingestions.

Respiratory symptoms of hydrocarbon ingestion typically resolve within 7 days, althoughchest radiograph abnormalities may be present for weeks. Complications such aspneumatoceles, pneumothorax, empyema, bacterial pneumonia, and respiratory distresssyndrome occur in a small number of patients. A few patients may develop chronic pulmonaryfunction abnormalities dues to proliferative alveolar thickening.

Another population in which hydrocarbon ingestion is important is adolescents. Adolescentsmay inhale hydrocarbons chronically and are at risk for long-term symptoms such as peripheralneuropathy and encephalopathy. They are also at risk for sudden cardiac death.

References:

Lee DC. Hydrocarbons. In: Marx JA, Hockberger RS, Walls RM, eds. Rosen’s EmergencyMedicine: Concepts and Clinical Practice. 5th ed. St. Louis, Mo: Mosby; 2002:2160-2163

McGuigan ME. Poisoning potpourri. Pediatr Rev. 2001;22:295-302. Available at:http://pedsinreview.aappublications.org/cgi/content/full/22/9/295

Tucker JF. Aliphatic hydrocarbons. In: Ford MD, Delaney KA, Ling LJ, Erickson T, eds. ClinicalToxicology. Philadelphia, Pa: WB Saunders Co; 2001:789-793

Copyright © 2007 by the American Academy of Pediatrics page 308

Page 309: AAP MCQ 2007

2007 PREP SA on CD-ROM

Question: 149

A 2-year-old boy who has been followed by your practice since birth presents for a healthsupervision visit. On careful physical examination, you have difficulty locating the right testis. Inreviewing your records, you note that this is a new finding.

Of the following, the MOST likely diagnosis is

A. agenesis of the testis

B. indirect inguinal hernia

C. pseudohermaphroditism

D. retractile testis

E. undescended testis

Copyright © 2007 by the American Academy of Pediatrics page 309

Page 310: AAP MCQ 2007

2007 PREP SA on CD-ROM

Preferred Response: DCritique: 149

Because the boy described in the vignette has had normal findings on previous testicularexaminations, it is likely that the cremasteric reflex is causing retractile testis, preventingexamination of the testis on this visit. A warm examination room, warm compresses to theinguinal area, or having the boy sit in the “tailor’s position” (sitting with knees flexed and anklescrossed on the examination table) or squatting may facilitate descent of the normal testis backinto the scrotal sac. Retractile testis is not common in early infancy or after puberty. Agenesis ofthe testis is unlikely because the testes were palpable on previous examinations. Indirectinguinal hernia would be associated with an inguinal mass, but the testis would still be palpableon examination.

True undescended testis is the single most common genitourinary problem in boys. Bilateralundescended testes may be associated with chromosomal or endocrine disorders, includingpseudohermaphroditism as well as other syndromes and should be evaluated with carefulattention to the presence of other malformations or metabolic disturbances. A normal-appearingpenis implies intact testicular tissue, but chromosomal and endocrine studies should beperformed. An endocrine consultation may be appropriate in an infant who has bilateralundescended testes, but it is not necessary for a child who has a unilateral undescended testis.

Failure of the testis to descend along the appropriate track from the abdomen through theinguinal ring and into the scrotal sac may be associated with infertility and up to a 40% increasein the risk for testicular cancer, which would be undetected in the abdominal location. Structuralchanges in the undescended testis have been documented as early as 2 years of age anddiminished sperm production as young as 6 years.

Although administration of beta human chorionic gonadotropin has been used infrequently toeffect testicular descent, it is not believed to be very effective. Adverse effects includepremature closure of the epiphyses and premature onset of secondary sex characteristics(reversible). Laparoscopic orchiopexy of the presumably present unilateral undescended testisis the current treatment of choice and usually is performed at 1 year of age.

References:

Callaghan P. In brief: undescended testis. Pediatr Rev. 2000;21:395. Available at:http://pedsinreview.aappublications.org/cgi/content/full/21/11/395

Roth DR, Gonzales ET. Disorders of renal development and anomalies of the collecting system,bladder, penis and scrotum. In: McMillan JA, DeAngelis CD, Feigin RD, Warshaw JB, eds.Oski’s Pediatrics: Principles and Practice. 3rd ed. Philadelphia, Pa: Lippincott, Williams & Wilkins;1999:1555-1556

Copyright © 2007 by the American Academy of Pediatrics page 310

Page 311: AAP MCQ 2007

2007 PREP SA on CD-ROM

Question: 150

A 15-year-old girl is rushed to the emergency department after slipping off a diving board andstriking her head on cement. On physical examination, her mental status evaluation results arecompletely normal. She can abduct her upper extremities at the shoulder, but cannot flex orextend her arms and minimally moves her fingers. She cannot move her limp lower extremities.You order emergent magnetic resonance imaging of the brain and spine.

Of the following, the MOST important therapy to implement before the patient is sent for imagingis

A. fosphenytoin 18 phenytoin equivalents/kg intravenously

B. dexamethasone 1 mg/kg orally

C. low-molecular weight heparin 1 mg/kg subcutaneously

D. mannitol 1 g/kg intravenously

E. methylprednisolone 30 mg/kg intravenously

Copyright © 2007 by the American Academy of Pediatrics page 311

Page 312: AAP MCQ 2007

2007 PREP SA on CD-ROM

Preferred Response: ECritique: 150

The possibility of occult or overt spinal trauma should be considered with any injury. For anynotable body trauma, the cervical and lower spine should be immobilized until spinal trauma hasbeen eliminated as a possibility. Common sites for fracture dislocation spinal cord injuries are C1through C2, C5 through C6, and T12 through L1 spinal segments. Fracture dislocations of thevertebral column are the most common causes of spinal cord injury, although injury can be assevere as complete cord transection without any visible trauma on radiography. Suddenwhiplash (flexion-extension) injuries may contuse (Item C150A) or lacerate the cervical cord.

Typical symptoms and signs of spinal trauma include focal pain along the spinal axis or anyweakness or sensory loss in the extremities or trunk. “Spinal shock” is characterized by loss ofall voluntary movement and sensation inferior to the site of injury. Reflex function in the cord islost temporarily, resulting in ileus and bladder distention.

While the spine is immobilized, the child should undergo a thorough neurologic examination.If there is high index of suspicion for spinal trauma, as for the girl described in the vignette,prompt initiation of methylprednisolone 30 mg/kg administered intravenously over 1 hour,followed by 5.4 mg/kg per hour the next 23 hours, is a protocol that can lead to improvedneurologic outcomes. The cervical spine typically is evaluated by radiographs (including lateral,anterior-posterior, and odontoid views), but if there is lingering concern, computed tomographyscan or magnetic resonance imaging is warranted.

Intravenous dexamethasone is useful for spinal cord compression with tumors, but itsefficacy has not been investigated fully for spinal trauma. Anticonvulsants sometimes areadministered prophylactically after severe head trauma, but this is controversial. Mannitol isuseful only if the patient displays signs of increased intracranial pressure. Low-molecular weightheparin to prevent deep venous thrombosis might be useful if the patient is not expected toregain ambulatory status for several days.

References:

Bracken MB, Shepard MJ, Collins WF, et al. A randomized, controlled trial of methylprednisoloneor naloxone in the treatment of acute spinal-cord injury. Results of the Second National AcuteSpinal Cord Injury Study. N Engl J Med. 1990;322:1405-1411. Abstract available at:http://www.ncbi.nlm.nih.gov/entrez/query.fcgi?cmd=Retrieve&db=pubmed&dopt=Abstract&list_uids=2278545&query_hl=19&itool=pubmed_docsum

Haslam RHA. Spinal cord disorders: spinal cord trauma. In: Behrman RE, Kliegman RM, JensonHB, eds. Nelson Textbook of Pediatrics. 17th ed. Philadelphia, Pa: WB Saunders Co; 2004:2050

Temkin NR, Dikmen SS, Wilensky AJ, Keihm J, Chabal S, Winn HR. A randomized, double-blindstudy of phenytoin for the prevention of post-traumatic seizures. N Engl J Med. 1990;323:497-502. Abstract available at:http://www.ncbi.nlm.nih.gov/entrez/query.fcgi?cmd=Retrieve&db=pubmed&dopt=Abstract&list_uids=2115976&query_hl=21&itool=pubmed_docsum

Copyright © 2007 by the American Academy of Pediatrics page 312

Page 313: AAP MCQ 2007

2007 PREP SA on CD-ROM

Question: 151

You care for a 17-year-old boy who is overweight. He has gained 44 lb (20 kg) in the last year,especially in his abdominal area. On examination today, his blood pressure is 158/90 mm Hgusing a large, appropriately sized cuff. His mother has a similar body habitus and is beingtreated for type 2 diabetes mellitus.

Of the following, the MOST likely abnormalities to expect in this patient if his present conditioncontinues into adulthood is

A. high triglyceride concentrations, low high-density lipoprotein cholesterol concentrations

B. hypoglycemia from insulin sensitivity

C. low concentrations of C-reactive protein in the blood

D. low fibrinogen concentrations with bleeding diatheses

E. low triglyceride concentrations, low low-density lipoprotein cholesterol concentrations

Copyright © 2007 by the American Academy of Pediatrics page 313

Page 314: AAP MCQ 2007

2007 PREP SA on CD-ROM

Preferred Response: ACritique: 151

The metabolic syndrome is a series of clinical and laboratory findings that include truncalobesity, elevated concentrations of serum triglycerides, decreased concentrations of high-density lipoprotein cholesterol, and an elevation in blood pressure. Sex- and age-specific valuesexist for each of these values in the adult population. The metabolic syndrome increases the riskfor type 2 diabetes and cardiovascular disease. In some recent studies of adult patients, theincidence of the metabolic syndrome was as high as 23%. The metabolic syndrome affectspeople from all ethnic and cultural backgrounds, but is most prevalent in the Hispanic population.The increasing incidence of the metabolic syndrome in the adult population is likely the result ofthe growing number of children who develop obesity. The growth in numbers of obese childrenhas paralleled the increased incidence in type 2 diabetes and impaired glucose tolerance.

The patient described in the vignette has abdominal obesity, hypertension, and a familyhistory of type 2 diabetes, which places him at risk for the metabolic syndrome and its attendantrisks for the development of cardiovascular disease as he ages. The most likely laboratoryabnormalities that will be identified with the metabolic syndrome are elevated triglycerideconcentrations and decreased concentrations of high-density lipoprotein cholesterol. Insulinresistance, which often is found in metabolic syndrome, may lead to hyperglycemia. C-reactiveprotein and fibrinogen values often are elevated in patients who have metabolic syndrome.

References:

Cruz M L, Weigensberg MJ, Huang TT, Ball G, Shaibi GQ, Goran MI. The metabolic syndromein overweight Hispanic youth and the role of insulin sensitivity. J Clin Endocrinol Metab.2004;89:108-113. Available at: http://jcem.endojournals.org/cgi/content/full/89/1/108

Ford ES, Giles WH, Dietz WH. Prevalence of the metabolic syndrome among US adults: findingsfrom the third National Health and Nutrition Examination Survey. JAMA. 2002;287:356-359.Abstract available at:http://www.ncbi.nlm.nih.gov/entrez/query.fcgi?orig_db=PubMed&db=PubMed&cmd=Search&term=%22JAMA+:++the+journal+of+the+American+Medical+Association%22[Jour]+AND+2002[pdat]+AND+Ford+E[author]

Copyright © 2007 by the American Academy of Pediatrics page 314

Page 315: AAP MCQ 2007

2007 PREP SA on CD-ROM

Question: 152

A newborn female has loose neck skin (Item Q152A) and nonpitting edema of the lowerextremities (Item Q152B).

Of the following, the MOST appropriate evaluation for this infant is

A. blood chromosome analysis

B. magnetic resonance imaging of the brain

C. slitlamp ophthalmologic examination

D. ultrasonography of the liver

E. voiding cystourethrography

Copyright © 2007 by the American Academy of Pediatrics page 315

Page 316: AAP MCQ 2007

2007 PREP SA on CD-ROM

Preferred Response: ACritique: 152

The finding of loose neck skin (Item C152A), which is suggestive of the presence of a cystichygroma in fetal life, and nonpitting edema of the lower extremities in a newborn female shouldraise the suspicion of Turner syndrome (TS). Congenital lymphedema (Item C152B), whichoccurs in up to 80% of affected females, typically disappears during infancy, leaving only a puffyappearance to the hands and feet, although in some patients it reappears when estrogentherapy is initiated. The redundant posterior neck skin can persist as the pterygium colli, orwebbed neck. The clinical diagnosis of TS should be confirmed by peripheral blood chromosomeanalysis, which will reveal monosomy X, mosaic monosomy X, or the presence of an abnormalX chromosome that contains a deletion. A buccal smear lacks sensitivity and should not be usedto make the diagnosis of TS.

Affected girls also have short stature, ovarian dysgenesis, a broad chest with wide-spacednipples (Item C152C), ear anomalies, cubitus valgus, and renal (eg, pelvic kidney, horseshoekidney) and cardiac (eg, bicuspid aortic valve, coarctation of the aorta) defects. Intelligencetypically is normal. Estrogen replacement therapy at the expected time of puberty is required inmost cases, and treatment with growth hormone also should be offered.

Magnetic resonance imaging of the brain, slitlamp ophthalmologic examination, andultrasonography of the liver are not routinely indicated. Although renal defects may be present,the most common abnormality is horseshoe kidney, which can be detected by ultrasonographyand usually has no clinical effects that would prompt the need for voiding cystourethrography.

References:

Frias JL, Davenport ML, Committee on Genetics and Section on Endocrinology. Healthsupervision for children with Turner syndrome. Pediatrics. 2003;111:692-702. Available at:http://pediatrics.aappublications.org/cgi/content/full/111/3/692

Sybert VP, McCauley E. Turner's syndrome. N Engl J Med. 2004;351:1227-1238

Copyright © 2007 by the American Academy of Pediatrics page 316

Page 317: AAP MCQ 2007

2007 PREP SA on CD-ROM

Question: 153

A 16-year-old boy presents for his annual sports physical before wrestling season. He has beenparticipating in a weight lifting program and proudly reports his increased muscle mass andstrength. You are concerned that he may be using performance-enhancing substances (eg,creatine).

Of the following, the use of creatine is MOST likely to be revealed by

A. abnormal liver enzyme concentrations

B. an interview with the patient

C. increase in hemoglobin concentration

D. presence of gynecomastia

E. serum drug screening

Copyright © 2007 by the American Academy of Pediatrics page 317

Page 318: AAP MCQ 2007

2007 PREP SA on CD-ROM

Preferred Response: BCritique: 153

Creatine and other substances such as anabolic steroids, androstenedione, growth hormone,and erythropoietin have been used by athletes because of their performance-enhancingreputations. Most adolescent users of performance-enhancing substances (PESs) seek toimprove their athletic performance, although some use these agents to enhance their physicalappearance by developing a more muscular body. All athletes, whether in organized sports orindividual training programs (eg, weight lifting), should be asked about the use of PESs. Thediagnosis of PES use is aided by a high index of suspicion and a history that includes therecognition of signs of use and adverse effects.

Creatine enhances the ability of muscles to maintain power in short maximal exerciseefforts. Creatine cannot work without strength training; those who lift weights achieve a greaterbenefit compared with those who do not lift. Adverse effects include muscle cramping andgastrointestinal symptoms of vomiting and diarrhea. The use of creatine is not recommended foradolescent athletes because its safety has not been established. The use of creatine and otherPESs is most likely to be revealed by an interview with an adolescent. There is no specificassay to identify creatine use. Creatine has not been reported to cause abnormal liver enzymeconcentrations, increased hemoglobin concentrations, or gynecomastia.

The lifetime prevalence of illegal steroid use among high school students in the United Stateshas been reported to be as high as 6%. Users of anabolic steroids anticipate gains in musclemass and strength. Deepening of the voice and hirsutism in a female and gynecomastia and ahigh voice in a male should heighten the clinician’s suspicion for the use of anabolic steroids.Virilization in the female also may result in hair loss. Hypogonadism or a decrease in testicularsize associated with oligospermia/azoospermia and decreased sexual function/desire mayoccur in male users. Acne, often severe, can be observed in both male and female abusers.Other adverse effects associated with anabolic steroid abuse include hypertension,hyperaggressiveness, mood swings, psychosis, and mania. Depression can occur duringwithdrawal. Prepubertal abusers of anabolic steroids are at risk for premature closure of theirepiphyseal growth plates. Nonspecific evidence of anabolic steroid use may be found inlaboratory study results, including elevated liver enzyme concentrations and decreased high-density and increased low-density lipoprotein cholesterol levels. Laboratory evidence of orallyadministered steroids can be present in the urine for days to weeks; injected steroids can bepresent for 6 months or longer.

Androstenedione has received considerable attention in the media and is purported toincrease muscle mass and strength. As a precursor to testosterone, use theoretically caninduce premature puberty or the premature closure of growth plates and cause other adverseeffects that have been attributed to anabolic steroid use. Assays (urine screens confirmed bygas chromatography) included in some mandatory drug screens of collegiate athletes areavailable to detect androstenedione by specialized laboratories.

Growth hormone (GH) use theoretically increases protein anabolism and lipolysis; thus, ithas a particular lure for bodybuilders. The anticipated signs of GH abuse are clinical findingsconsistent with acromegaly. These findings include coarsening of facial features, growth of facialbones (including the mandible, with increased separation of teeth), increase in headcircumference, broadening of the nose, enlargement of the tongue, and thickness of fingers andtoes. GH abuse may be associated with adverse behavior changes, hypertension, diabetes,cardiovascular disease, and peripheral neuropathy. There is no readily available, reliable test ormetabolic marker to detect the use of exogenous GH.

Erythropoietin increases red blood cell mass and improves oxygen delivery to skeletalmuscle, thereby improving exercise endurance. Specialized laboratory testing can identify theuse of natural and recombinant erythropoietin. The increased red blood cell mass also increasesblood viscosity and may cause vascular occlusions.

References:

Coupey SM. Specific drugs. In: Schydlower M, ed. Substance Abuse: A Guide for HealthProfessionals. 2nd ed. Elk Grove Village, Ill: American Academy of Pediatrics; 2002:191-276

Copyright © 2007 by the American Academy of Pediatrics page 318

Page 319: AAP MCQ 2007

2007 PREP SA on CD-ROM

Greydanus DE, Patel DR. Sports doping in the adolescent athlete: the hope, hype, andhyperbole. Pediatr Clin North Am. 2002;49:829-855. Abstract available at:http://www.ncbi.nlm.nih.gov/entrez/query.fcgi?orig_db=PubMed&db=PubMed&cmd=Search&term=%22Pediatric+clinics+of+North+America%22[Jour]+AND+829[page]+AND+2002[pdat]

Hergenroeder AC, Neinstein LS. Guidelines in sports medicine. In: Neinstein LS, ed. AdolescentHealth Care A Practice Guide. 4th ed. Philadelphia, Pa: Lippincott Williams & Wilkins; 2002:380-438

Koch JJ. Performance-enhancing substances and their use among adolescent athletes. PediatrRev. 2002;23:310-317. Available at:http://pedsinreview.aappublications.org/cgi/content/full/23/9/310

Copyright © 2007 by the American Academy of Pediatrics page 319

Page 320: AAP MCQ 2007

2007 PREP SA on CD-ROM

Question: 154

A 14-year-old boy has a body mass index that is greater than the 95th percentile for age and anaccelerating weight curve. His blood pressure is 135/85 mm Hg. His mother has type 2 diabetesthat developed during her first pregnancy, and several paternal relatives also have type 2diabetes. The family emigrated from the Caribbean when he was a toddler. He has dark velvetythickening (Item Q154A) of the skin on his neck and under his arms.

Of the following, the findings that are MOST supportive of a diagnosis of metabolic syndrome inthis young man are

A. acanthosis nigricans, hypertension, obesity

B. acanthosis nigricans and maternal history of diabetes

C. acanthosis nigricans, obesity

D. hypertension, obesity

E. obesity and maternal history of diabetes

Copyright © 2007 by the American Academy of Pediatrics page 320

Page 321: AAP MCQ 2007

2007 PREP SA on CD-ROM

Preferred Response: DCritique: 154

The constellation of findings that lead to the diagnosis of metabolic syndrome in adults varies,depending upon the criteria used, although hypertension and obesity are common to alldefinitions. The latest criteria from the International Diabetes Federation include ethnicity-specificwaist circumference criteria or a body mass index greater than 30 kg/M2 plus two of thefollowing:

•Fasting triglyceride concentration greater than 150 mg/dL (1.7 mmol/L) or treatment for

this finding•High-density lipoprotein cholesterol concentration less than 40 mg/dL (1.03 mmol/L) in

men and 50 mg/dL (1.29 mmol/L) in women •Hypertension manifested by systolic blood pressure of 130 mm Hg or greater and

diastolic pressure of 85 mm Hg or greater or previously treated hypertension•Fasting plasma glucose value of 100 mg/dL (5.5 mmol/L) or greater or previously

diagnosed type 2 diabetes

Definitions in childhood are not established, but usually consist of similar criteria adjusted toage-appropriate values greater than the 85th percentile for blood pressure, weight, and waistcircumference. A recent review of the criteria for the diagnosis of metabolic syndromeconcludes that “metabolic syndrome” may not be an entity in adults that is greater in severitythan the sum of its components.

Acanthosis nigricans is not included as a criterion for the diagnosis of metabolic syndrome,although it often is found. Maternal or family history of diabetes frequently is found and increasesthe risk of type 2 diabetes in the child, but it is not one of the criteria for the diagnosis ofmetabolic syndrome.

References:

Boney CM, Verma A, Tucker R, Vohr BR. Metabolic syndrome in childhood: association withbirth weight, maternal obesity, and gestational diabetes mellitus. Pediatrics. 2005;115:e290-e296.Available at: http://pediatrics.aappublications.org/cgi/content/full/115/3/e290

Ford ES. Prevalence of the metabolic syndrome defined by the International Diabetes Federationamong adults in the U.S. Diabetes Care. 2005;28:2745-2749. Available at:http://care.diabetesjournals.org/cgi/content/full/28/11/2745

Hirschler V, Aranda C, Calcagno Mde L, Maccalini G, Jadzinsky M. Can waist circumferenceidentify children with the metabolic syndrome? Arch Pediatr Adolesc Med. 2005;159:740-744.Abstract available at:http://www.ncbi.nlm.nih.gov/entrez/query.fcgi?orig_db=PubMed&db=PubMed&cmd=Search&term=%22Archives+of+pediatrics+%26+adolescent+medicine%22[Jour]+AND+740[page]+AND+2005[pdat]

Janssen I, Katzmarzyk PT, Srinivasan SR, et al. Combined influence of body mass index andwaist circumference on coronary artery disease risk factors among children and adolescents.Pediatrics. 2005;115:1623-1630. Available at:http://pediatrics.aappublications.org/cgi/content/full/115/6/1623

Kahn R, Buse J, Ferrannini E, Stern M; American Diabetes Association; European Associationfor the Study of Diabetes. The metabolic syndrome: time for a critical appraisal: joint statementfrom the American Diabetes Association and the European Association for the Study ofDiabetes. Diabetes Care. 2005;28:2289-2304. Available at:http://care.diabetesjournals.org/cgi/content/full/28/9/2289

Copyright © 2007 by the American Academy of Pediatrics page 321

Page 322: AAP MCQ 2007

2007 PREP SA on CD-ROM

Ten S, Maclaren N. Insulin resistance syndrome in children. J Clin Endocrinol Metab.2004;89:2526-2539. Available at: http://jcem.endojournals.org/cgi/content/full/89/6/2526

Copyright © 2007 by the American Academy of Pediatrics page 322

Page 323: AAP MCQ 2007

2007 PREP SA on CD-ROM

Question: 155

You are evaluating a 10-year-old boy who has behavior problems. He hits his younger brother,talks back to his parents, and refuses to perform his assigned household chores. He does wellin school, but is oppositional in the classroom. His parents ask what they can do to improve hisbehavior.

Of the following, your BEST response is that

A. he needs cognitive testing

B. he should be referred for individual counseling

C. medication is the preferred first-line treatment

D. school-based behavior programs are preferred to home-based programs

E. the parents should be trained in behavior modification techniques

Copyright © 2007 by the American Academy of Pediatrics page 323

Page 324: AAP MCQ 2007

2007 PREP SA on CD-ROM

Preferred Response: ECritique: 155

Behavior modification is a general category of therapy that refers to principles and techniquesbased on learning theory and used to change behavior. Behavioral techniques are used to eitherstrengthen or maintain desired behaviors or to decrease or eliminate undesired behaviors. Forexample, parental praise or token economies are examples of techniques used to reinforcebehavior, and time-out is a technique used to decrease or eliminate a behavior.

The child described in the vignette has oppositional behavior that may respond to behavioralmanagement. He is doing well in school and, therefore, probably does not need cognitive testing.Individual counseling may be used as an adjunct, but he is more likely to respond to consistentbehavior management. Behavioral approaches should be tried before using medication to controlbehavior. The behavioral plan should be instituted both at home and in school, withcommunication between teacher and parents to maintain consistency.

References:

Boris NW, Dalton R. Disruptive behavioral disorders. In: Behrman RE, Kliegman RM, JensonHB, eds. Nelson Textbook of Pediatrics. 17th ed. Philadelphia, Pa: WB Saunders Co; 2004:88-90

Vitulano LA. Child and adolescent behavior therapy. In: Lewis M, ed. Child and AdolescentPsychiatry: A Comprehensive Textbook. 3rd ed. Philadelphia, Pa: Lippincott Williams & Wilkins;2002:998-1014

Copyright © 2007 by the American Academy of Pediatrics page 324

Page 325: AAP MCQ 2007

2007 PREP SA on CD-ROM

Question: 156

A 14-month-old girl is brought to the emergency department with a 12-hour history of fever andrash. Her mother became frightened when it was difficult to arouse the girl after her nap.Findings on physical examination include a temperature of 104°F (40°C), a heart rate of 164beats/min, a respiratory rate of 42 breaths/min, and a blood pressure of 75/45 mm Hg. Thereare petechiae and purpura (Item Q156A) on the chest, arms, and legs. As you are establishingintravenous access and drawing blood for initial laboratory evaluations, you request animmediate dose of antibiotics.

Of the following, the MOST appropriate therapy is

A. ceftriaxone

B. penicillin

C. vancomycin

D. vancomycin plus ceftriaxone

E. vancomycin plus gentamicin

Copyright © 2007 by the American Academy of Pediatrics page 325

Page 326: AAP MCQ 2007

2007 PREP SA on CD-ROM

Preferred Response: DCritique: 156

The findings reported for the patient in the vignette suggest that she has sepsis due to Neisseriameningitidis, but nearly all pathogenic bacteria can cause petechiae and purpura. When treatingpatients who have potentially life-threatening infections, the initial choice of antimicrobial agentsshould be broad enough to cover the endemic pathogenic bacteria in the local community. Oncea bacterial pathogen is isolated and susceptibility data are known, the antimicrobial therapy canbe simplified. The treatment of sepsis varies with age of the patient.

Vancomycin and ceftriaxone represent the best choice for most patients older than 3months who have potentially life-threatening infections. Vancomycin provides coverage forpenicillin- and cephalosporin-resistant Streptococcus pneumoniae, Staphylococcus aureus, andEnterococcus, and ceftriaxone is effective against N meningitidis, most S pneumoniae, and othergram-negative organisms such as Haemophilus influenzae. Single agents (eg, ceftriaxone,penicillin, vancomycin) do not provide enough coverage in a critically ill patient. Vancomycin andgentamicin provide broad coverage, but gentamicin is not active against H influenzae and doesnot penetrate the central nervous system as effectively as ceftriaxone.

For most neonates, ampicillin in addition to cefotaxime or gentamicin is an appropriatechoice. Many experts suggest that if central nervous system disease has not or cannot be ruledout, cefotaxime is preferable because the cephalosporins penetrate the central nervous systembetter than the aminoglycosides. Vancomycin rarely is included in the treatment of neonates whohave community-acquired sepsis. Certain situations may call for the addition of acyclovir.

Patients between 1 and 3 months of age are more problematic. It is during this age that acrossover can occur from pathogens commonly seen in the neonatal period (eg, group BStreptococcus, Escherichia coli, Listeria monocytogenes, Enterococcus) to those usuallyencountered in children older than 3 months of age (eg, S pneumoniae, N meningitidis, Saureus). Clinicians should understand the common bacterial pathogens and their resistancepatterns in their community when making antibiotic choices. If there is concern for methicillin-resistant S aureus or penicillin- or cephalosporin-resistant S pneumoniae, vancomycin should bepart of the empiric regimen.

References:

Stormorken A, Powell KR. Sepsis and shock. In: Behrman RE, Kliegman RM, Jenson HB, eds.Nelson Textbook of Pediatrics. 17th ed. Philadelphia, Pa: WB Saunders Co; 2004:846-850

American Academy of Pediatrics. Meningococcal infections. In: Pickering LK, ed. Red Book:2006 Report of the Committee on Infectious Diseases. 27th ed. Elk Grove Village, Ill: AmericanAcademy of Pediatrics; 2006:452-460

Durbin WJ. Pneumococcal infections. Pediatr Rev. 2004;25:418-424. Available at:http://pedsinreview.aappublications.org/cgi/content/full/25/12/418

Copyright © 2007 by the American Academy of Pediatrics page 326

Page 327: AAP MCQ 2007

2007 PREP SA on CD-ROM

Question: 157

You are evaluating a newborn boy who has lax abdominal musculature (Item Q157A) andbilateral undescended testes. Other findings on physical examination are normal.

Of the following, the MOST likely urologic abnormality in this boy is

A. hydronephrosis

B. renal cysts

C. ureterocele

D. ureteropelvic junction obstruction

E. vesicoureteral reflux

Copyright © 2007 by the American Academy of Pediatrics page 327

Page 328: AAP MCQ 2007

2007 PREP SA on CD-ROM

Preferred Response: ACritique: 157

Prune belly (Eagle-Barrett) syndrome (PBS) is a relatively uncommon condition resulting frompoorly developed abdominal musculature. Affected children may have a variety of urinary tractanomalies, including undescended testes, hydronephrosis, posterior urethral valves (PUV), andbladder dysfunction.

The lax abdominal musculature (Item C157A) and bilateral undescended testes describedfor the boy in the vignette suggest the diagnosis of PBS. Abdominal ultrasonography and voidingcystourethrography are indicated in all children who have PBS to study the entire urinary tract.Hydronephrosis is a common anomaly associated with PBS. The hydronephrosis usually isobstructive and due to PUV or vesicoureteral reflux and ureteropelvic junction obstruction. Renalcysts are very uncommon, and ureteroceles are seen only occasionally in children who havePBS. Some affected children may have nonobstructive hydronephrosis. The best test todifferentiate obstructive from nonobstructive hydronephrosis is renal scintigraphy. In obstructivehydronephrosis, the tracer cannot escape the kidney, but in most kidneys with nonobstructivehydronephrosis, some amount of the tracer exits the kidney.

The renal outcome for children who have PBS is generally poor. Meticulous attention mustbe paid to the prevention of urinary tract infections (UTIs). Thus, it is imperative to instituteantibiotic prophylaxis if obstruction is present, and in some cases of moderate-to-severehydronephrosis with obstruction, early surgical repair is recommended. Urinary diversion via avesicostomy or Mitrofanoff procedure (using the appendix to connect the bladder to theabdomen, with exit near the umbilicus) may be indicated to prevent urinary stasis and UTIs.Unfortunately, renal growth usually is compromised because of multiple abnormalities within theurinary tract, and most children who have PBS develop renal insufficiency.

References:

Denes FT, Arap MA, Giron AM, Silva FA, Arap S. Comprehensive surgical treatment of prunebelly syndrome: 17 years' experience with 32 patients. Urology. 2004;64:789-793. Abstractavailable at:http://www.ncbi.nlm.nih.gov/entrez/query.fcgi?orig_db=PubMed&db=PubMed&cmd=Search&term=%22Urology%22[Jour]+AND+789[page]+AND+2004[pdat]

Jennings RW. Prune belly syndrome. Semin Pediatr Surg. 2000;9:115-120. Abstract available at:http://www.ncbi.nlm.nih.gov/entrez/query.fcgi?orig_db=PubMed&db=PubMed&cmd=Search&term=%22Seminars+in+pediatric+surgery%22[Jour]+AND+115[page]+AND+2000[pdat]

Strand WR. Initial management of complex pediatric disorders: prunebelly syndrome, posteriorurethral valves. Urol Clin North Am. 2004;31:399-415. Abstract available at:http://www.ncbi.nlm.nih.gov/entrez/query.fcgi?orig_db=PubMed&db=PubMed&cmd=Search&term=%22The+Urologic+clinics+of+North+America%22[Jour]+AND+399[page]+AND+2004[pdat]

Copyright © 2007 by the American Academy of Pediatrics page 328

Page 329: AAP MCQ 2007

2007 PREP SA on CD-ROM

Question: 158

A 15-year-old boy was camping with friends 1 week ago. Over the last 4 days, he hasdeveloped fever, worsening headache, increasing sleepiness, and combativeness. You suspectarboviral meningoencephalitis.

Of the following, the MOST frequently used method to confirm the diagnosis is

A. detection of myelin basic protein in the cerebrospinal fluid (CSF)

B. detection of viral antigen in brain biopsy tissue

C. detection of viral nucleic acid in the CSF by polymerase chain reaction

D. isolation of the virus by culture of the CSF

E. measurement of acute and convalescent antibody titers in serum or CSF

Copyright © 2007 by the American Academy of Pediatrics page 329

Page 330: AAP MCQ 2007

2007 PREP SA on CD-ROM

Preferred Response: ECritique: 158

Arboviruses are an important cause of encephalitis in the United States during the late springand summer months. These agents are transmitted by the bites of mosquitos, ticks, orsandflies. Examples of arboviral encephalitis encountered in North America include St. Louisencephalitis, La Crosse encephalitis, Western and Eastern equine encephalitis, Californiaencephalitis, West Nile encephalitis, and Colorado tick fever.

The diagnosis should be suspected in any child who presents with symptoms of fever,irritability or change in mental status, and headache. Cerebrospinal fluid (CSF) showspleocytosis (usually <500 cells with a mononuclear predominance), normal-to-mildly elevatedCSF protein concentration, and a normal CSF glucose concentration. Most arboviral infectionsresolve within 2 weeks, but there is a mortality rate of 10%. Eastern equine encephalitis has amuch higher mortality rate of up to 50%.

Confirming the diagnosis of arboviral encephalitis remains difficult because rapid diagnostictests such as polymerase chain reaction are unavailable, and culture of the CSF is rarelyhelpful. Isolation of virus or immunohistochemical detection of viral antigen from brain tissue mayprovide positive results, but this requires a brain biopsy, which rarely is performed.Measurement of myelin basic protein, which can be elevated in patients who have multiplesclerosis and other demyelinating conditions, is not helpful in the diagnosis of arboviralinfections. Serology continues to be the mainstay of diagnosis for arboviral infections, withseveral different serologic techniques available. The presence of virus-specific immunoglobulinM (IgM) detected by IgM-capture enzyme immunoassay in the CSF is diagnostic, and the findingof virus-specific IgM in the serum is presumptive evidence of infection. The demonstration of afourfold elevation of virus-specific IgG in acute and convalescent (4 to 6 weeks) sera, asmeasured by immunofluorescence, hemagglutination inhibition, complement fixation, orneutralization, is the most common method of confirming an arboviral infection. An elevated virus-specific IgG titer in a single serum sample is also strong evidence of a recent infection.

References:

American Academy of Pediatrics. Arboviruses (including California encephalitis [primarily LaCrosse], Eastern and Western equine encephalitis, Powassan encephalitis, St Louisencephalitis, Venezuelan equine encephalitis, West Nile encephalitis, Colorado tick fever,dengue, Japanese encephalitis, and yellow fever). In: Pickering LK, ed. Red Book: 2006 Reportof the Committee on Infectious Diseases. 27th ed. Elk Grove Village, Ill: American Academy ofPediatrics; 2006:211-217

Hayes EB, O’Leary DR. West Nile virus infection: a pediatric perspective. Pediatrics.2004;113:1375-1381. Available at:http://pediatrics.aappublications.org/cgi/content/full/113/5/1375

Solomon T. Flavivirus encephalitis. N Engl J Med. 2004;351:370-378. Abstract available at:http://www.ncbi.nlm.nih.gov/entrez/query.fcgi?orig_db=PubMed&db=PubMed&cmd=Search&term=%22The+New+England+journal+of+medicine%22[Jour]+AND+370[page]+AND+2004[pdat]

Copyright © 2007 by the American Academy of Pediatrics page 330

Page 331: AAP MCQ 2007

2007 PREP SA on CD-ROM

Question: 159

An 18-year-old girl who has asthma presents for an annual evaluation. She states that she hashad asthma for the past 6 years. Her symptoms include cough, wheezing, and chest tightness,primarily triggered by upper respiratory tract infections, strenuous exercise, and pollenexposure. She never has been hospitalized for her asthma, but she had to visit the emergencydepartment 2 years ago for an exacerbation. After reviewing her symptom frequency,medication use, and metered-dose inhaler technique, she performs a pulmonary function test(Item Q159A). The prebronchodilator FEV1 is 74% of expected and the FEV1/FVC is 79% ofexpected.

Of the following, the BEST characterization of her asthma severity, based on results of herpulmonary function test, is

A. mild obstructive with reversibility

B. mild restrictive without reversibility

C. moderate obstructive with reversibility

D. moderate restrictive without reversibility

E. normal

Copyright © 2007 by the American Academy of Pediatrics page 331

Page 332: AAP MCQ 2007

2007 PREP SA on CD-ROM

Preferred Response: CCritique: 159

Spirometry, also referred to as a pulmonary function test (PFT), can measure inspiratory andexpiratory respiratory effort. When interpreting spirometry, measurements of forced vitalcapacity (FVC), volume of air exhaled during the first second (FEV1), and the FEV1/FVC ratiohelp determine obstructive versus restrictive disease. If obstructive disease is suspected, theseverity of obstructive lung disease can be classified further as mild, moderate, or severe (ItemC159A). The adolescent in the vignette demonstrated a prebronchodilator FEV1 of 74% andFEV1/FVC of 79%, which is consistent with moderate obstructive severity.

Reversibility (Item C159B) is defined as an increase in FEV1 of 12% and 200 mL after ashort-acting bronchodilator. The adolescent in the vignette demonstrated a 35% increase (+700mL) in FEV1 after a bronchodilator treatment, which is consistent with airway reversibility.

Causes of obstructive lung disease include asthma and cystic fibrosis; restrictive lungdisease may indicate interstitial fibrosis, scoliosis, or neuromuscular disease.

References:

Guill MF. Asthma update: clinical aspects and management. Pediatr Rev. 2004;25:335-344.Available at: http://pedsinreview.aappublications.org/cgi/content/full/25/10/335

Liu AH, Spahn JD, Leung DYM. Childhood asthma. In: Behrman RE, Kliegman RM, Jenson HB,eds. Nelson Textbook of Pediatrics. 17th ed. Philadelphia, Pa: WB Saunders Co 2004:760-774

Copyright © 2007 by the American Academy of Pediatrics page 332

Page 333: AAP MCQ 2007

2007 PREP SA on CD-ROM

Question: 160

A 16-year-old boxer presents to your office 2 hours after a blow delivered to one ear duringsparring. The ear is swollen and painful. On physical examination, you note a 2-cm diameterarea of bruising and fluctuance along the upper aspect of the pinna anteriorly.

Of the following, the MOST appropriate next step in the management of this patient is to

A. apply ice to the ear to limit further swelling

B. apply a pressure dressing

C. arrange for consultation with an otorhinolaryngologist within 1 to 3 days

D. evacuate the hematoma with needle and syringe

E. provide antibiotic prophylaxis to prevent infection

Copyright © 2007 by the American Academy of Pediatrics page 333

Page 334: AAP MCQ 2007

2007 PREP SA on CD-ROM

Preferred Response: DCritique: 160

A tangential blow to the ear can cause a shearing force that separates the skin from theunderlying perichondrium or the perichondrium from the cartilage. The resulting hematoma (ItemC160A) may cause long-term disfigurement if not managed properly. The patient described inthe vignette has an auricular hematoma that should be evacuated by fine-needle aspirationbefore a clot forms.

The classic sequela of an auricular hematoma is the “cauliflower ear” commonly found inboxers, kickboxers, and wrestlers, which is caused by infiltration of fibroblasts into thehematoma and eventual neocartilage formation. Such migration can be prevented by early fine-needle evacuation of the hematoma. Larger hematomas and partially organized hematomas mayrequire an open approach. Another potential consequence of an auricular hematoma is necrosisof the cartilage due to separation of the perichondrium from the cartilage, which deprives thecartilage of its blood supply.

The clot can be evacuated by an open or closed method. Postevacuation care sometimesincludes splints or pressure dressings, but no randomized studies clearly demonstrate the needfor these additional measures or the optimal management strategy. Refractory auricularhematomas may require more aggressive treatment, such as open debridement.

Ice may be applied to the ear after evacuation of the hematoma, but it is not adequatetherapy by itself. Similarly, antibiotic prophylaxis against skin flora, including Staphylococcusaureus and group A staphylococci, is recommended, but only after hematoma evacuation.Consultation with an otorhinolaryngologist is appropriate, but the clot will have stabilized within 1to 3 days, making needle evacuation ineffective if delayed until then.

References:

Ghanem T, Rasamny JK, Park SS. Rethinking auricular trauma. Laryngoscope. 2005;115:1251-1255. Abstract available at:http://www.ncbi.nlm.nih.gov/entrez/query.fcgi?orig_db=PubMed&db=PubMed&cmd=Search&term=Laryngoscope[Jour]+AND+1251[page]+AND+2005[pdat]

Jones SEM, Mahendran S. Interventions for acute auricular haematoma. The CochraneDatabase of Systematic Reviews. 2004;2:CD004166. Available at:http://www.mrw.interscience.wiley.com/cochrane/clsysrev/articles/CD004166/frame.html

Kazahaya K, Handler SD. Otolaryngologic trauma. In: Fleisher GR, Ludwig S, Henretig FM, eds.Textbook of Pediatric Emergency Medicine. 5th ed. Philadelphia, Pa: Lippincott Williams &Wilkins; 2006:1497-1505

Copyright © 2007 by the American Academy of Pediatrics page 334

Page 335: AAP MCQ 2007

2007 PREP SA on CD-ROM

Question: 161

A 7-year-old child presents with a scalp lesion characterized by patchy areas of hair loss withpalpable suboccipital nodes (Item Q161A).

Of the following, the MOST likely diagnosis is

A. alopecia areata

B. nevus sebaceus

C. seborrheic dermatitis

D. tinea amiantacea

E. tinea capitis

Copyright © 2007 by the American Academy of Pediatrics page 335

Page 336: AAP MCQ 2007

2007 PREP SA on CD-ROM

Preferred Response: ECritique: 161

The scalp is a common site of infection by fungal dermatophytes. Organisms most frequentlyresponsible in children include Trichophyton tonsurans and Microsporum canis. T tonsurans isresponsible for 95% of infections in North America. Infection may produce one or severaldiscrete patches of hair loss (Item C161A) within which are scale and the remnants of brokenhairs within follicles (ie, black dot hairs). In some patients, infection may be more diffuse,resulting in widespread scaling (Item C161B) and less obvious alopecia (ie, the seborrheicform). Other manifestations include pustules or a tender boggy mass (ie, a kerion) (ItemC161C). The presence of enlarged suboccipital or posterior cervical lymph nodes in associationwith hair loss, as described for the child in the vignette, should raise suspicion for tinea capitis.

Alopecia areata (Item C161D) is characterized by the sudden appearance of sharplycircumscribed areas of complete hair loss. The scaling, black dot hairs, and lymphadenopathyseen in tinea capitis are not observed. Nevus sebaceus (of Jadassohn) (Item C161E) isassociated with alopecia but is present at birth as a slightly raised, oval or linear yellow-orangeplaque. Scalp involvement in seborrheic dermatitis is characterized by the accumulation ofgreasy scale; hair loss is uncommon. Tinea (pityriasis) amiantacea (Item C161F) is amanifestation of psoriasis that appears as thick, white asbestos-like scale that adheres to thehair and scalp. The thickness of the scale and absence of alopecia distinguish this disorder fromtinea capitis.

References:

Hogan PA. Papulosquamous disease: psoriasis vulgaris. In: Schachner LA, Hansen RC, eds.Pediatric Dermatology. 3rd ed. St. Louis, Mo: Mosby; 2003:643-652

Tay Y-K. Hair disorders: nonscarring alopecia. In: Schachner LA, Hansen RC, eds. PediatricDermatology. 3rd ed. St. Louis, Mo: Mosby; 2003:544-552

Weston WL, Lane AT, Morelli JG. Birthmarks. In: Color Textbook of Pediatric Dermatology. 3rded. St. Louis, Mo: Mosby; 2002:308-316

Weston WL, Lane AT, Morelli JG. Dermatitis: seborrheic dermatitis. In: Color Textbook ofPediatric Dermatology. 3rd ed. St. Louis, Mo: Mosby; 2002:39-40

Weston WL, Lane AT, Morelli JG. Fungal and yeast infections of the skin: hair infections (tineacapitis). In: Color Textbook of Pediatric Dermatology. 3rd ed. St. Louis, Mo: Mosby; 2002:63-66

Weston WL, Lane AT, Morelli JG. Hair disorders: common forms of hair loss. In: Color Textbookof Pediatric Dermatology. 3rd ed. St. Louis, Mo: Mosby; 2002:222-224

Copyright © 2007 by the American Academy of Pediatrics page 336

Page 337: AAP MCQ 2007

2007 PREP SA on CD-ROM

Question: 162

A 15-year-old girl is admitted to your facility with severe anorexia nervosa and amenorrhea. Sheweighs 35 kg and is 160 cm tall. She has bradycardia and orthostatic hypotension. You plan tostabilize her medically and begin nasogastric tube feeding.

Of the following, the electrolyte abnormality that is MOST likely to occur during the first week ofher treatment is

A. hypercalcemia

B. hyperphosphatemia

C. hypocalcemia

D. hyponatremia

E. hypophosphatemia

Copyright © 2007 by the American Academy of Pediatrics page 337

Page 338: AAP MCQ 2007

2007 PREP SA on CD-ROM

Preferred Response: ECritique: 162

The girl described in the critique is severely malnourished and about to undergo intensivenutritional rehabilitation with enteral feeding. Although this therapy is lifesaving, it also may resultin refeeding syndrome. In this syndrome, malnourished patients given oral, enteral, orintravenous nutrition develop fluid retention and electrolyte abnormalities. The most commonelectrolyte abnormality reported is hypophosphatemia, which can occur in approximately 25% ofpatients who have anorexia nervosa during refeeding. Other electrolyte and micronutrientabnormalities, including hyponatremia, hypocalcemia, hypokalemia, hyperglycemia, and thiaminedeficiency, also may occur, but are less prevalent than hypophosphatemia. Hypercalcemia andhyperphosphatemia generally do not occur. With careful monitoring of electrolytes and slowrefeeding, signs and symptoms of refeeding syndrome can be avoided. However, the clinicalmanifestations include edema, muscle weakness, and cardiac arrhythmias.

The precise mechanism by which phosphorus levels are lowered during refeeding has notyet been characterized fully. Malnourished patients are depleted in total body phosphorus,despite normal serum concentrations. The carbohydrate challenge during refeeding induces therelease of insulin, which causes fluid and electrolyte shifts. In addition, malnourished patientsbeing refed synthesize the phosphate-rich compounds creatine phosphokinase, 2,3diphosphoglycerate, and adenosine triphosphate. Thus, total body stores of phosphorus may bedepleted even further during energy synthesis.

To minimize the risk of refeeding syndrome, supplemental feedings should be introducedgradually to malnourished patients (beginning at 25% of recommended calories and advancingto full calories over 5 to 7 days). In addition, serum electrolytes, blood glucose, calcium,phosphorus, and magnesium should be measured at least daily for the first week of feeding.Many centers automatically prescribe a phosphorus supplement (500 mg twice a day) topatients who have anorexia nervosa and are being refed.

References:

Frates SE. Eating disorders. In: Hendricks KM, Duggan C, eds. Manual of Pediatric Nutrition. 4thed. Hamilton, Ontario, Canada: BC Decker; 2005:494-504

Marinella MA. Refeeding syndrome and hypophosphatemia. J Intensive Care Med. 2005;20:155-159. Abstract available at:http://www.ncbi.nlm.nih.gov/entrez/query.fcgi?cmd=Retrieve&db=pubmed&dopt=Abstract&list_uids=15888903&query_hl=2&itool=pubmed_docsum

Copyright © 2007 by the American Academy of Pediatrics page 338

Page 339: AAP MCQ 2007

2007 PREP SA on CD-ROM

Question: 163

You are present at the birth of an infant in whom bilateral hydronephrosis was diagnosed inutero. A fetal shunt was placed in each flank between the renal pelvis and the amniotic cavity.Nonetheless, the infant has bilaterally palpable flank masses, and the shunts are not apparent atbirth. The infant shows no dysmorphisms and has no respiratory distress. Renalultrasonography reveals bilateral hydronephrosis.

Of the following, the MOST correct statement regarding the fetus/neonate who has obstructiveuropathy is that

A. urinary tract infection, hydronephrosis, and respiratory distress all can be treated and resolvedwith a fetal shunt

B. urinary tract infection, hydronephrosis, and respiratory distress typically lead to fetal orneonatal death

C. urinary tract infection is common, hydronephrosis often persists, and respiratory distress isnot uncommon

D. urinary tract infection is uncommon, hydronephrosis resolves spontaneously, and respiratorydistress is uncommon

E. urinary tract infection is uncommon, hydronephrosis resolves spontaneously, and respiratorydistress results from apnea

Copyright © 2007 by the American Academy of Pediatrics page 339

Page 340: AAP MCQ 2007

2007 PREP SA on CD-ROM

Preferred Response: CCritique: 163

The infant described in the vignette has a prenatal diagnosis of hydronephrosis and palpablyenlarged kidneys on physical examination. Despite a history of shunting in utero, there are noapparent shunts present on examination. Because this is not uncommon when upper urinarytract obstruction is the problem, present techniques generally address direct drainage of renalcalyces or cysts by needle under ultrasonographic guidance.

Hydronephrosis in the fetus, which may be physiologic and transient, or pathologic, isobserved in 1 in 500 to 700 fetuses. Pathologic obstructive uropathy most frequently resultsfrom ureteropelvic junction obstruction or, less commonly, ureterovesical obstruction. In males,the most common obstructive uropathy is posterior urethral valves (Item C163A), an obstructionof the bladder outlet. Obstruction of the emptying of the renal collecting system, ureters, orbladder may result in retrograde hydrostatic pressure and destruction of the renal parenchyma.Rupture of the fetal urinary collecting system under these conditions is believed to cause urinaryascites. When an obstruction is identified in a fetus, a vesicoamniotic shunt to relieve pressureand preserve renal function may be considered. The optimal timing of delivery may bedetermined in balancing the effects of oligohydramnios and pulmonary hyperplasia againstprogressive renal parenchymal loss. Unfortunately, efforts to mitigate the obstruction andconsequential renal impairment by placing shunts have not met with great success. Fetal shuntsmay become dislodged or obstructed, the accumulation of urine continues, and their placementimposes risks of preterm labor and chorioamnionitis.

Amniotic fluid volumes may be diminished in the presence of fetal urinary tract obstruction,which may, in turn, lead to impaired fetal lung development, especially if the amniotic fluid volumeis reduced during the canalicular stage of lung development (16 to 24 weeks’ gestation).Postnatal death from pulmonary hypoplasia is well described in affected patients. Althoughcertain fetuses who have lower urologic obstructions may benefit from vesicoamniotic shunting,patient selection is difficult, and no large randomized trials have demonstrated clear benefit. Atpresent, placement is reserved for fetuses whose delivery is not imminent and who have a highrisk for pulmonary hypoplasia due to the presence of oligohydramnios.

Postnatally, infants who have obstructive uropathy are at increased risk for urinary tractinfection, vesicoureteral reflux, hydronephrosis, and possibly respiratory distress, but theseconditions do not typically lead to fetal or neonatal death. Respiratory distress may be related toimpingement of renal masses on the diaphragm, urinary ascites, or an element of pulmonaryhypoplasia. Although simple, transient, physiologic hydronephrosis may resolve spontaneously,obstructive uropathy does not.

References:

Chevalier RL. Perinatal obstructive nephropathy. Semin Perinatol. 2004;28:124-131. Abstractavailable at:http://www.ncbi.nlm.nih.gov/entrez/query.fcgi?cmd=Retrieve&db=pubmed&dopt=Abstract&list_uids=15200251&query_hl=38&itool=pubmed_docsum

Crombleholme TM. Surgical treatment of the fetus. In: Martin RJ, Fanaroff AA, Walsh MC, eds.Fanaroff and Martin’s Neonatal-Perinatal Medicine: Diseases of the Fetus and Infant. 8th ed.Philadelphia, Pa: Mosby-Elsevier; 2006:231-254

Kennedy WA II. Assessment and management of fetal hydronephrosis. NeoReviews.2002;3:e214-e219. Available at: http://neoreviews.aappublications.org/cgi/content/full/3/10/e214

Copyright © 2007 by the American Academy of Pediatrics page 340

Page 341: AAP MCQ 2007

2007 PREP SA on CD-ROM

Question: 164

During the health supervision visit of a 2-year-old boy who is new to your practice, you note aproductive cough that his mother says has been present for several months. His weight is belowthe 5th percentile. Fine crackles are present in his lungs bilaterally, and he has mild clubbing ofthe fingers. The remainder of his physical examination findings are unremarkable. A chestradiograph (Item Q164A) reveals poorly defined bronchovascular markings and a few cysticspaces bilaterally, prompting you to order computed tomography scan of the chest, whichreveals diffuse bronchiectasis.

Of the following, the MOST likely diagnosis is

A. allergic bronchopulmonary aspergillosis

B. cystic fibrosis

C. foreign body aspiration

D. Mycobacterium tuberculosis infection

E. primary ciliary dyskinesia

Copyright © 2007 by the American Academy of Pediatrics page 341

Page 342: AAP MCQ 2007

2007 PREP SA on CD-ROM

Preferred Response: BCritique: 164

Bronchiectasis is characterized by abnormal dilatation of the bronchi and bronchioles. It may belocalized or diffuse, and the changes in the bronchial tree are irreversible. The most commoncause of diffuse bronchiectasis in children is cystic fibrosis, and affected children also mayexhibit chronic diarrhea and failure to thrive. Recurrent infections and inflammation causechronic obstruction and hypertrophy of the wall of the bronchi. Less common causes of diffusebronchiectasis include allergic bronchopulmonary aspergillosis, primary ciliary dyskinesia,immunodeficiency, and infections such as tuberculosis, pertussis, and measles. Foreign bodyaspiration may lead to local bronchiectasis, but diffuse bronchiectasis is unusual. Other causesof local bronchiectasis include lobar pneumonia, right middle lobe syndrome, and extrinsiccompression by enlarged lymph nodes.

Clinical symptoms of bronchiectasis are cough productive of copious sputum, hemoptysis,and anorexia. Localized rales or wheezing may be present on physical examination. Chestradiographs typically reveal nonspecific findings such as atelectasis, focal pneumonitis, andincreased bronchovascular markings. The diagnosis may be made by high-resolution computedtomography (CT) scan of the chest, which reveals enlargement of the bronchial tree. CT scan isused to differentiate between the types of bronchiectasis, which are cylindrical, varicose, andsaccular (the most severe type). If bronchiectasis is diagnosed, an evaluation for cystic fibrosis,immunodeficiencies, and infections is warranted. In general, treatment of bronchiectasisconsists of chest physiotherapy, bronchodilators, and antibiotics.

References:

Barker AF. Bronchiectasis. N Engl J Med. 2002;346:1383-1393

Davis PB. Cystic fibrosis. Pediatr Rev. 2001;22:257-264. Available at:http://pedsinreview.aappublications.org/cgi/content/full/22/8/257

Lakser O. Bronchiectasis. In: Behrman RE, Kliegman RM, Jenson HB, eds. Nelson Textbook ofPediatrics. 17th ed. Philadelphia, Pa: WB Saunders Co; 2004:1436

Copyright © 2007 by the American Academy of Pediatrics page 342

Page 343: AAP MCQ 2007

2007 PREP SA on CD-ROM

Question: 165

A child presents for her 2-week evaluation after being delivered by a midwife at home. Theparents are concerned that they have never seen her turn her head, which makes it difficult forher to feed at the breast. They also note that her back does not appear normal. On physicalexamination, her hairline appears low posteriorly. You confirm that she does not turn her head,and when placed prone, does not turn her head to the side. Her right scapula appears to behigher than the left, and you note that the spine does not appear to be perfectly straight,suggesting congenital scoliosis. You obtain an anteroposterior radiograph of the cervical spine,which shows multilevel segmentation anomalies (Item Q165A) in the mid to lower cervical spine.

Of the following, the condition that is MOST consistent with these findings is

A. cleidocranial dysostosis

B. Klippel-Feil syndrome

C. Noonan syndrome

D. Sandifer syndrome

E. Turner syndrome

Copyright © 2007 by the American Academy of Pediatrics page 343

Page 344: AAP MCQ 2007

2007 PREP SA on CD-ROM

Preferred Response: BCritique: 165

The Klippel-Feil syndrome involves the fusion of cervical vertebrae (Item C165A) and occurs inapproximately 1 in 42,000 births, with a 65% female predominance. It is usually a sporadicevent. Due to neck immobility, affected individuals are at risk of cervical spine injury. Associateddefects may include deafness (conduction or sensorineural, occurring in up to 30% of patients),congenital heart defects (usually ventricular septal defect), rib defects, hemivertebrae, Sprengelanomaly (elevation of the scapula), scoliosis, and renal anomalies.

Noonan and Turner syndrome may be associated with a “webbed” neck, but the spine is notinvolved. Sandifer syndrome refers to the tonic extension of the neck associated withgastroesophageal reflux and is functional rather than anatomic. Cleidocranial dysostosispresents with absence of the clavicles, yielding a “droopy chest” rather than the elevatedscapula and shoulders that appear in Klippel-Feil syndrome.

References:

Klippel-Feil sequence. In: Jones KL. Smith’s Recognizable Patterns of Human Malformations. 6thed. Philadelphia, Pa: Elsevier Saunders; 2006:716-718

Weaver DD, Wheeler PG. Skeletal dysplasias. In: McMillan JA, DeAngelis CD, Feigin RD,Warshaw JB, eds. Oski’s Pediatrics: Principles and Practice. Philadelphia, Pa: LippincottWilliams & Wilkins; 1999:2134-2154

Copyright © 2007 by the American Academy of Pediatrics page 344

Page 345: AAP MCQ 2007

2007 PREP SA on CD-ROM

Question: 166

A mother brings her 12-month-old boy to you because he holds his head tilted to the right (ItemQ166A). She tells you that he periodically draws up both his legs and cries. The child hasdeveloped normally, but does not yet walk or cruise. On physical examination, he has mildlyincreased deep tendon reflexes in the upper and lower extremities, but other findings are normal.

Of the following, the study MOST likely to establish this boy's diagnosis is

A. audiometry

B. electroencephalography

C. magnetic resonance imaging of the brain and cervical spine

D. pH probe of the distal esophagus

E. radiographs of the cervical spine

Copyright © 2007 by the American Academy of Pediatrics page 345

Page 346: AAP MCQ 2007

2007 PREP SA on CD-ROM

Preferred Response: CCritique: 166

A head tilt commonly is produced by muscular torticollis, but the examiner always shouldconsider a number of other disorders that can produce a twisted neck.

Muscular torticollis results from bleeding within the sternocleidomastoid muscle shortly afterbirth. Swelling can be palpated within the muscle. The head is tipped toward the injured muscle,and the chin is rotated toward the other. In older children, muscular torticollis can result fromminor trauma or inflammation of the cervical muscles. However, a high index of suspicion shouldbe maintained for other processes producing head tilt.

Neurologic processes causing head tilt are perhaps the most worrisome. Posterior fossa orcervical cord tumor (Item C166A) should be considered, along with syringomyelia, dystonia(from drug or dystonia musculorum deformans), or visual disturbance (trochlear nerve palsymore commonly than abducens or oculomotor nerve paresis). Congenital disorders such ashemivertebrae and Klippel-Feil syndrome can masquerade as muscular torticollis in thenewborn. Other causes of head tilt include atlantoaxial rotary subluxation, lymphadenitis,Sandifer syndrome (gastroesophageal reflux), spasmus nutans, and paroxysmal torticollis.

Paroxysmal torticollis is a migraine variant with onset in the first year after birth. Infantsexperience recurrent attacks of head tilt for minutes, sometimes with pallor, vomiting, oragitation. Episodes cease by age 2 to 3 years. No treatment is indicated, although affectedchildren often develop migraines later in childhood.

The 12-month-old boy described in the vignette is on the borderline of being delayed incruising. Although his periodic withdrawal of arms and legs could be interpreted as due to reflux,such behavior also is seen with infantile spasms or cervical spine pain. The increased deeptendon reflexes point to an upper motor neuron process. Magnetic resonance imaging of thebrain and cervical spine is warranted, particularly to exclude a tumor; radiographs wouldvisualize only the bony anatomy. A pH probe would be useful only if gastroesophageal refluxwere more likely. Audiometry can exclude hearing loss causing a head tilt, andelectroencephalography can exclude seizures with head posturing.

References:

Robin NH. In brief: congenital muscular torticollis. Pediatr Rev. 1996;19:374-375

Thompson GH. The neck: torticollis. In: Behrman RE, Kliegman RM, Jenson HB, eds. NelsonTextbook of Pediatrics. 17th ed. Philadelphia, Pa: WB Saunders Co; 2004:2288-2289

Copyright © 2007 by the American Academy of Pediatrics page 346

Page 347: AAP MCQ 2007

2007 PREP SA on CD-ROM

Question: 167

You are seeing a 6-week-old infant who was born with trisomy 21 and a large atrioventricularseptal defect. Over the previous week, she has tired with feeding and has not gained weight.Her respiratory rate is 60 breaths/min and heart rate is 150 beats/min. Auscultation reveals mildretractions and a 2/6 systolic murmur with a gallop rhythm. The liver is palpable at 2 cm belowthe costal margin, and the perfusion is good. You decide to increase the caloric content of theformula to 24 kcal/oz, and you contact her pediatric cardiologist to discuss referral for surgicalrepair.

Of the following, the BEST therapeutic option while awaiting surgical repair is

A. captopril

B. furosemide

C. hydralazine

D. propranolol

E. verapamil

Copyright © 2007 by the American Academy of Pediatrics page 347

Page 348: AAP MCQ 2007

2007 PREP SA on CD-ROM

Preferred Response: BCritique: 167

Congestive heart failure (CHF) is a clinical syndrome that reflects the inability of the heart tomeet the metabolic requirements of the body, including those for growth. Congenital heartdefects are the most common reason for pediatric heart failure, and the failure develops mostfrequently during early infancy. CHF results from excessive workload imposed on the cardiacmuscle, usually caused by structural defects. The cardiac defects may impose an excessivevolume load on the left ventricle (eg, large ventricular septal defect, atrioventricular septaldefect), an excessive pressure load on the ventricle (eg, aortic stenosis), or a combination ofvolume and pressure load (eg, ventricular septal defect and coarctation). Less commonly, CHFmay result from an intrinsic alteration in myocardial performance, which could be due to aninflammatory or an infectious process directly affecting the myocardium and depressing itscontractile function.

The child described in the vignette presents with symptoms that have been developing overweeks and are typical of progressive CHF in a patient who has a large-volume shunting lesionsuch as an atrioventricular septal defect. Poor feeding results from the infant’s inability togenerate a prolonged suck while maintaining nasal breathing. This is complicated further bytachypnea caused by pulmonary congestion. Decreased caloric intake coupled with increasedcaloric expenditure due to tachypnea and tachycardia makes it difficult for the infant to gainweight. Indeed, weight loss is common. The murmur in this infant is consistent with mitralregurgitation, which often accompanies a complete atrioventricular septal defect (strongassociation with a cleft in the mitral valve). This defect leads to left-to-right shunting at the atrialand ventricular levels and is one of the most common forms of congenital heart disease in thechild who has Down syndrome. A gallop rhythm, usually an S3, results from a volume-loaded,dilated, stiff left ventricle. Liver enlargement reflects increased filling pressures in the right heart.

Treatment of CHF resulting from a left-to-right volume load in the infant is directed at diuresisto address the volume overload on the pulmonary circuit and left heart. Furosemide, a well-tolerated and effective loop diuretic, can provide the necessary diuresis. With some unloading ofthe lungs and volume of the left heart, feeding may improve, and caloric intake may increase.Although medical treatment may provide some immediate relief, patients who have CHF affectingtheir growth should be considered for surgical repair of the defect. Even when diuresis does notyield a significant improvement in symptoms, it may be useful in improving lung functionpreoperatively.

Afterload reduction with captopril can be effective in the management of CHF, but it does notprovide the same level of symptomatic improvement as diuresis. Hydralazine, anantihypertensive and vasodilator, generally is not used for the management of CHF in the infant.It can lead to sinus tachycardia that can exacerbate left heart filling and compound CHF. Theuse of beta blockers in CHF resulting from myocardial dysfunction may be warranted, but theygenerally are not indicated for patients who have large-volume shunting lesions. Verapamil, acalcium channel blocker, is contraindicated in infants younger than 1 year of age.

References:

Balfour I. Management of chronic congestive heart failure in children. Curr Treat OptionsCardiovasc Med. 2004;6:407-416. Abstract available at:http://www.ncbi.nlm.nih.gov/entrez/query.fcgi?orig_db=PubMed&db=PubMed&cmd=Search&term=%22Current+treatment+options+in+cardiovascular+medicine%22[Jour]+AND+2004[pdat]+AND+Balfour+I[author]

Dreyer WJ, Fisher DJ. Clinical recognition and management of chronic congestive cardiacfailure. In: Garson A Jr, Bricker JT, Fisher DJ, Neish SR, eds. The Science and Practice ofPediatric Cardiology. 2nd ed. Baltimore, Md: Williams & Wilkins; 1998:2309-2325

Talner NS, McGovern JJ, Carboni MP. Congestive heart failure. In: Moller JH, Hoffman JIE, eds.Pediatric Cardiovascular Medicine. Philadelphia, Pa: Churchill Livingstone; 2000:817-829

Copyright © 2007 by the American Academy of Pediatrics page 348

Page 349: AAP MCQ 2007

2007 PREP SA on CD-ROM

Question: 168

Numerous therapeutic agents are known to have teratogenic effects on the developing fetus.

Of the following, the findings in the newborn that are MOST suggestive of prenatal exposure toan angiotensin-converting enzyme inhibitor are

A. deafness and cataracts

B. microtia and conotruncal malformation

C. nasal hypoplasia and stippled epiphyses

D. neonatal anuria and patent ductus arteriosus

E. smooth philtrum and lip

Copyright © 2007 by the American Academy of Pediatrics page 349

Page 350: AAP MCQ 2007

2007 PREP SA on CD-ROM

Preferred Response: DCritique: 168

Angiotensin-converting enzyme (ACE) inhibitors are used in the treatment of hypertension andthe management of heart failure. Studies of this class of agents in pregnant animals haverevealed an increased incidence of renal defects and fetal death. In humans, the use of ACEinhibitors throughout pregnancy or during the second and third trimesters has been associatedwith a number of complications, including fetal oligohydramnios and neonatal anuria due topersistent inhibition of the reninangiotensin system as well as fetal hypotension with subsequentpoor perfusion of tissues. Patent ductus arteriosus also has been reported and has beenhypothesized to result from the effect of these agents on kinase II, which increases productionof prostaglandin. Hypoplasia of the skull, spontaneous abortion, and intrauterine demise alsohave been reported. Thus, ACE inhibitors should not be used during pregnancy.

Microtia and conotruncal malformations are associated with prenatal exposure to retinoicacid. Nasal hypoplasia and stippled epiphyses are characteristic of the fetal warfarin syndrome,which results from first-trimester exposure to coumarin derivatives. Deafness and cataracts(Item C168A) are associated with rubella infections during the first trimester of pregnancy.Smooth philtrum and a thin upper lip are part of the facial characteristics of infants who have fetalalcohol syndrome (Item C168B).

References:

August P, Rose BD. Angiotensin converting enzyme inhibitors and receptor blockers inpregnancy. UpToDate. 2006;14.1. Available at:http://www.utdol.com/utd/content/topic.do?topicKey=hyperten/6229&type=A&selectedTitle=9~212

Pietrement C, Malot L, Santerne B, Roussel B, Motte J, Morville P. Neonatal acute renal failuresecondary to maternal exposure to telmisartan, angiotensin II receptor antagonist. J Perinatol.2003;23:254-255. Abstract available at:http://www.ncbi.nlm.nih.gov/entrez/query.fcgi?orig_db=PubMed&db=PubMed&cmd=Search&term=%22Journal+of+perinatology+:+official+journal+of+the+California+Perinatal+Association%22[Jour]+AND+2003[pdat]+AND+Pietrement+C[author]

Copyright © 2007 by the American Academy of Pediatrics page 350

Page 351: AAP MCQ 2007

2007 PREP SA on CD-ROM

Question: 169

The director of a community after-school program for adolescent boys is organizing anorientation session for new mentors. He wants some information about current substance abusetrends.

Of the following, you are MOST likely to report that among adolescents,

A. athletes who use performance-enhancing substances are unlikely to use other illicit drugs

B. daily cigarette smoking by 8th graders has increased steadily in the last decade

C. homosexual youth are the least likely to engage in the use of alcohol and marijuana

D. inhalant abuse is more prevalent among students in the 8th grade than the 12th grade

E. marijuana use among 12th graders has decreased steadily since 1980

Copyright © 2007 by the American Academy of Pediatrics page 351

Page 352: AAP MCQ 2007

2007 PREP SA on CD-ROM

Preferred Response: DCritique: 169

The overall use of drugs of abuse (alcohol, tobacco, illicit drugs) among high school students inthe United States was at its highest during the late 1970s. Although substance abuse declinedduring the ensuing decade, the trend was reversed by increases in usage through much of the1990s. More recently, from 2001 to 2005, there has been a gradual decline in the use of mostdrugs of abuse. Nevertheless, substance abuse remains a major public health concern.

Two periodic surveys track national trends in use of alcohol, tobacco, and other drugs byhigh school students and have Internet sites that report their data: Monitoring the Future (MTF)and the Youth Risk Behavior Survey. Data are reported as the prevalence of substance use andpercentage of sample reporting use or percentage having used during a given period (eg,lifetime [ever used], use in past year [annual], or use in the past 30 days [30 day prevalence]).

During the 2005 MTF survey, 50% of 12th graders and more than one in five of 8th gradersreported they had tried (lifetime prevalence) an illicit drug. (Item C169A) lists the percentage of8th and 12th graders who reported use in the past 30 days of alcohol, cigarettes (tobacco), anyillicit drug, marijuana, and amphetamines. Inhalant abuse, unlike most substance abuse, is moreprevalent among students in the 8th than the 12th grade (Item C169B). Alcohol remained themost widely used drug among high school students. Of the 12th graders surveyed, 47%reported alcohol use in the past 30 days, reflecting a continued slow decline from 54% in 1991.The prevalence of binge drinking (having consumed five or more drinks in a row at least once inthe prior 2 weeks), however, has shown little change and was reported by 28% of 12th graders.

Cigarette smoking trends for 8th and 12th graders have shown a decline in use during thepast decade. Item (C169C) shows the decline in 30-day and daily prevalence reported by 8thgraders during the 1996 and 2005 MTF surveys.

The annual prevalence of marijuana use among 12th graders declined during the 1980s, butwas followed by increases in the 1990s. More recently, there has been a gradual decrease inthe annual prevalence, but little change in the 30-day prevalence of marijuana use (Item C169D).

Individual risk factors associated with adolescent substance abuse include low self-esteem,poor coping skills, depression, anxiety, alienation from conventional norms, and impulsivity.Homosexual adolescents have rates of substance abuse (including alcohol and marijuana) thatare significantly higher than heterosexual peers. The use of performance-enhancing substancesincreases the likelihood of use of other illicit drugs.

References:

Johnston LD, O’Malley PM, Bachman JG, Schulenberg JE. Monitoring the Future: NationalResults on Adolescent Drug Use: Overview of Key Findings, 2005. Bethesda, Md: NationalInstitute on Drug Abuse; 2006. Available at:http://www.monitoringthefuture.org/pubs/monographs/overview2005.pdf

Kulig JW, American Academy of Pediatrics Committee on Substance Abuse. Tobacco, alcohol,and other drugs: the role of the pediatrician in prevention, identification, and management ofsubstance abuse. Pediatrics. 2005;115:816-821. Available at:http://pediatrics.aappublications.org/cgi/content/full/115/3/816

Monitoring the Future Web Site. Available at: www.monitoringthefuture.org

National Survey on Drug Use and Health. Available at:http://www.drugabusestatistics.samhsa.gov/nsduh.htm

Youth Risk Behavior Surveillance—United States, 2003. MMWR Morbid Mortal Wkly Rep SurvSumm. 2004;53:No. SS-2. Available at: http://www.cdc.gov/mmwr/PDF/SS/SS5302.pdf

Copyright © 2007 by the American Academy of Pediatrics page 352

Page 353: AAP MCQ 2007

2007 PREP SA on CD-ROM

Question: 170

You are evaluating a 13-year-old boy who has a body mass index of 32 kg/m2 and a bloodpressure of 140/80 mm Hg. Initial laboratory studies reveal fasting serum triglyceride levels of180 mg/dL (2.0 mmol/L) (normal, 40 to 100 mg/dL [0.45 to 1.1 mmol/L]). You diagnose metabolicsyndrome.

Of the following, the MOST appropriate evaluation in addition to fasting blood glucose is

A. erythrocyte sedimentation rate

B. 5-hour oral glucose tolerance test

C. Holter monitoring

D. liver transaminases

E. serum alpha-fetoprotein

Copyright © 2007 by the American Academy of Pediatrics page 353

Page 354: AAP MCQ 2007

2007 PREP SA on CD-ROM

Preferred Response: DCritique: 170

The concept of “metabolic syndrome” as a unifying term for people who have obesity,hypertension, insulin resistance, and dyslipidemia composed of low high-density lipoprotein andelevated triglyceride concentrations is presently being questioned. Nonetheless, 20% to 40% ofindividuals who are obese and exhibit insulin resistance have nonalcoholic fatty liver disease(nonalcoholic steatohepatitis). Elevated transaminases associated with a mild hepaticinflammatory process in such patients can progress over time to frank cirrhosis.

The best treatment for children who have this disorder is weight loss, but trials areunderway to test the effect of drugs that decrease insulin resistance, such as metformin andthiazolidinediones such as rosiglitazone or pioglitazone. The latter two drugs are not approvedby the United States Food and Drug Administration for children.

The components of “metabolic syndrome” are common in obese children, but the risk fortype 2 diabetes is quite low. A 5-hour glucose tolerance test never is indicated for the diagnosisof diabetes; the glucose response 2 hours after oral glucose administration is sufficient to makethe diagnosis. This test is indicated if the fasting blood glucose is between 100 mg/dL and 126mg/dL (5.5 and 7 mmol/L). A fasting plasma glucose of 126 mg/dL (7 mmol/L) or greater isdiagnostic of diabetes using American Diabetes Association criteria. Holter monitoring,nonspecific measures of inflammation such as erythrocyte sedimentation rate and measurementof serum alpha-fetoprotein, a tumor marker, are not indicated.

References:

Cruz ML, Shaibi GQ, Weigensberg MJ, Spruijt-Metz D, Ball GD, Goran MI. Pediatric obesityand insulin resistance: chronic disease risk and implications for treatment and preventionbeyond body weight modification. Annu Rev Nutr. 2005;25:435-468. Available at:http://www.ncbi.nlm.nih.gov/entrez/query.fcgi?orig_db=PubMed&db=PubMed&cmd=Search&term=%22Annual+review+of+nutrition%22[Jour]+AND+435[page]+AND+2005[pdat]

Mandato S, Lucariello M, Licenziati MR, et al. Metabolic, hormonal, oxidative, and inflammatoryfactors in pediatric obesity-related liver disease. J Pediatr. 2005;147:62-66. Abstract available at:http://www.ncbi.nlm.nih.gov/entrez/query.fcgi?cmd=Retrieve&db=pubmed&dopt=Abstract&list_uids=16027697&query_hl=75&itool=pubmed_docsum

Schwimmer JB, McGreal N, Deutsch R, Finegold MJ, Lavine JE. Influence of gender, race, andethnicity on suspected fatty liver in obese adolescents. Pediatrics. 2005;115:e561-e565.Available at: http://pediatrics.aappublications.org/cgi/content/full/115/5/e561

Copyright © 2007 by the American Academy of Pediatrics page 354

Page 355: AAP MCQ 2007

2007 PREP SA on CD-ROM

Question: 171

You are evaluating a 6-year-old boy who has Duchenne muscular dystrophy. He is doing well ina regular classroom and will be attending second grade in a different school next year. Onphysical examination, you note a healthy-appearing boy who has pseudohypertrophy of the calfmuscles and uses a Gower maneuver (Item Q171A) to rise from the floor.

In gathering information to help this child's transition to a new school, you are MOST likely to askabout

A. augmented communication resources

B. recent pulmonary function testing

C. signs of sleep apnea

D. the number of floors in the school

E. wheelchair use

Copyright © 2007 by the American Academy of Pediatrics page 355

Page 356: AAP MCQ 2007

2007 PREP SA on CD-ROM

Preferred Response: DCritique: 171

The time of diagnosis and medical, social, and educational transitions are common stress pointsfor parents in the life of their child who has a developmental disability. The need for increasingmedical equipment and assistance, such as braces, a wheelchair, or a gastrostomy tube, maycause significant stress. Similarly, entering kindergarten, middle school, junior high school, andhigh school each present challenges. A child likely needs re-evaluation of school servicesprovided at those times and may require significant testing or accommodations for differingphysical facilities.

For the ambulatory boy in the vignette, the most likely difficulty he will encounter isnumerous stairs in a multilevel school that may be tiring for him to climb and descend. He ishealthy and is unlikely to have problems with sleep apnea or need pulmonary function tests untilhis disease has progressed further. Likewise, at his present level of function, he does not needa wheelchair or communication device. However, most children who have Duchenne musculardystrophy need educational support for related learning disabilities that may manifest over time.

References:

Perrin JM. Chronic illness in childhood. In: Behrman RE, Kliegman RM, Jenson HB, eds. NelsonTextbook of Pediatrics. 17th ed. Philadelphia, Pa: WB Saunders Co; 2004:135-138

Plauché Johnson C, Kastner TA, and the Committee/Section on Children With Disabilities.Helping families raise children with special health care needs at home. Pediatrics. 2005;115:507-511. Available at: http://pediatrics.aappublications.org/cgi/content/full/115/2/507

Copyright © 2007 by the American Academy of Pediatrics page 356

Page 357: AAP MCQ 2007

2007 PREP SA on CD-ROM

Question: 172

An 11-day-old infant presents to the clinic with a history of a temperature of 100.8°F (38.2°C)and a 1-day history of poor feeding. Findings on physical examination are normal. You initiate asepsis evaluation that includes a lumbar puncture. The cerebrospinal fluid results demonstrate awhite blood cell count of 6x103/mcL (6x109/L), with 68% neutrophils, 2% bands, and 30%lymphocytes. The protein concentration is 200 mg/dL (2 g/L), and the glucose value is 36 mg/dL(2.0 mmol/L). The abnormal findings prompt you to order magnetic resonance imaging, whichdemonstrates abnormal frontal lobes bilaterally that includes some degree of infarction but alsoabscesses and cerebritis (Item Q172A).

Of the following, the MOST likely pathogen is

A. Citrobacter koseri

B. Escherichia coli

C. Klebsiella pneumoniae

D. Listeria monocytogenes

E. Streptococcus agalactiae

Copyright © 2007 by the American Academy of Pediatrics page 357

Page 358: AAP MCQ 2007

2007 PREP SA on CD-ROM

Preferred Response: ACritique: 172

Organisms from the genus Citrobacter cause a chronic form of bacterial meningitis in theneonate that leads to brain abscess formation in approximately 80% of infected infants. Theabscesses tend to be large multiloculated areas that can be difficult to treat. The exactmechanism for the invasiveness of Citrobacter and its propensity to cause brain abscesses isunknown.

Citrobacter are gram-negative enteric rods closely related to Salmonella that include at least11 species, the most common of which are C freundii and C koseri. The organisms can bepassed vertically from mother to infant at delivery, but colonization from nosocomial sources(eg, hand contamination, contaminated infant formula) has been described. Most organismsfrom the genus Citrobacter are resistant to ampicillin but susceptible to the later-generationcephalosporins and aminoglycosides. Treatment of brain abscesses (Item C172A) usuallyinvolves long-term antimicrobial therapy (4 to 6 weeks after sterilization of the cerebrospinalfluid) with a combined regimen that includes a third- or fourth-generation cephalosporin and anaminoglycoside. The abscess area frequently must be drained, and follow-up cranial imaging isused to help assess the duration of therapy. Patients who have multiloculated abscess usuallyhave significant developmental problems.

Although Escherichia coli, Klebsiella pneumoniae, Listeria monocytogenes, or Streptococcusagalactiae (group B Streptococcus) can cause a brain abscess, Citrobacter is most likely to beresponsible for the indolent course described for the neonate in the vignette that includes brainabscess formation. Similar to Citrobacter, Enterobacter sakazakii causes neonatal meningitiswith brain abscess formation, but it is encountered less frequently than Citrobacter.

References:

Boyce TG, Gruber WC, Fisher RG. Citrobacter. In: Feigin RD, Cherry JD, Demmler GJ, KaplanSL, eds. Textbook of Pediatric Infectious Diseases. 5th ed. Philadelphia, Pa: WB Saunders Co;2004:1423-1426

Goodkin HP, Harper MB, Pomeroy SL. Intracerebral abscess in children: historical trends atChildren’s Hospital Boston. Pediatrics. 2004;113:1765-1770. Available at:http://pediatrics.aappublications.org/cgi/content/full/113/6/1765

Copyright © 2007 by the American Academy of Pediatrics page 358

Page 359: AAP MCQ 2007

2007 PREP SA on CD-ROM

Question: 173

You are evaluating a 10-year-old boy who has intermittent urinary incontinence. Voidingcystourethrography detects a urethral stricture.

Of the following, the MOST likely cause of this boy's urethral stricture is

A. carcinoma

B. chronic infection

C. congenital narrowing

D. intermittent urolithiasis

E. trauma

Copyright © 2007 by the American Academy of Pediatrics page 359

Page 360: AAP MCQ 2007

2007 PREP SA on CD-ROM

Preferred Response: ECritique: 173

Urethral stricture disease involves blockage or narrowing of the urethra that causes reducedurine flow during voiding. Most urethral strictures occur in males. The most common causes inchildren are previous trauma (eg, posterior urethral injury occurring as a consequence of pelvicfracture) or instrumentation of the urethra, infection (eg, gonorrhea), congenital anomaly,idiopathic, and as a complication of balanitis (balanitis xerotica obliterans).

Children who have urethral strictures can develop a variety of signs and symptoms,including straining to urinate, a decrease in the size and force of the urine stream, persistentsense of bladder fullness, urine dribbling, and frequency and urgency of urination. Overtphysical examination findings are generally absent. Diagnosis is confirmed by cystoscopy.Alternatively, voiding cystourethrography can be performed to determine the extent of thestricture or ultrasonography may be performed.

Treatment of a urethral stricture depends on the length, location, and persistence of thestricture. In general, initial urethral strictures that are short (<1.5 cm) can be treated with eitherendoscopic incision or dilation. Although this form of treatment has only a modest success rate,it is minimally invasive and associated with few procedure-related complications. Recurrentstrictures or strictures longer than 2 cm usually are not amenable to endoscopic incision ordilation, and this form of therapy proves to be a "temporizing" measure without much long-termsuccess. Severe urethral stricture in males may damage the bladder and causehydronephrosis.

Carcinoma of the urethra is rare in children, and the boy described in the vignette has nohistory of chronic infection. Moreover, chronic gonorrhea is uncommon at this age. The boy’sage effectively eliminates congenital narrowing as a cause; symptoms usually develop in earlychildhood. Chronic intermittent urolithiasis is a very rare cause of urethral stricture, especially inpatients who have no history of chronic abdominal or flank pain.

References:

Farhat W, McLorie G. Urethral syndromes in children. Pediatr Rev. 2001;22:17-21. Available at:http://pedsinreview.aappublications.org/cgi/content/full/22/1/17

Koraitim MM. Post-traumatic posterior urethral strictures: preoperative decision making.Urology. 2004;64:228-231. Abstract available at:http://www.ncbi.nlm.nih.gov/entrez/query.fcgi?orig_db=PubMed&db=PubMed&cmd=Search&term=%22Urology%22[Jour]+AND+228[page]+AND+2004[pdat]

Sugimoto M, Kakehi Y, Yamashita M, Matsuki T, Inui M, Taketa S. Ten cases of congenitalurethral stricture in childhood with enuresis. Int J Urol. 2005;12:558-562. Abstract available at:http://www.ncbi.nlm.nih.gov/entrez/query.fcgi?orig_db=PubMed&db=PubMed&cmd=Search&term=%22International+journal+of+urology+:+official+journal+of+the+Japanese+Urological+Association%22[Jour]+AND+558[page]+AND+2005[pdat]

Copyright © 2007 by the American Academy of Pediatrics page 360

Page 361: AAP MCQ 2007

2007 PREP SA on CD-ROM

Question: 174

A 3-year-old boy who has myelomeningocele and a history of recurrent urinary tract infectionspresents with a 1-day history of a temperature to 102ºF (38.9ºC) and cloudy urine. Laboratorytest results include a peripheral white blood cell count of 15x103/mcL (15x109/L), with 60%neutrophils, 30% lymphocytes, and 10% monocytes. Urine obtained by catheterization is cloudy;has a strong odor; and is positive for nitrites, leukocyte esterase, and blood. Microscopicanalysis shows too numerous-to-count white blood cells and 50 to 100 red blood cells, and gram-negative bacilli are seen on Gram stain. One day later, the urine culture is positive forPseudomonas aeruginosa.

Of the following, the MOST appropriate antibiotic for treatment of this patient is

A. ampicillin

B. ceftazidime

C. cefuroxime

D. trimethoprim-sulfamethoxazole

E. vancomycin

Copyright © 2007 by the American Academy of Pediatrics page 361

Page 362: AAP MCQ 2007

2007 PREP SA on CD-ROM

Preferred Response: BCritique: 174

Pseudomonas sp are gram-negative bacilli that are primarily waterborne and soilborneorganisms, have relatively low virulence, and are associated with a wide variety of infections. Ofthe pseudomonal species, P aeruginosa is the most common. It is classified as an opportunisticpathogen infrequently causing disease in healthy hosts, but it is a major cause of infection inpatients who have underlying conditions. Osteomyelitis or cellulitis associated with a puncturewound and urinary tract infections are the most common diseases caused by this organism inthe otherwise healthy host. Both of these conditions can be treated with a single antibiotic agentsuch as a fluoroquinolone (ciprofloxacin) for oral therapy or piperacillin, piperacillin/tazobactam,ceftazidime, a fluoroquinolone, or an aminoglycoside parenterally. Combination therapy ofpiperacillin or ceftazidime plus an aminoglycoside is also appropriate.

Serious P aeruginosa infections are more likely in patients who have underlying conditions,such as bacteremia, sepsis, and pneumonia (especially in patients who have cystic fibrosis).For these patients, therapy usually consists of two anti-Pseudomonas agents, such as a beta-lactam antibiotic (piperacillin, piperacillin/tazobactam, or ceftazidime) plus an aminoglycoside.Imipenem and ciprofloxacin also may be used successfully as single agents.

The treatment of pneumonia usually includes the beta-lactam antibiotics ticarcillin,ticarcillin/clavulanic acid, piperacillin, piperacillin/tazobactam, or ceftazidime, usually incombination with an aminoglycoside. Combination therapy often is used to ensure treatment ofresistant strains and to prevent selection of resistant mutants. The carbapenem class ofantibiotics (imipenem or meropenem) and the monobactam antibiotic aztreonam generally arereserved for the treatment of serious infections caused by organisms resistant to the other beta-lactam antibiotics or for patients who have moderate renal disease and are at risk foraminoglycoside-related nephrotoxicity.

Because pseudomonal species are found widely in the environment, it is important todistinguish between true infection and colonization of a surface because inappropriate use ofantibiotics does not treat colonization and predisposes the patient to developing a resistantorganism. In both situations, cultures are positive, but it is important to look at other factorsrelated to the patient’s clinical condition. A positive culture is much more likely to representcolonization if a patient’s clinical condition is unchanged or improving and there is no evidence ofinfection on physical examination or laboratory testing. Two examples of probable colonizationare: 1) a positive endotracheal tube culture in patient who is improving on a ventilator with fewerthan 25 white blood cells on Gram stain and no change in amount or color of secretions, and 2)a patient who has a positive urine culture but no change in urine color or odor, urinalysisnegative for nitrites and leukocyte esterase, and no findings on physical examination.

The patient in the vignette has an underlying condition that predisposes him to urinary tractinfections. Physical examination and laboratory findings indicate that he has a urinary tractinfection caused by P aeruginosa. Ceftazidime is the only agent listed that has activity againstthis organism.

References:

Berthelot P, Grattard F, Mahul P, et al. Prospective study of nosocomial colonization andinfection due to Pseudomonas aeruginosa in mechanically ventilated patients. Intensive CareMed. 2001;27:503-512. Abstract available at:http://www.ncbi.nlm.nih.gov/entrez/query.fcgi?orig_db=PubMed&db=PubMed&cmd=Search&term=%22Intensive+care+medicine%22[Jour]+AND+503[page]+AND+2001[pdat]

Giamarellou H, Antoniadou A. Antipseudomonal antibiotics. Med Clin North Am. 2001;85:19-42.Abstract available at:http://www.ncbi.nlm.nih.gov/entrez/query.fcgi?orig_db=PubMed&db=PubMed&cmd=Search&term=%22The+Medical+clinics+of+North+America%22[Jour]+AND+19[page]+AND+2001[pdat]

Prince AS. Pseudomonas aeruginosa. In: Long SS, Pickering LK, Prober CG, eds. Principlesand Practice of Pediatric Infectious Diseases. 2nd ed. New York, NY: Churchill Livingstone;

Copyright © 2007 by the American Academy of Pediatrics page 362

Page 363: AAP MCQ 2007

2007 PREP SA on CD-ROM

2003:857-860

Copyright © 2007 by the American Academy of Pediatrics page 363

Page 364: AAP MCQ 2007

2007 PREP SA on CD-ROM

Question: 175

A 7-year-old boy presents with a 3-year history of seasonal rhinorrhea and nasal congestion.He states that "as soon as the weather warms up," he experiences daily nasal symptoms thatare often severe enough to limit his outdoor activity. On physical examination, you noteinfraorbital swelling, a transverse nasal crease, boggy turbinates, and clear rhinorrhea. A nasalsmear demonstrates numerous eosinophils. You decide to treat him with a nasal spray.

Of the following, the MOST effective long-term treatment for this boy is a nasal spray containinga(n)

A. anticholinergic

B. corticosteroid

C. decongestant

D. mast cell stabilizer

E. saline solution

Copyright © 2007 by the American Academy of Pediatrics page 364

Page 365: AAP MCQ 2007

2007 PREP SA on CD-ROM

Preferred Response: BCritique: 175

The boy described in the vignette presents with classic symptoms of allergic rhinitis. Theapproach to medical management of allergic rhinitis should take into account the patient’sprimary symptoms, past experiences with medications, and any coexisting medical problems.Nasal corticosteroids have become a first-line therapy for allergic rhinitis, with studiesdemonstrating improvement in nasal pruritus, sneezing, nasal obstruction, and rhinorrhea.Further, nasal steroids typically are well tolerated and have minimal adverse effects.Anticholinergic nasal sprays are approved for nonallergic rhinitis and are suited best fortreatment of vasomotor rhinitis and atrophic rhinitis, conditions more commonly seen in adults.Nasal decongestants often are purchased without a prescription and markedly reduce nasalcongestion. However, recurrent or prolonged use of nasal decongestants can result in rhinitismedicamentosa, a condition of rebound nasal congestion. Nasal cromolyn is approved forallergic rhinitis, although it must be taken several times a day and has been shown to be inferiorto nasal steroids for treatment of allergic rhinitis. Nasal saline should be considered a routineadjunct for nasal irrigation, but it is generally considered inferior to nasal corticosteroids for thetreatment of allergic rhinitis.

The use of other medications to treat allergic rhinitis is based on patient symptoms.Sneezing, nasal pruritus, and ocular itching often can be managed with an oral antihistamine. Anophthalmic antihistamine or vasoconstricting eye drop can be added when ocular symptoms donot improve with oral antihistamines alone. Nasal antihistamines also have been approved forallergic rhinitis.

References:

Atkins D, Leung DYM. Principles of treatment of allergic disease. In: Behrman RE, Kliegman RM,Jenson HB, eds. Nelson Textbook of Pediatrics. 17th ed. Philadelphia, Pa: WB Saunders Co2004:752-758

Mahr TA, Sheth K. Update on allergic rhinitis. Pediatr Rev. 2005;26:284-289. Available at:http://pedsinreview.aappublications.org/cgi/content/full/26/8/284

Copyright © 2007 by the American Academy of Pediatrics page 365

Page 366: AAP MCQ 2007

2007 PREP SA on CD-ROM

Question: 176

An 11-year-old highly competitive gymnast presents to your office with complaints of increasingright wrist pain over the past 2 to 3 months. She recalls no specific trauma to the wrist or arm.On physical examination, you note no swelling of the distal forearm or wrist. She has normalrange of motion at the wrist. There is moderate-to-marked tenderness of the distal radius, but notenderness more distally over the wrist joint. A radiograph of the forearm shows slight wideningof the distal radial physis.

Of the following, you are MOST likely to explain to the child and family that

A. minor pains such as this are common in athletes and should not cause alarm

B. she must stop her training immediately

C. there is no evidence for trauma to her bones

D. this is a common injury in young gymnasts that can be overcome with an altered trainingregimen

E. this most likely represents a wrist sprain

Copyright © 2007 by the American Academy of Pediatrics page 366

Page 367: AAP MCQ 2007

2007 PREP SA on CD-ROM

Preferred Response: DCritique: 176

The combination of history, physical examination, and radiologic findings reported for the girl inthe vignette are most consistent with stress injury to the distal radial physis. This overuse injurymay lead to subsequent derangement in radial-ulnar growth. Until the injury has healed fully, thegirl should alter her training to remove stress on her distal radius by splinting her wrist, butcontinued aerobic training such as running still can be permitted.

Chronic wrist pain has been reported in up to 79% of young gymnasts. In one recent report,more than 50% of the gymnasts studied had evidence of stress injury to the distal radial physis,but only 7% had frank widening of the growth plate on radiography. The consequence of thisinjury, if left untreated, is distal radius physeal arrest with secondary ulna-radial lengthdifference.

The distal radial physis is located just proximal to the wrist within the distal radius. Thepresence of point tenderness over the distal radius and the lack of tenderness over the wrist arenot consistent with a sprained wrist. The abnormal radiograph showing widening of the physisconfirms the diagnosis.

References:

American Academy of Pediatrics Committee on Sports Medicine and Fitness. Intensive trainingand sports specialization in young athletes. Pediatrics. 2000;106:154-157. Available at:http://pediatrics.aappublications.org/cgi/content/full/106/1/154

Caine D, Howe W, Ross W, Bergman G. Does repetitive physical loading inhibit radial growth infemale gymnasts? Clin J Sport Med. 1997;7:302-308. Abstract available at:http://www.ncbi.nlm.nih.gov/entrez/query.fcgi?orig_db=PubMed&db=PubMed&cmd=Search&term=7[volume]+AND+302[page]+AND+1997[pdat]+AND+Caine+D[author]

DiFiori JP, Puffer JC, Aish B, Dorey F. Wrist pain, distal radial physeal injury, and ulnar variancein young gymnasts: does a relationship exist? Am J Sports Med. 2002;30:879-885. Abstractavailable at:http://www.ncbi.nlm.nih.gov/entrez/query.fcgi?orig_db=PubMed&db=PubMed&cmd=Search&term=Am+J+Sports+Med[Jour]+AND+879[page]+AND+2002[pdat]

Kolt GS, Kirkby RJ. Epidemiology of injury in elite and subelite female gymnasts: a comparison ofretrospective and prospective findings. Br J Sports Med. 1999;33:312-318. Abstract available at:http://www.ncbi.nlm.nih.gov/entrez/query.fcgi?orig_db=PubMed&db=PubMed&cmd=Search&term=Br+J+Sports+Med[Jour]+AND+312[page]+AND+1999[pdat]

Copyright © 2007 by the American Academy of Pediatrics page 367

Page 368: AAP MCQ 2007

2007 PREP SA on CD-ROM

Question: 177

You diagnose tinea capitis in a 7-year-old girl. She is otherwise healthy and has no knownallergies to medications. You plan to prescribe oral griseofulvin.

Of the following, it is MOST appropriate to prescribe the drug

A. after obtaining a baseline complete blood count

B. after obtaining baseline liver function tests

C. with clinical follow-up only at 4 to 6 weeks

D. with serial complete blood counts during therapy

E. with serial liver function tests during therapy

Copyright © 2007 by the American Academy of Pediatrics page 368

Page 369: AAP MCQ 2007

2007 PREP SA on CD-ROM

Preferred Response: CCritique: 177

Griseofulvin remains the preferred treatment for tinea capitis. The desired dose is 20 mg/kg perday for at least 6 weeks. Although many rare severe adverse effects, including agranulocytosisand aplastic anemia, have been reported, griseofulvin remains a safe drug for use in children,and laboratory monitoring at baseline or during therapy is not required in an otherwise healthychild. Clinical follow-up at 4 to 6 weeks is adequate surveillance for the adverse reactions.Newer antifungal agents have been introduced, but none have the longstanding safety profile ofgriseofulvin.

References:

Bennett ML, Fleischer AB, Loveless JW, Feldman SR. Oral griseofulvin remains the treatment ofchoice for tinea capitis in children. Pediatr Dermatol. 2000:17:304-309. Available at:http://www.blackwell-synergy.com/doi/abs/10.1046/j.1525-1470.2000.01784.x?prevSearch=allfield%3A%28%22Oral+griseofulvin+remains+the+treatment+of+choice+for+tinea+capitis+in+children%22%29

Friedlander S. Fungal infections: superficial fungal infections. In: Schachner LA, Hansen RC,eds. Pediatric Dermatology. 3rd ed. St. Louis, Mo: Mosby; 2003:1093-1106

Weston WL, Lane AT, Morelli JG. Fungal and yeast infections of the skin: hair infections (tineacapitis). In: Color Textbook of Pediatric Dermatology. 3rd ed. St. Louis, Mo: Mosby; 2002:63-67

Copyright © 2007 by the American Academy of Pediatrics page 369

Page 370: AAP MCQ 2007

2007 PREP SA on CD-ROM

Question: 178

A 16-year-old boy comes to your office with a 6-month history of abdominal cramping. He statesthat the cramps immediately precede a bowel movement and that passage of stool results inpain relief. There is no clear association with any type of food group. The patient's bowelmovements are variable, ranging from hard stools every other day to loose stools several timesa day. Weight and height are normal, as are physical examination findings and measurements ofstool guaiac, complete blood count, erythrocyte sedimentation rate, albumin, aspartateaminotransferase, alanine aminotransferase, immunoglobulin A, and tissue transglutaminase.Stool studies are negative for Giardia sp, Clostridium difficile, and enteric pathogens.

Of the following, the MOST appropriate next step is

A. colonoscopy and biopsy

B. fiber supplementation

C. observation

D. oral tegaserod

E. referral to a psychiatrist

Copyright © 2007 by the American Academy of Pediatrics page 370

Page 371: AAP MCQ 2007

2007 PREP SA on CD-ROM

Preferred Response: BCritique: 178

Irritable bowel syndrome (IBS) is a functional gastrointestinal disorder that typically occurs inteenagers and young adults. According to the Rome 2 criteria, IBS is defined as abdominaldiscomfort or pain that is relieved with defecation and is associated with either a change infrequency or consistency of stool. Typically, as described for the boy in the vignette, theaffected patient has a sudden abdominal cramp, develops an urge to "run to the bathroom," andpasses a mushy stool containing mucus. Although diarrhea is common, rectal bleeding does notoccur. Infectious and inflammatory causes of diarrhea should be excluded by physicalexamination (including stool for occult blood test) and by stool studies and laboratory testing(including complete blood count, erythrocyte sedimentation rate, and measurement ofimmunoglobulin A and tissue transglutaminase antibody).

The precise causes of IBS are poorly understood, but heredity, altered colonic motility,alterations in colonic microflora, and excessive gas production have been postulated to play arole. Psychological factors may also worsen symptoms and predispose to health-seekingbehavior, but stress is not believed to be the principal cause of the symptoms in most patients.

IBS has a different symptom complex from functional abdominal pain in the school-age child,which is typically periumbilical and not associated with disordered defecation. However,approximately 20% to 25% of children who have functional abdominal pain develop irritablebowel symptoms within 5 years after their initial evaluation.

Because of a paucity of randomized trials, treatment of IBS is highly individualized andpatient-specific. Treatment often begins with a trial of a fiber supplement, which has beenreported to help up to 50% of patients. Other treatments include antispasmodics (eg,hyoscyamine), antidiarrheals (loperamide), cholestyramine, and probiotics. The 5HT4 agonisttegaserod has been shown to be effective in adults who have constipation-predominant IBS, butis not first-line treatment and is not indicated in patients who have diarrhea. Psychotherapy,cognitive behavioral therapy, hypnosis, and other complementary therapies may be of benefit,but generally are not first-line therapy. Because IBS can be debilitating and is treated easily,observation alone generally is not recommended.

Patients who have persistent or refractory symptoms may need to undergo flexiblesigmoidoscopy or colonoscopy at some point to rule out microscopic colonic inflammation(microscopic colitis). Celiac disease should be excluded by serology or endoscopy. However,for a patient who has classic IBS and no "red flags," endoscopic evaluation can be postponeduntil after a trial of therapy.

References:

Hadley SK, Gaarder SM. Treatment of irritable bowel syndrome. Am Fam Physician.2005;72:2501-2506. Available at: http://www.aafp.org/afp/20051215/2501.html

Talley NJ, Spiller R. Irritable bowel syndrome: a little understood organic bowel disease? Lancet.2002;360:555-564. Available at:http://www.sciencedirect.com/science?_ob=ArticleURL&_aset=V-WA-A-W-A-MsSAYZW-UUA-U-AACCBABYDE-AACWYEVZDE-EEZVDZDUV-A-U&_rdoc=1&_fmt=summary&_udi=B6T1B-46HNW1W-10&_coverDate=08%2F17%2F2002&_cdi=4886&_orig=search&_st=13&_sort=d&view=c&_acct=C000052235&_version=1&_urlVersion=0&_userid=1300165&md5=1d7e48bc1cdde305fff9bbbe5787207d

Walker LS, Guite JW, Duke M, Barnard JA, Greene JW. Recurrent abdominal pain: a potentialprecursor of irritable bowel syndrome in adolescents and young adults. J Pediatr.1998;132:1010-1015. Abstract available at:http://www.ncbi.nlm.nih.gov/entrez/query.fcgi?cmd=Retrieve&db=pubmed&dopt=Abstract&list_uids=9627595&query_hl=6&itool=pubmed_docsum

Copyright © 2007 by the American Academy of Pediatrics page 371

Page 372: AAP MCQ 2007

2007 PREP SA on CD-ROM

Question: 179

An infant is born following a pregnancy complicated by no prenatal care and reduced fundalheight for gestation on examination during labor. Fetal heart rate tracings are nonreassuring.Physical examination of the infant reveals a birthweight of 1,800 g, flattened facies (Item Q179A),low-set ears, respiratory distress, a large flank mass on the left, and joint contractures. Renalultrasonography documents a single left multicystic and dysplastic kidney; the right kidney isabsent.

Of the following, the BEST explanation for these findings is

A. Alport disease

B. congenital nephrotic syndrome

C. congenital Wilms tumor

D. oligohydramnios sequence

E. Turner syndrome

Copyright © 2007 by the American Academy of Pediatrics page 372

Page 373: AAP MCQ 2007

2007 PREP SA on CD-ROM

Preferred Response: DCritique: 179

The infant described in the vignette is growth-restricted, has a flank mass consistent with anenlarged kidney, and has the characteristic phenotype of the fetal oligohydramnios sequence.Bilateral renal aplasia (true Potter syndrome), which occurs in 1 per 3,000 births, or severedysplasia, such as that seen in multicystic dysplastic or hereditary polycystic kidney diseaseand resulting in oligohydramnios (Potter sequence), both jeopardize fetal and neonatal well-being. Renal aplasia/dysplasia in the fetus results in oligohydramnios that alters amniotic fluiddynamics and interrupts normal development of the fetal lung, especially during the canalicularstage of lung development from 16 to 24 weeks’ gestation. Pulmonary hypoplasia results andmay prove fatal in the neonatal period despite efforts at assisted ventilation.

An additional complication of this aberrant renal development is reduced intrauterine volume,resulting in fetal constraint. In such instances, the growth restriction is generalized, the facies isflattened (Item C179A), the ears appear low-set, the arms and legs may be malpositioned withclubbing of the feet (Item C179B) and joint contractures, and breech presentation is notuncommon.

Alport disease is an inherited condition (X-linked in 85% and autosomal recessive in 15%)that involves the kidneys, cochlea, and eyes, leading to sensorineural deafness and nephritis.The defect affects type IV collagen, and clinical stigmata do not present in the newborn period.Congenital nephrotic syndrome is characterized by edema, ascites or hydrops, and associatedproteinuria, hyperlipidemia, and hypoalbuminemia. It is most common among families that haveFinnish ancestry. Wilms tumor, also known as a mesoblastic nephroma, may be congenital andmay be associated with aniridia, hemihypertrophy, or Beckwith-Wiedemann syndrome. Findingson the physical examination include hypertension, hematuria, and a flank mass. Turnersyndrome is due to an absence of part or all of one of the X chromosomes. Renal anomaliesseen in this condition include renal aplasia or hypoplasia, rotated or horseshoe kidneys, or renalduplication. The clinical phenotype includes growth restriction, lymphedema of the dorsalaspects of the hands and feet, cystic hygroma, and cardiovascular defects such as coarctationof the aorta and other left-sided cardiac outflow tract lesions.

References:

Davis ID, Avner ED. Isolated glomerular diseases with recurrent gross hematuria: Alportsyndrome. In: Behrman RE, Kliegman RM, Henson HB, eds. Nelson Textbook of Pediatrics.17th ed. Philadelphia, Pa: Saunders; 2004:1738-1739

Frías JL, Davenport ML, Committee on Genetics and Section on Endocrinology. Healthsupervision for children with Turner syndrome. Pediatrics. 2003;111:692-702. Available at:http://pediatrics.aappublications.org/cgi/content/full/111/3/692

Jaffe N, Huff V. Neoplasms of the kidney. In: Behrman RE, Kliegman RM, Henson HB, eds.Nelson Textbook of Pediatrics. 17th ed. Philadelphia, Pa: Saunders; 2004:1711-1714

Jones KL. Oligohydramnios sequence. In: Smith’s Recognizable Patterns of HumanMalformation. 6th ed. Philadelphia, Pa: Elsevier Saunders; 2006:726-727

Judge NE, Morrison SC. Perinatal ultrasound. In: Martin RJ, Fanaroff AA, Walsh MC, eds.Fanaroff and Martin’s Neonatal-Perinatal Medicine: Diseases of the Fetus and Infant. 8th ed.Philadelphia, Pa: Mosby-Elsevier; 2006:141-166

Rapaport R. Hypofunction of the ovaries: hypergonadotropic hypogonadism in the female(primary hypogonadism). In: Behrman RE, Kliegman RM, Henson HB, eds. Nelson Textbook ofPediatrics. 17th ed. Philadelphia, Pa: Saunders; 2004:1931-1934

Vogt BA, Avner ED. Nephrotic syndrome: congenital nephrotic syndrome. In: Behrman RE,Kliegman RM, Henson HB, eds. Nelson Textbook of Pediatrics. 17th ed. Philadelphia, Pa:

Copyright © 2007 by the American Academy of Pediatrics page 373

Page 374: AAP MCQ 2007

2007 PREP SA on CD-ROM

Saunders; 2004:1757

Copyright © 2007 by the American Academy of Pediatrics page 374

Page 375: AAP MCQ 2007

2007 PREP SA on CD-ROM

Question: 180

A 16-month-old girl presents with the acute onset of difficulty breathing after attending a birthdayparty. She is in moderate distress, with a respiratory rate of 56 breaths/min. She has slightlydiminished breath sounds on the right. You suspect foreign body aspiration. A plainanteroposterior chest radiograph appears normal.

Of the following, the MOST appropriate radiographic study to confirm your suspicion of foreignbody aspiration in this girl is

A. airway fluoroscopy

B. barium swallow

C. inspiratory and expiratory chest radiographs

D. magnetic resonance imaging of the chest

E. ventilation perfusion scan

Copyright © 2007 by the American Academy of Pediatrics page 375

Page 376: AAP MCQ 2007

2007 PREP SA on CD-ROM

Preferred Response: ACritique: 180

Young children are at high risk for foreign body aspiration, which causes a significant number ofpediatric deaths each year. A history of choking in these cases is not always available, so it isimportant for the clinician to maintain a high level of suspicion for aspiration in young childrenwho have acute respiratory distress. Physical findings associated with foreign body aspirationinclude asymmetric breath sounds and wheezing, cough, tachypnea, and retractions, but someaffected children have normal and equal breath sounds bilaterally.

Plain radiography of the chest should be the initial diagnostic test, keeping in mind that manyforeign bodies are food particles and, therefore, radiolucent. The chest radiograph findings maybe normal, as described for the child in the vignette. Abnormal findings may include atelectasis,consolidation, mediastinal shift, or air trapping. If clinical suspicion for foreign body aspiration ishigh, further radiologic testing is indicated, even with normal chest film results. Airwayfluoroscopy is an appropriate next test for the child in the vignette. Fluoroscopy enables theclinician to evaluate diaphragm and mediastinum movement and is a noninvasive and safeprocedure. High-resolution computed tomography scan is another tool that may aid in thediagnosis of foreign body aspiration, but may require sedation. This may be difficult in a patientwho has acute respiratory distress.

Older children may cooperate for inspiratory and expiratory radiographs (Item C180A),which may accentuate an area of air trapping, but these are impractical for young children.Lateral decubitus films may be useful in young patients. Barium swallow results may beabnormal following ingestion of a foreign body, but they will not confirm or rule out a diagnosis ofaspiration. Magnetic resonance imaging and ventilation perfusion scans are not useful in thediagnosis of foreign body aspiration.

References:

Long FR. Imaging evolution of airway disorders in children. Radiol Clin North Am. 2005;43:371-389. Available at: http://www.radiologic.theclinics.com/article/PIIS0033838904002209/abstract

Rovin JD, Rodgers BM. Pediatric foreign body aspiration. Pediatr Rev. 2000;21:86-90. Availableat: http://pedsinreview.aappublications.org/cgi/content/full/21/3/86

Copyright © 2007 by the American Academy of Pediatrics page 376

Page 377: AAP MCQ 2007

2007 PREP SA on CD-ROM

Question: 181

A 13-month-old boy presents to your office after his mother called for an urgent appointmentbecause he had a bad cough and noisy breathing. He has been previously healthy and is fullyimmunized. On physical examination, his temperature is 100.9°F (38.3°C), pulse rate is 142beats/min, respiratory rate is 36 breaths/min (crying), and pulse oximetry reading is 98% onroom air. The mother states that he went to sleep normally with only mild symptoms of an upperrespiratory tract infection and awakened at 4 am with noisy breathing. He tolerated sips of juicethis morning and has had no vomiting, diarrhea, or high fever. After the boy settles down fromhaving his vital signs measured, he has a "barking cough," and on auscultation, you noticestridor with every breath. Mild suprasternal retractions are visible on examination of the chest.He prefers to sit up and looks slightly anxious.

Of the following, the treatment MOST likely to provide improvement is

A. ceftriaxone intramuscularly

B. dexamethasone orally or intramuscularly

C. humidified oxygen by face mask

D. nebulized albuterol

E. nebulized budesonide

Copyright © 2007 by the American Academy of Pediatrics page 377

Page 378: AAP MCQ 2007

2007 PREP SA on CD-ROM

Preferred Response: BCritique: 181

The boy described in the vignette most likely has croup, the most common cause of upperairway obstruction in young children. The disease usually is due to a viral infection(parainfluenza types 1 and 2). Croup typically has a nocturnal presentation in a child 6 months to2 years of age, with either no prodrome or an upper respiratory tract infection. Noninfectious(spasmodic) croup may be seen in children who have atopy or a history of asthma.

The differential diagnosis of acute upper airway obstruction includes foreign bodyingestion/aspiration, bacterial tracheitis, and epiglottitis. Because of Haemophilus influenzae typeb immunization, epiglottitis is rare today. Severe tonsillitis or parapharyngeal abscesses alsomay cause acute upper airway obstruction, but these conditions generally afflict older childrenand adolescents.

Signs and symptoms of upper airway obstruction include preference for an upright posture(sometimes “tripoding,” with the hand placed on the examination table), breathing through anopen mouth, dysphagia or inability to swallow secretions, and stridor. It is common for virallaryngotracheobronchitis to cause wheezing or signs of lower airway obstruction, as well.

Assessment for respiratory distress includes evaluating for retractions, accessory muscleuse, hypoxia, and fatigue. Stridor at rest implies potential upper airway compromise due tosubglottic and vocal cord edema.

Therapy for airway obstruction is avoidance of agitation (venipuncture for laboratory studiesis not indicated or helpful) and early administration of oral or injectable steroids. Nebulizedbudesonide has not been shown to be superior to parenteral dexamethasone. Provision ofhumidified oxygen is indicated as a triage intervention for all children in respiratory distress, but itdoes not represent definitive management. Use of mist tents or nebulized saline also is notdefinitive therapy. Nebulized albuterol does not constrict edematous airway tissue and is nottherapeutic in croup, unless the condition is associated with wheezing or other evidence ofbronchoconstriction. Nebulized racemic epinephrine may have a short-term benefit but is notdefinitive treatment.

Antibiotics are not indicated in viral croup. Viral croup usually can be distinguished frombacterial tracheitis based on lower temperature, abrupt onset of symptoms, and the presence ofupper respiratory tract symptoms for a brief period prior to the development of croup.

If a child appears toxic, has severe respiratory distress, or fails therapy for croup, otherdiagnoses should be considered, such as epiglottitis, bacterial tracheitis, severe croup, orforeign body. These conditions are airway emergencies.

Most children who have mild-to-moderate croup are managed successfully as outpatients.Children who have stridor at rest, impending respiratory failure, or underlying disease should beobserved in the hospital.

References:

Cohen LF. In brief: stridor and upper airway obstruction in children. Pediatr Rev. 2000;21:4-5.Available at: http://pedsinreview.aappublications.org/cgi/content/full/21/1/4

Malhotra A, Krilov LR. Viral croup. Pediatr Rev. 2001;22:5-12. Available at:http://pedsinreview.aappublications.org/cgi/content/full/22/1/5

Russell K, Wiebe N, Saenz A, et al. Glucocorticoids for croup. The Cochrane Database ofSystematic Reviews. 2004;1:CD001955. Available at:http://www.mrw.interscience.wiley.com/cochrane/clsysrev/articles/CD001955/frame.html

Copyright © 2007 by the American Academy of Pediatrics page 378

Page 379: AAP MCQ 2007

2007 PREP SA on CD-ROM

Question: 182

A father brings his 2-year-old daughter to you because he has noticed "funny eye movements"over the past 2 weeks. Upon physical examination, you find that the child has pendularnystagmus (Item Q182A), her head is tilted to the right, and she nods her head. Other findingson the physical examination are normal.

Of the following, the MOST likely diagnosis is

A. congenital nystagmus

B. optic glioma

C. phenytoin intoxication

D. retinoblastoma

E. spasmus nutans

Copyright © 2007 by the American Academy of Pediatrics page 379

Page 380: AAP MCQ 2007

2007 PREP SA on CD-ROM

Preferred Response: ECritique: 182

Nystagmus is an involuntary, rhythmic oscillation of both eyes (rarely one) in which at least onephase is slow. Nystagmus reflects a deficit in gaze-holding mechanisms, analogous to brakesthat no longer stop an automobile appropriately. Jerk nystagmus is characterized by a slowphase in one direction, usually back to central position, followed by a fast, compensatorymovement in the opposite direction, back to the far lateral side. A few beats of jerk nystagmus inthe horizontal plane are physiologic, or normal, when a child gazes far laterally. Sustainednystagmus, especially if associated with other neurologic findings, is abnormal. The differentialdiagnosis includes drug toxicity, vestibular dysfunction, tumor, or infarction. Chiari malformationsproduce downbeating or vertical jerk nystagmus.

Pendular nystagmus is defined as slow movements in both phases, with the eyes moving“to and fro.” Pendular nystagmus can be a sign of brainstem infarction, spinocerebellardegeneration, multiple sclerosis, or rarely seizure. Spasmus nutans (Item C182A) is a benign,transient disorder without known cause that is characterized by pendular nystagmus,intermittent head tilt, and nodding or titubating of the head, as described for the girl in thevignette. The head tilt can be mistaken as muscular torticollis. The onset is between ages 1 and3 years, most often in the first 12 postnatal months, with spontaneous resolution within 1 to 2years.

A glioma in the optic pathway can cause jerk or pendular nystagmus, but it also producesloss of visual acuity or field, and over time, optic atrophy. Phenytoin intoxication produces jerknystagmus. Congenital nystagmus can have pendular to jerk movements that often are notnoted until several months or years after birth, and it frequently is characterized by some loss ofvisual acuity. This disorder can be inherited in variable pattern. Retinoblastoma at presentationdoes not produce nystagmus, but instead leukocoria (Item C182B) with loss of the red reflex.

References:

Hoyt CS, Good WV. Ophthalmic problems in childhood. In: Berg BO, ed. Child Neurology: AClinical Manual. 2nd ed. Philadelphia, Pa: JB Lippincott Co; 1994:241-255

Olitsky SE, Nelson LB. Disorders of eye movement and alignment. In: Behrman RE, KliegmanRM, Jenson HB, eds. Nelson Textbook of Pediatrics. 17th ed. Philadelphia, Pa: WB SaundersCo; 2004:2092-2096

Copyright © 2007 by the American Academy of Pediatrics page 380

Page 381: AAP MCQ 2007

2007 PREP SA on CD-ROM

Question: 183

You are evaluating a 6-hour-old male infant who was born after a term pregnancy and normaldelivery and weighs 4 kg. The infant is comfortable but exhibits mild tachypnea, with arespiratory rate of 50 breaths/min. His heart rate is regular at 150 beats/min. His oxygensaturation is 60% in all extremities and does not increase significantly with the administration ofoxygen by face mask. His lungs are clear, and there are no murmurs, gallops, or rub. Yoususpect transposition of the great arteries.

Of the following, the BEST management strategy is

A. diuretic therapy intravenously for pulmonary edema

B. increasing the left-to-right (aorta-to-pulmonary artery) shunt at the ductus arteriosus

C. increasing the right-to-left shunt at the foramen ovale

D. increasing the right-to-left (pulmonary artery-to-aorta) shunt at the ductus arteriosus

E. intubation and mechanical ventilation with an Fio2 of 1.0

Copyright © 2007 by the American Academy of Pediatrics page 381

Page 382: AAP MCQ 2007

2007 PREP SA on CD-ROM

Preferred Response: BCritique: 183

Understanding blood flow patterns in the patient who has transposition of the great arteries(TGA) requires a clear understanding of shunting patterns in the healthy fetus. Maintenance ofthe normal patterns can be essential to the management of the neonate who has TGA. In thenormal heart, the right atrium and right ventricle deliver desaturated blood to the organ ofoxygenation. In the fetus, this organ is the placenta, and its fetal blood supply is via the umbilicalartery that arises from the fetal descending aorta. The ductus arteriosus provides a fetalshunting pathway that allows the right side of the fetal heart to deliver desaturated blood to theorgan of oxygenation by shunting the blood away from the high-resistance pulmonary arteriesand into the descending aorta. This direction of flow occurs in part because the fetal pulmonaryvascular resistance is slightly higher than the fetal systemic vascular resistance because thedeveloping lungs are filled with fluid and the placenta is a low-resistance circuit.

At birth, when the lungs expand with air and the placenta is removed from the circulation, thepulmonary vascular resistance falls and the systemic vascular resistance increases. This leadsto a reversal of flow across the ductus arteriosus (from the systemic into the pulmonary circuit).Over the subsequent hours and days, the ductus arteriosus begins the process of spontaneousclosure.

The foramen ovale is an important fetal shunt that prenatally allows the relatively oxygenatedblood returning from the placenta to cross from the right atrium into the left atrium. In so doing,the blood with the highest oxygen content is directed to the coronary and cerebral circulations.Patency of the foramen in utero results from a slightly higher pressure in the right atrium than theleft due to very little (approximately 10% of the combined fetal cardiac output) blood returning tothe left atrium from the lungs. The pressure difference “pushes” the flap of the foramen into theleft atrium, creating the “hole” and allowing right-to-left blood flow. At birth, when the lungs areexpanded and the entire cardiac output is directed into the lungs, pressure in the left atriumincreases and rises slightly above that in the right atrium, forcing the flap of the foramen againstthe atrial septum and closing the “hole.”

TGA is the second most common form of cyanotic heart disease, but it is the most commonto present in the first day (or hours) after birth. Affected patients have atrioventricularconcordance and ventriculoarterial discordance (Item C183A); that is, the left atrium connects tothe left ventricle and the right atrium connects to the right ventricle, but the left ventricle connectsto the pulmonary artery and the right ventricle connects to the aorta. Thus, desaturated bloodreturning from the body enters the right heart, only to be pumped back to the aorta. Patients canexhibit profound cyanosis if there is no reliable method to mix left-sided blood into the right side(atrial septal defect or ventricular septal defect).

The volume of pulmonary blood flow in TGA is generally normal, but because of theanatomy, oxygenated blood from the left heart repeatedly is delivered to the lungs. To achieveeffective pulmonary blood flow and deliver desaturated blood to the lungs, the ductus arteriosusmust remain patent, allowing desaturated “blue” blood from the high-resistance systemic circuitto cross from the aorta into the pulmonary artery. This blood is oxygenated in the lungs (hence,the term effective pulmonary blood flow) and returns to the left atrium. As the total amount ofblood entering the lungs (from the left heart and via the ductus arteriosus) increases, so doesthe volume of blood in the left atrium. The atrial septum is stretched to accommodate theincreased volume, and the foramen ovale frequently “opens.” Because pressure in the leftatrium now exceeds that in the right atrium, the direction of shunting across the foramen is left-to-right, allowing oxygenated blood to enter the right heart, which in TGA, pumps to the aorta. Thus,the combination of the ductus arteriosus and the foramen ovale can be used to increaseoxygenated blood flow to the body in a patient who has TGA.

Patency of the ductus arteriosus can be maintained by prostaglandins such as PGE-1,administered as a continuous intravenous infusion. If the patency of the foramen ovale is notsufficient to maintain adequate oxygen saturation in the neonate, a balloon atrial septostomy, acatheter procedure to enlarge the atrial communication, can be performed.

Diuretics are not indicated for the infant in the vignette because there is normal pulmonaryblood flow into well-functioning lungs. Increasing right-to-left shunting at the foramen ovaledelivers more blue blood into the left heart, rather than the desired delivery of more oxygenated

Copyright © 2007 by the American Academy of Pediatrics page 382

Page 383: AAP MCQ 2007

2007 PREP SA on CD-ROM

blood to the aorta. Similarly, increasing right-to-left shunting at the ductus arteriosus can occuronly if the pulmonary vascular resistance exceeds the systemic vascular resistance, whichfurther diminishes effective pulmonary blood flow and reduces blood returned to the left atrium.Intubation and mechanical ventilation are not indicated because the patient does not haverespiratory distress or failure. Because the lungs work normally and the cause for the cyanosisis anatomic, high Fi2 would not change oxygen saturation significantly.

References:

Friedman AH, Fahey JT. The transition from fetal to neonatal circulation: normal responses andimplications for infants with heart disease. Semin Perinatol. 1993;17:106-121. Abstract availableat:http://www.ncbi.nlm.nih.gov/entrez/query.fcgi?cmd=Retrieve&db=pubmed&dopt=Abstract&list_uids=8327901&query_hl=12&itool=pubmed_DocSum

Teitel D. Right to left shunts. In: Rudolph CD, Rudolph AM, Hostetter MK, Lister G, Siegel NJ,eds. Rudolph’s Pediatrics. 21st ed. New York, NY: McGraw Hill; 2003:1814-1826

Copyright © 2007 by the American Academy of Pediatrics page 383

Page 384: AAP MCQ 2007

2007 PREP SA on CD-ROM

Question: 184

You have just diagnosed Ebstein anomaly in a newborn.

Of the following, the MOST likely prenatal exposure to result in this abnormality is

A. alcohol

B. coumarin

C. lithium

D. phenytoin

E. retinoic acid

Copyright © 2007 by the American Academy of Pediatrics page 384

Page 385: AAP MCQ 2007

2007 PREP SA on CD-ROM

Preferred Response: CCritique: 184

Several retrospective studies have provided evidence that lithium is a human teratogen, withmalformation rates as high as 12% in exposed infants. Among the most common defects arecardiac anomalies, including Ebstein anomaly, which is a rare malformation characterized by amalformed tricuspid valve and secondary abnormalities of the right ventricle and atrium. Atrialseptal defect is another commonly reported abnormality. More recent studies have providedconflicting data about the degree of risk associated with maternal lithium use, leading some tosuggest that it can be used cautiously in pregnancy among patients for whom other medicationsare not efficacious in controlling bipolar disorder. In general, even among those who wouldconsider the use of lithium during pregnancy, it is reserved for patients at substantial risk ofrelapse if lithium is discontinued. It still is discontinued in these patients in very early pregnancyduring embryogenesis and restarted if deterioration occurs. If lithium is administered during thesecond or third trimesters, care must be taken to avoid high serum concentrations; a neonatalconcentration of lithium equivalent to the maternal level leads to sedation in the newborn period.Thyroid function disturbances also have been reported in infants of mothers treated with lithium.Patients must be counseled in detail about the relative risks and benefits of this approach.Lactating women should be advised that breastfeeding also might produce significant lithiumconcentrations in the infant.

Fetal alcohol syndrome is characterized by prenatal-onset growth restriction and facialdysmorphisms. Although cardiac defects occur, they usually are septal defects. Fetal coumarinexposure can result in nasal hypoplasia, stippling of the epiphyses, and low birthweight.Phenytoin syndrome includes growth deficiency, a dysmorphic face (Item C184A), andhypoplasia of the distal phalanges (Item C184B). Cardiac anomalies are less frequent and do notinclude Ebstein anomaly. Retinoic acid embryopathy is a severe syndrome characterized bycentral nervous system defects, microtia, and conotruncal malformations.

References:

Cohen LS, Friedman JM, Jefferson JW, Johnson EM, Weiner ML. A reevaluation of risk of inutero exposure to lithium. JAMA. 1994;271:146-150. Abstract available at:http://www.ncbi.nlm.nih.gov/entrez/query.fcgi?orig_db=PubMed&db=PubMed&cmd=Search&term=%22JAMA+:+the+journal+of+the+American+Medical+Association%22[Jour]+AND+1994[pdat]+AND+Cohen+LS[author]

Jones KL. Dysmorphology. In: Behrman RE, Kliegman RM, Jenson HB, eds. Nelson Textbookof Pediatrics. 17th ed. Philadelphia Pa: Saunders; 2004:616-623

Stoll BJ, Kliegman RM. Metabolic disturbances. In: Behrman RE, Kliegman RM, Jenson HB, eds.Nelson Textbook of Pediatrics. 17th ed. Philadelphia PA: Saunders; 2004:609-613

Copyright © 2007 by the American Academy of Pediatrics page 385

Page 386: AAP MCQ 2007

2007 PREP SA on CD-ROM

Question: 185

A 15-year-old agitated and delusional boy presents to the emergency department. Physicalexamination reveals tachycardia, sweating, and pupils that are dilated and react slowly, but thereis no nystagmus, and the eyelids and conjunctivae are normal.

Of the following, the substance that MOST likely was abused by this boy is

A. alprazolam

B. heroin

C. marijuana

D. methamphetamine

E. phencyclidine

Copyright © 2007 by the American Academy of Pediatrics page 386

Page 387: AAP MCQ 2007

2007 PREP SA on CD-ROM

Preferred Response: DCritique: 185

The symptoms and signs exhibited by the 15-year-old boy described in the vignette are mostlikely the result of methamphetamine abuse. Expected short-term signs and symptoms due tothe direct effects of methamphetamines on adrenergic receptors include tachycardia,hypertension, diaphoresis, agitation, and fever. The typical eye signs are dilated pupils that areslow to react to light, normal eyelids and conjunctivae, and no nystagmus. Among the othersymptoms are nausea, vomiting, psychosis, hypervigilence, violent behavior, strokes, andseizures. High doses have been associated with ventricular irritability and life-threateningarrhythmias.

Frequent use of methamphetamines can cause insomnia and weight loss. Cerebrovasculardamage, symptoms similar to Parkinson disease, homicidal and suicidal thoughts, andpsychosis can result from chronic use. Psychotic symptoms (eg, paranoia, visual and auditoryhallucinations, delusions, and rages of violent behavior) can persist for months to years afterdiscontinuing use.

Although there are no physiologic manifestations of methamphetamine addiction/withdrawal,abusers can develop drug tolerance and experience withdrawal symptoms. Such symptoms,sometimes termed a withdrawal syndrome, may include depression, fatigue, anxiety,aggression, paranoia, problems with concentration and memory, and craving for the drug.Intermittent cravings for amphetamines may occur months to years after discontinuing use andmay be triggered by situations.

The anticipated physical signs of alprazolam (a benzodiazepine) abuse are sleepiness andsedation. Pupils are of normal size and may be slow to react. Under the influence of heroin, thepupils are constricted, and agitation is not typically observed. With marijuana use, the pupils,although slow to react, usually are of normal size, and conjunctival injection is common.Tachycardia and sweating are not present. Although phencyclidine use is associated withagitation and combativeness, the pupils are normal-sized, not constricted, and other eyeabnormalities may be observed, including retracted upper eyelid, swollen lid, and vertical andhorizontal nystagmus.

References:

Jenkins RR. Substance abuse: amphetamines. In: Behrman RE, Kliegman RM, Jenson HB, eds.Nelson Textbook of Pediatrics. 17th ed. Philadelphia, Pa: WB Saunders Co; 2004:660

MacKenzie RG, Heischober B. Methamphetamine. Pediatr Rev. 1997;18:305-309. Available at:http://pedsinreview.aappublications.org/cgi/content/full/18/9/305

Neinstein LS, Heischober BS. Miscellaneous drugs: stimulants, inhalants, opiates, depressants,and anabolic steroids. In: Neinstein LS, ed. Adolescent Health Care A Practical Guide. 4th ed.Philadelphia, Pa: Lippincott, Williams & Wilkins; 2002:1337-1369

Copyright © 2007 by the American Academy of Pediatrics page 387

Page 388: AAP MCQ 2007

2007 PREP SA on CD-ROM

Question: 186

A 14-year-old-boy has been followed yearly by another physician. When you see him for the firsttime, he brings records that you use to construct a growth curve (Item Q186A). His parentsstate that they are not worried about his growth because his 20-year-old brother was a slowgrower and still seems to be growing a little. Physical examination reveals 6-mL testes andSexual Maturity Rating 2 pubic hair. There are no other signs of puberty. His bone age onradiography is 12 years.

Of the following, the MOST appropriate suggestion for the family is that

A. a period of watchful waiting for 6 months is reasonable because he likely has delayed puberty

B. a short course of injected testosterone might help boost his growth and initiate puberty, whichis delayed

C. eating a high-calorie, high-protein meal once a day may initiate his puberty

D. laboratory studies to assess his endocrine status should be obtained immediately

E. little can be offered to improve his growth because he probably is approaching the end of hisgrowth phase

Copyright © 2007 by the American Academy of Pediatrics page 388

Page 389: AAP MCQ 2007

2007 PREP SA on CD-ROM

Preferred Response: DCritique: 186

Children who have constitutionally delayed maturation and are slow to progress through pubertybut otherwise normal have appropriate increments in linear growth that parallel the normalgrowth curve after the age of about 2 years. They then appear to fall from the normal growthcurve when they do not have a pubertal growth spurt at the usual age. In some, prolongedprepubertal growth attenuation, lasting 1 to 3 years, may lead to a perfectly reasoned medicaldecision to pursue further investigation. Children who have genetic short stature usually findtheir own growth curves by 1 to 2 years of age, and subsequently grow along those curves untilthey reach adult height, which although decreased, is commensurate with family heights.Children who have congenital growth hormone deficiency grow slowly in length from the age ofabout 9 months, but usually have normal weight and even appear a bit chubby. Acquired growthhormone deficiency leads to growth attenuation at a later age but usually is associated withweight maintenance. Children who have endocrine causes of growth attenuation(hypothyroidism, growth hormone deficiency, adrenocortical excess) generally are notunderweight.

Growth attenuation (failure to grow over the past several years), as documented in thegrowth curve constructed for the boy described in the vignette, is a worrisome sign of hiddenillness that demands immediate investigation. The investigation should look for hidden metabolicor gastrointestinal disease, including celiac disease, inflammatory bowel disease,hypothyroidism, growth hormone deficiency, and, if there is concern based on physicalappearance (changes may be subtle), Cushing disease. Laboratory studies for the boy in thevignette should assess his endocrine status.

Delayed puberty rarely is associated with prolonged growth attenuation, but suchattenuation requires evaluation; watchful waiting is not appropriate. Children who areunderweight for height commonly have metabolic/nutritional or gastrointestinal disorders. Thosewho maintain weight or are slightly heavy for height more commonly have hypothyroidism,Cushing disease, or growth hormone deficiency. The child in the vignette should not be offeredadjunctive therapies until a clear diagnosis has been made. Boys grow until their bones are fullyfused at a bone age of about 18 years; a boy whose bone age is 12 years still has his pubertalgrowth spurt ahead of him.

References:

Israel EJ, Levitsky LL, Anupindi SA, Pitman MB. Case Records of the Massachusetts GeneralHospital. Weekly clinicopathological exercises. Case 3-2005. A 14-year-old boy with recentslowing of growth and delayed puberty. N Engl J Med. 2005;352:393-403

Kemp S, Güngör N. Growth failure. eMedicine Specialties: Pediatrics: Endocrinology. 2005.Available at: http://www.emedicine.com/ped/topic902.htm

Reiter EO. Short stature/tall stature. In: Burg FD, Ingelfinger JR, Polin RA, Gershon AA, eds.Gellis & Kagan’s Current Pediatric Therapy. Philadelphia, Pa: WB Saunders; 2003:679-682

Copyright © 2007 by the American Academy of Pediatrics page 389

Page 390: AAP MCQ 2007

2007 PREP SA on CD-ROM

Question: 187

You see a 9-month-old girl for a health supervision visit. She is in her infant carrier with apropped bottle. The mother explains that the infant does not sleep through the night, which isdisrupting the mother's ability to function well at work. The infant appears well, with length andhead circumferences at the 25th percentile and weight at the 95th percentile. During theexamination, the infant cries. The mother hands you a bottle and asks if she may leave to checkon a referral for her older child.

Of the following, you are MOST likely to suggest to the mother

A. alteration of mother's work schedule

B. dental referral for the infant

C. establishment of a consistent bedtime routine for the infant

D. nutrition consultation for the infant

E. social work evaluation

Copyright © 2007 by the American Academy of Pediatrics page 390

Page 391: AAP MCQ 2007

2007 PREP SA on CD-ROM

Preferred Response: ECritique: 187

The relationship between an infant and parent is complex, and there are normal variations in howparent and child communicate, with great differences based on infant and child temperament andparental responsibilities and distractions. An infant’s ability to feed and a parent’s ability toprovide adequate nutrition for the infant are often reflective of the relationship between them.Common feeding practices that suggest a disordered relationship include propping the bottle andfeeding to quiet the infant.

The mother described in the vignette is working, sleep-deprived, and distracted during thechild’s examination. When her child cries, her response is to offer a bottle instead of looking for asource of discomfort for the infant. In this case, she is so distracted that she does not even tryto feed the infant herself, asking the clinician to feed her. Social work assistance in evaluatingthe family situation may allow the family to take advantage of community services that wouldhelp the mother to attend to her child with fewer distractions.

Alteration of mother’s work schedule probably will not address all of the issues in thissituation. Children who drink from a propped bottle are at risk for dental caries, but thisinformation could be shared with the mother without a dental visit. The mother could benefit fromparent education and counseling to help manage the infant’s sleeping schedule, but this is onlypart of the problem. Similarly, the infant’s weight likely is related to feeding practice and can beaddressed with parenting education.

References:

Block RW, Krebs NF, and the Committee on Child Abuse and Neglect and the Committee onNutrition. Failure to thrive as a manifestation of child neglect. Pediatrics. 2005;116:1234-1237.Available at: http://pediatrics.aappublications.org/cgi/content/full/116/5/1234

Fraser JJ Jr, McAbee GN, and Committee on Medical Liability. Dealing with the parent whosejudgment is impaired by alcohol or drugs: legal and ethical considerations. Pediatrics. 2004;114:869-873. Available at: http://pediatrics.aappublications.org/cgi/content/full/114/3/869

Kairys SW, Johnson CF, and Committee on Child Abuse and Neglect. The psychologicalmaltreatment of children—technical report. Pediatrics. 2002;109:e68. Available at:http://pediatrics.aappublications.org/cgi/content/full/109/4/e68

Copyright © 2007 by the American Academy of Pediatrics page 391

Page 392: AAP MCQ 2007

2007 PREP SA on CD-ROM

Question: 188

A 4-month-old boy presents to the clinic with a 2-day history of a temperature of 100.6°F(38.1°C) and vomiting. The infant was born at 26 weeks' gestation, and his birthweight was 960g. He remained in the hospital for 3 months after birth because of multiple complications,including bilateral grade 4 intraventricular hemorrhages necessitating the placement of aventriculoperitoneal shunt 6 weeks ago. Physical examination reveals an afebrile infant who hasa shunt bubble on the right temporo-occipital region and a gastrostomy button. You explain thathis shunt bubble requires aspiration to evaluate the cerebrospinal fluid for infection.

Of the following, the organism that is MOST likely to be isolated in the aspirate is

A. Enterococcus sp

B. Escherichia coli

C. nontypeable Haemophilus influenzae

D. Staphylococcus epidermidis

E. Streptococcus pneumoniae

Copyright © 2007 by the American Academy of Pediatrics page 392

Page 393: AAP MCQ 2007

2007 PREP SA on CD-ROM

Preferred Response: DCritique: 188

Although the pathogens that commonly cause community-acquired bacterial meningitis(Enterococcus sp, Escherichia coli, Haemophilus influenzae, or Streptococcus pneumoniae)may be responsible for the illness in the patient described in the vignette, the most commonorganism responsible for ventriculoperitoneal shunt (VPS) infections is Staphylococcusepidermidis. This organism is isolated in more than 50% of VPS infections, with S aureusaccounting for an additional 20%. Treatment of VPS infections includes specific antimicrobialtherapy directed at the causative organism and, in most instances, removal or replacement ofthe VPS.

S epidermidis is just one of the many species of coagulase-negative staphylococci thatcolonize humans. Only S epidermidis and S saprophyticus have demonstrated consistentpathogenicity for humans. Nosocomial infections with S epidermidis often are associated withimplanted hardware. Venous, arterial, and peritoneal catheters; cerebrospinal fluid and vascularshunts; and other prosthetic materials (eg, joints, valves) frequently are infected with thisorganism because they tend to produce a protective biofilm that surrounds the organism andleads to enhanced adhesion to foreign surfaces. In addition, immunocompromised and neonatalintensive care unit patients are at increased risk for serious infections with S epidermidis.

Most S epidermidis isolates are methicillin-resistant, but the organism remains susceptible tovancomycin. Therefore, the empiric antimicrobial agent of choice is vancomycin with or withoutrifampin or gentamicin (used for synergy in some difficult cases) until susceptibility data areavailable. Because of the biofilm production, foreign bodies that are infected often must beremoved for adequate treatment.

References:

Flynn PM, Barrett FF. Infection associated with medical devices. In: Behrman RE, Kliegman RM,Jenson HB, eds. Nelson Textbook of Pediatrics. 17th ed. Philadelphia, Pa: WB Saunders Co;2004:858-860

Kiang M, Cheng TL. In brief: Staphylococcus epidermidis. Pediatr Rev. 2003;24:430-431.Available at: http://pedsinreview.aappublications.org/cgi/content/full/24/12/430

Todd JK. Staphylococcus. In: Behrman RE, Kliegman RM, Jenson HB, eds. Nelson Textbook ofPediatrics. 17th ed. Philadelphia, Pa: WB Saunders Co; 2004:861-866

Copyright © 2007 by the American Academy of Pediatrics page 393

Page 394: AAP MCQ 2007

2007 PREP SA on CD-ROM

Question: 189

You are seeing a 10-year-old boy and his 13-year-old sister for the first time. When you reviewthe medical records provided by their mother, you find normal medical histories, vital signs, andphysical examination results for the children. However, the family history indicates that two ofthe children's uncles are receiving hemodialysis and are deaf and one grandfather died of kidneydisease. You obtain a screening urinalysis (UA) in both children. The boy's UA reveals moderateblood, negative protein, and 20 to 30 red blood cells/high-power field (RBC/hpf); the girl's UAreveals trace blood with 5 to 10 RBC/hpf.

Of the following, the MOST accurate statement regarding the prognosis for these children is that

A. the boy will develop end-stage renal disease (ESRD); the girl will not develop ESRD

B. the chances of developing ESRD are equal in the boy and girl

C. the boy will develop ESRD with hearing deficits; the girl will not develop ESRD

D. the boy will develop ESRD and esophageal leiomyomatosis; the girl will develop only hearingdeficits

E. the boy will develop ESRD and giant cell thrombocytosis; the girl will develop only ESRD

Copyright © 2007 by the American Academy of Pediatrics page 394

Page 395: AAP MCQ 2007

2007 PREP SA on CD-ROM

Preferred Response: CCritique: 189

Alport syndrome (hereditary nephritis) is an uncommon disease, affecting about 1 in 5,000persons. There are several modes of inheritance, the most common being X-linked. Althoughthere is a male-to-female ratio of 1:1, men are affected earlier and more severely than women.Other modes of inheritance include autosomal dominant, autosomal recessive, andheterogeneous. Females who have the defective gene on one of their two X chromosomesusually are asymptomatic, although they may exhibit some degree of minor renal insufficiency;most females who have Alport syndrome have some blood in the urine. Males who have thedefective gene usually develop renal failure between the second and third decades of life.

In most cases, the mutation lies in abnormal collagen type IV (COL4A) protein that is anessential part of the glomerular basement membrane. Other conditions that may be inherited andpresent with hematuria, such as benign familial hematuria and thin membrane disease, areassociated with significantly more favorable renal outcomes than is seen with Alport syndrome.

The primary characteristics of Alport syndrome are progressive renal failure, sensorineuralhearing deficits, and abnormalities of the lens of the eye (lenticonus). Uncommon associatedconditions include immunologic abnormality of skin, disorders of platelets, abnormalities of whiteblood cells, and smooth muscle tumors.

Alport syndrome can be diagnosed by several methods. Most commonly, it is diagnosedinitially by testing the urine (for blood) of persons who have an affected family member. Theurine result may be confirmed by performing a kidney biopsy and assessing for the presence orabsence of the COL4A protein. In certain cases, testing for the abnormal gene in Alportsyndrome may be performed.

The strong family history of renal disease with associated hearing deficits reported for theboy in the vignette is very suggestive of Alport syndrome. Based on the usual mode ofinheritance, it can be assumed that the boy will develop end-stage renal disease, but his sistermay develop only minor kidney disease or simply be a carrier of the gene. The family historysuggests that the boy likely also will develop some degree of hearing deficits. Unusualassociated anomalies, including esophageal leiomyomatosis and giant cell thrombocytopenia,are seen only occasionally in Alport syndrome and usually are associated with autosomalmodes of inheritance. The family history indicates that this boy has inherited the disease throughan X-linked pattern.

References:

Kashtan CE. Familial hematurias: what we know and what we don't. Pediatr Nephrol.2005;20:1027-1035. Abstract available at:http://www.ncbi.nlm.nih.gov/entrez/query.fcgi?orig_db=PubMed&db=PubMed&cmd=Search&term=%22Pediatric+nephrology+(Berlin,+Germany)%22[Jour]+AND+1027[page]+AND+2005[pdat]

Gross O, Netzer KO, Lambrecht R, Seibold S, Weber M. Meta-analysis of genotype-phenotypecorrelation in X-linked Alport syndrome: impact on clinical counselling. Nephrol Dial Transpl.2002;17:1218-1227. Available at: http://ndt.oxfordjournals.org/cgi/content/full/17/7/1218

Longo I, Porcedda P, Mari F, et al. COL4A3/COL4A4 mutations: from familial hematuria toautosomal-dominant or recessive Alport syndrome. Kidney Int. 2002;61:1947-1956. Abstractavailable at:http://www.ncbi.nlm.nih.gov/entrez/query.fcgi?cmd=Retrieve&db=pubmed&dopt=Abstract&list_uids=12028435&query_hl=59&itool=pubmed_docsum

Copyright © 2007 by the American Academy of Pediatrics page 395

Page 396: AAP MCQ 2007

2007 PREP SA on CD-ROM

Question: 190

A 16-year-old previously healthy boy presents with a 2-week history of intermittent elevatedtemperatures to 102ºF (39°C), headache, malaise, fatigue, myalgias, and a progressivelyworsening nonproductive cough. During history-taking, he reports that recently he exploredseveral caves while hiking in a forest preserve in Ohio. Physical examination reveals a tired-appearing adolescent who has a temperature of 102.4ºF (39.1°C), a dry cough, and diffuseintermittent rhonchi on chest auscultation. Laboratory findings include a white blood cell count of12x103/mcL (12x109/L), with 60% neutrophils, 2% band forms, and 38% lymphocytes. Chestradiograph (Item Q190A) shows patchy left upper and left lower lobe opacities and hilaradenopathy.

Of the following, the MOST likely pathogen causing this patient's condition is

A. Aspergillus fumigatus

B. Coccidioides immitis

C. Histoplasma capsulatum

D. Rhizopus sp

E. Sporothrix schenckii

Copyright © 2007 by the American Academy of Pediatrics page 396

Page 397: AAP MCQ 2007

2007 PREP SA on CD-ROM

Preferred Response: CCritique: 190

Histoplasmosis is a common granulomatous infection caused by the dimorphic fungusHistoplasma capsulatum. The clinical spectrum of disease ranges from asymptomatic in 50% ofcases to generalized fatal dissemination in less than 1% of cases. The infection occursworldwide but is most common in the Ohio, Mississippi, and Missouri River valleys in the UnitedStates. Soil contaminated by animal droppings, especially from birds and bats, plays a major rolein disseminating the organism. H capsulatum frequently is isolated from soil near chickenhouses, bird roosts, caves infested with bats, and silos inhabited by pigeons. Active disturbanceof the environment is necessary for dissemination of the spores. Acute pulmonaryhistoplasmosis is the most common form of disease. Asymptomatic primary infection occurs inabout 50% of adults and children exposed to H capsulatum spores. Symptomatic diseasepresents as an influenzalike illness, with abrupt onset of fever, malaise, headache, myalgia, andnonproductive cough, as reported for the boy in the vignette. Chest radiographs reveal patchypneumonic infiltrates and hilar adenopathy.

Aspergillus is a fungus that is ubiquitous in the environment and is found in soil, hay,compost piles, decaying vegetation, water, house dust, bedding, food, medications, surgicaldressings, and potted plants. The clinical features of aspergillosis depend on the host responseto the disease, but differ from those exhibited by the boy described in the vignette. The lungsand paranasal sinuses are the most frequent sites of infection, but disseminated infection mayaffect several body organs.

Disease may be due to hypersensitivity to the organism or invasion of tissues by thefungus. Allergic pulmonary aspergillosis (due to hypersensitivity) is seen in patients who haveasthma; they present with wheezing, fever, dyspnea, and productive cough. Invasive disease(Item C190A) almost exclusively occurs in patients who have underlying conditions,immunocompromised patients who have neutropenia, after medication use that causesneutrophil dysfunction, or after cytotoxic chemotherapy or immunosuppressive therapy. Thecondition is characterized by cough, hemoptysis, obstructive airway signs, and other pulmonarysymptoms. Hematogenous dissemination is seen commonly in the severelyimmunocompromised host, usually in association with invasive pulmonary aspergillosis andneutropenia, with the brain, gastrointestinal tract, heart, liver, and kidneys being the mostfrequently involved sites. The signs and symptoms are related to impaired function of theinvolved organ.

Sporotrichosis is caused by the dimorphic fungus Sporothrix schenckii. It is a subacute orchronic fungal infection that can involve both the superficial and deep tissues. The organism isfound primarily in warm, temperate zones worldwide. In the United States, most cases occur instates bordering the Mississippi and Missouri Rivers. Cases generally follow intradermalinoculation of spores after contact with contaminated thorns, splinters, reeds, and grasses; floralnursery and tree farm personnel are at the highest risk. Lymphocutaneous sporotrichosis is themost common presentation. It initially appears as a small, painless, movable subcutaneousnodule that enlarges to become a fluctuant mass and progresses to ulceration, with theappearance of addition painless subcutaneous nodules along the lymphatic channels. Theclassic clinical picture is an ulcer on the finger or wrist with an associated chain of nodules alongdraining lymphatics. Joints or bone may become infected via dissemination through thebloodstream from a cutaneous focus. Bone lesions are often multiple, lytic, and destructive, andthe tibia is involved most commonly. In joint lesions, the synovium is thickened, and the cartilagedegenerates. The knee is the most commonly affected joint.

Although Coccidioides immitis may produce the symptoms reported for the patient in thevignette, this organism is found in the western desert states of the United States, not in themidwest. Rhizopus sp are opportunistic fungi that cause disease in severelyimmunocompromised individuals.

References:

American Academy of Pediatrics. Aspergillosis. In: Pickering LK, ed. Red Book: 2006 Report ofthe Committee on Infectious Diseases. 27th ed. Elk Grove Village, Ill: American Academy of

Copyright © 2007 by the American Academy of Pediatrics page 397

Page 398: AAP MCQ 2007

2007 PREP SA on CD-ROM

Pediatrics; 2006:219-222

American Academy of Pediatrics. Histoplasmosis. In: Pickering LK, ed. Red Book: 2006 Reportof the Committee on Infectious Diseases. 27th ed. Elk Grove Village, Ill: American Academy ofPediatrics; 2006:371-374

American Academy of Pediatrics. Sporotrichosis. In: Pickering LK, ed. Red Book: 2006 Report ofthe Committee on Infectious Diseases. 27th ed. Elk Grove Village, Ill: American Academy ofPediatrics; 2006:595-597

Hughes WT. Aspergillosis. In: McMillan JA, DeAngelis CD, Feigin RD, Warshaw JB, eds. Oski’sPediatrics Principles and Practice. 3rd ed. Philadelphia, Pa: Lippincott Williams & Wilkins;1999:1155-1156

Leggiadro RJ. Histoplasmosis. In: McMillan JA, DeAngelis CD, Feigin RD, Warshaw JB, eds.Oski’s Pediatrics Principles and Practice. 3rd ed. Philadelphia, Pa: Lippincott Williams & Wilkins;1999:1163-1164

McGinnis MR. Sporotrichosis. In: McMillan JA, DeAngelis CD, Feigin RD, Warshaw JB, eds.Oski’s Pediatrics Principles and Practice. 3rd ed. Philadelphia, Pa: Lippincott Williams & Wilkins;1999:1164-1165

Copyright © 2007 by the American Academy of Pediatrics page 398

Page 399: AAP MCQ 2007

2007 PREP SA on CD-ROM

Question: 191

The parents of an 8-year-old girl who has asthma would like to know more about possibletriggers. The girl has had mild persistent asthma since age 5 years and frequently hasincreased symptoms in the fall that coincide with starting school. During the fall, she experiencesincreased rhinorrhea, sneezing, and ocular pruritus. These nasal and ocular symptoms haveoccurred each fall for the past 3 years but generally do not occur at other times of the year.

Of the following, the MOST likely trigger for the patient's asthma exacerbations is

A. allergic rhinitis

B. exercise

C. gastroesophageal reflux

D. psychogenic cough

E. sinusitis

Copyright © 2007 by the American Academy of Pediatrics page 399

Page 400: AAP MCQ 2007

2007 PREP SA on CD-ROM

Preferred Response: ACritique: 191

The girl described in the vignette most likely is experiencing asthma exacerbations in the fall dueto seasonal allergic rhinitis. Viral illnesses and allergic rhinitis are two common asthma triggersfor school-age children. Further, approximately 80% of children who have asthma developallergic rhinitis and demonstrate positive skin tests to aeroallergens.

Exercise may result in asthma symptoms, but the relative absence of symptoms duringother times of year for the girl in the vignette makes this less likely. Gastroesophageal reflux(GER) can cause cough and exacerbate asthma. Eliciting the complaint of heartburn orregurgitation can be difficult in a child, but the relative seasonality of symptoms for this girlmakes GER unlikely. Stress related to starting school may result in a psychogenic cough orschool absence, but psychogenic cough is a diagnosis of exclusion. Also, the lack of identifiablestressors at school (eg, bullying) or home (eg, divorce) and the ocular pruritus and sneezingmake psychogenic cough unlikely. Sinusitis may be a complication of uncontrolled allergicrhinitis, but the absence of purulent rhinorrhea, sinus tenderness, or fever makes sinusitis lesslikely than seasonal allergic rhinitis.

References:

Liu AH, Spahn JD, Leung DYM. Childhood asthma. In: Behrman RE, Kliegman RM, Jenson HB,eds. Nelson Textbook of Pediatrics. 17th ed. Philadelphia, Pa: WB Saunders Co 2004:760-774

Chan EY, Dundas I, Bridge PD, Healy MJ, McKenzie SA. Skin-prick testing as a diagnostic aidfor childhood asthma. Pediatr Pulmonol. 2005;39:558-562. Abstract available at:http://www.ncbi.nlm.nih.gov/entrez/query.fcgi?orig_db=PubMed&db=PubMed&cmd=Search&term=Pediatr+Pulmonol[Jour]+AND+558[page]+AND+2005[pdat]

Copyright © 2007 by the American Academy of Pediatrics page 400

Page 401: AAP MCQ 2007

2007 PREP SA on CD-ROM

Question: 192

A 16-year-old male was involved in a brawl with other teens. He was struck a number of timeswith fists and clubs, and he admits to punching some of his adversaries as well. He presents 6hours later with complaint of pain in his wrist. Physical examination reveals moderate swellingand tenderness on the dorsum of the hand in the region of the base of the thumb and indexfinger. A radiograph of the hand reveals a fracture.

Of the following, the fracture that is associated with the POOREST prognosis for healing is a

A. first metacarpal fracture

B. fracture to the proximal phalanx of the thumb

C. lunate fracture

D. Salter II fracture of the distal radius

E. scaphoid fracture

Copyright © 2007 by the American Academy of Pediatrics page 401

Page 402: AAP MCQ 2007

2007 PREP SA on CD-ROM

Preferred Response: ECritique: 192

Carpal bone fractures in children are rare due to the largely cartilaginous nature of these bonesin early childhood. Most of the carpal bones are ossified by early adolescence, resulting infracture patterns beyond that age being similar to adult injuries. The scaphoid (Item C192A) isthe most commonly fractured carpal bone in children, usually as a result of a fall ontooutstretched hands. The classic physical finding is tenderness in the “anatomic snuffbox” justdistal to the radius where the scaphoid articulates with the radius.

A high index of suspicion must be maintained when evaluating a hand or wrist injury in achild due to a significant risk of nonunion of the fractured scaphoid if it is not immobilized properlyin a plaster cast. Unacceptable functional defects can result, requiring surgical interventions torepair the damage. The rate of scaphoid nonunion in children who are properly immobilized maybe as low as 1%, significantly lower than the 5% to 12% rate found in adults.

Carpal lunate fractures are exceedingly rare injuries in children, with only isolated casereports in the literature. There are no data indicating a high rate of complications. Fractures tothe first metacarpal, proximal phalanx, and distal radius are relatively common. Except in casesof severe disruption of the anatomy or open fractures, uneventful healing is the expectedoutcome.

References:

Goddard N. Carpal fractures in children. Clin Orthop Relat Res. 2005;432:73-76. Abstractavailable at:http://www.ncbi.nlm.nih.gov/entrez/query.fcgi?orig_db=PubMed&db=PubMed&cmd=Search&term=432[volume]+AND+73[page]+AND+2005[pdat]+AND+Goddard+N[author]

Toh S, Miura H, Arai K, Uasumura M, Wada M, Tsubo K. Scaphoid fractures in children:problems and treatment. J Pediatr Orthop. 2003;23:216-221. Abstract available at:http://www.ncbi.nlm.nih.gov/entrez/query.fcgi?orig_db=PubMed&db=PubMed&cmd=Search&term=J+Pediatr+Orthop[Jour]+AND+216[page]+AND+2003[pdat]

Henderson B, Letts M. Operative management of pediatric scaphoid fracture nonunion. J PediatrOrthop. 2003;23:402-406. Abstract available at:http://www.ncbi.nlm.nih.gov/entrez/query.fcgi?orig_db=PubMed&db=PubMed&cmd=Search&term=J+Pediatr+Orthop[Jour]+AND+402[page]+AND+2003[pdat]

Dias JJ, Wildin CJ, Bhowal B, Thompson JR. Should acute scaphoid fractures be fixed? Arandomized controlled trial. J Bone Joint Surg. 2005;87:2160-2168. Abstract available at:http://www.ncbi.nlm.nih.gov/entrez/query.fcgi?orig_db=PubMed&db=PubMed&cmd=Search&term=J+Bone+Joint+Surg[Jour]+AND+2160[page]+AND+2005[pdat]

Copyright © 2007 by the American Academy of Pediatrics page 402

Page 403: AAP MCQ 2007

2007 PREP SA on CD-ROM

Question: 193

A 7-year-old girl is hospitalized after the acute onset of fever, rapid development of hypotension,diffuse erythema of the skin (Item Q193A), rapidly accelerating renal failure, and multisystemorgan involvement. Toxic shock syndrome (TSS) is diagnosed.

Of the following, the MOST likely finding associated with TSS caused by toxin-producingStreptococcus pyogenes is

A. foreign body at the site of infection

B. necrotizing fasciitis

C. recurrent episodes of S pyogenes infection

D. severe myalgias

E. widespread blistering of the skin

Copyright © 2007 by the American Academy of Pediatrics page 403

Page 404: AAP MCQ 2007

2007 PREP SA on CD-ROM

Preferred Response: BCritique: 193

Toxic shock syndrome (TSS) is a multisystem disease caused by toxin-producing strains ofStaphylococcus aureus or Streptococcus pyogenes. Both forms are characterized by shockand widespread macular erythema. Soft-tissue infections often are seen in association withstreptococcal TSS. Necrotizing fasciitis is a particularly serious complication of S pyogenes TSSthat constitutes a surgical emergency.

Profuse watery diarrhea, vomiting, conjunctival injection, and severe myalgias areassociated with S aureus-mediated TSS, but are less common in the streptococcal form of thedisease. If a focus of infection is identified, the presence of a foreign body at this site is commonwith S aureus-mediated TSS, but not with S pyogenes-mediated TSS. Neither type of TSS isassociated with widespread blistering of the skin. Recurrent episodes of S pyogenes infectionare not a common outcome following TSS.

References:

American Academy of Pediatrics. Toxic shock syndrome. In Pickering LK, ed. Red Book: 2006Report of the Committee on Infectious Diseases. 27th ed. Elk Grove Village, Ill: AmericanAcademy of Pediatrics; 2006:660-665

Darmstadt G. Bacterial infections: streptococcal and staphylococcal infections. In: SchachnerLA, Hansen RC, eds. Pediatric Dermatology. 3rd ed. St. Louis, Mo: Mosby; 2003:989-1011

Gerber MA. Group A Streptococcus. In: Behrman RE, Kliegman RM, Jenson HB, eds. NelsonTextbook of Pediatrics. 17th ed. Philadelphia, Pa: WB Saunders Co; 2004:874-878

Copyright © 2007 by the American Academy of Pediatrics page 404

Page 405: AAP MCQ 2007

2007 PREP SA on CD-ROM

Question: 194

A 2-month-old exclusively breast-fed infant presents to your office because his mother thinksthat he is irritable. His mother reports that the infant has been passing loose stools and crieswhen he has a bowel movement. He is generally happy at other times. Physical examinationdemonstrates a healthy, afebrile, vigorous infant who has normal skin color. Cardiac, pulmonary,and abdominal examination findings are all normal. Anal inspection demonstrates no fissures. Astool specimen has reddish flecks, and the guaiac test is positive.

Of the following, the BEST next step is to

A. begin therapy with oral amoxicillin

B. institute a trial of lansoprazole

C. obtain an upper gastrointestinal radiography series

D. remove milk products from the maternal diet

E. send stool for Clostridium difficile toxin testing

Copyright © 2007 by the American Academy of Pediatrics page 405

Page 406: AAP MCQ 2007

2007 PREP SA on CD-ROM

Preferred Response: DCritique: 194

The infant described in the vignette is passing small amounts of blood in the stool, but appearsotherwise healthy without any sign of infection. Although bacterial colitis is a possibility, the mostlikely diagnosis is allergic colitis. The first-line treatment of this condition is dietary restriction ofmilk protein from the mother's diet. Oral amoxicillin is not indicated unless there is evidence ofcolitis caused by an amoxicillin-sensitive pathogen. Lansoprazole, a proton pump inhibitor, is fortreatment of gastroesophageal reflux. An upper gastrointestinal barium radiograph is used todiagnose malrotation, pyloric stenosis, and inflammatory bowel disease, but this presentation isnot consistent with those conditions. Clostridium difficile is present in the stool of 25% of healthyterm infants, but it rarely, if ever, causes colitis in this age group.

Infantile allergic proctocolitis typically presents in infants younger than 3 months of age andis characterized by loose stool that contains small amounts of blood and mucus. If performed,endoscopic examination demonstrates an erythematous colon with lymphoid nodules, andbiopsy shows an inflammatory infiltrate comprised primarily of eosinophils (Item C194A). Themost common offending antigens triggering the inflammatory response are milk and soy, butother antigens, including wheat, eggs, corn, fish, and nuts, also may play a role. Allergic colitismay occur in both breastfed and bottle-fed infants; in the case of breastfed infants, it is believedthe immunogenic peptides come from the maternal diet and pass into the mother’s milk. Althoughgenerally believed to be a benign condition that resolves by 1 year of age, rare cases of severeanemia and colonic stricture have been reported with allergic colitis. Therefore, the initialrecommended management involves restriction of milk or soy from the infant’s and mother'sdiets. In most infants, the rectal bleeding resolves within 3 weeks after dietary restriction,although small amounts of microscopic bleeding may persist. By 1 year of age, most infants canbe rechallenged successfully with milk or soy.

References:

Chang J-W, Wu T-C, Wang K-S, Huang I-F, Huang B, Yu I-T. Colon mucosal pathology ininfants under three months of age with diarrhea disorders. J Pediatr Gastroenterol Nutr.2002;35:387-390. Available at: http://www.jpgn.org/pt/re/jpgn/abstract.00005176-200209000-00031.htm;jsessionid=G1XZJHD0FWF6q3h0vtvs5tQTH2PFpGn5Z7s5YBXrxTQtRcWslPJl!1941873617!-949856145!8091!-1?index=1&database=ppvovft&results=1&count=10&searchid=1&nav=search

Odze RD, Wershil BK, Leichtner AM, Antonioli DA. Allergic colitis in infants. J Pediatr.1995;126:163-170. Abstract available at:http://www.ncbi.nlm.nih.gov/entrez/query.fcgi?cmd=Retrieve&db=pubmed&dopt=Abstract&list_uids=7844660&query_hl=10&itool=pubmed_docsum

Copyright © 2007 by the American Academy of Pediatrics page 406

Page 407: AAP MCQ 2007

2007 PREP SA on CD-ROM

Question: 195

You are called to evaluate a male infant at 50 hours of age because he has not voided. He wasborn at term and has breastfed poorly, but has passed stool. He appears uncomfortable onphysical examination, with a large abdomen and seemingly palpable bladder. There is norespiratory distress. The external genitalia are normal, and both testes descended.

Of the following, the MOST appropriate initial step in this infant's evaluation is

A. consultation with a urologist

B. intravenous pyelography

C. nuclear renal scan

D. passing of a urinary catheter

E. renal ultrasonography

Copyright © 2007 by the American Academy of Pediatrics page 407

Page 408: AAP MCQ 2007

2007 PREP SA on CD-ROM

Preferred Response: DCritique: 195

The causes of acute renal failure (ARF) in the newborn include: dehydration, sepsis, congestiveheart failure, toxic effects of certain drugs (angiotensin-converting enzyme inhibitors,indomethacin, amphotericin), acute tubular necrosis, congenital renal dysplasia, renal veinthrombosis, and obstructive uropathies (posterior urethral valves, neurogenic bladder, bilateralureteropelvic junction or ureterovesical junction obstruction).

The infant described in the vignette, who has not passed urine 50 hours after birth and whohas a palpably full bladder, should be evaluated thoroughly. Anuria in a newborn for more than24 hours warrants an evaluation that includes a thorough history (including obstetric history anddrug exposure), review of any prenatal ultrasonography images, assessment for evidence ofperinatal asphyxia or sepsis, and a family history of renal disease. The physical examinationshould address the genitalia, abdomen, and flanks as well as any signs of edema oroligohydramnios sequence. Laboratory evaluation should attend to serum electrolytes, ureanitrogen and creatinine, urinalysis, urine culture, and urinary electrolytes and creatinine. Toobtain urine and complete the evaluation of the patient, a urinary catheter must be placed. Doingso will answer two questions:

•Are the urethra and bladder outlet patent or obstructed?•Have the kidneys produced any urine?

Once urine is obtained, the laboratory evaluation can be completed and a fractionalexcretion of sodium (FENa) calculated to assist in distinguishing prerenal ARF (FENa <1.0) fromintrinsic ARF (FENa >1.0). If no urine can be obtained by catheterization, ultrasonography of thekidneys, ureters, and bladder should be performed. Neither nuclear renal scan nor intravenouspyelography should be performed until urine flow is established. A urologist may be consulted,but bladder catheterization should be performed first.

References:

Drukker A, Guignard J-P. Renal aspects of the term and preterm infant: a selective update. CurrOpin Pediatr. 2002;14:175-182. Available at: http://www.co-pediatrics.com/pt/re/copeds/abstract.00008480-200204000-00006.htm;jsessionid=Ej9NXjYM7YHFOQLxSd9nIX3QlfWf1CTVttYNXrcN5328kqV1ogrB!-1861031787!-949856144!9001!-1

Gouyon JB, Guignard JP. Management of acute renal failure in newborns. Pediatr Nephrol.2000;14:1037-1044. Abstract available at:http://www.ncbi.nlm.nih.gov/entrez/query.fcgi?cmd=Retrieve&db=pubmed&dopt=Abstract&list_uids=10975322&query_hl=42&itool=pubmed_docsum

Haycock GB. Management of acute and chronic renal failure in the newborn. Semin Neonatol.2003;8:325-334. Abstract available at:http://www.ncbi.nlm.nih.gov/entrez/query.fcgi?cmd=Retrieve&db=pubmed&dopt=Abstract&list_uids=15001136&query_hl=44&itool=pubmed_docsum

Vogt VA, Dell KM, Davis ID. The kidney and urinary tract. In: Martin RJ, Fanaroff AA, WalshMC, eds. Fanaroff and Martin’s Neonatal-Perinatal Medicine: Diseases of the Fetus and Infant.8th ed. Philadelphia, Pa: Mosby-Elsevier; 2006:1659-1684

Copyright © 2007 by the American Academy of Pediatrics page 408

Page 409: AAP MCQ 2007

2007 PREP SA on CD-ROM

Question: 196

An African-American mother brings her previously healthy 2-year-old son to the emergencydepartment because he looked pale when he woke up this morning. She also reports fever anddecreased activity over the last few days. He was diagnosed with a urinary tract infection andgiven trimethoprim-sulfamethoxazole 3 days ago. On physical examination, he is markedly paleand appears jaundiced. His heart rate is 170 beats/min, and his respiratory rate is 30breaths/min. The rest of his physical examination findings are unremarkable. Laboratoryevaluation reveals a hemoglobin concentration of 5 g/dL (50 g/L).

Of the following, the MOST likely cause of this boy's anemia is

A. aplastic anemia

B. glucose-6-phosphate dehydrogenase deficiency

C. hereditary spherocytosis

D. sickle cell hemolytic crisis

E. transient erythroblastopenia of childhood

Copyright © 2007 by the American Academy of Pediatrics page 409

Page 410: AAP MCQ 2007

2007 PREP SA on CD-ROM

Preferred Response: BCritique: 196

The sudden onset of pallor and jaundice in a healthy child is suggestive of acute hemolyticanemia, and the history of recent use of trimethoprim-sulfamethoxazole, as reported for the boyin the vignette, should raise the suspicion of glucose-6-phosphate dehydrogenase (G6PD)deficiency. G6PD is an enzyme used for glucose metabolism. Deficiency of the enzyme resultsin a decrease in reduced glutathione, which is important for the removal of oxygen radicals.When oxidant drugs are administered to those who have G6PD deficiency, the oxygen radicalscannot be removed adequately, and the red cell membrane is damaged. Trimethoprim-sulfamethoxazole is an oxidant drug commonly prescribed for children. Other oxidant drugsinclude chloramphenicol, nitrofurantoin, primaquine, and acetylsalicylic acid. Moderate-to-severeillness may be another cause of oxidant stress without medication use and may be an incitingfactor for acute hemolysis in those who have G6PD deficiency.

There are many forms of G6PD deficiency, which is X-linked in inheritance. The A-minustype is seen in African-American males. In this type, the activity of the G6PD enzyme diminisheswith the age of the red blood cell. Affected patients are asymptomatic until they take an oxidantdrug or become moderately ill, at which time they develop pallor, jaundice, and fatigue.Hemoglobin and hematocrit concentrations fall dramatically, and reticulocytosis occurs. Theyoung reticulocytes produced by patients who have A-minus type G6PD deficiency contain anadequate amount of G6PD, so resolution occurs spontaneously. The Mediterranean type ofG6PD deficiency also is characterized by severe hemolysis with exposure to oxidant stress.However, enzyme deficiency in this type is seen in red blood cells of all ages, so reticulocytosisdoes not result in spontaneous recovery.

Aplastic anemia and transient erythroblastopenia of childhood are disorders of red cellproduction, so jaundice is not a typical feature. Children who have hereditary spherocytosiseither present with neonatal hyperbilirubinemia or later with splenomegaly and mild-to-moderateanemia. Children who have sickle cell disease are likely to develop hemolytic crisis with illness,but symptoms of vaso-occlusive crisis usually occur in the first postnatal year, making this anunusual diagnosis in a previously healthy 2-year-old.

References:

Segel GB, Hirsh MG, Feig SA. Managing anemia in pediatric office practice: part 1. Pediatr Rev.2002;23:75-84. Available at: http://pedsinreview.aappublications.org/cgi/content/full/23/3/75

Segel GB, Hirsch MG, Feig SA. Managing anemia in pediatric office practice: part 2. Pediatr Rev.2002;23:111-121. Available at: http://pedsinreview.aappublications.org/cgi/content/full/23/4/111

Copyright © 2007 by the American Academy of Pediatrics page 410

Page 411: AAP MCQ 2007

2007 PREP SA on CD-ROM

Question: 197

A 2-year-old boy is pulled from the pool by the lifeguard after slipping into the water while hismother was caring for another child poolside.

Of the following, the factor MOST predictive of a favorable prognosis is

A. no seizure activity

B. no vomiting occurred

C. palpable pulse on arrival at the hospital after cardiopulmonary resuscitation en route

D. palpable pulse when emergency medical services arrive at scene

E. estimated submersion time of less than 3 minutes

Copyright © 2007 by the American Academy of Pediatrics page 411

Page 412: AAP MCQ 2007

2007 PREP SA on CD-ROM

Preferred Response: DCritique: 197

Near-drowning events among children generally have one of two outcomes: survival withvirtually no neurologic sequelae or death soon after arrival at the hospital. The single mostimportant predictor of outcome for a near-drowning in a pediatric patient is whether cardiacarrest has occurred. Those who survive cardiac arrest do so only if cardiopulmonaryresuscitation is required for fewer than 10 minutes. Thus, patients who have palpable pulses atthe scene are most likely to survive and have a good prognosis.

Other factors in the prognosis include presence of coma, hyperglycemia, and pupillaryresponse. Persistence of coma or lack of purposeful movements by 24 hours after thesubmersion event correlates with poor neurologic outcome.

The presence of seizure activity may signify central nervous system injury from hypoxia ormay be the cause of the submersion injury. Vomiting is common in patients who survive near-drowning and may be associated with aspiration and lung injury. Length of submersion isimportant as a prognostic sign but is rarely known definitively. Thus, absence of seizure activity,presence of vomiting, or submersion time of less than 3 minutes are not predictive of a favorableprognosis.

Submersion injury associated with marked hypothermia (core temperature less than 82.4°F[28°C]) may help protect the central nervous system and myocardium. Resuscitation in suchpatients should continue without interruption during rewarming to a core temperature to 89.6°F(32°C).

References:

Fiore M, Heidemann S. Near drowning. eMedicine Specialties: Pediatrics: Critical Care. 2004.Available at: http://www.emedicine.com/ped/topic2570.htm

Glaze DG. The comatose child. In: McMIllan JA, DeAngelis CD, Feigin RD, Warshaw JB, eds.Oski’s Pediatrics: Principles and Practice. 3rd ed. Philadelphia, Pa: Lippincott, Williams & Wilkins;1999:1952-1956

Quan L. Near-drowning. Pediatr Rev. 1999;20:255-260. Available at:http://pedsinreview.aappublications.org/cgi/content/full/20/8/255

Copyright © 2007 by the American Academy of Pediatrics page 412

Page 413: AAP MCQ 2007

2007 PREP SA on CD-ROM

Question: 198

A 4-month-old is brought to clinic by his parents for evaluation of bilateral droopy eyes. Hismother believes this has developed just over the last week. The child recently started takingcereal in addition to breastfeeding and has been constipated. Physical examination revealsdroopy eyelids (Item Q198A) and 1+ deep tendon reflexes diffusely.

Of the following, the MOST likely diagnosis is

A. botulism

B. blepharophimosis syndrome

C. congenital ptosis

D. muscular dystrophy

E. myasthenia gravis

Copyright © 2007 by the American Academy of Pediatrics page 413

Page 414: AAP MCQ 2007

2007 PREP SA on CD-ROM

Preferred Response: ACritique: 198

Ptosis may be congenital or acquired. Congenital ptosis (Item C198A) is associated with a widevariety of lid dysfunctions, may be unilateral or bilateral, and may have a neural or muscularorigin. Ptosis may arise from a congenital or new third nerve origin, with severe lid weaknessand often associated with superior rectus weakness. Ptosis resulting from sympatheticdysfunction usually presents unilaterally, with no more than 2- to 3-mm lid weakness. Theresponsible lesion arises anywhere along the course of the sympathetic pathway from thehypothalamus to the orbit. Such ptosis usually is accompanied by miosis and anhidrosis (Hornersyndrome) and can be congenital or newly acquired. Neuromuscular causes for ptosis thathave a new-onset neuropathic or myopathic basis include myasthenia gravis, mitochondrialdisease (chronic progressive external ophthalmoplegia), and toxin exposures, such as botulism,diphtheria, tick paralysis, insecticides, and vincristine.

The bilateral ptosis, combined with constipation and diminished reflexes, all developingduring the introduction of solid foods described for the infant in the vignette is classic forbotulism. Congenital ptosis would have been present since birth, and myasthenia producesvariable ptosis, not seen in this baby. Most muscular dystrophies, except myotonic dystrophy,do not produce ptosis. Blepharophimosis syndrome is an autosomal dominant condition that hashigh penetrance, producing bilateral and severe ptosis as well as other abnormalities, such asconstricted lids, increased distance between the medial canthi, absent epicanthal folds, flat nasalbridge, and low-set ears.

References:

Hoyt CS, Good WV. Ophthalmic problems in childhood. In: Berg BO, ed. Child Neurology: AClinical Manual. 2nd ed. Philadelphia, Pa: JB Lippincott Co; 1994:241-255

Olitsky SE, Nelson LB. Abnormalities of the lids. In: Behrman RE, Kliegman RM, Jenson HB,eds. Nelson Textbook of Pediatrics. 17th ed. Philadelphia, Pa: WB Saunders Co; 2004:2097-2098

Copyright © 2007 by the American Academy of Pediatrics page 414

Page 415: AAP MCQ 2007

2007 PREP SA on CD-ROM

Question: 199

A 3-year-old girl is brought to your office for re-evaluation of a fever that began 6 days ago. Hermother tells you that her daughter's temperature has been as high as 102.2°F (39°C). Herphysical examination was unremarkable when you examined her 3 days ago, but today you noteinjected sclera; cracked, red lips (Item Q199A); a strawberry appearance of her tongue; and aswollen, nontender, cervical node. You tell her mother that you believe this is Kawasaki disease.

Of the following, the MOST appropriate statement to make to the mother is that

A. an exercise stress test should be performed as a baseline study

B. aspirin therapy will be used until the fever subsides

C. cardiac involvement may include abnormalities of the coronary arteries or the myocardium

D. echocardiography should be performed to evaluate for the presence of coronary aneurysms

E. immediate treatment with intravenous immune globulin will eliminate the chance of coronaryinvolvement

Copyright © 2007 by the American Academy of Pediatrics page 415

Page 416: AAP MCQ 2007

2007 PREP SA on CD-ROM

Preferred Response: CCritique: 199

Kawasaki disease (KD) is believed to be a multisystem illness characterized by vasculitis ofsmall- and medium-size blood vessels, including the coronary arteries. Although the precisecause remains unclear, recent investigations suggest that at least some of the cases are relatedto an infectious organism (New Haven coronavirus). The median age of patients affected by KDis 2.3 years, and nearly 80% of cases occur in children younger than 4 years of age. Morecases are reported in the winter and spring than during the summer and fall, and males areaffected approximately 1.5 times more frequently than females. KD can be seen in patients of allethnicities, races, and cultures, but it is most prevalent in those of Asian descent.

KD is diagnosed using a set of diagnostic criteria that include the presence of fever for atleast 5 days and at least four of the five following clinical findings:

•Changes in the extremities (edema, erythema, desquamation [Item C199A])

•Polymorphous exanthem (Item C199B) that usually is truncal

•Conjunctival injection (Item C199C)

•Erythema or fissuring of the lips (Item C199D), oral cavity, and tongue•Cervical lymphadenopathy

In addition, the diagnosis of KD requires that the constellation of findings with which thepatient presents cannot be explained by any other known disease process. Some children mayhave an incomplete form of the disease but still are at risk for its complications.

KD has an acute phase that lasts 1 to 2 weeks after the onset of fever, which is when thediagnostic criteria typically are present. The subacute phase follows from 2 to 8 weeks afteronset. Patients may demonstrate desquamation of the fingers and toes. It is during this phasewhen coronary aneurysms may develop, particularly in those who have not been treated withintravenous gamma globulin during the acute phase. The convalescent phase lasts for monthsfollowing the illness.

Patients who have KD may have involvement of the cardiovascular system that can includemyocarditis, valvulitis, or arteritis. The latter is diagnosed by the presence of edematouscoronary walls on echocardiography, usually between days 5 and 8 of the illness. Coronaryartery aneurysms can develop after the acute phase and typically in the subacute phase. Theincidence of coronary aneurysms may be as high as 22% to 25% in untreated patientscompared with an incidence of 3% to 5% in those who receive intravenous immune globulintherapy in the acute phase.

Exercise stress testing is not part of the initial evaluation and cannot be performed reliably ina child as young as the one in the vignette; it usually is reserved for older, more cooperativechildren toward the age of 7 to 8 years. High-dose aspirin therapy (80 to 100 mg/kg per day) isadministered until the patient is afebrile for 48 hours, at which time the dose is decreased to 3 to5 mg/kg per day for 6 to 8 weeks or until platelet concentrations normalize. Echocardiographyusually is performed at the time of diagnosis to assess for the presence of a subclinicalmyocarditis, to evaluate for coronary arteritis, and to serve as a baseline for future studies.Aneurysms of the coronary arteries are not seen in the acute phase and, therefore, theirabsence on echocardiography should reassure neither the parents nor the pediatrician.

References:

Esper F, Shapiro ED, Weibel C, Ferguson D, Landry ML, Kahn JS. Association between a novelhuman coronavirus and Kawasaki disease. J Infect Dis. 2005;191:499-502. Available at:http://www.journals.uchicago.edu/JID/journal/issues/v191n4/33623/33623.html

Newburger JW, Takahashi M, Gerber MA, et al. Diagnosis, treatment, and long-termmanagement of Kawasaki disease: a statement for health professionals from the Committee onRheumatic Fever, Endocarditis, and Kawasaki Disease, Council on Cardiovascular Disease in

Copyright © 2007 by the American Academy of Pediatrics page 416

Page 417: AAP MCQ 2007

2007 PREP SA on CD-ROM

the Young, American Heart Association. Pediatrics. 2004;114:1708-1733. Available at:http://pediatrics.aappublications.org/cgi/content/full/114/6/1708

Copyright © 2007 by the American Academy of Pediatrics page 417

Page 418: AAP MCQ 2007

2007 PREP SA on CD-ROM

Question: 200

A newborn female has an open neural tube defect, low-set ears, ventricular septal defect, andrib and vertebral column malformations.

Of the following, the MOST likely maternal condition that was present during this pregnancy is

A. alcoholism

B. diabetes mellitus

C. hypothyroidism

D. iodine deficiency

E. syphilis

Copyright © 2007 by the American Academy of Pediatrics page 418

Page 419: AAP MCQ 2007

2007 PREP SA on CD-ROM

Preferred Response: BCritique: 200

Maternal diabetes mellitus can have profound effects on the development and health of the fetus.The severity of maternal disease and the subsequent fetal effects can vary considerably.Because the maternal diabetic state can be present from the time of conception, early prenataleffects can result in malformations, growth deficiency, and stillbirth. There is a threefold increasein malformations among offspring of diabetic mothers; the incidence is correlated with theseverity and level of control of the maternal illness. The most common defects involve the heart,central nervous system, kidneys, and skeleton, as described for the infant in the vignette. Of thecardiac defects, ventricular septal defect, transposition of the great vessels, and dextrocardiaare most common. Central nervous system defects can range from anencephaly orholoprosencephaly to spina bifida and hydrocephalus. Malformations of the lower spine alsooccur and are termed the caudal regression syndrome. The spine may be segmenteddefectively or terminate in the sacral or lumbar region, resulting in abnormal neurologic functionbelow the level of the defect. Rib defects also may be seen.

Infants of diabetic mothers also may present in the newborn period with macrosomia due tohyperinsulinemia and excessive glucose availability. The macrosomia affects both linear growthand weight. Alternatively, if the diabetic mother has substantial vascular disease, fetal growthcan be impaired, resulting in growth restriction. Other complications in infants of diabetic mothersinclude hyperbilirubinemia, hypoglycemia, vascular thromboses, respiratory distress, and birthinjury due to macrosomia.

Fetal alcohol syndrome is characterized by prenatal growth deficiency, microcephaly, andcardiac defects. Neural tube and vertebral column defects are not common features. Maternalhypothyroidism has little effect on the fetus, which produces its own thyroid hormone; womenwho have untreated hypothyroidism have been reported to give birth to healthy offspring.Maternal iodine deficiency can cause fetal deficiency of the mineral, which results in goiter, signsof cretinism, retarded bone growth, constipation, umbilical hernia, and mottling in the newborn.Prompt treatment with iodine is necessary to prevent mental retardation. Maternal syphilis canaffect the fetal skin, mucous membranes, liver, central nervous system, and bones. Cardiacanomalies and open neural tube defects are not common features.

References:

Jones KL. Dysmorphology. In: Behrman RE, Kliegman RM, Jenson HB, eds. Nelson Textbookof Pediatrics. 17th ed. Philadelphia Pa: Saunders; 2004:616-623

Riskin A, Haney PM. Infant of a diabetic mother. UpToDate. 2006;14.1. Available at:http://www.utdol.com/utd/content/topic.do?topicKey=neonatol/7268&type=A&selectedTitle=1~12

Stoll BJ, Kliegman RM. The endocrine system: infants of diabetic mothers. In: Behrman RE,Kliegman RM, Jenson HB, eds. Nelson Textbook of Pediatrics. 17th ed. Philadelphia Pa:Saunders; 2004:613-614

Copyright © 2007 by the American Academy of Pediatrics page 419

Page 420: AAP MCQ 2007

2007 PREP SA on CD-ROM

Question: 201

The mother of a 14-year-old boy arranges to meet with you privately before the boy's annualhealth supervision visit. She is concerned because he is quiet, has no athletic interests, and hasonly a few friends. He is content to spend the weekend shopping, cooking, reading, and listeningto music. Although he doesn't like school, he is an honor student. The mother also tells you herhusband's youngest brother recently disclosed his homosexuality and wants to introduce hismale partner to the extended family.

Of the following, while counseling the mother, you are MOST likely to include a statement that

A. compared with heterosexual peers, gay high school students are more likely to abusesubstances

B. self-awareness of sexual orientation is established by age 5 years

C. sexual orientation is culturally determined

D. sexual play with same-sex friends is a clear marker for homosexuality

E. she should explain to her son that he is free to choose his sexual orientation

Copyright © 2007 by the American Academy of Pediatrics page 420

Page 421: AAP MCQ 2007

2007 PREP SA on CD-ROM

Preferred Response: ACritique: 201

Pediatricians should be aware that some adolescents may have concerns about their sexualorientation or that of siblings, friends, parents, or other relatives. Likewise, parents may havequestions or concerns about the sexual orientation of their children. The term “sexualorientation” refers to an individual’s pattern of physical and emotional arousal toward otherpersons. Homosexual individuals are attracted to persons of the same sex. Gender identity isthe knowledge of oneself as being male or female. Sexual orientation is not synonymous withsexual activity or sexual behavior. Homosexual individuals may engage in sexualactivities/behaviors with persons of the same or opposite sex. Sexual orientation, which isbiologically based, is not determined by any one factor, but by a combination of genetic,hormonal, and environmental influences. Theories that adverse life events (eg, abnormalparenting, sexual abuse) influence sexual orientation have not been substantiated. Homosexualyouth are at a higher risk of school drop-out, homelessness, and substance abuse. They alsoare more likely to be threatened at school. Homosexual males are more likely to attempt suicidethan heterosexual males.

Although sexual orientation is believed to be established in early childhood, a time whenhomosexual individuals experience a sense of being different, a self-awareness of homosexualidentity develops during the adolescent years. Cultural taboos or social intolerance may inhibithomosexual disclosure and behavior, but same-sex experimentation in early adolescence is notuncommon and is not a marker for homosexuality. Sexual behavior/activity is a choice; sexualorientation is inherent and is not a choice.

Sexuality education should be an integrated component of comprehensive health care andthe longitudinal relationship that pediatricians develop with children, adolescents, and theirfamilies. During the early adolescent years, both adolescents and their parents will havequestions about puberty and sexuality that offer counseling opportunities. Important points forpediatricians who are counseling parents about their adolescent’s emerging sexuality are listedin Item C201A.

Parents should be encouraged to allow their fifth and sixth grade children to participate inschool sexual health education programs that provide information about puberty and balanceabstinence messages with decision making, including contraception and sexually transmitteddisease prevention strategies. Successful programs provide opportunities for youngadolescents to practice communication and negotiation skills and include information aboutaccess to services.

References:

Committee on Psychosocial Aspects of Child and Family Health and Committee onAdolescence. Sexuality education for children and adolescents. Pediatrics. 2001;108:498-502.Available at: http://pediatrics.aappublications.org/cgi/content/full/108/2/498

Frankowski BL and Committee on Adolescence. Sexual orientation and adolescents. Pediatrics.2004;113:1827-1832. Available at:http://pediatrics.aappublications.org/cgi/content/full/113/6/1827

Gutgesell ME, Payne N. Issues of adolescent psychological development in the 21st century.Pediatr Review. 2004;25:79-85. Available at:http://pedsinreview.aappublications.org/cgi/content/full/25/3/79

Sieving RE, Oliphant JA, Blum RW. Adolescent sexual behavior and sexual health. Pediatr Rev.2002;23:407-416. Available at: http://pedsinreview.aappublications.org/cgi/content/full/23/12/407

Copyright © 2007 by the American Academy of Pediatrics page 421

Page 422: AAP MCQ 2007

2007 PREP SA on CD-ROM

Question: 202

You are evaluating a short 14-year-old boy who is underweight for height. His growth curve isshown in Item Q202A. He says he is feeling well. His mother worries that he does not have asmuch stamina as he did at age 12 years.

Of the following, the laboratory study that is MOST likely to be useful in assessing the reason forpoor growth is measurement of

A. free thyroxine

B. insulin-like growth factor 1

C. insulin-like growth factor binding protein 3

D. tissue transglutaminase antibody

E. urine free cortisol

Copyright © 2007 by the American Academy of Pediatrics page 422

Page 423: AAP MCQ 2007

2007 PREP SA on CD-ROM

Preferred Response: DCritique: 202

Most children who have short stature and slow growth, such as the boy described in thevignette, do not have endocrine disorders. Healthy children may have a slow linear growthvelocity during middle childhood and early adolescent ages if they have constitutional growthdelay. However, severe attenuation of weight gain preceding attenuation of linear growth almostalways is a sign of a severe underlying disorder.

The most common pathologic causes of slow growth are an underlying metabolic, renal, orgastrointestinal disorder. Laboratory tests directed at identifying these disorders can helpdistinguish between constitutional delay and other conditions. Celiac disease is a common causeof short stature associated with underweight. If immunoglobulin A concentrations are normal,measurement of tissue transglutaminase antibody is the best screening study for celiac disease.Endocrine causes of growth attenuation, including hypothyroidism, growth hormone deficiency,and Cushing syndrome, are not associated with decreased weight for height. Measures ofinsulin-like growth factor-1 (IGF-1) and insulin-like growth factor binding protein 3 are goodscreening studies for growth hormone deficiency, although hepatic IGF-1 production also isstimulated by insulin and may be low in underweight children. A 24-hour urine free cortisolmeasurement is an excellent screening study for Cushing syndrome, but this disorder isassociated with at least mild obesity.

References:

Kemp S, Güngör N. Growth failure. eMedicine Specialties: Pediatrics: Endocrinology. 2005.Available at: http://www.emedicine.com/ped/topic902.htm

Reiter EO. Short stature/tall stature. In: Burg FD, Ingelfinger JR, Polin RA, Gershon AA, eds.Gellis & Kagan’s Current Pediatric Therapy. Philadelphia, Pa: WB Saunders; 2003:679-682

Israel EJ, Levitsky LL, Anupindi SA, Pitman MB. Case Records of the Massachusetts GeneralHospital. Weekly clinicopathological exercises. Case 3-2005. A 14-year-old boy with recentslowing of growth and delayed puberty. N Engl J Med. 2005;352:393-403

van Rijn JC, Grote FK, Oostdijk W, Wit JM. Short stature and the probability of coeliac disease,in the absence of gastrointestinal symptoms. Arch Dis Child. 2004;89:882-883. Available at:http://adc.bmjjournals.com/cgi/content/full/89/9/882

Copyright © 2007 by the American Academy of Pediatrics page 423

Page 424: AAP MCQ 2007

2007 PREP SA on CD-ROM

Question: 203

You are evaluating a 2-year-old boy because he bangs his head on his bed before he goes tosleep. His mother has no other concerns about his behavior. There have been no recentchanges in the family or stresses that she can identify. Physical examination findings are normalexcept for excoriated skin on the boy's forehead and minimal ecchymosis.

Of the following, the MOST likely diagnosis is

A. autism

B. brain injury

C. childhood depression

D. global developmental delay

E. normal variant behavior

Copyright © 2007 by the American Academy of Pediatrics page 424

Page 425: AAP MCQ 2007

2007 PREP SA on CD-ROM

Preferred Response: ECritique: 203

Head banging is one of several common rhythmic movements that include body rocking andthumb sucking. As many as 20% of children may bang their heads. It is more common in boys(3:1) and occurs most commonly between 6 months and 4 years of age. The cause of headbanging is unknown, but it usually resolves without intervention. In fact, parental attention to thehead banging may prolong the practice. Head banging may cause abrasions or mild bruising, asnoted in the vignette; children who have significant injury require further evaluation.

Although head banging is common in typically developing children, it is more frequent inchildren who have developmental disorders such as mental retardation and autism as well as inchildren who have hearing or vision impairment.

Because the mother describes no other concerns regarding her child’s development orbehavior, the boy in the vignette likely has head banging behavior that will resolve withoutintervention. His mother should be counseled to ignore the behavior or distract her son withother activities. Children who are depressed may perform rhythmic behaviors, but they typicallyhave other manifestations, such as change from their previous routines of play and eatingpatterns. Children who have global developmental delay have cognitive delays as well as delaysin adaptive behavior, such as difficulty with eating independently and cooperating with dressing.There is no history of head injury in the child in the vignette, so head banging as a manifestationof brain injury is unlikely, and neuroimaging is unnecessary. Children who have autism typicallyhave difficulty with communication (eg, language delays, lack of pointing and gaze-following),impaired social skills (lack of pretend play), and rigid or repetitive behavior and play patterns thatare not described in the vignette.

References:

American Psychiatric Association. Stereotypic movement disorder. Diagnostic and StatisticalManual of Mental Disorders. 4th ed revised. Washington, DC: American Psychiatric Association;2000:131-133

Boyce WT, Shonkoff JP. Compulsive behaviors. In: Rudolph CD, Rudolph AM, Hostet6ter MK,Lister G, Siegel NJ, eds. Rudolph’s Pediatrics. 21st ed. New York, NY: McGraw-Hill; 2003:459-462

Copyright © 2007 by the American Academy of Pediatrics page 425

Page 426: AAP MCQ 2007

2007 PREP SA on CD-ROM

Question: 204

You are asked to review a case for morbidity and mortality conference. The infant was born atterm to a 19-year-old gravida 1, para 1 woman by normal spontaneous vaginal delivery. Themother was known to be group B Streptococcus-negative, but she did have genital warts. TheApgar scores were 9 at 1 minute and 10 at 5 minutes. On the seventh postnatal day, the infantdeveloped a temperature of 103°F (39.4°C) and was brought to the emergency department. Atthis time, the infant was in shock and required mechanical ventilation. Physical examinationrevealed scleral icterus and hepatosplenomegaly but no skin lesions. A lumbar puncture couldnot be performed. Laboratory results include:

•White blood cell count of 2.34x103/mcL (2.34x109/L), with 32% lymphocytes, 41%

neutrophils, 8% bands, 15% monocytes, 3% eosinophils, and 1% basophils

•Hemoglobin of 7.1 g/dL (71 g/L)

•Hematocrit of 21% (0.21)

•Platelet count of 40x103/mcL (40x109/L)

•Prothrombin time of 41.2 seconds

•Activated partial thromboplastin time of >106 seconds

•Aspartate aminotransferase concentration of 3,086 U/L

•Alanine aminotransferase concentration of 456 U/L•Total bilirubin of 4.4 mg/dL (75.2 mcmol/L)

The chest radiograph demonstrated diffuse interstitial infiltrates bilaterally (Item Q204A). Thepatient did poorly over the next 3 days and died despite aggressive management in a pediatricintensive care unit.

Of the following, the MOST likely cause of this patient's death is

A. adenovirus

B. Escherichia coli

C. group B Streptococcus

D. herpes simplex virus

E. Listeria monocytogenes

Copyright © 2007 by the American Academy of Pediatrics page 426

Page 427: AAP MCQ 2007

2007 PREP SA on CD-ROM

Preferred Response: DCritique: 204

Neonates presenting with shocklike symptoms during the first 7 to 10 days after birth are adiagnostic challenge. The differential diagnosis includes five major categories: 1) bacterialsepsis, 2) inborn errors of metabolism, 3) ductal-dependent complex congenital heart disease,4) nonaccidental trauma, and 5) viral sepsis. Clinicians must make rapid treatment decisionsbased on history and physical examination. The fever, hepatosplenomegaly, leukopenia, andabnormal findings on chest radiography reported for the patient in the vignette suggest infectionas the most likely cause of the infant’s illness. Group B Streptococcus, Escherichia coli, andListeria monocytogenes are causes of neonatal septic shock, but the interstitial pulmonaryinvolvement and hepatitis reported in this case make a virus more likely. Although adenoviralinfection can have this type of presentation, the most likely viral pathogen is herpes simplex.

Neonatal infections with the herpes simplex virus (HSV) may be due to either HSV type 1(25%) or HSV type 2 (75%). Approximately 80% of mothers of affected infants have no signs orsymptoms of HSV infection prior to delivery. The virus commonly is transmitted to the infant fromthe mother because of the large amount of virus present and the prolonged shedding. If theprimary infection affects only the cervix, the mother is usually asymptomatic. The infantsbecome colonized with HSV at delivery and usually present with active disease between 5 and10 days after birth. Neonatal disease may present as: 1) disease localized to the skin, eye, ormouth (SEM); 2) disseminated disease involving the liver, lungs, adrenal glands, central nervoussystem, and skin; and 3) localized central nervous system disease. In disseminated disease,approximately 20% of neonates never develop skin lesions. Therefore, the lack of skin lesionsdoes not eliminate HSV as a potential pathogen for a sick infant. The diagnosis of neonatal HSVcan be confirmed with: 1) viral cultures of skin lesions, the nasopharynx, or cerebrospinal fluid;2) direct fluorescent antibody staining of vesicle scrapings; or 3) DNA detection by polymerasechain reaction test of the cerebrospinal fluid. The Tzanck preparation can be helpful but onlyindicates the presence of a DNA virus and is not specific for HSV.

All neonatal HSV infections should be treated with intravenous acyclovir. The dosage is 60mg/kg per day in three divided doses given intravenously for 14 days for SEM disease and for atleast 21 days for disseminated or central nervous system disease.

References:

Kimberlin DW, Lin CY, Jacobs RF, et al. Natural history of neonatal herpes simplex virusinfections in the acyclovir era. Pediatrics. 2001;108:223-229. Available at:http://pediatrics.aappublications.org/cgi/content/full/108/2/223

Waggoner-Fountain LA, Grossman LB. Herpes simplex virus. Pediatr Rev. 2004;25:86-93.Available at: http://pedsinreview.aappublications.org/cgi/content/full/25/3/86

Copyright © 2007 by the American Academy of Pediatrics page 427

Page 428: AAP MCQ 2007

2007 PREP SA on CD-ROM

Question: 205

A 10-year-old girl presents to the emergency department with a 1-day history of brown urine.She reports no dysuria, urgency, frequency, or abdominal or flank pain. Her vital signs include:temperature, 98.8°F (37.1°C); blood pressure, 165/97 mm Hg; heart rate, 84 beats/min; andrespiratory rate, 20 breaths/min. On physical examination, moderate periorbital edema isevident, but there are no other abnormalities. Urinalysis reveals moderate blood and 4+ protein.The serum complement 3 (C3) concentration is low, and the C4 concentration is normal.

Of the following, the MOST likely cause of this girl's hematuria is

A. focal segmental glomerulosclerosis

B. immunoglobulin A nephropathy

C. lupus nephritis

D. membranoproliferative glomerulonephritis

E. postinfectious acute glomerulonephritis

Copyright © 2007 by the American Academy of Pediatrics page 428

Page 429: AAP MCQ 2007

2007 PREP SA on CD-ROM

Preferred Response: ECritique: 205

The girl in the vignette has painless gross hematuria without fever. Fever would suggestpossible infection, and abdominal pain would hint at infection, stone, or renal malformation(tumor, cyst). She has an elevated blood pressure and periorbital edema, which may be due tohypoalbuminemia or fluid retention. Her urinalysis shows blood and protein. The most significantclue to the cause of her renal disease is her low serum complement 3 (C3) concentration andnormal C4 concentration. She has strong evidence of nephritis (gross hematuria, hypertension,periorbital edema), decreased C3, and normal C4 values, which are consistent withpostinfectious acute glomerulonephritis (PIAGN). If both the C3 and C4 values were low, thenephritis more likely would be membranoproliferative GN or lupus nephritis. Children who haveeither focal segmental glomerulosclerosis or immunoglobulin A nephropathy may develop grosshematuria, hypertension, edema, and nephritis, but because of the pathogenesis of thesediseases, the serum complement values would be normal.

PIAGN is a common but generally self-limited renal disease that usually occurs in childhood.Among the several known pathogenetic organisms associated with PIAGN, the most common isgroup A beta-hemolytic Streptococcus. Most children who have PIAGN recover complete renalfunction and exhibit normalization of C3 levels by 6 weeks, although some continue to exhibithematuria and/or proteinuria for prolonged periods after initial presentation. Some also havehypertension, primarily due to salt and water retention, for up to 3 months. Occasionally, thedisease may assume a rapidly progressive course, resulting in acute renal failure and the needfor treatment with high-dose intravenous corticosteroids and possibly intravenouscyclophosphamide or dialysis if renal failure persists. In these patients, the renal outcome isguarded.

References:

Berrios X, Lagomarsino E, Solar E, Sandoval G, Guzman B, Riedel I. Post-streptococcal acuteglomerulonephritis in Chile--20 years of experience. Pediatr Nephrol. 2004;19:306-312. Abstractavailable at:http://www.ncbi.nlm.nih.gov/entrez/query.fcgi?cmd=Retrieve&db=pubmed&dopt=Abstract&list_uids=14689289&query_hl=61&itool=pubmed_docsum

Kasahara T, Hayakawa H, Okubo S, et al. Prognosis of acute poststreptococcalglomerulonephritis (APSGN) is excellent in children, when adequately diagnosed. Pediatr Int.2001;43:364-367. Abstract available at:http://www.ncbi.nlm.nih.gov/entrez/query.fcgi?orig_db=PubMed&db=PubMed&cmd=Search&term=%22Pediatrics+international+:+official+journal+of+the+Japan+Pediatric+Society%22[Jour]+AND+364[page]+AND+2001[pdat]

Pinto SW, Sesso R, Vasconcelos E, Watanabe YJ, Pansute AM. Follow-up of patients withepidemic poststreptococcal glomerulonephritis. Am J Kidney Dis. 2001;38:249-255. Abstractavailable at:http://www.ncbi.nlm.nih.gov/entrez/query.fcgi?orig_db=PubMed&db=PubMed&cmd=Search&term=am+j+kidney+dis[Jour]+AND+249[page]+AND+2001[pdat]

Copyright © 2007 by the American Academy of Pediatrics page 429

Page 430: AAP MCQ 2007

2007 PREP SA on CD-ROM

Question: 206

A 5-month-old female infant presents with a 1-day history of fever to 102ºF (38.9ºC), emesis,and multiple episodes of greenish diarrhea with streaks of blood. Her mother states that theinfant is less active, sleepier, and has no interest in feeding. Physical examination reveals alistless infant who has a sunken anterior fontanelle, dry mucous membranes with decreasedskin turgor, and skin irritation in the diaper area.

Of the following, the MOST likely cause of this patient's gastroenteritis is

A. astrovirus

B. Escherichia coli

C. Norwalk virus

D. rotavirus

E. Salmonella sp

Copyright © 2007 by the American Academy of Pediatrics page 430

Page 431: AAP MCQ 2007

2007 PREP SA on CD-ROM

Preferred Response: DCritique: 206

Diarrheal diseases in children younger than 5 years of age remain one of the leading causes ofmorbidity and mortality in developing countries and are important causes of morbidity indeveloped countries. In the United States, children 5 years of age and younger have between 35and 40 million episodes of diarrhea annually, resulting in about 170,000 hospitalizations and 300deaths due to complications. Item C206A shows the most common viral and bacterial diarrhealpathogens in children by age group.

Most of the infectious organisms that cause diarrhea are spread via the fecal-oral route.Organisms such as Shigella, Giardia, and Campylobacter also may be transmitted by person-to-person contact because of their low infective inoculum dose. Foodborne diseases affect all agegroups. Common causes include: E coli 0157:H7 (EHEC) associated with ingestion ofundercooked meat, Salmonella and Campylobacter associated with contaminated poultryproducts and unpasteurized milk, Yersinia enterocolitica associated with contaminated porkproducts, and Norwalk virus associated with the consumption of raw seafood. Water also hasbeen documented as a vehicle of transmission for Giardia lamblia, Campylobacter,Cryptosporidium, and Norwalk virus.

Child care centers serve as important reservoirs for transmission of infectious diarrhealagents, with the peak incidence of diarrhea occurring in children younger than 3 years of agewho are not toilet trained. Rotavirus (seen most commonly in children younger than 12 monthsof age), Shigella, and Giardia have been associated with outbreaks in child care centers.

The child described in the vignette has diarrhea caused by rotavirus, the most commoncause of viral diarrheal disease in infants and toddlers. Norwalk virus and E coli are not commoncauses of diarrhea in young infants. Astrovirus is a cause of diarrhea in children younger than 4years of age, but most of the infections are asymptomatic. Salmonella sp also may causediarrhea in infants, but it is much less common than a viral etiology, especially in developedcountries.

References:

American Academy of Pediatrics. Escherichia coli diarrhea (including hemolytic-uremicsyndrome). In: Pickering LK, ed. Red Book: 2006 Report of the Committee on InfectiousDiseases. 27th ed. Elk Grove Village, Ill: American Academy of Pediatrics; 2006:291-296

American Academy of Pediatrics. Rotavirus infections. In: Pickering LK, ed. Red Book: 2006Report of the Committee on Infectious Diseases. 27th ed. Elk Grove Village, Ill: AmericanAcademy of Pediatrics; 2006:572-574

Ramaswamy K, Jacobson K. Infectious diarrhea in children. Gastroenterol Clin North Am.2001;30:611-624. Abstract available at:http://www.ncbi.nlm.nih.gov/entrez/query.fcgi?orig_db=PubMed&db=PubMed&cmd=Search&term=%22Gastroenterology+clinics+of+North+America%22[Jour]+AND+611[page]+AND+2001[pdat]

Thielman NM, Guerrant RL. Clinical practice. Acute infectious diarrhea. N Engl J Med.2004;350:38-47

Copyright © 2007 by the American Academy of Pediatrics page 431

Page 432: AAP MCQ 2007

2007 PREP SA on CD-ROM

Question: 207

A mother brings in her 2-year-old boy for food avoidance education after he was seen in theemergency department last week for anaphylaxis following peanut ingestion. This is the thirdanaphylactic episode after peanut ingestion since he was diagnosed with a peanut food allergyat age 1 year. At that time, he was evaluated by a pediatric allergist and determined to havepositive skin tests and an elevated serum immunoglobulin (Ig) E to peanut antigen, consistentwith an IgE-mediated allergy. The mother is frustrated and would like advice regarding peanutfood allergy.

Of the following, the BEST advice at this time is to

A. perform an oral challenge to determine the maximum peanut amount tolerated by the patient

B. recommend that the patient avoid hot-pressed peanut oil

C. recommend strict avoidance of all legumes (eg, peanuts, beans, soy)

D. recommend strict avoidance of peanuts and tree nuts

E. start daily antihistamine prophylaxis for accidental peanut ingestion

Copyright © 2007 by the American Academy of Pediatrics page 432

Page 433: AAP MCQ 2007

2007 PREP SA on CD-ROM

Preferred Response: DCritique: 207

Peanut (Arachis hypogaea) allergy is one of the most common food allergies in the UnitedStates. The initial approach to peanut food allergy is strict avoidance of all peanut food productsand rapid administration of intramuscular epinephrine if a severe allergic reaction occurs.Although tree nuts are not related to the legume family (ie, peanuts, soy, beans), patients whohave peanut food allergies should be counseled to avoid tree nuts for two reasons. First,approximately 25% to 50% of patients who are allergic to peanuts also are allergic to tree nuts.Second, tree nut foods may contain trace amounts of peanut because these foods frequentlyare processed in the same factory. Interestingly, children who have peanut food allergy rarely(approximately 5%) react to soy. Also, sunflower oil is tolerated in peanut-allergic patients.However, patients should be counseled to avoid unrefined (eg, cold-pressed, crude) and somegourmet peanut oils that may contain significant amounts of residual peanut protein. Studieshave demonstrated that refined peanut oil (eg, hot-pressed) does not contain significant residualproteins that bind immunoglobulin (Ig) E from sera in peanut-allergic patients.

Oral challenges typically are reserved to rule out unlikely foods that may cause a reaction orto determine if a person has outgrown a food allergy based on either history (eg, no longerexperiencing a reaction after accidental ingestion), skin testing (eg, negative skin testing result),or serum IgE testing (eg, a peanut IgE <0.35 IU/mL has a strong negative predictive value). Anoral challenge has been used only for research purposes to determine the maximum peanuttolerated in known allergic individuals; its use is strongly discouraged in any other scenario.Daily antihistamine therapy may lessen pruritus associated with an allergic reaction, but it is notrecommended to “prevent” a reaction.

In the past, peanut food allergy was believed to be lifelong, but newer studies suggest thatup to 20% of previously sensitized individuals may outgrow their peanut food allergy. Allergies totree nuts and shellfish are usually lifelong.

References:

Sampson HA, Leung DYM. Adverse reactions to foods. In: Behrman RE, Kliegman RM, JensonHB, eds. Nelson Textbook of Pediatrics. 17th ed. Philadelphia, Pa: WB Saunders Co 2004:789-792

Sampson HA. Clinical practice. Peanut allergy. N Engl J Med. 2002;346:1294-1299

Copyright © 2007 by the American Academy of Pediatrics page 433

Page 434: AAP MCQ 2007

2007 PREP SA on CD-ROM

Question: 208

A 4-year-old child who has Duchenne muscular dystrophy requires operative intervention undergeneral anesthesia for inguinal hernia repair.

Of the following, the operative complication that is MORE likely to occur in this child than in ahealthy child is

A. atlantoaxial rotary subluxation

B. corneal damage

C. malignant hyperthermia

D. seizure

E. surgical site infection

Copyright © 2007 by the American Academy of Pediatrics page 434

Page 435: AAP MCQ 2007

2007 PREP SA on CD-ROM

Preferred Response: CCritique: 208

Children who have neuromuscular disorders, including muscular dystrophy, myotonicdystrophy, and channelopathies such as hyperkalemic periodic paralysis, are at increased riskfor complications during the perioperative period. These complications include apnea due to theirunderlying weakness, cardiac arrhythmias, and malignant hyperthermia (MH). The agents mostcommonly associated with these complications are depolarizing neuromuscular blocking agentssuch as succinylcholine and inhaled anesthetic agents such as halothane.

MH is characterized by muscle rigidity and acute muscle necrosis. Affected patientsdevelop a hypermetabolic state leading to metabolic acidosis, hyperthermia, cardiac arrhythmia,markedly elevated creatine kinase concentrations, and myoglobinuria. This autosomal dominantcondition is caused by excessive release of intracellular calcium by the sarcoplasmic reticulum.Several genetic mutations have been described that result in MH, but the common feature is alower threshold for calcium release. The standard treatment is the muscle relaxer dantrolene,which decreases calcium release by the sarcoplasmic reticulum. Preoperative use of dantroleneis suggested for patients at high risk for MH.

Atlantoaxial rotary subluxation, corneal damage, seizure, and surgical site infection are nomore common in children who have Duchenne muscular dystrophy than in the generalpopulation.

References:

Bertorini TE. Perisurgical management of patients with neuromuscular disorders. Neurol Clin.2004;22:293.313. Abstract available at:http://www.ncbi.nlm.nih.gov/entrez/query.fcgi?orig_db=PubMed&db=PubMed&cmd=Search&term=Neurol+Clin[Jour]+AND+2004[pdat]+AND+Bertorini+TE[author]

Klingler W, Lehmann-Horn F, Jurkat-Rott K. Complications of anaesthesia in neuromusculardisorders. Neuromusc Disord. 2005;15:195-206. Abstract available at:http://www.ncbi.nlm.nih.gov/entrez/query.fcgi?orig_db=PubMed&db=PubMed&cmd=Search&term=Neuromusc+Dis[Jour]+AND+195[page]+AND+2005[pdat]

Copyright © 2007 by the American Academy of Pediatrics page 435

Page 436: AAP MCQ 2007

2007 PREP SA on CD-ROM

Question: 209

Several weeks ago, a boy presented with fever, malaise, headache, and a skin lesion. Thelesion began as a red papule and slowly enlarged to form a large annular ring with a flaterythematous border (Item Q209A). Today he complains of recurrent joint pains that areparticularly severe in his knees.

Of the following, the MOST likely diagnosis is

A. Epstein-Barr virus infection

B. human parvovirus B19 infection

C. Lyme disease

D. juvenile rheumatoid arthritis

E. Rocky Mountain spotted fever

Copyright © 2007 by the American Academy of Pediatrics page 436

Page 437: AAP MCQ 2007

2007 PREP SA on CD-ROM

Preferred Response: CCritique: 209

The earliest manifestation of Lyme disease is the unique skin eruption, erythema chronicummigrans (ECM). ECM appears 1 to 3 weeks after the bite of a tick that is carrying the spirocheteBorrelia burgdorferi. A red papule erupts at the site of the tick bite and slowly progresses overseveral weeks to form a large, annular, erythematous lesion (Item C209A) that exceeds 5 cm indiameter, as described for the boy in the vignette. The lesion has a flat red border and centralclearing. Untreated ECM appears to resolve spontaneously, but multiple lesions may appear 3 to5 weeks after the tick bite as a manifestation of early disseminated disease. Othermanifestations of this stage of infection include cranial nerve palsies, meningitis, conjunctivitis,arthritis, headaches, fatigue, myalgias, and low-grade fever. These symptoms reflectspirochetemia. Late disease is characterized most commonly by recurrent arthritis that usuallyis pauciarticular and affects the large joints, particularly the knees. Arthritis may occur without ahistory of earlier stages of illness. Recurrent rash and joint symptoms have resulted in confusionof Lyme disease with juvenile rheumatoid arthritis.

Early Lyme disease (eg, when erythema chronicum migrans is present) is diagnosedclinically. Although serologic testing is available, immunoglobulin (Ig) M-specific antibody peaks 3to 6 weeks and IgG antibody weeks to months after infection. Because erythema chronicummigrans appears 1 to 2 weeks after a tick bite, results of serologic testing at this time may benegative. Diagnosis may be difficult in patients who have later stages of the disease and shouldbe based on clinical findings and the results of serologic testing.

Infectious mononucleosis caused by Epstein-Barr virus infection) may be accompanied by arash that is pink and morbilliform. If a patient is treated with amoxicillin, a bright red, generalizedmorbilliform eruption (Item C209B) often occurs. Cutaneous findings in human parvovirus B19infection are the classic erythematous “slapped-cheek” appearance (Item C209C) followed by alacylike rash (Item C209D) on proximal extremities. Systemic juvenile rheumatoid arthritis ischaracterized by the appearance of a salmon-pink macular eruption (Item C209E) thattransiently appears in association with fever spikes. Cutaneous manifestations of RockyMountain spotted fever typically consist of a maculopapular, petechial rash (Item C209F) thatusually begins on the wrists and ankles and spreads within hours to the trunk. Palms and solestypically are involved.

References:

American Academy of Pediatrics. Lyme disease (Borrelia burgdorferi infection). In Pickering LK,ed. Red Book: 2006 Report of the Committee on Infectious Diseases. 27th ed. Elk Grove Village,Ill: American Academy of Pediatrics; 2006:428-433

Weston WL, Lane AT, Morelli JG. Bacterial infections (pyodermas) and spirochetal infections ofthe skin: spirochetal diseases. In: Color Textbook of Pediatric Dermatology. 3rd ed. St. Louis,Mo: Mosby; 2002:56-57

Bodemer C. Epstein-Barr virus (EBV) infections. In: Schachner LA, Hansen RC, eds. PediatricDermatology. 3rd ed. St. Louis, Mo: Mosby; 2003:1071-1076

Miller ML, Cassidy JT. Juvenile rheumatoid arthritis. In: Behrman RE, Kliegman RM, Jenson HB,eds. Nelson Textbook of Pediatrics. 17th ed. Philadelphia, Pa: WB Saunders Co; 2004:799-805

American Academy of Pediatrics. Epstein-Barr virus infections (infectious mononucleosis). InPickering LK, ed. Red Book: 2006 Report of the Committee on Infectious Diseases. 27th ed. ElkGrove Village, Ill: American Academy of Pediatrics; 2006:286-288

American Academy of Pediatrics. Rocky Mountain spotted fever. In Pickering LK, ed. Red Book:2006 Report of the Committee on Infectious Diseases. 27th ed. Elk Grove Village, IL: AmericanAcademy of Pediatrics; 2006:570-572

Copyright © 2007 by the American Academy of Pediatrics page 437

Page 438: AAP MCQ 2007

2007 PREP SA on CD-ROM

Galen W. Rickettsial infection. In: Schachner LA, Hansen RC, eds. Pediatric Dermatology. 3rded. St. Louis, Mo: Mosby; 2003:1025-1030

Weston WL, Lane AT, Morelli JG. Viral infections: viral exanthems: the morbilliform eruptions. In:Color Textbook of Pediatric Dermatology. 3rd ed. St. Louis, Mo: Mosby; 2002:89-100

Copyright © 2007 by the American Academy of Pediatrics page 438

Page 439: AAP MCQ 2007

2007 PREP SA on CD-ROM

Question: 210

The family of a diabetic patient in your practice requests evaluation for celiac disease. Theyhave heard from other families of children who have diabetes that patients who have type 1diabetes are at increased risk for this condition.

Of the following, a TRUE statement regarding celiac disease screening is that

A. empiric gluten withdrawal is the diagnostic test of choice

B. initial screening should include serum immunoglobulin A (IgA) and tissue transglutaminaseantibody

C. patients who have selective IgA deficiency have a lower rate of celiac disease than thegeneral population

D. the most specific antibody test for celiac screening is the antigliadin IgG antibody

E. the prevalence of celiac disease in children who have type 1 diabetes mellitus is greater than10%

Copyright © 2007 by the American Academy of Pediatrics page 439

Page 440: AAP MCQ 2007

2007 PREP SA on CD-ROM

Preferred Response: BCritique: 210

The patient described in the vignette has type 1 diabetes mellitus, and it is now recognized thatsuch patients are at increased risk of celiac disease (nontropical sprue, gluten-sensitiveenteropathy). The prevalence of celiac disease in children who have diabetes is approximately3% to 4% in those screened at diabetes onset, and the overall prevalence is approximately 6%.Early diagnosis of celiac disease (sprue) has the potential to reduce later complications ofuntreated sprue, such as growth failure, osteopenia, and intestinal lymphoma. For thesereasons, screening for celiac disease should be considered for all patients who have type 1diabetes mellitus. Other risk factors for celiac disease, and in which screening should beconsidered, include Down syndrome, chronic lymphocytic (Hashimoto) thyroiditis, selectiveimmunoglobulin (Ig)A deficiency, and a first-degree relative who has celiac disease.

The classic signs and symptoms of celiac disease are age-dependent. Toddlers presentwith abdominal distention, diarrhea, irritability, and failure to thrive. In contrast, older children andadolescents more commonly present with abdominal pain, anemia, growth attenuation, andarthritis. Currently, however, many high-risk but asymptomatic children are detected byserologic screening.

Because celiac disease is a lifelong condition that requires permanent dietary restriction,empiric withdrawal of gluten from the diet without a definitive diagnosis is not appropriate. Theinitial serologic screening test suggested by the North American Society for PediatricGastroenterology and Nutrition is the measurement of serum tissue transglutaminase IgA (TTG)antibody, in conjunction with a measurement of total serum IgA. The antibody screening test hasa sensitivity of greater than 95%, although it may have false-negative results in patients who areIgA-deficient. The IgA antiendomysial antibody test also has excellent sensitivity and specificity,but it is more expensive and labor-intensive. Tests for antibodies to gliadin (IgG and IgA) andreticulin are also available, but because they have a large number of false-positive results (ie,poor specificity), they should be used only under selected circumstances.

Currently, celiac disease cannot be diagnosed definitively by serology alone. Therefore, apatient who has a positive TTG antibody test result should have the diagnosis verified throughendoscopy and duodenal biopsy. In celiac disease, the duodenal biopsy demonstrates varyingdegrees of inflammation and villous atrophy. Patients whose duodenal biopsy results areconsistent with celiac disease should be placed on a lifelong gluten-free diet. In most patients,compliance can be documented by return of the tissue transglutaminase antibody levels tonormal within 6 to 12 months, and follow-up biopsy is usually not necessary.

References:

Barera G, Bonfanti R, Viscardi M, et al. Occurrence of celiac disease after onset of type 1diabetes: a 6 year prospective longitudinal study. Pediatrics. 2002;109:833-838. Available at:http://pediatrics.aappublications.org/cgi/content/full/109/5/833

Farrell RJ, Kelly CP. Celiac sprue. N Engl J Med. 2002;346:180-188. Available at:http://content.nejm.org/cgi/content/extract/346/3/180

Hill ID, Dirks MH, Liptak GS, et al. Guideline for the diagnosis and treatment of celiac disease inchildren: recommendations of the North American Society for Pediatric Gastroenterology,Hepatology and Nutrition. J Pediatr Gastroenterol Nutr. 2005;40:1-19. Available at:http://www.jpgn.org/pt/re/jpgn/abstract.00005176-200501000-00001.htm;jsessionid=G1XZJHD0FWF6q3h0vtvs5tQTH2PFpGn5Z7s5YBXrxTQtRcWslPJl!1941873617!-949856145!8091!-1?index=1&database=ppvovft&results=1&count=10&searchid=2&nav=search

Copyright © 2007 by the American Academy of Pediatrics page 440

Page 441: AAP MCQ 2007

2007 PREP SA on CD-ROM

Question: 211

A newborn in your neonatal intensive care unit has had intermittent seizures for 72 hours. Youhave been unable to control the seizures with phenobarbital, hydantoin, and lorazepam.Electrolyte, calcium, magnesium, and glucose concentrations are normal. The infantsubsequently becomes apneic, comatose, and unresponsive.

Of the following, the BEST laboratory test to determine the cause of coma in this infant is

A. analysis of whole blood lead concentration

B. evaluation of urine for reducing substances

C. measurement of serum amino acids, organic acids, lactate, and ammonia

D. measurement of serum cortisol, thyroxine, and thyroid-stimulating hormone

E. measurement of very long-chain fatty acids

Copyright © 2007 by the American Academy of Pediatrics page 441

Page 442: AAP MCQ 2007

2007 PREP SA on CD-ROM

Preferred Response: CCritique: 211

Seizures in the neonatal period often are due to hypoxic-ischemic encephalopathy or intracranialpathology such as malformation or hemorrhage. Most are controlled readily with a single or,possibly, two anticonvulsant therapies. The patient described in the vignette has refractoryseizures that have not responded to three anticonvulsants, and the infant’s condition progressesto coma. This presentation is not uncommon in metabolic diseases, where seizures may befocal or multifocal clonic jerks or generalized and tonic. Such events typically occur after the first48 postnatal hours. Because transient abnormalities in glucose and electrolytes are not presentin this patient, inborn errors of metabolism need to be ruled out.

A categorical approach to the laboratory evaluation of the comatose newborn or younginfant is helpful. Toxins and drugs can be evaluated by history and blood, urine, or meconiumtoxicology screens. Intracranial pathology can be evaluated by ultrasonography or computedtomography scan. Hypoxia-ischemia mechanisms can be discerned by history, blood gasanalysis, evidence of multiple organ system dysfunction, the neurologic examination, and thetiming of the onset of seizures (generally in the first 24 hours of postnatal life). Infection may besystemic or focal (central nervous system) and can be evaluated by cultures, antigen testing,and use of specific viral polymerase chain reaction tests of the blood, urine, and cerebrospinalfluid.

The diagnosis of inborn errors of metabolism requires consideration of broad categories ofmetabolism: problems with amino acid metabolism that produce elevated serum concentrationsof certain amino acids (eg, phenylketonuria with excessive serum phenylalanine and urinaryexcretion of phenylalanine and its metabolites); carbohydrate metabolism that may result inhypoglycemia, organic acidemias, hyperammonemia, and acidurias; and problems in fatty acidmetabolism that result in hypoglycemia with corresponding hypoketosis and often elevatedserum transaminases and ammonia concentrations. For these reasons, the evaluation shouldinclude measurement of serum electrolytes, glucose, lactic acid, ammonia, and amino acids aswell as organic acids from the serum and urine.

Lead toxicity is a consideration for coma in the older infant, toddler, or child exposed toenvironmental lead. Measurement of urinary reducing substances may be helpful ingalactosemia, but is insufficient when evaluating encephalopathy and coma in the newborn oryoung infant in whom an inborn error of metabolism is being ruled out. Serum cortisol, thyroxine,and thyroid-stimulating hormone may be important in evaluating congenital hypotonia, persistenthypoglycemia, or suspected panhypopituitarism. Total and free carnitine and acylcarnitine profileresults are abnormal in infants who have fatty acid oxidation defects. Affected infants presentwith hypoglycemia, cardiomyopathy, and urinary hypoketosis.

References:

Burton BK. Inborn errors of metabolism in infancy: a guide to diagnosis. Pediatrics.1998;102:e69. Available at: http://pediatrics.aappublications.org/cgi/content/full/102/6/e69

Enns GM, Packman S. Diagnosing inborn errors of metabolism in the newborn: clinical features.NeoReviews. 2001;2:e183-e191. Available at:http://neoreviews.aappublications.org/cgi/content/full/2/8/e183

Lemley KV, Hintz SR, Enns GM. Continuous renal replacement therapy in the initial managementof neonatal hyperammonemia due to urea cycle defects. NeoReviews. 2000;1:e173-e179.Available at: http://neoreviews.aappublications.org/cgi/content/full/1/9/e173

Menkes JH, Wilcox WR. Inherited metabolic diseases of the nervous system. In: Menkes JH,Sarnat HB, Maria BL, eds. Child Neurology. 7th ed. Philadelphia, Pa: Lippincott Williams &Wilkins; 2000:29-142

Zinn AB. Inborn errors of metabolism. In: Martin RJ, Fanaroff AA, Walsh MC, eds. Fanaroff andMartin’s Neonatal-Perinatal Medicine: Diseases of the Fetus and Infant. 8th ed. Philadelphia, Pa:

Copyright © 2007 by the American Academy of Pediatrics page 442

Page 443: AAP MCQ 2007

2007 PREP SA on CD-ROM

Mosby-Elsevier; 2006:1597-1658

Copyright © 2007 by the American Academy of Pediatrics page 443

Page 444: AAP MCQ 2007

2007 PREP SA on CD-ROM

Question: 212

An 18-month-old girl is brought to your clinic because her mother feels she is pale. She has norelevant findings on past medical history and eats a regular diet. She is alert and interactive buthas significant pallor. The remainder of the physical examination results are normal. A completeblood count reveals a normal white blood cell count and platelet count. The hemoglobinconcentration is 4.5 g/dL (45 g/L) and mean cell volume is 74 fL.

Of the following, the MOST likely diagnosis is

A. acute lymphoblastic leukemia

B. Diamond-Blackfan anemia

C. glucose-6-phosphate dehydrogenase deficiency

D. iron deficiency anemia

E. transient erythroblastopenia of childhood

Copyright © 2007 by the American Academy of Pediatrics page 444

Page 445: AAP MCQ 2007

2007 PREP SA on CD-ROM

Preferred Response: ECritique: 212

The previously healthy child described in the vignette presents with severe, normocytic anemiaand otherwise normal findings, features that are most consistent with transienterythroblastopenia of childhood (TEC). TEC is a disease of unknown cause that results intransient failure of the erythroid cell line. A history of preceding viral illness may be elicited. Theaverage age of affected children is 2 years; the condition rarely occurs in the first year afterbirth. Findings on the physical examination usually are normal, except for marked pallor andother signs of anemia, such as tachycardia, tachypnea, and fatigue. Laboratory findings includenormocytic anemia and reticulocytopenia. Red cell adenosine deaminase activity is normal, and iantigen expression can be normal or increased. Treatment consists of erythrocyte transfusionfor children who have hemoglobin values less than 5 g/dL (50 g/L). Those who have hemoglobinlevels higher than 5 g/dL (50 g/L) may recover without transfusion. The prognosis is excellent.

Diamond-Blackfan anemia is caused by insensitivity to erythropoietin and usually presentsin infants younger than 1 year of age with pallor and fatigue. Other features, such as facialabnormalities, thumb abnormalities, and congenital heart disease, are present in up to 25% ofaffected patients. Laboratory findings include macrocytic anemia and reticulocytopenia. Red celladenosine deaminase activity and i antigen expression are increased.

Iron deficiency causes a microcytic, hypochromic anemia. Children who have acutelymphoblastic leukemia rarely present with isolated anemia, and other findings, such ashepatosplenomegaly, lymphadenopathy, neutropenia, neutrophilia, or thrombocytopenia, areexpected. Glucose-6-phosphate dehydrogenase deficiency causes the acute onset of pallorwith jaundice, which is not described in the child in the vignette.

References:

Segel GB, Hirsh MG, Feig SA. Managing anemia in pediatric office practice: part 1. Pediatr Rev.2002;23:75-84. http://pedsinreview.aappublications.org/cgi/content/full/23/4/111

Segel GB, Hirsch MG, Feig SA. Managing anemia in pediatric office practice: part 2. Pediatr Rev.2002;23:111-121. Available at: http://pedsinreview.aappublications.org/cgi/content/full/23/4/111

Copyright © 2007 by the American Academy of Pediatrics page 445

Page 446: AAP MCQ 2007

2007 PREP SA on CD-ROM

Question: 213

A 12-year-old girl presents to the emergency department with abdominal pain. Her parentsreport that she awakened with a temperature of 101°F (38.4°C) this morning, complained ofabdominal pain, and has vomited twice. There is no diarrhea, and there are no sick contacts.She reports nausea and no interest in food or drink. She was previously entirely healthy.Urinalysis reveals a specific gravity of 1.015 and is otherwise normal.

Of the following, the finding that MOST indicates the need for immediate surgical intervention is

A. abdominal distention

B. hyperactive bowel sounds

C. pain in the right lower quadrant

D. rigidity of the abdominal wall

E. voluntary guarding

Copyright © 2007 by the American Academy of Pediatrics page 446

Page 447: AAP MCQ 2007

2007 PREP SA on CD-ROM

Preferred Response: DCritique: 213

The differential diagnosis of abdominal pain in children is an almost daily challenge for thepediatrician. Young children are especially challenging because they cannot verbally reportmany of the symptoms elicited in history-taking or even respond to questions regardingpalpation of the abdomen. The approach to the abdominal examination should elicit as muchpatient cooperation as possible and prevent crying and movement, which may confuse findings.

Examination findings such as tenderness may occur in children who have both nonsurgicalabdominal complaints and peritoneal inflammation. The presence of bowel sounds is usuallyreassuring. Exquisite tenderness elicited by walking, buttoning pants, and climbing on theexamination table strongly suggests a “surgical abdomen.” Distention with tenderness, rigidity ofthe abdominal wall to palpation, and rebound tenderness are more specific signs of peritonealirritation and, therefore, a surgical abdomen.

Simple abdominal distention may occur with constipation, gaseous distention, or pregnancy.Hyperactive bowel sounds usually suggest increased peristalsis against a bowel obstruction orgastroenteritis and are not generally present in peritonitis. Pain in the right lower quadrant issuggestive when other signs of peritonitis are present, but it also may occur with ovarian cyst ortorsion, gastroenteritis, constipation, pneumonia, and other disease processes. Voluntaryguarding is common on examination, even in children and adults who have no abdominal pain.

References:

Ashcraft KW. Consultation with the specialist: acute abdominal pain. Pediatr Rev. 2000;21:363-367. Available at: http://pedsinreview.aappublications.org/cgi/content/full/21/11/363

Pegoli W. Appendicitis. In: McMIllan JA, DeAngelis CD, Feigin RD, Warshaw JB, eds. Oski’sPediatrics: Principles and Practice. 3rd ed. Philadelphia, Pa: Lippincott, Williams & Wilkins;1999:1702-1703

Copyright © 2007 by the American Academy of Pediatrics page 447

Page 448: AAP MCQ 2007

2007 PREP SA on CD-ROM

Question: 214

During clinic rounds with medical students, you are seeing a 2-year-old girl who has otitis media.One of the students asks about the potential complications of otitis media.

Of the following, the statement you are MOST likely to make is that

A. cholesteatoma is more common when chronic otitis media with effusion is treated withtympanostomy tubes rather than no tubes

B. ear drainage is an uncommon complication after myringotomy and tympanostomy tubeinsertion

C. intracranial lateral sinus thrombosis occurs in 8% of ear infections, especially with amoxicillin-resistant bacteria

D. sensorineural hearing loss is prevented best by placement of tympanostomy tubes

E. warning symptoms and signs of intracranial complication include vomiting and blurred vision

Copyright © 2007 by the American Academy of Pediatrics page 448

Page 449: AAP MCQ 2007

2007 PREP SA on CD-ROM

Preferred Response: ECritique: 214

Since the advent of antibiotics, intracranial complications of otitis media have becomeincreasingly uncommon, but the incidence of infratemporal complications of the middle ear,mastoid air space, and adjacent structures remains unchanged. Hearing loss is the mostcommon complication. Although tympanostomy tubes often are placed for chronic otitis media,whether they prevent hearing loss remains controversial. Other complications of otitis mediainclude perforation of the tympanic membrane, otorrhea, and mastoiditis. Chronic infection maylead to cholesteatoma, and tympanostomy prevents this complication. Ear drainage is commonafter tympanostomy and myringotomy.

Intracranial suppurative complications of otitis media, such as meningitis, lateral sinusthrombosis, epidural and subdural abscess or empyema, and otitic hydrocephalus resemblingpseudotumor cerebri, now occur in far less than 1% of children. The affected child may presentwith fever, irritability, malaise, and signs of increased intracranial pressure, such as headache,vomiting, diplopia, or papilledema. A focal seizure also may occur.

References:

Hendley JO. Clinical practice: otitis media. N Engl J Med. 2002;347:1169-1174. Available at:http://content.nejm.org/cgi/content/extract/347/15/1169

Paradise JL. Otitis media. In: Behrman RE, Kliegman RM, Jenson HB, eds. Nelson Textbook ofPediatrics. 17th ed. Philadelphia, Pa: WB Saunders Co; 2004:2138-2148

Copyright © 2007 by the American Academy of Pediatrics page 449

Page 450: AAP MCQ 2007

2007 PREP SA on CD-ROM

Question: 215

You receive a call from a teenage patient, who tells you that she is having palpitations and feelssomewhat lightheaded. You refer her to your local emergency department, where no symptomsare discerned and physical examination findings are normal. The emergency department sendsa copy of the patient's electrocardiogram by facsimile to your office for you to review (ItemQ215A).

Of the following, the BEST interpretation of the girl's electrocardiogram is

A. premature atrial contractions

B. premature ventricular contractions

C. supraventricular tachycardia

D. ventricular tachycardia

E. Wolff-Parkinson-White syndrome

Copyright © 2007 by the American Academy of Pediatrics page 450

Page 451: AAP MCQ 2007

2007 PREP SA on CD-ROM

Preferred Response: BCritique: 215

The myocardium is an electrical generator that propagates electrical impulses throughout itstissue. In the healthy heart, the sinus node, situated in the right atrium, initiates the cardiacimpulse with which depolarization causes the atrium to contract. This is represented as a Pwave on the surface electrocardiogram (ECG). The impulse travels to the atrioventricular node(AVN), where it is slightly delayed before it is passed along to the His Purkinje system. The PRinterval represents the time of delay of the AVN. When the impulse reaches ventricularmyocardium, ventricular contraction occurs, which appears as the QRS complex on surfaceECG. The process completes as the ventricles repolarize and the T wave is seen on surfaceECG.

When the atria depolarize early and from tissue outside of the sinus node, a “premature”atrial contraction is seen on surface ECG. When such atrial activity is not associated with aventricular contraction, it is referred to as a blocked P wave. When ventricular cells depolarizespontaneously, early, and in isolation, a widened QRS complex appears on surface ECG. Thisis referred to as a premature ventricular contraction (Item C215A) and is not related to apreceding P wave. More than two of these ventricular beats occurring in a row at a rate that isfaster than the sinus rate represents ventricular tachycardia (Item C215B). This rhythm can bedeleterious and may lead to cardiovascular collapse if it is sustained. Supraventriculartachycardia (Item C215C) describes an abnormal heart rhythm that originates in the atrium.Most often it is a narrow QRS complex rhythm, often with rates between 240 and 300 beats/min.Wolff-Parkinson-White syndrome (Item C215D) is characterized by a complex of findings on thesurface ECG, including a shortened PR interval and ventricular pre-excitation (earlydepolarization of the QRS complex) with a delta wave.

The ECG for the girl in the vignette reveals a sinus rhythm, with a single wide QRS complexthat occurs in the absence of a P wave and is early compared with the preceding heart rate.This is interpreted as premature ventricular contractions.

References:

Kaltman J, Shah M. Evaluation of the child with an arrhythmia. Pediatr Clin North Am.2004;51:1537-1551. Abstract available at:http://www.ncbi.nlm.nih.gov/entrez/query.fcgi?orig_db=PubMed&db=PubMed&cmd=Search&term=%22Pediatric+clinics+of+North+America%22[Jour]+AND+1537[page]+AND+2004[pdat]

Liebman J. Electrocardiography. In: Moller JH, Hoffman JIE, eds. Pediatric CardiovascularMedicine. Philadelphia, Pa: Churchill Livingston; 2000:111-141

Copyright © 2007 by the American Academy of Pediatrics page 451

Page 452: AAP MCQ 2007

2007 PREP SA on CD-ROM

Question: 216

A newborn female has a cardiac murmur. Before the cardiologist arrives to evaluate her, shehas a seizure. Results of laboratory testing include a serum calcium concentration of 5.0 mg/dL(1.25 mmol/L). Subsequently, echocardiography reveals an aortic arch anomaly.

Of the following, the MOST appropriate test to obtain to aid in the diagnosis of this infant is

A. brainstem auditory evoked potentials

B. electroencephalography

C. fluorescent in situ hybridization analysis of chromosome 22

D. peripheral blood chromosome analysis

E. thyroid function testing

Copyright © 2007 by the American Academy of Pediatrics page 452

Page 453: AAP MCQ 2007

2007 PREP SA on CD-ROM

Preferred Response: CCritique: 216

The infant described in the vignette has a number of features of 22q11 deletion syndrome, whichincludes most infants affected by DiGeorge syndrome (80% to 90%) and velocardiofacialsyndrome (95%). DiGeorge syndrome results from abnormal cervical neural crest migration intothe derivatives of the third and fourth pharyngeal arches and pouches during earlyembryogenesis. The pattern of malformations includes hypoplasia or aplasia of the thymus andparathyroid glands and structural abnormalities of the great vessels. The defects lead to theclassic clinical manifestations of deficient cellular immunity due to T-cell dysfunction, causingincreased susceptibility to infection; hypocalcemia due to absent parathyroid hormone, resultingin hypocalcemic seizures; and aortic arch abnormalities. Some affected infants also havedysmorphic facial features, which can include short palpebral fissures, short philtrum,micrognathia, and ear abnormalities.

Most patients who have DiGeorge syndrome are found to have partial monosomy for theproximal long arm of chromosome 22 due to microdeletion of 22q11.2. This microdeletion can bedetected by fluorescent in situ hybridization (FISH) using a molecular probe specific for theregion. Although the deletion occasionally is evident on routine chromosome analysis, the FISHstudy is much more sensitive. Most cases represent de novo deletion events, although familialcases have been described.

The 22q11.2 deletion is associated with a variety of other phenotypes, including thevelocardiofacial syndrome, conotruncal anomaly-face syndrome, and others. Velocardiofacialsyndrome denotes patients who have the features of DiGeorge syndrome plus craniofacial(unusual palpebral fissures, broad nasal root, narrow upper lip) and palatal (cleft uvula,submucous cleft palate, cleft palate) abnormalities. Of interest, all of these phenotypes can beevident in a single family in which a heritable chromosome 22 deletion is present. It has beensuggested that this constellation of related syndromes that have the common etiologic factor of achromosome 22q11.2 deletion should be termed "22q11 deletion syndrome." Because deafnessis not a major feature of DiGeorge syndrome, obtaining brainstem auditory evoked potentials isnot indicated at this time and will not aid in diagnosis. The cause of the seizure in the infantdescribed in the vignette is hypocalcemia, and electroencephalography would not provideadditional useful information for management, which should include administration of calcium.Thyroid function usually is preserved in DiGeorge syndrome.

References:

American Academy of Pediatrics Committee on Genetics. Molecular genetic testing in pediatricpractice: a subject review. Pediatrics. 2000;106:1494-1497. Available at:http://pediatrics.aappublications.org/cgi/content/full/106/6/1494

Beck AE, Hudgins L. Congenital cardiac malformations in the neonate: isolated or syndromic?NeoReviews. 2003;4:e105. Available at:http://neoreviews.aappublications.org/cgi/content/full/4/4/e105

Jones KL. Deletion 22q11.2 syndrome. In: Smith's Recognizable Patterns of HumanMalformation. 6th ed. Philadelphia, Pa: Elsevier Saunders Co; 2006:298-301

Lin RJ, Cherry AM, Bangs CD, Hoyme HE. FISHing for answers: the use of molecularcytogenetic techniques in neonatology. NeoReviews. 2003;4:e94. Available at:http://neoreviews.aappublications.org/cgi/content/full/4/4/e94

Copyright © 2007 by the American Academy of Pediatrics page 453

Page 454: AAP MCQ 2007

2007 PREP SA on CD-ROM

Question: 217

An 18-year-old girl presents for a health supervision visit prior to leaving for college. She has aletter from the college she plans to attend instructing entering freshman who choose to live in thedormitories to obtain a meningococcal vaccination. You review vaccine information and obtainconsent for the immunization.

Of the following, an ACCURATE statement about recommendations for the use of themeningococcal MCV4 vaccine is that it

A. is administered subcutaneously as a single dose

B. protects against serogroup B

C. requires revaccination every 3 to 5 years

D. should be administered routinely to 11- to 12-year-olds

E. should be administered to high-risk children older than age 2 years

Copyright © 2007 by the American Academy of Pediatrics page 454

Page 455: AAP MCQ 2007

2007 PREP SA on CD-ROM

Preferred Response: DCritique: 217

In 2005, the American Academy of Pediatrics (AAP) issued recommendations for use ofmeningococcal vaccines. Currently, there are two available meningococcal vaccines: MSPSV4,(Menomune-A/C/Y/W-135; Sanofi Pasteur, Swiftwater, Pa.) licensed for use in children olderthan 2 years, and MCV4, (Menactra; Sanofi Pasteur), licensed for use in ages 11 to 55 years.Both vaccines contain capsular polysaccharides from Neisseria meningitidis serogroups A, C,Y, and W-135. MSPSV4 contains 50 mg of each serogroup, and MCV4 contains 4 mg of eachserogroup conjugated to 48 mg of diphtheria toxoid. Conjugation changes the expression of theimmune response from T cell-independent to T cell-dependent, providing an improved primaryresponse and a strong response at re-exposure. Thus, the conjugated vaccine MCV4 providespotential clinical advantages over MSPSV4. In addition to the longer-lasting immunity, MCV4reduces nasopharyngeal carriage, which should create a herd immunity effect. Neither vaccineprovides protection against serogroup B, a more common cause of disease in infants thanadolescents in the United States.

The AAP recommendations (2005) for meningococcal vaccine use include theadministration of MCV4 for:

•Routine immunization at the 11- to 12-year-old health supervision visit •Immunization of those who did not previously receive MCV4 at two points: high school

entry (age 15 years) and college entry for those who will live in dormitories•Children 11 years or older who are at increased risk, including those who have terminal

complement deficiency, who have anatomic or functional asplenia, or who travel tocountries on the Centers for Disease Control and Prevention (CDC) advisory list for the

vaccine•The possible reimmunization of adolescents 3 to 5 years after immunization with the

MPSV4 vaccine if they remain at increased risk for the disease

MCV4 is administered intramuscularly; MPSV4 is administered subcutaneously. There areno recommendations to revaccinate children every 3 to 5 years; the duration of protection withMCV4 currently is not known but is expected to be longer than 3 years. Information from MCV4evaluation studies will guide future revaccination policies. Although clinical trials of MCV4 areongoing in young children, MPSV4 is recommended for use in high-risk children from 2 to 10years of age. Updates on meningococcal vaccination recommendations can be found on theAAP and CDC Web sites.

References:

Centers for Disease Control and Prevention. Prevention and control of meningococcal disease:recommendations of the Advisory Committee on Immunization Practices (ACIP). Morbid MortalWkly Rep MMWR. 2005;54(RR-7):1-21. Available at: http://www.cdc.gov/mmwr/pdf/rr/rr5407.pdf

Committee on Infectious Diseases. Prevention and control of meningococcal disease:recommendations for use of meningococcal vaccines in pediatric patients. Pediatrics.2005;116:496-505. Available at: http://pediatrics.aappublications.org/cgi/content/full/116/2/496

Copyright © 2007 by the American Academy of Pediatrics page 455

Page 456: AAP MCQ 2007

2007 PREP SA on CD-ROM

Question: 218

A 15-year-old girl who is hirsute has had menses six times in the past year. She is overweight(body mass index of 35 kg/M2) and has a blood pressure of 110/75 mm Hg. Her cholesterolconcentration is 170 mg/dL (4.4 mmol/L), with a high-density lipoprotein cholesterolconcentration of 55 mg/dL (1.4 mmol/L) and fasting triglyceride value of 74 mg/dL (0.84 mmol/L).Her fasting blood glucose value is 108 mg/dL (6.0 mmol/L), and a 2-hour post-oral glucose bloodglucose value is 115 mg/dL (6.4 mmol/L).

Of the following, the MOST likely diagnosis for this patient is

A. idiopathic hirsutism

B. impaired glucose tolerance

C. Cushing syndrome

D. metabolic syndrome

E. polycystic ovary syndrome

Copyright © 2007 by the American Academy of Pediatrics page 456

Page 457: AAP MCQ 2007

2007 PREP SA on CD-ROM

Preferred Response: ECritique: 218

The diagnosis of metabolic syndrome currently is undergoing re-evaluation. There are presentlyfive consensus criteria for this diagnosis in adults and no formal consensus criteria for thediagnosis in childhood. Adult criteria include: variable combinations of obesity, increased waistcircumference, hypertension, elevated fasting triglycerides, and low high-density lipoproteincholesterol as well as impaired glucose tolerance or diabetes and evidence of insulin resistance.Although polycystic ovary syndrome (PCOS) may be seen in women who have thisconstellation of findings, it is not part of the diagnostic criteria in adults. Age- and sex-specificcriteria for these different components have been adapted by various investigators for use inchildren and adolescents.

The diagnosis of PCOS is clinical, based on two of the following three findings: evidence ofhyperandrogenism or measured elevated androgen levels, irregular menses or amenorrhea,and the presence of polycystic ovaries on ultrasonography, in the absence of other definedcauses, such as congenital adrenal hyperplasia or other androgenic disorders. Generally, girlswho have PCOS also have hyperinsulinism and are obese. Many affected girls havecomponents of the metabolic syndrome, including hypertension, obesity, high fasting triglycerideconcentrations, low high-density lipoprotein values, and glucose intolerance or frank type 2diabetes.

The girl described in the vignette is overweight, but has normal blood pressure and lipidvalues. Although her fasting blood glucose is slightly high, she has normal glucose tolerance.Therefore, she does not meet the adult criteria for metabolic syndrome. Because she hasirregular menses, she cannot be said to have idiopathic hirsutism. Although Cushing syndromecan be suspected in a girl who has oligomenorrhea, obesity, and hirsutism, it is a veryuncommon disorder in adolescents. Statistically, PCOS is the most common disorderassociated with the findings reported for the girl. A 24-hour urine free cortisol, overnightdexamethasone-suppressed serum cortisol, or an evening salivary cortisol measurement in thenormal range could rule out Cushing syndrome.

References:

Coviello AD, Legro RS, Dunaif A. Adolescent girls with polycystic ovary syndrome have anincreased risk of the metabolic syndrome associated with increasing androgen levelsindependent of obesity and insulin resistance. J Clin Endocrinol Metab. 2006;91:492-497.Abstract available at:http://www.ncbi.nlm.nih.gov/entrez/query.fcgi?cmd=Retrieve&db=pubmed&dopt=Abstract&list_uids=16249280&query_hl=79&itool=pubmed_docsum

Janssen I, Katzmarzyk PT, Srinivasan SR, et al. Combined influence of body mass index andwaist circumference on coronary artery disease risk factors among children and adolescents.Pediatrics. 2005;115:1623-1630. Available at:http://pediatrics.aappublications.org/cgi/content/full/115/6/1623

Kahn R, Buse J, Ferrannini E, Stern M; American Diabetes Association; European Associationfor the Study of Diabetes. The metabolic syndrome: time for a critical appraisal: joint statementfrom the American Diabetes Association and the European Association for the Study ofDiabetes. Diabetes Care. 2005;28:2289-2304. Available at:http://care.diabetesjournals.org/cgi/content/full/28/9/2289

Palmert MR, Gordon CM, Kartashov AI, Legro RS, Emans SJ, Dunaif A. Screening for abnormalglucose tolerance in adolescents with polycystic ovary syndrome. J Clin Endocrinol Metab.2002;87:1017-1023. Available at: http://jcem.endojournals.org/cgi/content/full/87/3/1017

Copyright © 2007 by the American Academy of Pediatrics page 457

Page 458: AAP MCQ 2007

2007 PREP SA on CD-ROM

Question: 219

You are seeing a 6-year-old girl whose mother is concerned about the girl masturbating. For thepast several months, the girl has touched her genitals while watching television with her mother.The parents have been divorced for 2 years, and the girl spends weekends with her father. Youquestion her mother further.

Of the following, the response that MOST increases your suspicion for behavior that is out of thenorm is

A. a history of urinary tract infection as an infant

B. play-acting intercourse

C. the practice of taking bubble baths

D. shyness with her mother in the bathroom

E. wearing of tight jeans

Copyright © 2007 by the American Academy of Pediatrics page 458

Page 459: AAP MCQ 2007

2007 PREP SA on CD-ROM

Preferred Response: BCritique: 219

Masturbation begins in infancy and is a nearly universal activity. Parents may be encouraged tocounsel their children on keeping the behavior private, but dramatically negative reactionsshould be avoided. Physical conditions such as vulvovaginitis or penile irritation as well asurethral irritation caused by wearing tight jeans may trigger increased masturbation; the behaviorusually decreases when the disorder is treated.

The child described in the vignette may be bored or may have vulvovaginitis related tobubble baths or urethral irritation causing her masturbatory behavior. She should be evaluatedmatter of factly, and private behavior should be discussed. Children of this age are becomingmore aware of differences between sexes and may become shy with parents in the bathroom. Ahistory of a single urinary tract infection in infancy is not concerning and is unrelated to the girl’sbehavior.

In contrast, behavior that is indicative of sexual exploitation, such as excessive orcompulsive masturbation or mimicking intercourse deserves further evaluation. A history of anyof these activities in the child in the vignette warrants involvement of the appropriate socialservices to ensure her safety.

References:

Dixon SD. Gender and sexuality: normal development to problems and concerns. In: RudolphCD, Rudolph AM, Hostetter MK, Lister G, Siegel NJ, eds. Rudolphs Pediatrics. 21st ed. NewYork, NY: McGraw-Hill; 2003:470-476.

Meyer TL, Cheng TL. In brief: unveiling the secrecy behind masturbation. Pediatr Rev.2002;23:148-149. Available at: http://pedsinreview.aappublications.org/cgi/content/full/23/4/148

Copyright © 2007 by the American Academy of Pediatrics page 459

Page 460: AAP MCQ 2007

2007 PREP SA on CD-ROM

Question: 220

Just before clinic starts, you find your two visiting medical students talking about a movie theysaw over the weekend in which some of the characters suffered from the "black death" orbubonic plague. They involve you in the conversation and ask you what you know about thisdisease of antiquity. They are astounded to hear that plague still exists in the world today. Youdecide to use this time to make a teaching point and query them about the appropriateantimicrobial therapy for plague.

Of the following, the MOST appropriate antimicrobial agent is

A. ampicillin

B. aztreonam

C. ceftriaxone

D. gentamicin

E. meropenem

Copyright © 2007 by the American Academy of Pediatrics page 460

Page 461: AAP MCQ 2007

2007 PREP SA on CD-ROM

Preferred Response: DCritique: 220

The most appropriate antimicrobial agents to treat patients who have plague (Yersinia pestis)are streptomycin and gentamicin. Because streptomycin has become difficult to obtain in recentyears, gentamicin has become the drug of choice. Alternative agents include doxycycline,tetracycline, and chloramphenicol. The usual duration of therapy is 7 to 10 days. Ampicillin,aztreonam, ceftriaxone, and meropenem are not effective.

Plague commonly presents with fever and painful, swollen regional lymph nodes that aretermed buboes. The buboes usually are found in the inguinal region but also can be present inthe axillary or cervical areas. Besides bubonic plague, patients may present with the septicemicform (hypotension, respiratory distress, intravascular coagulopathy), the pneumonic form, ormeningeal plague.

Plague is a zoonotic disease that can be transmitted by contact with sick or dying animals orby the bite of their fleas. Disease in the United States is found predominately in New Mexico,Arizona, Colorado, and California.

References:

American Academy of Pediatrics. Plague. In: Pickering LK, ed. Red Book: 2006 Report of theCommittee on Infectious Diseases. 27th ed. Elk Grove Village, Ill: American Academy ofPediatrics; 2006:523-525

Goldstein MD. Plague (Yersinia pestis). In: Feigin RD, Cherry JD, Demmler GJ, Kaplan SL, eds.Textbook of Pediatric Infectious Diseases. 5th ed. Philadelphia, Pa: WB Saunders Co;2004:1487-1492

Patt HA, Feigin RD. Diagnosis and management of suspected cases of bioterrorism: a pediatricperspective. Pediatrics. 2002;109:685-692. Available at:http://pediatrics.aappublications.org/cgi/content/full/109/4/685

Copyright © 2007 by the American Academy of Pediatrics page 461

Page 462: AAP MCQ 2007

2007 PREP SA on CD-ROM

Question: 221

A 14-year-old boy presents to the emergency department with a 2-week history of bilateral legedema and a 3-day history of abdominal swelling. His vital signs are: temperature, 98.4°F(36.9°C); blood pressure, 125/67 mm Hg; heart rate, 84 beats/min; and respiratory rate, 20breaths/min. Physical examination shows moderate ascites and 2+ leg edema. His urinalysisreveals negative blood and 4+ protein. Serum complement concentrations are ordered andfound to be normal.

Of the following, the MOST likely cause of his edema and proteinuria is

A. immunoglobulin A nephropathy

B. lupus nephritis

C. membranous nephropathy

D. membranoproliferative glomerulonephritis

E. postinfectious acute glomerulonephritis

Copyright © 2007 by the American Academy of Pediatrics page 462

Page 463: AAP MCQ 2007

2007 PREP SA on CD-ROM

Preferred Response: CCritique: 221

Nephrotic syndrome (NS) is a constellation of symptoms that include proteinuria,hypoalbuminemia, edema, and hyperlipidemia. The most important aspect is proteinuria; theother symptoms simply are outcomes of excessive urinary protein leak. Proteinuria is not theonly cause of hypoalbuminemia; other causes include advanced liver disease, severemalnutrition, and gastrointestinal losses.

If NS is suspected, a urinalysis may provide clues to the cause. Among the many causes ofNS in children, the most common are minimal-change NS (MCNS), focal segmentalglomerulosclerosis (FSGS), membranous nephropathy (MN), any cause of acuteglomerulonephritis (AGN), membranoproliferative GN (MPGN), lupus GN, and immunoglobulin Anephropathy (IgAN). The practitioner’s goal is to determine the cause of NS to devise the bestmanagement plan.

Although most glomerular diseases are not inherited, a family history of renal disease maysuggest FSGS, LN, or IgAN. Urinalysis may be helpful, with the presence of blood makingMCNS, FSGS, and MN less likely, although any of these conditions may feature protein andblood in the urine. Measuring serum electrolytes is essential to determine overall renal function inany child who has NS, but it is unlikely to provide any clues to the cause. An important clue maybe obtained by assessing serum levels of complement 3 and 4. Normal serum complementvalues, as reported for the boy in the vignette, generally eliminate postinfectious AGN, MPGN,and Tupus nephritis due to Henoch-Schönlein purpura as the cause of NS.

The older age of the boy described in the vignette, absence of hematuria, and normal serumcomplement values suggest MN as the most likely cause of his renal disease. Another commoncause could be FSGS. MCNS is most common in toddlers and early school-age children; MN,MPGN, and IgAN generally present in older (>10 years of age) children.

References:

Appel GB, Cook HT, Hageman G, et al. Membranoproliferative glomerulonephritis type II (densedeposit disease): an update. J Am Soc Nephrol. 2005;16:1392-1403. Abstract available at:http://www.ncbi.nlm.nih.gov/entrez/query.fcgi?orig_db=PubMed&db=PubMed&cmd=Search&term=%22Journal+of+the+American+Society+of+Nephrology+:+JASN%22[Jour]+AND+1392[page]+AND+2005[pdat]

Perfumo F, Martini A. Lupus nephritis in children. Lupus. 2005;14:83-88. Abstract available at:http://www.ncbi.nlm.nih.gov/entrez/query.fcgi?orig_db=PubMed&db=PubMed&cmd=Search&term=%22Lupus%22[Jour]+AND+83[page]+AND+2005[pdat]

Rana K, Isbel N, Buzza M, et al. Clinical, histopathologic, and genetic studies in nine families withfocal segmental glomerulosclerosis. Am J Kidney Dis. 2003;41:1170-1178. Abstract available at:http://www.ncbi.nlm.nih.gov/entrez/query.fcgi?orig_db=PubMed&db=PubMed&cmd=Search&term=%22American+journal+of+kidney+diseases+:+the+official+journal+of+the+National+Kidney+Foundation%22[Jour]+AND+1170[page]+AND+2003[pdat]

Copyright © 2007 by the American Academy of Pediatrics page 463

Page 464: AAP MCQ 2007

2007 PREP SA on CD-ROM

Question: 222

A 3-year-old boy who has spina bifida has a history of recurrent urinary tract infections (UTIs).He is currently being treated for a UTI. Monitoring laboratory evaluation shows a blood ureanitrogen of 23 mg/dL (8.2 mmol/L) and a creatinine concentration of 1.1 mg/dL (97.2 mcmol/L).

Of the following, the class of antibiotics that is MOST commonly associated with renal toxicity is

A. aminoglycosides

B. azalides

C. carbapenems

D. cephalosporins

E. penicillins

Copyright © 2007 by the American Academy of Pediatrics page 464

Page 465: AAP MCQ 2007

2007 PREP SA on CD-ROM

Preferred Response: ACritique: 222

Multiple antibiotic classes can cause renal toxicity, but the class that is the most likely to causenephrotoxicity is the aminoglycosides (gentamicin, tobramycin, amikacin, streptomycin, andkanamycin). The renal toxicity of these agents is related to their concentrative uptake by theproximal renal tubular cells and their capacity to interact with critical intracellular targets. Riskfactors associated with the development of such nephrotoxicity include prolonged course oftherapy, supertherapeutic doses, concurrent use of other nephrotoxic medications, dehydration,and underlying liver disease. The development of nephrotoxicity is characterized by the gradualonset of partial-to-complete, reversible, nonoliguric renal failure. The patient may exhibithypertension, and laboratory evaluation demonstrates elevations in blood urea nitrogen andcreatinine values as well as elevated protein concentrations on urinalysis. Nephrotoxicity rarelyis associated with the other antibiotics listed.

References:

Beauchamp D, Labrecque G. Aminoglycoside nephrotoxicity: do time and frequency ofadministration matter? Curr Opin Crit Care. 2001;7:401-408. Abstract available at:http://www.ncbi.nlm.nih.gov/entrez/query.fcgi?orig_db=PubMed&db=PubMed&cmd=Search&term=%22Current+opinion+in+critical+care%22[Jour]+AND+401[page]+AND+2001[pdat]

Kaloyanides GJ. Antibiotic-related nephrotoxicity. Nephrol Dial Transplant. 1994;9(suppl 4):130-134. Abstract available at:http://www.ncbi.nlm.nih.gov/entrez/query.fcgi?orig_db=PubMed&db=PubMed&cmd=Search&term=%22Nephrology,+dialysis,+transplantation+:+official+publication+of+the+European+Dialysis+and+Transplant+Association+-+European+Renal+Association%22[Jour]+AND+130[page]+AND+1994[pdat]

Mingeot-Leclercq MP, Tulkens PM. Aminoglycosides: nephrotoxicity. Antimicrob AgentsChemother. 1999;43:1003-1012. Available at:http://aac.asm.org/cgi/content/full/43/5/1003?view=long&pmid=10223907

Nagai J, Takano M. Molecular aspects of renal handling of aminoglycosides and strategies forpreventing nephrotoxicity. Drug Metab Pharmacokinet. 2004;19:159-170. Available at:http://www.jstage.jst.go.jp/article/dmpk/19/3/159/_pdf

Copyright © 2007 by the American Academy of Pediatrics page 465

Page 466: AAP MCQ 2007

2007 PREP SA on CD-ROM

Question: 223

A 9-year-old girl is stung on the left leg by a hornet while playing outside. Within 10 minutes, sheexperiences diffuse pruritus, and a large area of erythema develops at the sting site. Fiveminutes later, extensive raised welts appear on her trunk and extremities. Her parents rush herto the emergency department, where intravenous antihistamines and steroids are administered.During this episode, she denies difficulty breathing or swallowing. No tongue or uvula edema isevident during examination, and her symptoms gradually resolve over 4 hours.

Of the following, the NEXT recommended step in her management is to

A. admit the girl to the hospital for 24 hours

B. discharge the girl to home

C. perform skin testing to hornet and other stinging insects within the next 72 hours

D. prescribe a daily oral antihistamine for the girl to take when playing outside

E. start allergy shots to hornet

Copyright © 2007 by the American Academy of Pediatrics page 466

Page 467: AAP MCQ 2007

2007 PREP SA on CD-ROM

Preferred Response: BCritique: 223

The reaction described for the girl in the vignette is consistent with an immunoglobulin (Ig)E-mediated systemic reaction limited to cutaneous symptoms. Children younger than 16 years ofage who experience reactions limited to urticaria or angioedema have an overall risk for futuresystemic reactions of approximately 5% to 10%, a rate similar to individuals experiencing largelocal reactions.

After appropriate initial management, the girl in the vignette can be discharged home withappropriate counseling. She does not meet current guidelines for further testing or allergy shots.Hospital admission rarely is required for such patients unless symptoms persist despiteadministration of epinephrine.

Certain sting reactions warrant skin testing and potentially allergen immunotherapy. Skintesting is recommended for individuals younger than 16 years experiencing systemic reactionsthat involve more than cutaneous symptoms and in individuals older than 16 years who haveany type of systemic reaction, even cutaneous symptoms only. Individuals in these groups havea 30% to 60% chance for a systemic reaction when stung again by the same type of insect.Allergen immunotherapy is beneficial in these high-risk patients, lowering the risk of futuresystemic reaction to less than 5%.

When indicated, skin testing should be delayed for 2 to 4 weeks after the reaction; testingprior to this period may result in false-negative results. Antihistamines may improve localsymptoms or pruritus, but they do not have a role in daily prophylaxis for insect sting reactions.

References:

Sicherer SH, Leung DYM. Insect allergy. In: Behrman RE, Kliegman RM, Jenson HB, eds.Nelson Textbook of Pediatrics. 17th ed. Philadelphia, Pa: WB Saunders Co 2004:786-788

Golden DB, Kagey-Sobotka A, Norman PS, Hamilton RG, Lichtenstein LM. Outcomes of allergyto insect stings in children, with and without venom immunotherapy. N Engl J Med. 2004;351:668-674. Abstract available at:http://www.ncbi.nlm.nih.gov/entrez/query.fcgi?orig_db=PubMed&db=PubMed&cmd=Search&term=N+Engl+J+Med[Jour]+AND+668[page]+AND+2004[pdat]

Moffitt JE, Golden DBK, Reisman RE, et al. Stinging insect hypersensitivity: a practice parameterupdate. J Allergy Clin Immunol. 2004;114:869-886

Copyright © 2007 by the American Academy of Pediatrics page 467

Page 468: AAP MCQ 2007

2007 PREP SA on CD-ROM

Question: 224

An anxious 16-year-old girl presents to the emergency department complaining of a suddenonset of feeling that she is unable to breathe. She also complains of tingling around her lips andfingertips. Upon further questioning, you learn that she has had frequent similar episodes in thepast. She has no history of asthma, and she has no chest pain or fever. Physical examinationreveals a respiratory rate of 30 to 40 breaths/min, oxygen saturation on room air of 99%, nointercostal retractions, and no fever. No wheezing or crackles are evident on chest auscultation;breath sounds are good bilaterally. The balance of findings on the physical examination,including a complete neurologic evaluation, are normal.

Of the following, the symptom that BEST distinguishes the correct diagnosis from compensatoryhyperventilation is

A. chronicity of the complaint

B. lack of chest pain

C. lack of fever

D. oxygen saturation

E. respiratory rate

Copyright © 2007 by the American Academy of Pediatrics page 468

Page 469: AAP MCQ 2007

2007 PREP SA on CD-ROM

Preferred Response: ACritique: 224

The chronicity of the complaints reported by the girl in the vignette, particularly in the absence ofany signs or symptoms of organic disorders, suggests hyperventilation syndrome rather thancompensatory hyperventilation.

A rapid respiratory rate may be a compensation for organic disease such as pneumonia,asthma, diabetic ketoacidosis, increased intracranial pressure, congestive heart failure, or toxiningestion or it may be a manifestation of dysfunctional breathing. Hyperventilation syndrome is aform of dysfunctional breathing that is defined as breathing in excess of metabolic requirementsthat leads to an acute decrease in arterial carbon dioxide values (hypocapnia). Symptoms ofhypocapnia, such as paresthesias of the face and hands, generally appear when the P2declines to 20 mm Hg. Other symptoms include dyspnea, chest pain or tightness, musclespasm, dizziness, and palpitations.

The cause of hyperventilation syndrome is unclear; in fact, some authors doubt itsexistence. The cause is probably multifactorial. Early literature often used “hyperventilationsyndrome” and “panic attack” interchangeably, but it has become clear that they are notnecessarily the same. One recent study showed that patients who have hyperventilationsyndrome experience an accentuated increase in ventilation after a change in body positionfrom supine to standing, suggesting increased sensitivity of the baroreceptor response. Otherstudies have shown a strong association between hyperventilation syndrome and asthma,suggesting that some affected patients may be experiencing mild asthma attacks.

The acute evaluation of a patient presenting with isolated tachypnea should focus on rulingout acute organic causes. A history and physical examination help to rule out obvious problemssuch as congestive heart failure, asthma, pneumonia, or increased intracranial pressure.Additional suggested measures to rule out an organic cause in the acute setting include chestradiography, electrocardiography, arterial blood gas determination, and measurement of serumelectrolytes and hemoglobin. A trial of bronchodilators also can be ordered. Priorrecommendations to ask the patient to breathe into a paper bag in an attempt to reverse thehypocapnia have been discouraged by recent authors, who point out that this practice could bedangerous in the patient who has hyperventilation due to hypoxia.

Patients with hyperventilation syndrome due to anxiety often show spontaneous resolutionof their symptoms after assurance that there is no organic cause for the symptoms. Selectedpatients who have unremitting symptoms may require a ventilation perfusion scan to rule out thepresence of a pulmonary embolus or more detailed pulmonary evaluation.

Organic problems such as congestive heart failure and pneumonia may be present in theabsence of fever or chest pain. Blood oxygen saturation is normal in patients who have diabeticketoacidosis and increased intracranial pressure. Thus, fever, chest pain, and oxygensaturation do not reliably distinguish compensatory hyperventilation from hyperventilationsyndrome. However, the recurrent nature of this patient’s complaints would not be expected incompensatory hyperventilation due to any of these organic etiologies because spontaneousrecovery from any of them is unlikely without specific therapy. Impending respiratory failureshould be suspected in patients who present with tachypnea, diminished level of consciousness,poor skeletal muscle tone, and cyanosis.

References:

American Heart Association. Airway, ventilation, and management of respiratory distress andfailure. In: PALS Provider Manual. Dallas, Tex: American Heart Association; 2002:81-126

Gardner WN. The pathophysiology of hyperventilation disorders. Chest. 1996;109:516-534.Available at: http://www.chestjournal.org/cgi/reprint/109/2/516

Keeley D, Osman L. Dysfunctional breathing and asthma: it is important to tell the difference[editorial]. Br Med J. 2001;322:1075-1076. Available at:http://bmj.bmjjournals.com/cgi/content/full/322/7294/1075

Copyright © 2007 by the American Academy of Pediatrics page 469

Page 470: AAP MCQ 2007

2007 PREP SA on CD-ROM

Laffey JG, Kavanagh BP. Medical progress: hypocapnia. N Engl J Med. 2002;347:43-53

Malmberg LP, Tamminen K, Sovijärvi AR. Orthostatic increase of respiratory gas exchange inhyperventilation syndrome. Thorax. 2000;55:295-301. Available at:http://thorax.bmjjournals.com/cgi/content/full/55/4/295

Saisch SGN, Wessely S, Gardner WN. Patients with acute hyperventilation presenting to aninner-city emergency department. Chest. 1996;110:952-957. Available at:http://www.chestjournal.org/cgi/reprint/110/4/952

Copyright © 2007 by the American Academy of Pediatrics page 470

Page 471: AAP MCQ 2007

2007 PREP SA on CD-ROM

Question: 225

The mother of a 4-month-old infant is planning a winter trip to the tropics with her infant and asksabout the use of sunscreens and the safe amount of sun exposure for the infant.

Of the following, the MOST appropriate advice for the infant is to

A. apply waterproof sunscreen with a UVB SPF of 30 or greater at least every 30 minutes

B. avoid mid-day sun and apply sunscreen with a UVB SPF of 15 or greater

C. avoid all but incidental sun exposure because of decreased sweating and the risk of heatstroke

D. comply strictly with the use of physical sun blocks such as zinc oxide and titanium dioxidepaste

E. use only special sun-protective clothing

Copyright © 2007 by the American Academy of Pediatrics page 471

Page 472: AAP MCQ 2007

2007 PREP SA on CD-ROM

Preferred Response: CCritique: 225

The proper advice for management of the 4-month-old infant described in the vignette is to avoidall but incidental sun exposure; infants younger than 6 months of age should be kept out of directsunlight. Because they are not mobile and cannot remove themselves from uncomfortable lightand heat, they should be moved under a tree, umbrella, or stroller canopy, although an umbrellaor canopy in the vicinity of reflective surfaces (eg, the beach) may reduce ultraviolet radiationexposure by only 50%. Many infants have impaired functional sweating, and exposure to theheat of the sun may increase the risk of heatstroke. Sunburn may occur readily because aninfant's skin has less melanin than at any other time in life.

For infants and toddlers older than 6 months, mid-day sun avoidance remains the bestmethod of sun protection. Special sun-protective clothing as well as sunscreen agents that havesolar protection factor (SPF) of 30 or greater are recommended as adjuncts to other sun-protective practices. When engaged in swimming or water sports, sunscreens should bereapplied regularly throughout the period of sun exposure. Physical sun blocks containing zincoxide and titanium dioxide are becoming more cosmetically acceptable and provide excellentsun protection.

More than 80% of lifetime sun exposure is received before 18 years of age. Sun safetyhabits should be instilled early in life, and all children and adolescents should use protectivemeasures. Sunburn is the most obvious sign of sun damage, and cellular damage to the skinmay occur even with the use of sunscreens. For all ages, sun avoidance between 10 and 4 ispreferable.

References:

American Academy of Pediatrics. Committee on Environmental Health. Ultraviolet light: a hazardto children. Pediatrics. 1999;104:328-333. Available at:http://pediatrics.aappublications.org/cgi/content/full/104/2/328

Raimer S, Raimer BG. Physical injury: solar radiation. In: Schachner LA, Hansen RC, eds.Pediatric Dermatology. 3rd ed. St. Louis, Mo: Mosby; 2003:1227-1234

Weston WL, Lane AT, Morelli JG. Sun sensitivity: sunburn. In: Color Textbook of PediatricDermatology. 3rd ed. St. Louis, Mo: Mosby; 2002:144-147

Copyright © 2007 by the American Academy of Pediatrics page 472

Page 473: AAP MCQ 2007

2007 PREP SA on CD-ROM

Question: 226

A 5-month-old infant presents with a history of vomiting between 10 and 20 times a day. She isgrowing and developing normally. There is no blood in the vomitus, no respiratory symptoms,and no history of apnea. The parents are frustrated from cleaning up after the baby vomits andwant something done. Physical examination and upper gastrointestinal radiograph evaluationresults are normal.

Of the following, the MOST accurate statement about this patient is that she

A. is at increased risk of sudden infant death syndrome

B. is likely to develop an esophageal stricture in later life

C. probably will outgrow the condition by 1 year of age

D. should be referred for a head computed tomography scan

E. should undergo endoscopy to rule out eosinophilic esophagitis

Copyright © 2007 by the American Academy of Pediatrics page 473

Page 474: AAP MCQ 2007

2007 PREP SA on CD-ROM

Preferred Response: CCritique: 226

The infant described in the vignette has frequent vomiting, but she has no signs suggestingpathologic reflux, such as recurrent pneumonia, hematemesis, or failure to thrive. Therefore, it islikely that she will outgrow her reflux by 1 year of age. Gastroesophageal reflux rarely has beenreported to cause esophageal strictures, but this complication generally is seen in children whohave other medical conditions (eg, hiatal hernia, tracheoesophageal fistula, cerebral palsy) andnot in healthy term infants. There is no strong evidence in the medical literature to suggest thatinfants who have benign reflux are at risk of sudden infant death syndrome. Although intracraniallesions always should be considered in a child who is vomiting, nothing in this child'spresentation warrants computed tomography scan of the head at this time. Endoscopy to ruleout eosinophilic esophagitis is not necessary unless there is a strong history of other atopicdisease.

Infantile gastroesophageal reflux, defined as the regurgitation of gastric contents out of themouth, occurs in approximately 50% of term infants and peaks between 4 and 6 months of age.According to one study, approximately 20% of infants vomit five times a day or more between 4and 6 months of age, but by 12 months of age, the reflux resolves in 90% to 95% of infants.Therefore, observation and education are appropriate interventions for healthy babies who haveno signs of pathologic reflux. A small subset of infants may have reflux-associated pathology,including esophagitis, recurrent pneumonia, or reactive airway disease. For these infants,additional evaluation (with esophageal pH study or endoscopy) and therapy (with histaminereceptor antagonists, proton pump inhibitors, or prokinetics) may be of benefit until they outgrowthe condition. In contrast, although reflux is less common in older children (prevalence ofapproximately 3%), older children are less likely to outgrow the condition and are more likely torequire long-term therapy.

References:

Nelson SP, Chen EH, Syniar GM, Christoffel KK. Prevalence of symptoms of GE reflux duringinfancy. A pediatric practice-based survey. Arch Pediatr Adolesc Med. 1997;151:569-572.Abstract available at:http://www.ncbi.nlm.nih.gov/entrez/query.fcgi?cmd=Retrieve&db=pubmed&dopt=Abstract&list_uids=9193240&query_hl=18&itool=pubmed_docsum

Nelson SP, Chen EH, Syniar GM, Christoffel KK. Prevalence of symptoms of gastroesophagealreflux during childhood: a pediatric practice-based survey. Arch Pediatr Adolesc Med.2000;154:150-154. Abstract available at:http://www.ncbi.nlm.nih.gov/entrez/query.fcgi?cmd=Retrieve&db=pubmed&dopt=Abstract&list_uids=10665601&query_hl=20&itool=pubmed_docsum

Rudolph CD, Mazur LJ, Liptak GS, et al. Guidelines for evaluation and treatment ofgastroesophageal reflux in infants and children: recommendations of the North American Societyfor Pediatric Gastroenterology and Nutrition. J Pediatr Gastroenterol Nutr. 2001;32(suppl 2):S1-S31. Available at: http://www.jpgn.org/pt/re/jpgn/abstract.00005176-200100002-00001.htm;jsessionid=EjrOla6kqCUYONz2cAq2M3Da4Bz2hAxBakJXhuVwuU1Aecpv0uEz!-1070481199!-949856145!9001!-1

Copyright © 2007 by the American Academy of Pediatrics page 474

Page 475: AAP MCQ 2007

2007 PREP SA on CD-ROM

Question: 227

Retinopathy of prematurity (ROP) is the leading neonatal cause of blindness in the UnitedStates.

Of the following, the population at GREATEST risk for severe ROP comprises infants

A. admitted to a neonatal intensive care unit for any cause

B. born at less than 37 weeks' gestation who require parenteral nutrition

C. born at less than 32 weeks' gestation who require oxygen therapy

D. born at 28 weeks' gestation regardless of the need for assisted ventilation

E. who have a family history of myopia

Copyright © 2007 by the American Academy of Pediatrics page 475

Page 476: AAP MCQ 2007

2007 PREP SA on CD-ROM

Preferred Response: DCritique: 227

Retinopathy of prematurity (ROP) is the leading neonatal cause of blindness in the UnitedStates. Up to 400 infants each year are blinded by this condition. It affects 20% to 50% of allneonatal intensive care unit (NICU) patients weighing less than 1,500 g, with particularpredilection for the smallest and most immature newborns. Blindness is greatly increased ininfants who have stage 3 or 4 disease. The theories of contributing causal factors includeextreme prematurity, very low birthweight, exposure to supplemental environmental oxygen,possible oxidizing agent exposure (including iron), and the generation of inflammatory cytokines.

Perturbation of programmed angiogenesis in the developing retina ex utero andcorresponding metabolic changes result in certain vascularized and ischemic zones of theretina. Vascular endothelial growth factor is altered, and a proliferative vasculopathy that hasimmature, leaky, and tortuous tufts of new vessels jeopardizes perfusion of the retina andimparts biomechanical traction upon it. Results of these processes include impaired peripheralvision in the peripheral zones of vascularity and potential retinal detachment with correspondingblindness or visual field deficits.

The greatest risk for ROP is in infants born at less than 29 weeks’ gestation who weigh lessthan 1,200 g. Because retinal vascularity generally is completed by 37 to 44 weeks’ gestation,prematurity itself confers risk, but the risk is shifted toward the extreme end of prematurity.Oxygen therapy is a contributing risk factor, but imposes the greatest risk in the smallest andmost immature infants. Myopia is more common in very low-birthweight preterm infants than interm infants, but a family history of myopia has no bearing on the risk of ROP.

In view of these facts and findings about ROP, the American Academy of Pediatrics, theAmerican Academy of Ophthalmology, and the American Association for PediatricOphthalmology and Strabismus jointly published guidelines for ROP screening examination ofpreterm infants. Candidates for screening include infants whose birthweights are less than 1,500g, whose gestational ages are less than 32 weeks, and selected infants weighing 1,500 to 2,000g at birth who have an unstable clinical course if judged to be at risk by the attending pediatricianor neonatologist. The examination should be performed by a qualified and experiencedophthalmologist, and the results should be reported using the International Classification ofRetinopathy of Prematurity. The first examination should be performed between 4 and 6 weeksof postnatal age or at a corresponding postconceptual age of 31 to 34 weeks, whichever is later.

References:

Olitsky SE, Nelson LB. Disorders of the retina and vitreous. In: Behrman RE, Kliegman RM,Jenson HB, eds. Nelson Textbook of Pediatrics. 17th ed. Philadelphia, Pa: Saunders; 2004:2113-2118

Phelps DL. Retinopathy of prematurity: practical clinical approach. NeoReviews. 2001;2:e174-e179. Available at: http://neoreviews.aappublications.org/cgi/content/full/2/7/e174

Phelps DL. The eye (part 3): retinopathy of prematurity. In: Fanaroff AA, Martin RJ, eds.Neonatal-Perinatal Medicine: Diseases of the Fetus and Infant. 7th ed. Baltimore, Md: Mosby,Inc; 2002:1595-1602

Phelps DL and the ETROP Study Group. The early treatment for retinopathy of prematuritystudy: better outcomes, changing strategy [commentary]. Pediatrics. 2004;114:490-491.Available at: http://pediatrics.aappublications.org/cgi/content/full/114/2/490

Section on Ophthalmology American Academy of Pediatrics, American Academy ofOphthalmology, American Association for Pediatric Ophthalmology and Strabismus. Screeningexamination of premature infants for retinopathy of prematurity. Pediatrics. 2006;117:572-576.Available at: http://pediatrics.aappublications.org/cgi/content/full/117/2/572

Copyright © 2007 by the American Academy of Pediatrics page 476

Page 477: AAP MCQ 2007

2007 PREP SA on CD-ROM

Question: 228

You are seeing a 6-month-old boy for a health supervision visit. On physical examination, younote bilateral, nontender scrotal swelling (Item Q228A). The scrotum transilluminates (ItemQ228B). The remainder of the physical examination findings are normal.

Of the following, the MOST likely diagnosis is

A. hydrocele

B. inguinal hernia

C. orchitis

D. testicular germ cell tumor

E. varicocele

Copyright © 2007 by the American Academy of Pediatrics page 477

Page 478: AAP MCQ 2007

2007 PREP SA on CD-ROM

Preferred Response: ACritique: 228

Scrotal swelling is a common complaint in the pediatric population. Many of the causes are aresult of abnormal testicular descent. When the testes descend into the scrotum from theabdomen, they bring with them a portion of the peritoneum, known as the processus vaginalis,which normally obliterates (Item C228A). The tunica vaginalis subsequently forms around thetestes in the scrotum. Failure of the processus vaginalis to obliterate proximally results in fluidaccumulation around the testes, which forms a hydrocele. Physical findings include painless,bilateral swelling of the scrotum and transillumination, as seen in the infant described in thevignette. Most hydroceles resolve within 1 year; persistence beyond 1 year of age warrantssurgical evaluation.

Failure of distal obliteration of the processus vaginalis causes an inguinal hernia, in whichabdominal contents descend into the inguinal canal. This is characterized by an intermittentbulge (Item C228B) in the inguinal canal, which is made worse with straining or crying. Avaricocele is caused by dilatation of the pampiniform venous plexus and rarely is seen ininfancy. A characteristic “bag of worms” appearance (Item C228C) is noted when the patientstands upright. Orchitis usually is associated with a viral infection in postpubescent boys.Features include tender swelling of the testis, with erythema of the overlying scrotum (ItemC228D). Testicular germ cell tumors may be seen in infancy, but the typical physical finding is apainless scrotal mass.

References:

Elder JS. Acute and chronic scrotal swelling. In: Kliegman RM, Greenbaum LA, Lye PS, eds.Practice Strategies in Pediatric Diagnosis and Therapy. 2nd ed. Philadelphia, Pa: ElsevierSaunders; 2004:465-474

Schneck FX, Bellinger MF. Abnormalities of the testes and scrotum and their surgicalmanagement. In: Walsh PC, Retik AB, Vaughan ED Jr, Wein AJ, eds. Campbell’s Urology. 8thed. Philadelphia, Pa: Elsevier; 2002:2353-2356

Copyright © 2007 by the American Academy of Pediatrics page 478

Page 479: AAP MCQ 2007

2007 PREP SA on CD-ROM

Question: 229

A 14-year-old boy who had a sore throat and fever 2 weeks ago presents to the emergencydepartment still dressed in his football gear from the practice field, where he complained of acuteabdominal pain. He exhibits tachypnea, tachycardia, and mild hypotension and complains ofintense pain in the left upper quadrant.

Of the following, the MOST definitive study to diagnose this child's condition is

A. abdominal computed tomography scan

B. abdominal ultrasonography

C. complete blood count

D. diagnostic peritoneal lavage

E. partial thromboplastin time

Copyright © 2007 by the American Academy of Pediatrics page 479

Page 480: AAP MCQ 2007

2007 PREP SA on CD-ROM

Preferred Response: ACritique: 229

Splenic rupture in children almost always is due to blunt trauma, although susceptibility may beenhanced in children who have underlying disease, such as Epstein-Barr viral infection withsplenomegaly. Because trauma to the upper abdomen may be suspected in children who haveblunt trauma due to sports, motor vehicle accident, or assault, the approach to the child in whomsplenic injury is suspected should be the same as for all trauma victims.

Attention to airway, breathing, and circulation should be the initial step in evaluation. For thechild who has suspected splenic rupture, hemodynamic stability should be ensured.

Peritoneal lavage almost never is indicated in children because computed tomography scanof the abdomen with contrast provides more specific and sensitive information about the extentof abdominal injury. Limited evidence suggests that ultrasonography is sensitive and specific inaccurately diagnosing blunt abdominal trauma, but, as with other ultrasonographic procedures,results may be related to the skill of the operator.

Although many children do not require surgical intervention for splenic laceration, splenicrupture or laceration associated with hemodynamic instability may require such measures.Baseline complete blood count and coagulation studies (eg, partial thromboplastin time) shouldbe obtained, but they will not establish the diagnosis of splenic injury. In addition, an intravenousline should be placed, and the patient should not be fed until surgical clearance or several hoursof observation indicate no need for surgery. Support with intravenous fluids and blood productsis indicated for the patient described in the vignette, who has hypotension and tachycardia.

Children who require splenic resection are at increased risk for postsplenectomy sepsis,although the occurrence of this complication may be waning because of improved immunizationagainst encapsulated organisms.

References:

Haller JA Jr. Blunt trauma to the abdomen. Pediatr Rev. 1996;17:29-31

Sills RH. Spleen and lymph nodes. In: McMillan JA, DeAngelis CD, Feigin RD, Warshaw JB, eds.Oski’s Pediatrics: Principles and Practices. 3rd ed. Philadelphia, Pa: Lippincott, Williams &Wilkins; 1999:1465-1472

Sharma OP, Oswanski MF, Singer D, Raj SS, Daoud YA. Assessment of nonoperativemanagement of blunt spleen and liver trauma. Am Surg. 2005;71:379-386. Abstract available at:http://www.ncbi.nlm.nih.gov/entrez/query.fcgi?cmd=Retrieve&db=pubmed&dopt=Abstract&list_uids=15986966&query_hl=1&itool=pubmed_docsum

Stengel D, Bauwens K, Sehouli J, et al. Emergency ultrasound-based algorithms for diagnosingblunt abdominal trauma. The Cochrane Database of Systematic Reviews. 2005;2:CD004446.Available at:http://www.mrw.interscience.wiley.com/cochrane/clsysrev/articles/CD004446/frame.html

Copyright © 2007 by the American Academy of Pediatrics page 480

Page 481: AAP MCQ 2007

2007 PREP SA on CD-ROM

Question: 230

A 4-year-old girl remains intubated, mechanically ventilated, and completely unresponsive in theintensive care unit following a massive subarachnoid hemorrhage of unknown cause. She ishemodynamically stable and not receiving any sedating medications. She has a temperature of98.1ºF (36.7ºC) and has no evidence of infection. The girl has not exhibited any brainstem orcerebral function for more than 24 hours.

Of the following, the statement you are MOST likely to make while on rounds with the residentteam is that

A. a nuclear medicine blood flow study is the best method to determine whether the girl is braindead

B. electroencephalography must be performed to ascertain whether this girl is dead

C. even though this child is brain dead, her stable cardiac status precludes a declaration of death

D. no further testing is indicated because this child is brain dead

E. organ donation is contraindicated in this child once she is declared dead

Copyright © 2007 by the American Academy of Pediatrics page 481

Page 482: AAP MCQ 2007

2007 PREP SA on CD-ROM

Preferred Response: ACritique: 230

Brain death is equivalent to cardiorespiratory death, and a child is legally dead at the time thecriteria for brain death are fulfilled. The determination of brain death is most helpful to theprocess of organ procurement for transplantation.

Clinical criteria for brain death include a known, irreversible cause, along with completeabsence of cerebral and brainstem function for more than 12 hours (for children older than 1year) determined by at least two physicians. The patient must be normothermic andnormotensive, and there must be no toxins, drugs, or metabolic disorders present that couldobscure neurologic examination findings. Absence of brainstem function is confirmed byphysical findings plus an apnea test, with arterial P2 greater than 60 mm Hg rising by at least 20mm Hg over 8 to 10 minutes while the patient receives 100% oxygen.

Ancillary neurodiagnostic studies may be helpful in diagnosing brain death.Electroencephalography with electrocerebral silence is consistent with but not pathognomonicfor brain death. A cerebral radionuclide scan or conventional angiography showing completeabsence of blood flow in the vertebral and carotid arteries is pathognomonic for brain death.Such flow studies may be helpful when facial injury precludes examination or there is difficultyestablishing a known cause, as in the vignette.

Without further evaluation, the 4-year-old child described here cannot be declared braindead yet by current criteria because the cause of her subarachnoid hemorrhage is unknown. Anuclear medicine blood flow study can document brain death by demonstrating no blood flow inthe intracranial major arteries. Her stable cardiac status does not preclude a declaration of braindeath, although further testing is indicated. Electroencephalography alone, even if devoid of anyactivity, does not definitively establish brain death. Once declared brain dead, this child likely canbe a candidate for organ donation.

References:

Determination of brain death. Ad Hoc Committee on Brain Death. J Pediatr. 1987;110:15-19

Frankel LR, Mathers LH. Withdrawal or withholding of life support, brain death, and organprocurement. In: Behrman RE, Kliegman RM, Jenson HB, eds. Nelson Textbook of Pediatrics.17th ed. Philadelphia, Pa: WB Saunders Co; 2004:340-341

Report of Special Task Force. Guidelines for the determination of brain death in children.American Academy of Pediatrics Task Force on Brain Death in Children Pediatrics. 1987;80:298-300

Copyright © 2007 by the American Academy of Pediatrics page 482

Page 483: AAP MCQ 2007

2007 PREP SA on CD-ROM

Question: 231

You are evaluating a 12-year-old boy who recently moved to your community. His family historyreveals that his father, who is 36 years old, suffers from extremely high blood cholesterol levels(>500 mg/dL [12.9 mmol/L]). The boy's mother states that her husband's brother has the "sameproblem." Findings on the boy's physical examination are normal.

Of the following, the BEST next step for your patient is to

A. begin therapy with a lipid-lowering medication

B. institute a low-fat diet plan and follow up in 3 months

C. measure cholesterol and triglyceride concentrations

D. obtain a blood sample for genetic testing

E. order a baseline electrocardiogram

Copyright © 2007 by the American Academy of Pediatrics page 483

Page 484: AAP MCQ 2007

2007 PREP SA on CD-ROM

Preferred Response: CCritique: 231

There is good scientific evidence that the process of atherosclerosis begins in childhood.Children who have high cholesterol concentrations have a greater risk of having elevatedcholesterol values as adults than does the general population. In addition, several geneticsyndromes affect the normal metabolic processes of cholesterol-rich lipoproteins. These familialdiseases are characterized by early onset of frequently severe atherosclerotic changes in thecoronary artery system. Among such syndromes are familial hypercholesterolemia, familialcombined hyperlipidemia, familial hypertriglyceridemia, familial dysbetalipoproteinemia, andfamilial decreased high-density lipoprotein. Although the enzyme or protein abnormality varies ineach of these rare entities, the end result is similar, with affected children being at high risk forthe development of coronary heart disease at a young age.

Population approaches to lowering cholesterol concentrations in children and adolescentshave focused on the adoption of diets that are lower in saturated fat, total fat, and cholesterol.However, in cases such as the at-risk boy described in the vignette, an individualized approachis required in an attempt to facilitate early diagnosis and management. Therefore, children andadolescents who have a family history of premature cardiovascular disease or at least oneparent who has a high blood cholesterol reading should be considered for selective screening.Children who have a family history of any of the genetic hyperlipidemias also should undergoearly screening of cholesterol and triglycerides.

Initiation of therapy with lipid-lowering medications is not appropriate until a diagnosis hasbeen made. Similarly, genetic testing, which often is expensive and not available for all disorders,may not be of value as a first step. Electrocardiography does not have a diagnostic orprognostic role in the asymptomatic patient. Instituting a low-fat diet plan may not be wrong, but itdoes not aid in reaching a diagnosis.

References:

Cohen MS. Fetal and childhood onset of adult cardiovascular diseases. Pediatr Clin North Am.2004;51:1697-1719. Abstract available in:http://www.ncbi.nlm.nih.gov/entrez/query.fcgi?orig_db=PubMed&db=PubMed&cmd=Search&term=%22Pediatric+clinics+of+North+America%22[Jour]+AND+1697[page]+AND+2004[pdat]

Kwiterovich RC. Disorders of lipid and lipoprotein metabolism. In: Rudolph CD, Rudolph AM,Hostetter MK, Lister G, Siegel NJ, eds. Rudolph’s Pediatrics. 21st ed. New York, NY: McGrawHill; 2003:693-711

Lauer RM, Snetselaar L, Muhonen LF. Hyperlipidemia in children and adolescents. In: Moller JH,Hoffman JIE, eds. Pediatric Cardiovascular Medicine. Philadelphia, Pa: Churchill Livingston;2000:793-803

Copyright © 2007 by the American Academy of Pediatrics page 484

Page 485: AAP MCQ 2007

2007 PREP SA on CD-ROM

Question: 232

A 4-month-old child is admitted to the hospital for evaluation of failure to thrive and generalizedseizures. On physical examination, the child appears wasted and has a protuberant abdomenand marked hepatomegaly. Laboratory evaluation reveals fasting hypoglycemia, lactic acidosis,hyperuricemia, and hyperlipidemia. The boy's parents are first cousins.

Of the following, the BEST long-term management of this disorder is

A. oral dietary supplementation with long-chain fatty acids

B. oral dietary supplementation with protein

C. regular intravenous administration of 10% dextrose in water

D. regular intravenous administration of glucagon

E. regular oral administration of cornstarch

Copyright © 2007 by the American Academy of Pediatrics page 485

Page 486: AAP MCQ 2007

2007 PREP SA on CD-ROM

Preferred Response: ECritique: 232

The child described in the vignette has glycogen storage disease type I (GSD I) or von Gierkedisease. GSD I is an autosomal recessive disorder resulting from deficiency of glucose-6-phosphatase. Consequently, the final step in the catabolism of glycogen to glucose is impeded.Excess accumulation of the substrate glucose-6-phosphate results in increased lactateproduction. Hepatomegaly, which is sometimes evident at birth, is progressive, and linear growthis delayed.

Infants who have GSD I typically present when they begin to sleep through the night. Theprolonged fasting associated with the increased sleep pattern results in hypoglycemia, andaffected children may present with irritability, pallor, and seizures. In addition to hypoglycemia,metabolic derangements include lactic acidosis, hyperuricemia, hyperlipidemia, and ketonuria,as reported for the boy in the vignette. A number of renal complications can occur, and somepatients develop renal failure.

Diagnosis of GSD I is based on the near-to-complete absence of the enzyme glucose-6-phosphatase in liver tissue. Open-liver biopsy frequently is preferred to needle biopsy to ensureattainment of an appropriate specimen and to control bleeding, which may be excessive. Oncethe diagnosis is established, molecular genetic testing is available.

Treatment is twofold: avoidance of fasting and frequent administration of carbohydrates.Supplementation with long-chain fatty acids and protein is not necessary because affectedindividuals do not have difficulty metabolizing them. In infants, nasogastric feedings may beuseful at night. Uncooked cornstarch provides for the slow release of glucose and is a mainstayof treatment in infants and children. Glucagon administration is not helpful due to the inability tobreak down glycogen effectively. Intravenous 10% dextrose and water is cumbersome anddifficult over the long term.

References:

Chen Y-T. Defects in metabolism of carbohydrates: glycogen storage diseases. In: BehrmanRE, Kliegman RM, Jenson HB, eds. Nelson Textbook of Pediatrics. 17th ed. Philadelphia Pa:Saunders; 2004:469-475

Craigen WJ, Darras BT. Overview of disorders of glycogen metabolism. UpToDate. 2006;14.1.Available at:http://www.utdol.com/utd/content/topic.do?topicKey=dis_chld/8136&type=A&selectedTitle=2~11

Rake JP, Visser G, Labrune P, et al. Guidelines for management of glycogen storage diseasetype I: European Study on Glycogen Storage Disease Type I (ESGSD I). Eur J Pediatr.2002;161(suppl 1):S112-S119. Abstract available at:http://www.ncbi.nlm.nih.gov/entrez/query.fcgi?cmd=Retrieve&db=pubmed&dopt=Abstract&list_uids=12373584&query_hl=8&itool=pubmed_docsum

Copyright © 2007 by the American Academy of Pediatrics page 486

Page 487: AAP MCQ 2007

2007 PREP SA on CD-ROM

Question: 233

A 17-year-old boy presents for a sports physical. He has a learning disability and is shy. Hisheight is at the 75th percentile, and his body mass index is at the 85th percentile. Physicalexamination findings include minimal facial hair, bilateral gynecomastia (breast >4 cm indiameter), and small testes (testicular volume of 6 mL).

Of the following, the MOST likely cause of this patient's gynecomastia is

A. constitutional delay of puberty

B. incomplete androgen insensitivity syndrome

C. Klinefelter syndrome

D. obesity

E. pubertal gynecomastia

Copyright © 2007 by the American Academy of Pediatrics page 487

Page 488: AAP MCQ 2007

2007 PREP SA on CD-ROM

Preferred Response: CCritique: 233

Pubertal gynecomastia (Item C233A) is a benign and self-limited increase in glandular andstromal tissue that is believed to result from a temporary alteration in the ratio of estrogen totestosterone. Approximately two thirds of adolescent males develop pubertal gynecomastia. Themean age of onset is 13 years, and 50% of affected boys are at Sexual Maturity Rating stage 3when breast enlargement begins. Although the condition initially is unilateral, eventually morethan 75% of affected boys develop bilateral gynecomastia. A mobile, firm, rubbery nodule ispalpated beneath the areola in boys who have pubertal gynecomastia. This breast tissue mayextend beyond the areolar border, but it is rare for glandular tissue to exceed 4 cm in diameter.In addition, patients who have pubertal gynecomastia have normal testicular size. Pubertalgynecomastia usually resolves within 12 to 18 months; in only 8% of boys does it persist beyond2 years.

Adolescent males, such as the one described in the vignette, who are tall and havegynecomastia and small testes probably have Klinefelter syndrome (impaired androgenproduction), not pubertal gynecomastia. Klinefelter syndrome, the clinical manifestation of anextra X chromosome in males, occurs in 1 in 1,000 live male births. Other causes of nonpubertalgynecomastia include conditions that increase serum estrogen concentration, increase thebioavailability of estrogen, decrease serum androgen concentrations, or alter estrogen andandrogen receptors. A history and physical examination guide the search for associated clinicalconditions that include drugs (eg, spironolactone, ketoconazole, digitalis, anabolic steroids,cimetidine), liver and kidney disease, hyperthyroidism, androgen insensitivity syndromes, andneoplasms.

Constitutional delay of puberty is not a cause of gynecomastia and is associated with shortstature. Androgen insensitivity is an infrequent cause of gynecomastia that usually presents asa cause of primary amenorrhea in phenotypic females who have a 46,XY karyotype. Testes inaffected individuals may be located in the labioscrotal folds. Obesity alone does not explain theclinical finding of gynecomastia for the boy in the vignette, who has a body mass index of 85%(at risk for obesity) and small testes. Although obese males may have pubertal gynecomastiaand other conditions associated with nonpubertal gynecomastia, it is not uncommon for adiposetissue to be confused with breast tissue. When adipose tissue is present, palpation in the supineposition often reveals a subareolar depression surrounded by firm tissue (the doughnut sign).

References:

Neinstein LS, Joffe A. Gynecomastia. In: Neinstein LS, ed. Adolescent Health Care a PracticalGuide. 4th ed. Philadelphia, Pa: Lippincott Williams & Wilkins; 2002:264-270

Rapaport R. Gynecomastia. In: Behrman RE, Kliegman RM, Jenson HB, eds. Nelson Textbookof Pediatrics. 17th ed. Philadelphia, Pa: 2004:1930-1931

Copyright © 2007 by the American Academy of Pediatrics page 488

Page 489: AAP MCQ 2007

2007 PREP SA on CD-ROM

Question: 234

You are evaluating a 14-year-old boy who has a body mass index of 40 kg/m2. His mother and25-year-old sister have type 2 diabetes. A fasting blood glucose concentration for the boy is 110mg/dL (6.1 mmol/L).

Of the following, the MOST appropriate next step to screen for diabetes is to

A. measure glycosylated hemoglobin

B. measure serum insulin and C-peptide concentrations

C. perform a 2-hour oral glucose tolerance test

D. measure blood glucose 1 hour after a high-carbohydrate breakfast

E. repeat a fasting blood glucose measurement

Copyright © 2007 by the American Academy of Pediatrics page 489

Page 490: AAP MCQ 2007

2007 PREP SA on CD-ROM

Preferred Response: CCritique: 234

The boy described in the vignette is at high risk for type 2 diabetes because of his elevated bodymass index and strong family history. His elevated fasting blood glucose value is worrisome butnot diagnostic of diabetes. Criteria for the diagnosis of diabetes are two random blood glucosevalues greater than 200 mg/dL (11.1 mmol/L) or one random value of 200 mg/dL (11.1 mmol/L)with symptoms, a fasting plasma glucose greater than 126 mg/dL (6.7 mmol/L), or a 2-hour post-glucose tolerance test glucose value of greater than 200 mg/dL (11.1 mmol/L). Therefore, a 2-hour oral glucose tolerance test should be performed for the boy in the vignette to determinewhether he meets the criteria for diabetes.

Although glycosylated hemoglobin values are elevated in diabetes that is not perfectlycontrolled, measurement of this compound is not a good screening test for diabetes. Elevatedinsulin and C-peptide concentrations are found in any individual who has insulin resistance.Levels may help differentiate type 1 (low insulin and C-peptide) from type 2 (high insulin and C-peptide) diabetes, but are not diagnostic of diabetes itself. Many overweight children haveelevated insulin levels because they are insulin-resistant. There are no standards for thediagnosis of diabetes by measurement of blood glucose after a poorly defined high-carbohydratebreakfast.

Similarly, a repeat fasting glucose is likely to give an indeterminate value; only a 2-hour oralglucose tolerance test is definitive for the diagnosis of diabetes.

References:

American Diabetes Association. Type 2 diabetes in children and adolescents. Pediatrics.2000;105:671-680. Available at: http://pediatrics.aappublications.org/cgi/content/full/105/3/671

Kaufman FR. Type 2 diabetes in children and youth. Endocrinol Metab Clin North Am.2005;34:659-676. Abstract available at:http://www.ncbi.nlm.nih.gov/entrez/query.fcgi?orig_db=PubMed&db=PubMed&cmd=Search&term=%22Endocrinology+and+metabolism+clinics+of+North+America%22[Jour]+AND+659[page]+AND+2005[pdat]

Lipton RB, Drum M, Burnet D, et al. Obesity at the onset of diabetes in an ethnically diversepopulation of children: what does it mean for epidemiologists and clinicians? Pediatrics.2005;115:e553-e560. Available at:http://pediatrics.aappublications.org/cgi/content/full/115/5/e553

Copyright © 2007 by the American Academy of Pediatrics page 490

Page 491: AAP MCQ 2007

2007 PREP SA on CD-ROM

Question: 235

You are evaluating a 5-year-old boy who has cerebral palsy and mental retardation, is fedthrough a gastrostomy tube, and is dependent for all his care. He will be attending a full-dayprogram at the school in which he previously was enrolled. His parents are divorced, and hismother is his primary caretaker. She will begin working while he is in school. He has a 10-year-old brother with whom he shares a room and who alerts his mother when his brother needs helpat night.

Of the following, the concern you are MOST likely to address is

A. family stress

B. need for nursing services during the night

C. need for the mother to be available during school hours

D. potential for child abuse in school

E. vulnerability to communicable diseases

Copyright © 2007 by the American Academy of Pediatrics page 491

Page 492: AAP MCQ 2007

2007 PREP SA on CD-ROM

Preferred Response: ACritique: 235

Chronic illness has a significant impact on an affected child and his or her family. Familymembers struggle to balance the needs of individuals with the increased time and energydevoted to the care of a child who has the disability. The increased emotional and economicburden on the family can cause intolerable stresses that lead to divorce, depression, and otherpsychosocial difficulties. Although much of a family’s energy is devoted to the child who has achronic illness, other healthy children in the family are aware of the stresses in the family, andtheir needs and concerns also must be addressed. The pediatrician can be an invaluableresource to families who need assistance accessing the appropriate community resources tohandle the emotional and financial stresses of caring for a child who has a disability or chronicillness.

Children who have disabilities are at increased risk of abuse and neglect in educational andchild care settings, but the child described in the vignette is in a stable educational environmentwith caregivers who likely know him well. His mother needs to be available in case of anemergency during school hours, but she should be supported if she chooses to work. The childis unlikely to qualify for home nursing services. He should receive routine and additionalimmunizations, such as influenza, depending on his risk for serious sequelae of infection.

References:

Perrin JM. Chronic illness in childhood. In: Behrman RE, Kliegman RM, Jenson HB, eds. NelsonTextbook of Pediatrics. 17th ed. Philadelphia, Pa: WB Saunders Co; 2004:135-138

Michaud LJ. Therapy services and anticipatory guidance for the child with a disability. In:Rudolph CD, Rudolph AM, Hostetter MK, Lister G, Siegel NJ, eds. Rudolphs Pediatrics. 21st ed.New York, NY: McGraw-Hill; 2003:544-553

Copyright © 2007 by the American Academy of Pediatrics page 492

Page 493: AAP MCQ 2007

2007 PREP SA on CD-ROM

Question: 236

You are discussing diarrheal diseases with a group of medical students interested ininternational health. You advise them that there are more than 2,000 serovars of Salmonella.

Of the following, the serovar that has the MOST public health implications is

A. Heidelberg

B. Newport

C. Paratyphi

D. Typhi

E. Typhimurium

Copyright © 2007 by the American Academy of Pediatrics page 493

Page 494: AAP MCQ 2007

2007 PREP SA on CD-ROM

Preferred Response: DCritique: 236

The serovar of Salmonella that has the most public health implications is Typhi, the causativeagent for typhoid fever. Because humans are the only reservoir for Salmonella serovar Typhi,contact with an infected person is necessary for infection to occur. Salmonella serovarParatyphi may cause an enteric fever, but it is recovered less commonly than Salmonellaserovar Typhi. Salmonella serovar Heidelberg, Newport, and Typhimurium can becomeextraintestinal and cause bacteremia or other clinical illness, but they do not have the publichealth implications of Salmonella serovar Typhi.

In a patient who has symptoms suggestive of Salmonella infection, the diagnosis may bemade by culture of the stool or normally sterile body fluids (eg, blood, cerebrospinal fluid). Othermethods to detect Salmonella serovar Typhi have included rapid serum tests using enzymeimmunoassays and latex agglutination, but these have met with little success. DNA detectionthrough the use of polymerase chain reaction testing is both sensitive and specific but is notwidely available. The Widal test (febrile agglutinins; antibody detection of O and H antigens fromSalmonella serovar Typhi) is not reliable and should not be used.

References:

American Academy of Pediatrics. Salmonella infections. In: Pickering LK, ed. Red Book: 2006Report of the Committee on Infectious Diseases. 27th ed. Elk Grove Village, Ill: AmericanAcademy of Pediatrics; 2006:579-584

Cleary TG. Salmonella. In: Behrman RE, Kliegman RM, Jenson HB, eds. Nelson Textbook ofPediatrics. 17th ed. Philadelphia, Pa: WB Saunders Co; 2004:912-918

Copyright © 2007 by the American Academy of Pediatrics page 494

Page 495: AAP MCQ 2007

2007 PREP SA on CD-ROM

Question: 237

A 12-year-old African-American girl presents to your office with a 2-day history of grosshematuria. She describes the urine as brown. She states that she has had an upper respiratorytract infection for about 3 days. She denies dysuria, urgency, or frequency. Her vital signs andphysical examination findings are normal. Urinalysis reveals: specific gravity, 1.025; pH, 6.5;large blood; no protein; too numerous-to-count red blood cells; and 0 to 2 white blood cells.Serum electrolyte concentrations are normal.

Of the following, the MOST likely cause of her gross hematuria is

A. focal segmental glomerulosclerosis

B. immunoglobulin A nephropathy

C. lupus nephritis

D. membranoproliferative glomerulonephritis

E. papillary necrosis

Copyright © 2007 by the American Academy of Pediatrics page 495

Page 496: AAP MCQ 2007

2007 PREP SA on CD-ROM

Preferred Response: BCritique: 237

Immunoglobulin A nephropathy (IgAN) or Berger disease is the most common cause of primaryglomerulonephritis in adults in the world. In contrast, only about 5% to 10% of children who haveglomerular disease exhibit IgAN. Although once considered to be a random disease with nogenetic component, the prevailing opinion now is that there may be a strong familial componentto IgAN.

The pathogenesis of IgAN remains elusive. It is not a classically defined autoimmunedisease, although the disease develops from deposition of IgA-containing immune complexes inthe kidney. It is tempting to assume that the pathogenesis is related to mucosal IgA because thedisease often follows an upper respiratory tract infection, but deposited IgA is predominantlypolymeric IgA1. The association of some cases of IgAN with syndromes that affect therespiratory or gastrointestinal (GI) tracts supports the relationship between IgAN and theimmune system. Moreover, it has been shown that the gross hematuria worsens during or afterupper respiratory tract or GI infections. Serum IgA concentrations may be elevated in about50% of patients who have IgAN, making this finding relatively nonspecific and not diagnostic.The natural history of the disease varies. About one third of patients have a benign course, onethird experience slow progression to renal failure, and one third have a more progressive coursein which renal failure develops within 20 years. IgAN is more common in whites and Asians thanother ethnic groups and is observed frequently in American Indians. IgAN is more common inmales than females.

The girl in the vignette has the classic presentation of IgAN: a brief history of painless, grosshematuria following an upper respiratory tract infection. Her urinalysis demonstrates toonumerous-to-count red blood cells but no protein, and her renal function is normal. Children whohave focal segmental glomerulosclerosis may develop gross hematuria, but this is not common.Patients who have lupus nephritis may develop either microscopic or gross hematuria, but theyoften have associated symptoms (eg, rashes, arthritis, anorexia). Children who havemembranoproliferative glomerulonephritis may develop gross hematuria, but they often presentwith proteinuria, edema, and hypertension. Finally, children who have papillary necrosis usuallyhave sickle cell disease or trait, and the gross hematuria generally is associated with pain,typically abdominal or flank pain.

The follow-up of a patient who has IgAN includes at least quarterly assessment of urineprotein by urinalysis and determination of a random protein-to-creatinine ratio (P/C). A risingurine P/C is highly predictive of progressive renal disease. Frequent measurement of serumelectrolyte values is vital, especially for patients who have evidence of significant proteinuria orhypertension. Treatment typically begins with agents to reduce proteinuria: angiotensin-converting enzyme inhibitors or angiotensin receptor blockers. Oral corticosteroids may beeffective in some cases but generally do not alter the final outcome significantly. Use ofcalcineurin inhibitors such as cyclosporine or mycophenolate mofetil has been shown to slowthe progression of IgAN in adults, but larger trials are necessary to determine their efficacy fully.

References:

Mitsioni A. IgA nephropathy in children. Nephrol Dial Transpl. 2001;16(suppl 6):123-125.Available at: http://ndt.oxfordjournals.org/cgi/reprint/16/suppl_6/123

Utsunomiya Y, Koda T, Kado T, et al. Incidence of pediatric IgA nephropathy. Pediatr Nephrol.2003;18:511-515. Abstract available at:http://www.ncbi.nlm.nih.gov/entrez/query.fcgi?orig_db=PubMed&db=PubMed&cmd=Search&term=%22Pediatric+nephrology+(Berlin,+Germany)%22[Jour]+AND+511[page]+AND+2003[pdat]

Yoshikawa N, Tanaka R, Iijima K. Pathophysiology and treatment of IgA nephropathy in children.Pediatr Nephrol. 2001;16:446-457. Abstract available at:http://www.ncbi.nlm.nih.gov/entrez/query.fcgi?orig_db=PubMed&db=PubMed&cmd=Search&term=%22Pediatric+nephrology+(Berlin,+Germany)%22[Jour]+AND+446[page]+AND+2001[pdat]

Copyright © 2007 by the American Academy of Pediatrics page 496

Page 497: AAP MCQ 2007

2007 PREP SA on CD-ROM

Question: 238

You are seeing a 3-year-old boy for follow-up 2 days after cystoscopy to remove a piece of atoy that he inserted into his urethra. His mother states that he has been doing well at home otherthan occasionally complaining of pain when he urinates. Physical examination reveals only mildredness at the urethral meatus. Urinalysis is positive for leukocyte esterase, and microscopicexamination of the urine shows 10 to 20 white blood cells. Gram stain of the urine is negativeexcept for a few white blood cells. Urine culture is negative at 24 hours.

Of the following, the MOST likely cause of this patient's pyuria is

A. cystitis

B. Kawasaki disease

C. pyelonephritis

D. urethral instrumentation

E. urethral stone

Copyright © 2007 by the American Academy of Pediatrics page 497

Page 498: AAP MCQ 2007

2007 PREP SA on CD-ROM

Preferred Response: DCritique: 238

Urethritis is the response of the urethra to inflammation of any cause, both infectious andnoninfectious. The most common clinical presentation is dysuria, urinary frequency, and urethraldischarge or itching. Gram stain of the urethral secretions or of a first voided urine specimenmay show polymorphonuclear leukocytes. Urethral instrumentation or surgery and the presenceof a foreign body are two common causes of non-sexually transmitted urethritis. In these cases,the signs and symptoms are usually due to irritation and mild inflammation and generally resolvespontaneously once the offending agent is removed. Treatment is symptomatic. Secondaryinfection should be ruled out with urinalysis and urine culture. If an infection is present, antibiotictherapy should be directed toward the organism that is isolated.

The absence of fever, systemic symptoms, and red cells on urinalysis as well as thenegative urine culture in the patient described in the vignette make cystitis, pyelonephritis, andurethral stone unlikely. Although sterile pyuria may occur in Kawasaki disease, the child’ssymptoms are not consistent with this diagnosis.

References:

Krieger JN. Urethritis: etiology, diagnosis, treatment, and complications. In: Gillenwater JY,Grayhack JT, Howards SS, Mitchell ME, eds. Adult and Pediatric Urology. 4th ed. Philadelphia,Pa: Lippincott Williams & Wilkins; 2002:1849-1881

Copyright © 2007 by the American Academy of Pediatrics page 498

Page 499: AAP MCQ 2007

2007 PREP SA on CD-ROM

Question: 239

A 12-month-old male infant presents for an ear re-evaluation 1 month after being treated for hisfourth episode of otitis media. His parents describe a normal birth history and normaldevelopment. The child is breastfed and does not attend child care. His immunizations are up todate through 6 months of age, including three doses of the conjugated pneumococcal vaccine.There is no history of sinusitis, pneumonia, sepsis, meningitis, or urinary tract infections. Afterthe boy's last otitis media infection, your colleague measured the child's serum immunoglobulin(Ig) concentrations, and results included a low IgG of 150 mg/dL (1.5 g/L), a normal IgM of 80mg/dL (0.8 g/L), and a normal IgA of 40 mg/dL (0.4 g/L).

Of the following, the next BEST laboratory test to evaluate this infant's antibody function is

A. B- and T-cell flow cytometry

B. delayed-type hypersensitivity testing

C. isohemagglutinins

D. nitroblue tetrazolium test

E. serum protein electrophoresis

Copyright © 2007 by the American Academy of Pediatrics page 499

Page 500: AAP MCQ 2007

2007 PREP SA on CD-ROM

Preferred Response: CCritique: 239

The infant described in the vignette most likely has transient hypogammaglobulinemia of infancy,but the recurrent infections raise the possibility of an underlying immunodeficiency. Assessmentof antibody function is prudent for a child who presents with recurrent sinopulmonary infectionsand a low serum immunoglobulin (Ig) G concentration. The assessment may be either bymeasurement of serum isohemmaglutinins (ie, antibodies against blood antigens) in patients whodo not have type AB blood or comparison of pre- and postantibody responses to protein (eg,tetanus, diphtheria, measles, mumps, hepatitis B) and polysaccharide (eg, 23-valentpneumococcal) vaccines. Isohemmaglutinin testing is readily available because it is used inblood typing. Flow cytometry sorts cells by their shape, granularity, and cell surface markers.Low-to-absent B lymphocytes are seen with X-linked (Bruton) agammaglobulinemia, but furtherstudies would be needed to assess the functionality of the B lymphocyte.

Delayed-type hypersensitivity testing is a useful screening tool for T-lymphocyte function.This test involves intradermal application of an antigen (eg, tetanus, mumps, Candida,trichophyton), with interpretation 48 to 72 hours later.

The nitroblue tetrazolium test is a reduction assay that measures the superoxide burst ofneutrophils. The inability to create superoxide anions, as seen with chronic granulomatousdisease, results in ineffective bacterial phagocytosis, primarily for catalase-positive organisms(eg, Staphylococcus aureus, Aspergillus, Nocardia, Burkholderia cepacia).

Serum protein electrophoresis (SPEP) separates proteins based on their size, shape, andelectrical charge. A SPEP performed in the patient described in the vignette would show a smallgamma band, reflecting the patient’s low serum IgG level, but it does not assess qualitativeantibody function.

References:

Buckley RH. Primary defects in antibody production. In: Behrman RE, Kliegman RM, Jenson HB,eds. Nelson Textbook of Pediatrics. 17th ed. Philadelpha, Pa: WB Saunders Co 2004:689-692

Kidon MI, Handzel ZT, Schwartz R, Altboum I, Stein M, Zan-Bar I. Symptomatichypogammaglobulinemia in infancy and childhood - clinical outcome and in vitro immuneresponses. BMC Fam Pract. 2004;5:23. Available at: http://www.biomedcentral.com/1471-2296/5/23

Copyright © 2007 by the American Academy of Pediatrics page 500

Page 501: AAP MCQ 2007

2007 PREP SA on CD-ROM

Question: 240

A father carries his 5-year-old child into the emergency department after a motor vehicle crashin which his vehicle rolled over several times. The accident occurred 1 minute away. The boywas restrained by lap and shoulder belts. He did lose consciousness. In his father's arms, thechild moans softly but does not open his eyes.

Of the following, the MOST dangerous consequence of transporting this injured child to theemergency department in this manner is

A. delay in instituting intravenous fluid resuscitation

B. delay in providing supplemental oxygen

C. exacerbation of a cervical spine injury

D. hospital personnel not having adequate time to prepare for his arrival

E. vascular injury to the extremities

Copyright © 2007 by the American Academy of Pediatrics page 501

Page 502: AAP MCQ 2007

2007 PREP SA on CD-ROM

Preferred Response: CCritique: 240

The child described in the vignette has lost consciousness as a result of head trauma inflictedduring a motor vehicle collision. He should be assumed to have a cervical spine injury untilproven otherwise. Full immobilization with a cervical collar and a long spine board are indicatedduring transportation to a medical facility. Failure to immobilize his neck may cause or worsen acervical spine injury.

Motor vehicle accidents are the leading cause of spinal trauma in children, followed by falls.The incidence of spinal cord injury was approximately 1.5% in two studies that used nationaltrauma registry data. Approximately one third of children who have spinal trauma haveassociated spinal cord damage. Unlike adults, in whom recovery from spinal cord damage israre, children show a high rate of partial neurologic recovery from such injuries.

The initial evaluation of an accident victim should focus on the ABCs (airway, breathing, andcirculation), followed by a brief neurologic examination to assess for disability or neurologicdeficits. As long as the spine is protected, exclusion of a spinal cord injury may be safelydeferred until systemic instability has been addressed.

Evaluation for cervical spine injury should be undertaken in the conscious patient whoexhibits tenderness to palpation over the spine and all patients who have altered levels ofconsciousness. Lateral, anteroposterior, and odontoid radiographs identify up to 92% of allcervical spine fractures. The lateral films should include the base of the skull, all seven cervicalvertebrae, and the first thoracic vertebra. If complete visualization of this region is not possiblewith the lateral view, a “swimmer’s view” of the lower cervical and upper thoracic area should beobtained. If there is still difficulty in visualizing this area, computed tomography scan of the neckshould be ordered. In the presence of a significant fracture in a comatose patient or a neurologicdeficit in a conscious patient, magnetic resonance imaging should be obtained to provide thehighest quality images of the spinal cord.

Pharmacologic therapy with methylprednisolone after acute cervical spinal cord injury hasbeen recommended in the past after initial studies appeared to show benefit. Later studies havenot confirmed the benefit of methylprednisolone. This is a controversial area, with no clearconsensus on the standard of care. The current Advanced Trauma Life Support Manualcontinues to recommend its use.

The transfer of the child in the vignette via private automobile should not cause a delay in theadministration of either intravenous fluids or supplemental oxygen. It is unlikely that a vascularinjury would be worsened by this mode of transport. Finally, it is helpful but not essential to notifyemergency department personnel of an arriving trauma victim.

References:

American College of Surgeons. Spine and spinal cord trauma. In: Advanced Trauma LifeSupport® Program for Doctors. 6th ed. Chicago, Ill: American College of Surgeons; 1997:215-230

Carreon LY, Glassman SD, Campbell MJ. Pediatric spine fractures: a review of 137 hospitaladmissions. J Spinal Disord Tech. 2004;17:477-482. Abstract available at:http://www.ncbi.nlm.nih.gov/entrez/query.fcgi?orig_db=PubMed&db=PubMed&cmd=Search&term=J+Spinal+Disord+Tech[Jour]+AND+477[page]+AND+2004[pdat]

Martin BW, Dykes E, Lecky FE. Patterns and risks in spinal trauma. Arch Dis Child. 2004;89:860-865. Available at: http://adc.bmjjournals.com/cgi/content/full/89/9/860

Pharmacological therapy after acute cervical spinal cord injury. Neurosurgery.2002;50(suppl):S63-S72. Abstract available at:http://www.ncbi.nlm.nih.gov/entrez/query.fcgi?cmd=Retrieve&db=pubmed&dopt=Abstract&list_uids=12431289&query_hl=38&itool=pubmed_docsum

Wang MY, Hoh DJ, Leary SP, Griffith P, McComb JG. High rates of neurological improvement

Copyright © 2007 by the American Academy of Pediatrics page 502

Page 503: AAP MCQ 2007

2007 PREP SA on CD-ROM

following severe traumatic pediatric spinal cord injury. Spine. 2004;29:1493-1497. Abstractavailable at:http://www.ncbi.nlm.nih.gov/entrez/query.fcgi?orig_db=PubMed&db=PubMed&cmd=Search&term=Spine[Jour]+AND+1493[page]+AND+2004[pdat]

Copyright © 2007 by the American Academy of Pediatrics page 503

Page 504: AAP MCQ 2007

2007 PREP SA on CD-ROM

Question: 241

A 1-week-old infant presents for his first newborn evaluation. He had been dischargedapparently well and thriving at 48 hours of age. He now exhibits grouped vesicles (Item Q241A)on an erythematous base that were not present at birth. Wright stain of scrapings from the floorof the vesicles reveals multinucleated giant cells (Item Q241B) and balloon cells.

Of the following, the MOST likely diagnosis is

A. bullous impetigo

B. congenital varicella

C. herpes simplex virus infection

D. incontinentia pigmenti

E. recessive dystrophic epidermolysis bullosa

Copyright © 2007 by the American Academy of Pediatrics page 504

Page 505: AAP MCQ 2007

2007 PREP SA on CD-ROM

Preferred Response: CCritique: 241

Grouped vesicles (Item C241A) on an erythematous base, as described for the infant in thevignette, suggests neonatal herpes simplex virus (HSV) infection. Lesions may erupt anywhere,but vesicles on the scalp or buttocks are particularly common. Monitoring electrodes mayproduce sufficient skin trauma to expedite invasion by HSV and induce skin lesions. Vesiclesmay be present at birth, but onset after delivery is more likely. Up to 30% of infants who haveneonatal HSV do not exhibit skin lesions. Mucous membrane involvement is common. HSV innewborns may present as: 1) disseminated disease involving multiple organs; 2) localizedcentral nervous system disease; or 3) disease localized to the skin, eyes, and mouth. In theabsence of skin lesions, it is difficult to diagnose neonatal HSV infection. In neonates, HSVinfection should be considered in the differential diagnosis of fever, irritability, and abnormalcerebrospinal fluid findings. Neonatal HSV infections are very serious and associated with highmortality and morbidity rates. Initial symptoms of HSV infection may present between birth and 1month of age.

Bullous impetigo (Item C241B) is a common skin manifestation of Staphylococcus aureusinfection. As the name suggests, lesions are bullae (or crusted round erosions if the bullaerupture), not vesicles. Gram stain of the contents of a bulla reveals clusters of gram-positivecocci, and a bacterial culture can confirm the presence of Staphylococcus aureus. Congenitalvaricella is rare, but may mimic HSV infection in the newborn. Affected infants exhibit vesicles inassociation with abnormalities involving the eye, central nervous system, gastrointestinalsystem, or limbs. A linear arrangement of vesicles that have minimal erythema at the base ischaracteristic of incontinentia pigmenti (Item C241C). Rapid development of the “warty”hyperkeratotic stage distinguishes incontinentia pigmenti from HSV infection. Hemorrhagic bullaeappear at sites of even minimal skin trauma in newborns who have recessive dystrophicepidermolysis bullosa (RDEB). Removal of the blister roof leaves a raw, bleeding base that healswith scar formation. Early loss of finger nails as a result of blistering and scarring ischaracteristic of RDEB.

References:

American Academy of Pediatrics. Herpes simplex. In Pickering LK, ed. Red Book: 2006 Reportof the Committee on Infectious Diseases. 27th ed. Elk Grove Village, Ill: American Academy ofPediatrics; 2006:361-371

Kohl S. Herpes simplex virus. In: Behrman RE, Kliegman RM, Jenson HB, eds. Nelson Textbookof Pediatrics. 17th ed. Philadelphia, Pa: WB Saunders Co; 2004:1051-1056

Weston WL, Lane AT, Morelli JG. Genodermatoses: epidermolysis bullosa, ectodermaldysplasias, incontinentia pigmenti. In: Color Textbook of Pediatric Dermatology. 3rd ed. St.Louis, Mo: Mosby; 2002:274-283

Weston WL, Lane AT, Morelli JG. Skin diseases in newborns: transient skin disease. In: ColorTextbook of Pediatric Dermatology. 3rd ed. St. Louis, Mo: Mosby; 2002:299-308

Copyright © 2007 by the American Academy of Pediatrics page 505

Page 506: AAP MCQ 2007

2007 PREP SA on CD-ROM

Question: 242

The father of three children in your practice recently was diagnosed with Crohn disease. Hiswife does not have Crohn disease. He asks you if his children, ages 10, 12, and 16 years, are atincreased risk for developing the same illness.

Of the following, you are MOST likely to advise the father that

A. although his children are at increased risk of developing Crohn disease, their risk of developingulcerative colitis is decreased

B. Crohn disease in childhood usually presents in children younger than age 5 years

C. each of his children has at least a 20% chance of developing Crohn disease during his or herlifetime

D. most patients who have Crohn disease can be diagnosed by genetic testing

E. smoking is associated with an increased risk of developing Crohn disease

Copyright © 2007 by the American Academy of Pediatrics page 506

Page 507: AAP MCQ 2007

2007 PREP SA on CD-ROM

Preferred Response: ECritique: 242

Crohn disease is an illness characterized by regional intestinal inflammation, most commonlyinvolving the terminal ileum or colon. However, the upper small bowel and stomach also may beinvolved. Presenting features of Crohn disease include abdominal pain, diarrhea, rectal bleeding,growth failure, or perianal inflammation. Diagnosis typically is established by a combination ofradiologic imaging and endoscopy.

Similar to coronary artery disease or diabetes, inflammatory bowel diseases (Crohn diseaseand ulcerative colitis) are complex polygenic conditions that result from interactions betweengenetic predisposition and the environment. People of northern European and Jewish descentare at increased risk of Crohn disease. Family history is a known risk factor for bothinflammatory bowel diseases, and probands who have one type (eg, Crohn) may have a relativewho has the other type (eg, ulcerative colitis). Thus, if a father has Crohn disease, the risk ofhis children developing either Crohn disease or ulcerative colitis is increased over the generalpopulation. Between 5% and 20% of patients who have Crohn disease have a similarly affectedfirst-degree relative. Crohn disease typically presents in adolescence and young adulthood andis rare in children younger than age 5 years.

The genetics and epidemiology of Crohn disease are under active investigation. In 2001,mutations in the NOD2 gene were identified in approximately 25% of patients who had Crohndisease. This gene is found primarily in human macrophages and is involved in the response ofthe immune system to bacterial lipid antigens (innate immunity). It is, therefore, theorized that theintestinal inflammation in the disease occurs in patients genetically predisposed to immuneactivation by common bacterial antigens. Currently, although genetic testing for NOD2 mutationsis available, there is no genetic test that detects all types of Crohn disease.

Environmental triggers may play a role in initiating the disease. The best establishedenvironmental risk factor for Crohn disease is smoking, with many epidemiologic studiessuggesting a twofold relative risk in smokers.

References:

Hyams JS. Inflammatory bowel disease. Pediatr Rev. 2005;26:314-320 . Available at:http://pedsinreview.aappublications.org/cgi/content/full/26/9/314

Griffiths A, Hugot J-P. Crohn disease. In: Walker WA, Goulet O, Kleinman RE, Sherman PM,Shneider BL, Sanderson IR, eds. Pediatric Gastrointestinal Disease. 4th ed. Hamilton, Ontario,Canada: BC Decker; 2005:789-824

Copyright © 2007 by the American Academy of Pediatrics page 507

Page 508: AAP MCQ 2007

2007 PREP SA on CD-ROM

Question: 243

You are admitting a 35-week gestation newborn to the neonatal intensive care unit forrespiratory distress at 4 hours of age. She requires assisted ventilation. The resident workingwith you asks what test of pulmonary function is preferred in your initial assessment of thisnewborn.

Of the following, the BEST response is

A. arterial blood gas testing

B. capillary blood gas testing

C. end-tidal carbon dioxide monitoring

D. pulse oximetry monitoring

E. transcutaneous Pao2 monitoring

Copyright © 2007 by the American Academy of Pediatrics page 508

Page 509: AAP MCQ 2007

2007 PREP SA on CD-ROM

Preferred Response: ACritique: 243

A newborn who has respiratory distress and needs assisted ventilation, such as the patientdescribed in the vignette, requires monitoring of the adequacy of both oxygenation (Pa2) andventilation (Pa2). Numerous monitoring methods have emerged over the years, but thepreferred method for assessing overall pulmonary function is arterial blood gas testing.

Standard methods exist for sampling arterial, venous, or arterialized capillary blood tomeasure blood gases. Such measures (and normal newborn ranges) include the pH (7.35 to7.45), Pa2 (35 to 45 mm Hg), and Pa2 (50 to 80 mm Hg). Standard nomograms employingthese measurements can calculate the serum bicarbonate and the base excess (or deficit).Using a co-oximeter, whole blood hemoglobin saturation also can be measured. Obviously, suchblood gas samples must be obtained via an invasive arterial or venous catheter, arterialpuncture, venipuncture, or heel-stick. The best representative of adequate pulmonary function isthe arterial blood gas.

The capillary blood gas sample is the least reliable manner to measure physiologic andbiochemical stability of the acutely ill newborn who has respiratory distress. The limits imposedby this technique include adequacy of tissue perfusion at the site of collection (heel), which isaddressed by heel warming to 39° to 42°F for several minutes; circulatory volume; andvasoconstriction due to circulatory shock or hypothermia. Capillary blood also rarely can beacquired without disturbing the infant. Hence, even the best sample of arterialized capillary bloodmay yield spuriously elevated Pa2. The P2 measurement from the capillary bed is not the sameas arterial Pa2 and only approximates it at best. In the presence of a dynamically changingrespiratory status in an ill newborn, the capillary blood gas can be used only to estimate grosschanges in Pa2 or pH. Oxygenation is monitored best by arterial blood gas sampling orcontinuous pulse oximetry.

Pulse oximetry is a measure of the arterial hemoglobin saturation acquired by infrared light-emitting diode technology across an extremity (finger, toe, hand) that is read by a detector onthe same extremity. The absorption of light by saturated hemoglobin contributes to thecalculated and averaged reading that is typically displayed in a digital readout on a bedsidemonitor. Hemoglobin saturation and its correlation with arterial Pa2 may be affected bytemperature, acid-base balance, adequacy of 2,3-diphosphoglycerol content of theerythrocytes, and the presence of fetal hemoglobin. Normal values in the newborn are 88% to96% saturation. The normal hemoglobin-oxygen saturation curve is widely available in standardtextbooks. It must be correlated with an arterial blood gas sample at some point in time to knowbest how adequately the pulse oximetry value represents the true arterial oxygenation of thepatient. This methodology does not provide any assessment of ventilation (no measure of Pa2).

Transcutaneous Pa2 monitoring is a limited and somewhat dated technology that relies onheated skin allowing for capillary blood oxygen to be measured across a membrane by a silverelectrode. The technology requires frequent machine calibration, changing the electrode siteevery 4 hours to avoid skin damage, and correlation with arterial blood gas Pa2. It does notprovide a measure of ventilation.

End-tidal P2 may be measured using an adapter on the connecting apparatus for theendotracheal tube to the ventilator. The gas typically is sampled through a side port, andaveraged values are read on a monitor. It is best used to follow trends in P2; real-timecorrelation with arterial blood gas Pa2 varies. It is subject to error with the accumulation ofmoisture in the circuit over time. It does not provide a measure of oxygenation.

References:

Durand DJ, Phillips B, Boloker J. Blood gases: technical aspects and interpretation. In:Goldsmith JP, Karotkin EH, eds. Assisted Ventilation of the Neonate. 4th edition. Philadelphia,Pa: Saunders; 2003:279-292

Copyright © 2007 by the American Academy of Pediatrics page 509

Page 510: AAP MCQ 2007

2007 PREP SA on CD-ROM

Hay WW Jr. History of pulse oximetry in neonatal medicine. NeoReviews. 2005;6:e533-e538.Available at: http://neoreviews.aappublications.org/cgi/content/full/6/12/e533

Henning R, South M. Respiratory failure. In: Taussig LM, Landau LI, eds. Pediatric RespiratoryMedicine. St. Louis, Mo: Mosby; 1999:404-430

Wood BR. Physiologic principles. In: Goldsmith JP, Karotkin EH, eds. Assisted Ventilation of theNeonate. 4th ed. Philadelphia, Pa: Saunders; 2003:15-40

Copyright © 2007 by the American Academy of Pediatrics page 510

Page 511: AAP MCQ 2007

2007 PREP SA on CD-ROM

Question: 244

A 2-year-old girl is brought to your clinic because her mother noticed blood in her underwear thatmorning. She has otherwise been doing well, and she recently has been toilet trained. Findingson the physical examination, including the hymen and external genitalia, are normal. There is asmall amount of purulent, bloody discharge at the vaginal introitus.

Of the following, the MOST likely cause of her bleeding is

A. penetrating trauma

B. precocious puberty

C. sarcoma botryoides

D. urethral prolapse

E. vaginal foreign body

Copyright © 2007 by the American Academy of Pediatrics page 511

Page 512: AAP MCQ 2007

2007 PREP SA on CD-ROM

Preferred Response: ECritique: 244

The differential diagnosis of vaginal bleeding in preadolescent girls varies with age. Thepresence of maternal estrogens can cause scant, often bloody, vaginal discharge in neonates inthe first few weeks after birth. In toddlers and older children, a foreign body in the vagina(frequently toilet paper) is a common cause of vaginal bleeding. Foul-smelling, bloody dischargeis evident on physical examination, and the foreign body may not be seen initially. Vulvovaginitis,which may be nonspecific or associated with group A streptococcal infection, also may result invaginal bleeding. Other less common causes that may not be evident on physical examinationinclude endodermal carcinoma, clear cell adenocarcinoma, and capillary venous malformation.

Evidence of penetrating trauma may not always be present in the case of sexual abuse, soa thorough history should be obtained and a high index of suspicion should be maintaineddespite normal examination findings. Precocious puberty can cause vaginal bleeding, but othersigns such as thelarche would be expected. Urethral prolapse is often confused with vaginalbleeding. Physical examination findings include a doughnut-shaped mass (Item C244A) at thevaginal introitus and blood in the underwear. Sarcoma botryoides is a polypoid form ofrhabdomyosarcoma that may be apparent on physical examination as a grapelike mass at theintroitus.

References:

Lang ME, Darwish A, Long AM. Vaginal bleeding in the prepubertal child. Can Med Assoc J.2005;172:1289-1290. Available at: http://www.cmaj.ca/cgi/content/full/172/10/1289

Sanfilippo JS. Bleeding. In: Behrman RE, Kliegman RM, Jenson HB, eds. Nelson Textbook ofPediatrics. 17th ed. Philadelphia, Pa: WB Saunders Co; 2004:1833

Copyright © 2007 by the American Academy of Pediatrics page 512

Page 513: AAP MCQ 2007

2007 PREP SA on CD-ROM

Question: 245

During a health supervision visit of a 6-year-old child, you ask the mother if there are any gunsin the home. She states that her husband is a hunter, but he keeps his shotgun in his pickuptruck.

Of the following, the BEST anticipatory guidance with regard to firearm safety is to tell the motherto

A. enroll herself and her child in gun safety classes

B. ensure that she specifically asks if other guns are in the home

C. ensure that there are gun safety locks on the shotgun

D. insist that the gun be stored in a locked gun cabinet or safe with ammunition locked separately

E. teach the child to use the gun properly at the earliest possible age

Copyright © 2007 by the American Academy of Pediatrics page 513

Page 514: AAP MCQ 2007

2007 PREP SA on CD-ROM

Preferred Response: DCritique: 245

Potential for injury in a household with children that also contains firearms is a significantconcern. According to the American Academy of Pediatrics policy statement on firearm injuriesthat was reaffirmed in 2004, unintentional shootings account for 24% of firearm-related deaths inchildren younger than 5 years of age, 26% in children 5 through 9 years of age, 21% in children10 through 14 years of age, and 5% in adolescents 15 through 19 years of age. It is imperativethat pediatricians address the presence of guns in the home at health supervision visits and thatanticipatory guidance include discussion of best safety practices. Clearly, for young children,such as the boy described in the vignette, locking unloaded firearms in a cabinet, withammunition stored separately in another locked location, is best.

Due to the potential for nonaccidental firearm injury among adolescents, surveillance forsigns of depression and behavioral concerns, in addition to maintaining the previously noted gunstorage safety practices, is paramount. Suicide is the leading cause of death in adolescence andhas a higher incidence when both guns and alcohol are accessible.

Recent studies suggest that the mother may be unfamiliar with the ownership of and storagepractices for firearms in the household. All parents should be encouraged to ask other familymembers if they own firearms and where they are located. Gun safety locks are helpful but notas effective as storing the gun safely in a locked gun cabinet away from ammunition. Gun safetyclasses have not been shown to reduce the risk of firearm injury for children. Indeed, they mayincrease the risk, perhaps because of decreased parental vigilance after the child hascompleted the course.

References:

American Academy of Pediatrics Committee on Injury and Poison Prevention. Firearm-relatedinjuries affecting the pediatric population. Pediatrics. 2000;105:888-895. Available at:http://pediatrics.aappublications.org/cgi/content/full/105/4/888

Azrael D, Miller M, Hemenway D. Are household firearms stored safely? It depends on whomyou ask. Pediatrics. 2000;106:e31. Available at:http://pediatrics.aappublications.org/cgi/content/full/106/3/e31

Brent RL, Weitzman M. The pediatrician’s role and responsibility in educating parents aboutenvironmental risks. Pediatrics. 2004;113:1167-1172. Available at:http://pediatrics.aappublications.org/cgi/content/full/113/4/S1/1167

Copyright © 2007 by the American Academy of Pediatrics page 514

Page 515: AAP MCQ 2007

2007 PREP SA on CD-ROM

Question: 246

You are called to see a hospitalized 9-year-old girl who suddenly has become dystonic, with herneck hyperextended, and is unable to move her eyes, now superiorly deviated. The nursesrelate that this girl has non-Hodgkin lymphoma and has been receiving highly emetogenicchemotherapy.

Of the following, the drug MOST likely to have caused this girl's symptoms and signs is

A. aprepitant

B. diphenhydramine

C. lorazepam

D. metoclopramide

E. ondansetron

Copyright © 2007 by the American Academy of Pediatrics page 515

Page 516: AAP MCQ 2007

2007 PREP SA on CD-ROM

Preferred Response: DCritique: 246

Metabolic disorders, degenerative diseases, and infections can produce movement disorders,but drug exposures also should be considered as causes of ataxia, tremor, and dystonia. Ataxiacan be triggered by anticonvulsant toxicity from phenytoin, phenobarbital, and evencarbamazepine. Alcohol and thallium also can lead to ataxia. Drugs that can cause tremorinclude amphetamines, valproic acid, neuroleptics such as phenothiazines, tricyclicantidepressants, caffeine, and theophylline. Stimulant medications can unmask Tourettesyndrome, but by themselves do not produce tics.

Certain drugs are capable of producing an acute dystonic reaction in children. Therapeuticdoses of phenytoin or carbamazepine rarely cause progressive dystonia in children who haveepilepsy and an underlying structural abnormality of the brain. Children may have anidiosyncratic reaction to neuroleptic drugs, characterized by acute dystonic posturing that maybe confused with encephalopathy. Intravenous diphenhydramine, 1 to 2 mg/kg per dose, mayreverse the drug-related dystonia rapidly. Severe rigidity combined with high fever and deliriumalso may occur as part of the neuroleptic malignant syndrome a few days after the initiation ofneuroleptic drugs.

The 9-year-old girl described in the vignette is experiencing a classic acute dystonicreaction, with posturing and even oculogyric crisis. Metoclopramide is in the family of neurolepticdrugs, although used most often for nausea or gastrointestinal motility and most likely is thecause of the dystonic reaction. The antiemetics lorazepam and ondansetron do not inducemovement disorders, and diphenhydramine is used to reverse dystonia from metoclopramide.Aprepitant is a new substance P/neurokinin-1 antagonist used for chemotherapy-inducednausea and vomiting. It can cause weakness or dizziness, but does not cause movementdisorders.

References:

Johnston MV. Movement disorders. In: Behrman RE, Kliegman RM, Jenson HB, eds. NelsonTextbook of Pediatrics. 17th ed. Philadelphia, Pa: WB Saunders Co; 2004:2019-2023

Pranzatelli MR. Movement disorders in childhood. Pediatr Rev. 1996;17:388-394

Copyright © 2007 by the American Academy of Pediatrics page 516

Page 517: AAP MCQ 2007

2007 PREP SA on CD-ROM

Question: 247

You are called to evaluate an 18-month-old infant who was found playing with a medicationbottle. The top was open, and many of the pills were on the floor. The prescription was for atricyclic antidepressant used by the patient's older brother. On physical examination, the boyappears clumsy but awake and flushed. He has a heart rate of 150 beats/min, a respiratory rateof 28 breaths/min, and dilated pupils. He is breathing easily and has strong pulses.

Of the following, the MOST important next step in this patient's management is

A. administration of syrup of ipecac

B. electrocardiography and continuous cardiac monitoring

C. lidocaine therapy to prevent ventricular arrhythmias

D. measurement of serum drug levels for tricyclic antidepressants

E. measurement of serum electrolyte levels

Copyright © 2007 by the American Academy of Pediatrics page 517

Page 518: AAP MCQ 2007

2007 PREP SA on CD-ROM

Preferred Response: BCritique: 247

Tricyclic antidepressants (TCAs) are a commonly prescribed group of drugs for severaldisorders, although the introduction of the newer classes of antidepressants, serotonin reuptakeinhibitors, may reduce the use of TCAs. Nonetheless, toxicity from TCAs remains a major causeof morbidity and mortality in children. With their widespread use in older children, adolescents,and adults, toddlers continue to be at risk for accidental ingestion of TCAs.

When TCAs are ingested in toxic amounts, they primarily affect the central nervous andcardiovascular systems. Central nervous system signs and symptoms of TCA toxicity includeirritability, euphoria, seizures, and unresponsiveness. There also may be autonomic nervoussystem symptoms, such as mydriasis, dry skin, dry mouth, urinary retention, and tachycardia.Among the direct effects on the cardiac system are a delay in signal conduction through thebundle of His, depression of myocardial contractile function, and prolongation of the QRS andthe QT intervals. These latter cardiac effects may potentiate arrhythmia formation.

Electrocardiography (ECG) with subsequent cardiac monitoring is essential in the evaluationof the child suspected of having a TCA toxic ingestion. Monitoring the ECG complex should becontinued for several hours because cardiac dysrhythmias may occur late after the ingestion.The voltage intervals should be measured, with particular attention to the QRS duration and theQT interval. A QRS duration of greater than 100 msec is associated with the development ofseizures; a QRS duration of more than 160 msec is associated with ventricular dysrhythmiasthat may be particularly difficult to treat. Measuring serum drug concentrations of TCAs is nothelpful in the prognosis or management of TCA ingestion. Serum electrolytes may be measured,but the results do not carry predictive value in the management of TCA toxicity. Providing syrupof ipecac is not indicated in patients who have ingested TCAs. Lidocaine therapy is not indicatedin the absence of ventricular arrhythmias.

References:

Kerr GW, McGuffie AC, Wilkie S. Tricyclic antidepressant overdose: a review. Emerg Med J.2001;18:236-241. Available at: http://emj.bmjjournals.com/cgi/content/full/18/4/236

Michael JB, Sztajnkrycer MD. Deadly pediatric poisons: nine common agents that kill at lowdoses. Emerg Med Clin North Am. 2004;22:1019-1050. Abstract available at:http://www.ncbi.nlm.nih.gov/entrez/query.fcgi?db=pubmed&cmd=Retrieve&dopt=AbstractPlus&list_uids=15474780

Pérez-Fontán J, Lister G. The acutely ill infant and child. In: Rudolph CD, Rudolph AM, HostetterMK, Lister G, Siegel NJ, eds. Rudolph’s Pediatrics. 21st ed. New York, NY: McGraw-Hill;2003:271-400

Copyright © 2007 by the American Academy of Pediatrics page 518

Page 519: AAP MCQ 2007

2007 PREP SA on CD-ROM

Question: 248

In examining a 4-year-old girl who is new to your practice, you discover that she hasrudimentary thumbs and is well below the 5th percentile for both weight and height. You alsoobserve irregular hyperpigmentation on the trunk and anogenital areas.

Of the following, the MOST likely hematologic disorder associated with these findings is

A. acute lymphoblastic leukemia

B. Bloom syndrome

C. Diamond-Blackfan anemia

D. Fanconi anemia

E. thrombocytopenia and absent radii (TAR) syndrome

Copyright © 2007 by the American Academy of Pediatrics page 519

Page 520: AAP MCQ 2007

2007 PREP SA on CD-ROM

Preferred Response: DCritique: 248

Approximately 80% of patients who have Fanconi anemia exhibit congenital anomalies.Recognizing these anomalies is important because the aplastic anemia usually is not evidentuntil 4 to 14 years of age. Pancytopenia at birth is unusual. Among the unusual featuresdescribed are hyperpigmentation (60% of patients); short stature (57%); upper limb anomalies(48%) that include absent, hypoplastic, supernumerary, or bifid thumbs and aplasia of the firstmetacarpal or the radius; hypogonadism (40%); and microcephaly (27%). Almost 20% ofaffected patients manifest a variety of other skeletal anomalies. The only potential cure for thisautosomal recessive disorder is bone marrow transplantation, and early diagnosis allows forbetter donor identification and patient preparation. It is also important to make the diagnosis toenable parents to choose prenatal diagnosis for future pregnancies. Several complementationgroups have been identified in Fanconi anemia, with type A accounting for almost 66% of cases.If the specific complementation group can be identified in the index case, DNA analysis can beused for prenatal diagnosis. If a specific DNA defect is not identified, increased chromosomalbreakage in peripheral blood lymphocytes in the presence of cross-linking agents can be used.

Thrombocytopenia and absent radii (TAR) syndrome is also known as congenitalamegakaryocytic thrombocytopenia. The classic finding of absent radii and present thumbsdifferentiates it from Fanconi anemia (Item C248A). Anomalies of hands, shoulders, and lowerlimbs also may be seen. TAR presents with isolated thrombocytopenia in the neonate, but 45%of patients ultimately develop aplastic anemia. Inheritance is complex but is usually autosomalrecessive.

Diamond-Blackfan anemia, or congenital hypoplastic anemia, is associated with physicalabnormalities in approximately 25% of patients. Anomalies include dysmorphic facial features,micrognathia, flattening of the thenar eminences, and triphalangeal thumbs.

Bloom syndrome is a rare chromosomal breakage syndrome in which several skeletalanomalies may be found; the major hematologic risk is development of leukemia and othermalignancies. Only mild anemia is typical.

Acquired disorders also may be associated with both hematologic and skeletalabnormalities. Children who have newly diagnosed acute lymphoblastic leukemia often presentwith bone pain and generalized radiologic signs, but they do not have congenital abnormalities.

References:

D’Andrea AD. The constitutional pancytopenias. In: Behrman RE, Kliegman RM, Jenson HB,eds. Nelson Textbook of Pediatrics. 17th ed. Philadelphia Pa: Saunders; 2004:1642-1644

Wong W, Glader B. Approach to the newborn who has thrombocytopenia. NeoReviews.2004;5:e444. Available at: http://neoreviews.aappublications.org/cgi/content/full/5/10/e444

Copyright © 2007 by the American Academy of Pediatrics page 520

Page 521: AAP MCQ 2007

2007 PREP SA on CD-ROM

Question: 249

A 14-year-old girl presents with complaints of lower abdominal cramps that have occurred ondays 1 and 2 of her menstrual cycle for the past year. The cramps usually are associated withbloating and loose bowel movements. The symptoms have caused school absences on the firstday of the past three periods. Use of acetaminophen has provided no relief. Menarche was atage 12 years. Her menstrual cycle is regular; menses last 5 days and are characterized bymoderate flow.

Of the following, the MOST appropriate management of these symptoms is the use of

A. acetaminophen with codeine

B. fluoxetine

C. montelukast

D. naproxen sodium

E. tramadol hydrochloride

Copyright © 2007 by the American Academy of Pediatrics page 521

Page 522: AAP MCQ 2007

2007 PREP SA on CD-ROM

Preferred Response: DCritique: 249

Dysmenorrhea is the most common menstrual disorder among adolescents. Most studies reportthat more than 50% of menstruating 12- to 17-year-old girls experience dysmenorrhea with eachmenstrual cycle. Primary dysmenorrhea is defined as crampy lower abdominal and pelvic painthat occurs with menses and is not associated with pelvic pathology. It is the most common formof dysmenorrhea and is caused by prostaglandins that are produced during ovulatory cyclesthat usually begin 6 to 12 months after menarche. Prostaglandins increase resting myometrialtone and stimulate uterine contractions that lead to myometrial ischemia and pain. Systemiccirculation of prostaglandins causes headaches, nausea, vomiting, and diarrhea. Numerousstudies have documented the effectiveness of nonsteroidal anti-inflammatory drugs (NSAIDs)that inhibit prostaglandins for the treatment of dysmenorrhea; the most commonly recommendedof which are ibuprofen and naproxen sodium. When initiated early for cramps and usedconsistently, they relieve symptoms in most adolescent patients who have dysmenorrhea. Foradolescents who have primary dysmenorrhea and fail to improve with adequate NSAID therapy,a contraceptive containing estrogen and progesterone may be beneficial.

The lack of responsiveness to acetaminophen experienced by the girl in the vignette isexpected; acetaminophen is a weak prostaglandin inhibitor. The use of acetaminophen withcodeine may provide pain relief, but fails to address the underlying mechanism of pain indysmenorrhea and may be associated with undesired adverse effects.

Fluoxetine, a selective serotonin reuptake inhibitor, has been beneficial for the treatment ofpremenstrual syndrome and menstrual dysphoric disorder in women. Montelukast, a leukotrienereceptor antagonist, has an anti-inflammatory effect that is useful for the treatment of asthma,but not dysmenorrhea. Tramadol hydrochloride is an analgesic that does not inhibitprostaglandins and, therefore, is not useful in the management of dysmenorrhea.

References:

Braverman PK, Neinstein LS. Dysmenorrhea and premenstrual syndrome. In: Neinstein LS, ed.Adolescent Health Care A Practical Guide.. 4th ed. Philadelphia, Pa: Lippincott Williams &Wilkins; 2002:952-965

Davis AR, Westhoff CL. Primary dysmenorrhea in adolescent girls and treatment with oralcontraceptives. J Pediatr Adolesc Gynecol. 2001;14:3-8. Abstract available at:http://www.ncbi.nlm.nih.gov/entrez/query.fcgi?cmd=Retrieve&db=pubmed&dopt=Abstract&list_uids=11358700&query_hl=40&itool=pubmed_docsum

Jenkins RR. Menstrual problems. In: Behrman RE, Kliegman RM, Jenson HB, eds. NelsonTextbook of Pediatrics. 17th ed. Philadelphia, Pa: Saunders; 2004:663-667

Laufer MR, Goldstein DP. Gynecologic pain: dysmenorrhea, acute and chronic pelvic pain,endometriosis, and the premenstrual syndrome. In: Emans SJH, Laufer MR, Goldstein DP, eds.Pediatric and Adolescent Gyencology.. 5th ed. Philadelphia, Pa: Lippincott, Williams & Wilkins;2004:417-476

Copyright © 2007 by the American Academy of Pediatrics page 522

Page 523: AAP MCQ 2007

2007 PREP SA on CD-ROM

Question: 250

During a health supervision visit with a 12-year-old girl, you palpate a firm thyroid gland (ItemQ250A). Measurement of free thyroxine is 2.0 ng/dL (25.7 pmol/L) (normal, 0.6 to 1.6 ng/dL [7.7to 20.6 pmol/L]), thyroid-stimulating hormone is less than 0.01 mcIU/mL (0.01 mcIU/L) (normal,0.5 to 5.0 mcIU/mL [0.5 to 5.0 mcIU/L]), and triiodothyronine is 468 pg/dL (7.2 pmol/L) (normal,60 to 180 pg/dL [0.92 to 2.8 pmol/L]). She has fine, smooth skin and finger and tongue tremor.

Of the following, the MOST likely additional finding on physical examination is

A. hepatomegaly

B. lymphadenopathy

C. positive Chvostek sign

D. pretibial myxedema

E. thyroid bruit

Copyright © 2007 by the American Academy of Pediatrics page 523

Page 524: AAP MCQ 2007

2007 PREP SA on CD-ROM

Preferred Response: ECritique: 250

The laboratory results reported for the girl in the vignette are indicative of hyperthyroidism.Physical findings of hyperthyroidism can include a thyroid bruit because of the blood coursingthrough a hyperactive and simulated thyroid gland and, in extremely rare instances in children,pretibial myxedema. The latter finding is more common in adults, but still very rare. Othersymptoms and signs include widened palpebral fissures or actual ophthalmopathy (lesscommon in children than adults), nervousness and irritability, sweatiness, myopathy, difficultyconcentrating in school and sleeping, daytime tiredness, weight loss, diarrhea, and heatintolerance.

Hyperthyroidism should not cause hypocalcemia or intensify the Chvostek sign.Hepatomegaly is not a sign of hyperthyroidism, although occasional individuals who experiencea “thyroid storm” may have hepatic disease, including jaundice. Lymphadenopathy is not afinding in thyrotoxicosis.

References:

Gold JG, Sadeghi-Nejad A. Hyperthyroidism. eMedicine Specialties: Pediatrics: Endocrinology.2006. Available at: http://www.emedicine.com/ped/topic1099.htm

LaFranchi S. Hyperthyroidism in childhood and adolescence. UptoDate. 2006; 14.1. Available at:http://uptodateonline.com/utd/content/topic.do?file=pediendo/5570&type=A&selectedTitle=3~131

Levitsky LL. Graves disease. eMedicine Specialities: Pediatrics: Endocrinology. 2004. Availableat: http://www.emedicine.com/ped/topic899.htm

Copyright © 2007 by the American Academy of Pediatrics page 524

Page 525: AAP MCQ 2007

2007 PREP SA on CD-ROM

Question: 251

You are seeing a new patient who is 11 years old and has Duchenne muscular dystrophy. Hisuncle died of the disorder 2 years ago. His mother asks to see you privately and tells you thather son does not know his diagnosis; she asks that you refer to his "walking problem." Onphysical examination, you note that the boy is in a wheelchair and is unable stand withoutsupport. You ask to speak with the mother after evaluating the boy.

Of the following, you are MOST likely to tell her that

A. he likely knows his diagnosis

B. you agree to her plan

C. you are recommending psychological evaluation for the boy

D. you are referring the family to child protective services

E. you are required to tell the boy his diagnosis

Copyright © 2007 by the American Academy of Pediatrics page 525

Page 526: AAP MCQ 2007

2007 PREP SA on CD-ROM

Preferred Response: ACritique: 251

Family members of a child who has a serious illness grieve for their child’s illness and the loss ofnormal childhood. Denial is common, and physicians can help families accept a diagnosis whilemaintaining hope for improvement or recovery. It is important to ask about the parent’sunderstanding of the diagnosis. Although family responses vary, based on history and culture,most parents require time to process and review new information, especially when they arebeing given an unexpected and unwanted diagnosis.

The boy in the vignette likely knows his diagnosis because he has an uncle who recentlydied with the same disorder. His mother is trying to protect him from the diagnosis and herselffrom having to discuss the diagnosis and prognosis with him. A primary care physician canfacilitate discussions regarding diagnosis and prognosis in cooperation with the family.Appropriate supportive personnel differ for specific situations, but social workers, counselors,community support groups, and clergy are examples of people who may be supportive to thefamily during this process.

Although there is no requirement to tell the boy his diagnosis, doing so would be beneficial.Therefore, working with the mother to achieve this goal is desirable. Counseling may beappropriate for both child and family members, but there is no specific need for psychologicalevaluation. Likewise, there is no need to refer the boy to child protective services unless he isnot receiving appropriate care. Rather than agreeing to the mother’s plan, the clinician shouldhelp her discuss the diagnosis with her son.

References:

Perrin JM. Chronic illness in childhood. In: Behrman RE, Kliegman RM, Jenson HB, eds. NelsonTextbook of Pediatrics. 17th ed. Philadelphia, Pa: WB Saunders Co; 2004:135-138

Copyright © 2007 by the American Academy of Pediatrics page 526

Page 527: AAP MCQ 2007

2007 PREP SA on CD-ROM

Question: 252

A 5-year-old girl presents after having a brief generalized seizure. Her mother reports that thechild has had a 3-day history of fever, tenesmus, and bloody diarrhea. On physical examination,you find a mildly toxic-appearing child who has a temperature of 104°F (40°C) and diffuseabdominal tenderness. The rectal examination produces significant pain. Stool from her rectumis guaiac-positive. You tell her mother that you believe the diarrhea has an infectious cause.

Of the following, the MOST likely pathogen is

A. Cryptosporidium sp

B. rotavirus

C. Salmonella sp

D. Shigella sp

E. Yersinia sp

Copyright © 2007 by the American Academy of Pediatrics page 527

Page 528: AAP MCQ 2007

2007 PREP SA on CD-ROM

Preferred Response: DCritique: 252

Patients who have symptoms of infections due to Cryptosporidium sp or rotavirus usually do notappear toxic, have fever, or have blood in their stool. Infections with Salmonella or Yersinia spcan produce these symptoms, but the degree of toxicity and tenesmus described for the patientin the vignette suggests an infection caused by Shigella sp.

Four species of Shigella have been identified: S sonnei, S flexneri, S boydii, and Sdysenteriae. In industrialized countries, S sonnei and S flexneri are the most commonpathogens. The virulence trait shared by all species of Shigella is the ability to invade intestine,which generally is more marked than with Salmonella infection. The pathologic changes ofinfection occur primarily in the colon, with mucosal edema, ulcerations, bleeding tissues, andexudates present upon inspection. Humans are the only natural host for this organism, and theprimary mode of transmission is fecal-oral contamination.

Neurologic manifestations (eg, seizures) are the most common extraintestinal manifestationof illness and occur in as many as 40% of patients. Such neurologic manifestations do not seemto be accounted for by the presence of Shiga toxin alone; the exact mechanism by which theyoccur is not well understood.

As with other forms of gastroenteritis, clinicians should address fluid and electrolyteproblems prior to considering antimicrobial therapy for patients who have Shigella infections.Unlike infections with other pathogens (eg, Salmonella), most experts suggest providingantimicrobial therapy for patients who are ill with infections due to Shigella. Resistance patternsvary by region, and the choice of antimicrobial agent should be guided by the resistance patternof the infection organism. Children who attend child care must have negative stool cultures priorto their return.

References:

American Academy of Pediatrics. Shigella infections. In: Pickering LK, ed. Red Book: 2006Report of the Committee on Infectious Diseases. 27th ed. Elk Grove Village, Ill: AmericanAcademy of Pediatrics; 2006:589-591

Cleary TG. Shigella. In: Behrman RE, Kliegman RM, Jenson HB, eds. Nelson Textbook ofPediatrics. 17th ed. Philadelphia, Pa: WB Saunders Co; 2004:919-920

Copyright © 2007 by the American Academy of Pediatrics page 528

Page 529: AAP MCQ 2007

2007 PREP SA on CD-ROM

Question: 253

A 3-month-old boy is admitted to the hospital for evaluation of failure to thrive. His birthweightwas at the 50th percentile and length at the 75th percentile. Currently, his weight is below the 5thpercentile and length is at the 25th percentile. His vital signs and physical examination resultsare otherwise normal. He appears well hydrated. Measurement of serum electrolytes reveals:sodium, 139 mEq/L (139 mmol/L); potassium, 4.7 mEq/L (4.7 mmol/L); chloride, 114 mEq/L (114mmol/L); bicarbonate 12 mEq/L (12 mmol/L); blood urea nitrogen, 8 mg/dL (2.9 mmol/L); andcreatinine, 0.3 mg/dL (26.5 mcmol/L). A consulting nephrologist recommends measurement ofurine pH (which is 7.5) and urine ammonium (which is 12,000 mcM/L) (normal, >60,000 mcM/L).

Of the following, the MOST likely cause of this child's acidosis is

A. inborn error of metabolism

B. lactic acidosis

C. type I (distal renal tubular) acidosis

D. type II (proximal renal tubular) acidosis

E. type IV renal tubular acidosis

Copyright © 2007 by the American Academy of Pediatrics page 529

Page 530: AAP MCQ 2007

2007 PREP SA on CD-ROM

Preferred Response: CCritique: 253

The infant described in the vignette presents with failure to thrive, and measurement of hisserum electrolytes reveals a normal anion gap and hyperchloremic metabolic acidosis. The keyelement in his presentation is the urine pH, which is 7.5, indicating that it is highly alkaline. Inproximal renal tubular acidosis (RTA), as the urine leaves the proximal tubule, the relativeinability to reabsorb bicarbonate results in a very alkaline pH of the tubular fluid. However, thedistal tubule in patients who have proximal RTA is healthy, and it extrudes high quantities of acid,dropping the tubular pH to a lower value (=6.0). In contrast, because the distal nephronrepresents the last real opportunity for the kidney to regulate acid-base balance, children whohave distal RTA cannot adequately compensate for the lack of acid extrusion, resulting in veryalkaline urine. The urine ammonium value in the child in the vignette is very low at 12,000 mcM/L(normal >60,000 mcM/L), which is consistent with reduced acid (as part of ammonium) secretionand strongly predictive of distal RTA.

RTA is a common cause of metabolic acidosis in childhood. There are two primary types ofRTA: proximal or type II and distal or type I. Type IV RTA, also called hyperkalemic RTA, isuncommon and will be discussed only briefly.

A review of basic renal physiology in the maintenance of acid-base balance is necessary tounderstand RTA. The primary responsibility of the proximal tubule in the preservation of normalacid-base homeostasis is to reabsorb filtered bicarbonate. It achieves this goal by reabsorbingwater and carbon dioxide from the renal (apical membrane) tubule into the cell and thenconverting them to hydrogen and bicarbonate within the cell. Hydrogen is extruded back into thetubule, and bicarbonate is reabsorbed at the other end (basolateral membrane) of the cell. Thus,the reabsorption of base (bicarbonate) is linked with the expulsion of acid (hydrogen). Incontrast, the primary role of the distal tubule is to secrete acid, either as free hydrogen ion, aspart of ammonium (ammonia plus hydrogen), or linked to phosphorous. The distal nephron doesplay a role in bicarbonate reabsorption, but its primary role is to secrete acid.

Because children who have proximal (type II) RTA have difficulty reabsorbing bicarbonate,the serum bicarbonate concentration falls, resulting in acidosis. In distal (type I) RTA, the distaltubule struggles to secrete acid. This excess acid must be buffered in the blood, principally bybicarbonate, which also results in a decline in the serum bicarbonate or acidosis. In distal RTA,there may be concomitant proximal tubule disturbances. The level of the serum bicarbonate ineither proximal or distal RTA may vary, depending on the extent of the deficit. To maintainelectroneutrality in RTA, the loss of bicarbonate is compensated by the retention of chloride, alsoa negatively charged ion. Thus, the anion gap ([sodium]-[chloride + bicarbonate]) is maintainedat 10 to 16. In other types of acidosis, such as lactic acidosis, the excess accumulation of aciddoes not induce an increase in chloride absorption and, thus, the anion gap is elevated.

Given the normal anion gap, this child cannot have either an inborn error of metabolism orlactic acidosis, which typically results in a high (>16) anion gap. Additionally, the normal serumpotassium value effectively eliminates type IV RTA as a cause of the acidosis because the lackof aldosterone or renal resistance to aldosterone in this condition induces acidosis andhyperkalemia.

References:

Adedoyin O, Gottlieb B, Frank R, et al. Evaluation of failure to thrive: diagnostic yield of testingfor renal tubular acidosis. Pediatrics. 2003;112:e463-e466. Available at:http://pediatrics.aappublications.org/cgi/content/full/112/6/e463

Roth KS, Chan JC. Renal tubular acidosis: a new look at an old problem. Clin Pediatr (Phila).2001;40:533-543. Abstract available at:http://www.ncbi.nlm.nih.gov/entrez/query.fcgi?orig_db=PubMed&db=PubMed&cmd=Search&term=%22Clinical+pediatrics%22[Jour]+AND+533[page]+AND+2001[pdat]

Watanabe T. Proximal renal tubular dysfunction in primary distal renal tubular acidosis. PediatrNephrol. 2005;20:86-88. Abstract available at:

Copyright © 2007 by the American Academy of Pediatrics page 530

Page 531: AAP MCQ 2007

2007 PREP SA on CD-ROM

http://www.ncbi.nlm.nih.gov/entrez/query.fcgi?cmd=Retrieve&db=pubmed&dopt=Abstract&list_uids=15549407&query_hl=72&itool=pubmed_docsum

Copyright © 2007 by the American Academy of Pediatrics page 531

Page 532: AAP MCQ 2007

2007 PREP SA on CD-ROM

Question: 254

You are asked to evaluate two children, ages 3 and 9 years, on the pediatric hematology-oncology inpatient unit who have developed fever, cough, increased work of breathing, andnodular lesions on their chest radiographs. The children are isolated in private rooms and havedifferent nurses and doctors caring for them. The children's rooms are located adjacent to anarea where a new playroom is being constructed.

Of the following, the MOST likely pathogen causing these patients' pneumonia is

A. Aspergillus sp

B. Candida parapsilosis

C. Legionella pneumophila

D. Pseudomonas aeruginosa

E. respiratory syncytial virus

Copyright © 2007 by the American Academy of Pediatrics page 532

Page 533: AAP MCQ 2007

2007 PREP SA on CD-ROM

Preferred Response: ACritique: 254

Transmission-based precautions in the hospital are designed for patients documented orsuspected to be colonized or infected with pathogens spread by the airborne, droplet, or contactroutes. Such patients require additional precautions beyond standard precautions to interrupttransmission or spread of disease.

Airborne transmission occurs by dissemination of airborne droplet nuclei (small-particleresidue of evaporated droplets containing microorganisms that remain suspended in the air forlong periods), dust particles containing the infectious agent, or fungal spores. Microorganismsspread by the airborne route can be dispersed widely by air currents (eg, ventilation systems,particularly during times of construction) and may be inhaled by or deposited on a susceptiblehost within the same room or a long distance from the source patient, depending onenvironmental factors. Therefore, special air handling and ventilation are required to preventairborne transmission. Spores of filamentous fungi, such as Aspergillus, represent aconsiderable environmental hazard, especially to severely immunosuppressed patients, such asthe children described in the vignette. Other microorganisms transmitted by airborne dropletnuclei include Mycobacterium tuberculosis, measles virus, varicella virus, and disseminatedzoster. Specific recommendations in airborne precautions include: the use of private rooms;negative air-pressure ventilation with externally exhausted or HEPA-filtered air, if recirculated;and wearing of respiratory protective devices in cases of suspected or proven tuberculosis.

Droplet transmission occurs when droplets containing microorganisms generated frominfected persons, primarily during coughing, sneezing, and talking, are propelled a short distanceand deposited on the host’s conjunctivae, nasal mucosa, or mouth. Because these relativelylarge droplets do not remain suspended in the air, special air handling and ventilation are notrequired to prevent transmission. Contact transmission is the most frequent route oftransmission of nosocomial infections and can occur by direct or indirect contact with theinfected or colonized surface.

Candida parapsilosis, Legionella pneumophila, and Pseudomonas aeruginosa are nottransmitted by the airborne route, and only standard precautions are needed to prevent spread.Respiratory syncytial virus is transmitted by direct or close contact with contaminatedsecretions, and limiting its spread in the hospital only requires contact precautions.

References:

American Academy of Pediatrics. Infection control for hospitalized children. In: Pickering LK, ed.Red Book: 2006 Report of the Committee on Infectious Diseases. 27th ed. Elk Grove Village, Ill:American Academy of Pediatrics; 2006:153-164

Edmond M. Isolation. Infect Control Hosp Epidemiol. 1997;18:58-64. Abstract available at:http://www.ncbi.nlm.nih.gov/entrez/query.fcgi?orig_db=PubMed&db=PubMed&cmd=Search&term=%22Infection+control+and+hospital+epidemiology+:+the+official+journal+of+the+Society+of+Hospital+Epidemiologists+of+America%22[Jour]+AND+58[page]+AND+1997[pdat]

Garner JS. The Hospital Infection Control Practices Advisory Committee. Guideline for isolationprecautions in hospitals. Infect Control Hosp Epidemiol. 1996;17:53-80

Copyright © 2007 by the American Academy of Pediatrics page 533

Page 534: AAP MCQ 2007

2007 PREP SA on CD-ROM

Question: 255

A 10-year-old boy presents to the emergency department with a severe asthma exacerbationthat requires intubation and admission to the intensive care unit. The child's parents state thatthis is the third hospital admission for asthma since he was diagnosed at age 6 years. Theparents are worried about the severity of their son's asthma flares and would like to know theunderlying risk factors for fatal asthma. His current medications include an oral antihistamine, aninhaled corticosteroid, and an as-needed beta-2 agonist.

Of the following, the risk factor MOST associated with fatal and near-fatal asthma is

A. coexisting allergic rhinitis

B. female sex

C. high socioeconomic status

D. inhaled corticosteroid use

E. poor perception of symptoms

Copyright © 2007 by the American Academy of Pediatrics page 534

Page 535: AAP MCQ 2007

2007 PREP SA on CD-ROM

Preferred Response: ECritique: 255

In the past 30 years, asthma-related case-fatality rates in industrialized countries have variedfrom 1.5 to 8.5 per 100,000 persons. Patients who have severe disease are believed to be atgreatest risk, but those who have mild disease may have near-fatal or fatal asthma attacks.Identified risk factors for near-fatal and fatal asthma attacks include marked circadian variationin lung function, male sex, psychosocial disturbances, frequent visits to the emergencydepartment, hospitalization for asthma within the past year, poor perception of hypoxia or airwayobstruction, use of two or more beta-agonist inhalers per month, low socioeconomic statuscoupled with a low educational level, and previous life-threatening asthma attacks. Neitherincreased inhaled corticosteroid use nor allergic rhinitis has been linked to near-fatal or fatalasthma attacks.

References:

Guill MF. Asthma update: clinical aspects and management. Pediatr Rev. 2004;25:335-344.Available at: http://pedsinreview.aappublications.org/cgi/content/full/25/10/335

Liu AH, Spahn JD, Leung DYM. Childhood asthma. In: Behrman RE, Kliegman RM, Jenson HB,eds. Nelson Textbook of Pediatrics. 17th ed. Philadelphia, Pa: WB Saunders Co 2004:760-774

Copyright © 2007 by the American Academy of Pediatrics page 535

Page 536: AAP MCQ 2007

2007 PREP SA on CD-ROM

Question: 256

A 17-year-old boy is brought to the emergency department after he drove his car head-on intoanother motor vehicle. He was not restrained and was ejected from the vehicle. On physicalexamination, he has a Glasgow Coma Scale score of 5 and severe head trauma with an openscalp wound that is actively bleeding. He is breathing spontaneously, but a portion of the rightside of his chest seems to move paradoxically, moving in with inspiration and out with expiration.On auscultation, he has decreased breath sounds on the affected side.

Of the following, the MOST likely cause for his chest examination abnormalities is

A. flail chest

B. hemothorax

C. lung contusion with splinting

D. open pneumothorax

E. tension pneumothorax

Copyright © 2007 by the American Academy of Pediatrics page 536

Page 537: AAP MCQ 2007

2007 PREP SA on CD-ROM

Preferred Response: ACritique: 256

The boy described in the vignette has a flail chest, a condition that occurs after blunt traumafractures two or more ribs in two or more places, interrupting the continuity of this segment ofthe chest wall with the rest of the thoracic cage. The injury typically results in paradoxic motionof the chest wall, as demonstrated by the patient in the vignette. Negative intrapleural pressureduring the inspiratory phase of breathing causes the segment to be pulled down into the chestcavity with chest expansion, and positive intrapleural pressure during the expiratory phasecauses the segment to bulge out from the thoracic cage with chest wall relaxation.

Flail chest can lead to respiratory insufficiency due to splinting from pain or to airway edemafrom an underlying pulmonary contusion. At the scene of the injury, the patient should be placedwith the injured segment down, allowing better expansion and ventilation of the unaffected side. Apatient who has flail chest and is in respiratory distress should undergo endotracheal intubationand be supported with positive pressure ventilation. This protects the airway and providesoptimal expansion and splinting of the affected segment. Most patients who have flail chest donot require intubation and can be managed with analgesics and aggressive pulmonaryphysiotherapy.

Hemothorax refers to the accumulation of blood in the pleural space. An open pneumothoraxis characterized by a penetrating injury to the thoracic wall that permits air to move in and out ofthe thoracic cavity with each breath. A tension pneumothorax (Item C256A) is caused by an airleak within the lung in the absence of a penetrating injury to the thoracic wall. This causes anincreasing amount of air under increasing pressure within the pleural space, leading to collapseof the ipsilateral lung. A patient who has splinting due to lung contusion limits ipsilateral chest wallmovement during both phases of respiration to decrease pain. Any of these conditions can bepresent in conjunction with a flail chest, but only flail chest causes paradoxic motion of asegment of the thoracic wall.

References:

American College of Surgeons. Thoracic trauma. In: Advanced Trauma Life Support® Programfor Doctors. Chicago, Ill: American College of Surgeons; 1997:125-141

Kadish HA. Thoracic trauma. In: Fleisher GR, Ludwig S, Henretig FM, eds. Textbook of PediatricEmergency Medicine. 5th ed. Philadelphia, Pa: Lippincott Williams & Wilkins; 2006:1433-1452

Copyright © 2007 by the American Academy of Pediatrics page 537